Вы находитесь на странице: 1из 493

LECTURE NOTES

ON

HEAT & MASS TRANSFER

1
11 ME 354: HEAT TRANSFER (4 0 0 4)

Course Objectives:

1. Develop the ability to apply energy balance to heat transfer systems to solve engineering problems.
2. Learn the ability to apply the definitions of conduction, convection and radiation heat transfer to
engineering systems.
3. Predict the thermal response of engineering systems to energy transfer mechanisms for transient
and steady state situations.
4. To have working knowledge of variety of engineering correlations to heat transfer analysis and
thermal design of engineering components and systems.

Course Contents

UNIT I 12 Hours

Chapter 1: Mechanisms of Heat Transfer- Basic laws governing each mechanism; combined
mechanisms; Illustrative examples
Chapter 2: Conduction Basic Equations and Boundary Conditions: Conduction Basic Equations:
one dimensional conduction equation in rectangular, cylindrical and spherical coordinates; thermal
diffusivity;3-dimensional conduction equation in Cartesian coordinates; boundary conditions of first,
second and third kind; radiation boundary condition ;illustrative examples on formulation of
conduction problems.
Chapter 3: One Dimensional Steady State Conduction: Analysis of one dimensional steady state
conduction in slab, radial conduction in cylinders and spheres without and with heat generation; one-
dimensional steady state conduction in composite medium- concept of thermal potential and thermal
resistance; contact resistance; critical thickness of insulation for cylinders and spheres; analysis of
fins of uniform cross section with different tip conditions; one-dimensional steady state conduction in
solids with variable thermal conductivity. Illustrative examples

UNIT II 10 Hours

Chapter 4 : Transient Conduction: Lumped system analysis with illustrative examples; Criterion
for neglecting internal temperature gradients in transient conduction analysis ;One dimensional
transient conduction in a slab subjected to convective boundary condition- solution of this problem in
the form of Transient-Temperature Chart. Similar charts for radial transient conduction in an infinite
cylinder and in a sphere; illustrative examples; Use of these charts to solve multi-dimensional
transient conduction problems-illustrative examples. Transient temperature charts for semi-infinite
solids; Conduction shape factor: Illustrative examples
Chapter 5: Finite Difference Methods for Conduction: Finite Difference equations for one
dimensional steady state conduction in slabs, cylinders and spheres; Finite difference equations for
two dimensional steady state conduction; Explicit finite difference equations for one dimensional
transient conduction in slabs, cylinders and spheres; Implicit Scheme for one dimensional transient
conduction; Illustrative examples.

UNIT III 12 Hours

Chapter 6:Convection-Basic Concepts: Basic concepts for flow over bodies-Velocity boundary
layer, thermal boundary layer, drag coefficient, general expression for heat transfer coefficient in
terms of temperature gradient; illustrative examples. Concepts for flow through duct- hydraulic

2
diameter; hydrodynamically developing and developed flow; thermally developing and thermally
developed flow; general expression for pressure drop and heat transfer coefficient for flow through
ducts; illustrative examples; Concept of turbulence- Prandtls mixing length theory; velocity
distribution in turbulent flow through tubes; Dimensionless Parameters in Forced Convection Flow
and their physical significance.
Chapter 7: Forced convection for flow inside ducts: Analysis of hydro dynamically and thermally
developed laminar flow-COUETTE flow, flow inside a circular tube; expressions for friction factor
and pressure drop for hydro dynamically and thermally developed laminar and turbulent flows;
hydrodynamic and thermal entrance lengths; use of correlations to determine pressure drop and heat
transfer coefficient for hydro dynamically and thermally developed flow through tubes; illustrative
examples.
Chapter 8: Forced convection for flow over bodies:Integral method of analysis for laminar
incompressible boundary layer over a flat plate; Correlations for drag coefficient and heat transfer
coefficient for flow over a flat plate, flow across a cylinder, flow across a sphere and flow across
tube bundles

UNIT IV 08 Hours

Chapter 9:Free convection :- Dimensionless parameters for free convective heat transfer problems;
Correlations for free convection from vertical plane surfaces, vertical cylinders, horizontal and
inclined plane surfaces, horizontal cylinders, enclosed spaces; combined forced and free convection;
illustrative examples
Chapter 10: Boiling & Condensation:- Nusselts theory for laminar film condensation on a vertical
plane surface; correlations for determining heat transfer coefficient for laminar and turbulent film
condensation on a plane vertical surface and horizontal tubes. Illustrative examples..

UNIT V 10 Hours

Chapter 11: Heat Exchangers: Classification of heat exchangers; overall heat transfer coefficient;
expressions for mean temperature difference for parallel flow, counter flow heat exchangers;
correction factors for other type of heat exchangers; Limitations of LMTD method ;Effectiveness-
NTU method for heat exchanger analysis; illustrative examples
Chapter 12: Radiation Heat Transfer Among Surfaces in a Non-Participating Medium: Basic
concepts and terms used in radiation heat exchange analysis; Plancks law,Stefan-Boltzman law,
Weins displacement Law,Kirchoffs law and Lamberts Law; Radiation heat exchanger between two
parallel infinite black and gray surfaces; Radiation exchange between two finite surfaces Concept of
View Factor; View factor algebra; Hottels Cross string formula; Net work method for analysis of
radiation heat exchange in an enclosure.

Text Book :
Fundamentals of Heat & Mass Transfer, F.P.Incropera, D.P.Dewitt, T.L.Bergman ,A.S.Lavine,
K.N.Seetharamu, T.R.Seetharam,Wiley IndiaPvt.Ltd,New Delhi, Sixth Edition, 201

Reference Books:
1. Heat Transfer A basic Approach,M.Necati Ozisik, McGraw Hill International Edition, 1985
2. Principles of Heat Transfer, Frank Kreith & Mark S. Bohn , Sixth Edition, Thomson Learning,
2001

3
CHAPTER 1

INTRODUCTORY CONCEPTS AND BASIC LAWS OF


HEAT TRANSFER
1.1. Introduction:- We recall from our knowledge of thermodynamics that heat is a form
of energy transfer that takes place from a region of higher temperature to a region of lower
temperature solely due to the temperature difference between the two regions. With the
knowledge of thermodynamics we can determine the amount of heat transfer for any system
undergoing any process from one equilibrium state to another. Thus the thermodynamics
knowledge will tell us only how much heat must be transferred to achieve a specified change
of state of the system. But in practice we are more interested in knowing the rate of heat
transfer (i.e. heat transfer per unit time) rather than the amount. This knowledge of rate of
heat transfer is necessary for a design engineer to design all types of heat transfer equipments
like boilers, condensers, furnaces, cooling towers, dryers etc.The subject of heat transfer
deals with the determination of the rate of heat transfer to or from a heat exchange equipment
and also the temperature at any location in the device at any instant of time.
The basic requirement for heat transfer is the presence of a temperature
difference. The temperature difference is the driving force for heat transfer, just as the
voltage difference for electric current flow and pressure difference for fluid flow. One of the
parameters ,on which the rate of heat transfer in a certain direction depends, is the magnitude
of the temperature gradient in that direction. The larger the gradient higher will be the rate of
heat transfer.

1.2. Heat Transfer Mechanisms:- There are three mechanisms by which heat transfer can
take place. All the three modes require the existence of temperature difference. The three
mechanisms are: (i) conduction, (ii) convection and (iii) radiation

1.2.1Conduction:-It is the energy transfer that takes place at molecular levels. Conduction is
the transfer of energy from the more energetic molecules of a substance to the adjacent less
energetic molecules as a result of interaction between the molecules. In the case of liquids
and gases conduction is due to collisions and diffusion of the molecules during their random
motion. In solids, it is due to the vibrations of the molecules in a lattice and motion of free
electrons.
Fouriers Law of Heat Conduction:- The empirical law of conduction based on
experimental results is named after the French Physicist Joseph Fourier. The law states that
the rate of heat flow by conduction in any medium in any direction is proportional to the area
normal to the direction of heat flow and also proportional to the temperature gradient in that
direction. For example the rate of heat transfer in x-direction can be written according to
Fouriers law as

4
Qx Ax .(1.1)

Qx = kx Ax .. ..(1.2)


Or

In equation (1.2), Qx is the rate of heat transfer in positive x-direction through area Ax (Ax
can vary in the direction of x, for example, radial conduction in cylinders and spheres) of the
medium normal to x-direction, (dT/dx) is the temperature gradient and kx is the constant of
proportionality and is a material property called thermal conductivity of the material for

temperature, has to be negative in the direction of heat transfer. Therefore negative sign

conduction in x-direction.Since heat transfer has to take place in the direction of decreasing


has to be introduced in equation (1.2) to make Qx positive in the direction of decreasing
temperature, thereby satisfying the second law of thermodynamics. If equation (1.2) is
divided throughout by Ax we have

qx = (Qx / Ax) = kx ..(1.3)




qx is called the heat flux in x-direction.

Thermal Conductivity:- The constant of proportionality in the equation of Fouriers law of


conduction, kx is a material property called the thermal conductivity for conduction along x-
direction (For isotropic solids thermal conductivity is independent of direction; i.e. kx = ky =
kz = k, where as for an-isotropic solids kx ky kz ).The units of thermal conductivity can be

obtained from equation (1.2) as follows:


Solving for kx from Eq. (1.3) we have kx =

Therefore units of kxin SI system of units = (W/m2 ) * (m/ K) = W / (m K) or
W / (m 0 C). Thermal conductivity is a measure of a materials ability to conduct heat. The
thermal conductivities of materials vary over a wide range as shown in Fig. 1.1.
It can be seen from this figure that the thermal conductivities of gases such as air
vary by a factor of 10 4 from those of pure metals such as copper. The kinetic theory of gases
predicts and experiments confirm that the thermal conductivity of gases is proportional to the
square root of the absolute temperature, and inversely proportional to the square root of the
molar mass M. Hence, the thermal conductivity of gases increases with increase in
temperature and decrease with increase in molar mass. It is for these reasons that the thermal
conductivity of helium (M=4) is much higher than those of air (M=29) and argon (M=40).For
wide range of pressures encountered in practice the thermal conductivity of gases is
independent of pressure.
The mechanism of heat conduction in liquids is more complicated due to the fact
that the molecules are more closely spaced, and they exert a stronger inter-molecular force
field. The values of k for liquids usually lie between those for solids and gases. Unlike gases,
the thermal conductivity for most liquids decreases with increase in temperature except for
water. Like gases the thermal conductivity of liquids decreases with increase in molar mass.

In the case of solids heat conduction is due to two effects: the vibration of lattice
induced by the vibration of molecules positioned at relatively fixed positions , and energy

5
transported due to the motion of free electrons. The relatively high thermal conductivities of
pure metals are primarily due to the electronic component. The lattice component of thermal
conductivity strongly depends on the way the molecules are arranged. For example, diamond,
which is highly ordered crystalline solid, has the highest thermal conductivity at room
temperature.

Silver
1000
Copper

Solid
metals
100 Sodium

Liquid
metals
Steel
Oxides
10
Non-
k [W/(m-K)] Mercury Metallic
solids
1.0 Water
Fibres
Non- He, H
Metallic Insul 2

liquids ating Non-


0.1 Mate Meta
Plastics rials Evacuated
llic Insulating
Wood
Oils gases materials
Foams
CO
2
0.01
Fig. 1.1: Typical range of thermal conductivities of various materials

Unlike metals, which are good electrical and heat conductors, crystalline solids
such as diamond and semiconductors such as silicon are good heat conductors but poor
electrical conductors. Hence such materials find widespread use in electronic industry.
Despite their high price, diamond heat sinks are used in the cooling of sensitive electronic
components because of their excellent thermal conductivity. Silicon oils and gaskets are
commonly used in the packaging of electronic components because they provide both good
thermal contact and good electrical insulation.

One would expect that metal alloys will have high thermal
conductivities, because pure metals have high thermal conductivities. For example one would
expect that the value of the thermal conductivity k of a metal alloy made of two metals with

6
thermal conductivities k1 and k2 would lie between k1 and k2.But this is not the case. In fact k
of a metal alloy will be less than that of either metal.

The thermal conductivities of materials vary with temperature. But for


some materials the variation is insignificant even for wide temperature range.At temperatures
near absolute zero, the thermal conductivities of certain solids are extremely large. For
example copper at 20 K will have a thermal conductivity of 20,000 W / (m-K), which is
about 50 times the conductivity at room temperature. The temperature dependence of thermal
conductivity makes the conduction heat transfer analysis more complex and involved. As a
first approximation analysis for solids with variable conductivity is carried out assuming
constant thermal conductivity which is an average value of the conductivity for the
temperature range of interest.

Thermal Diffusivity:- This is a property which is very helpful in analyzing transient heat
conduction problem and is normally denoted by the symbol . It is defined as follows.

Heat conducted k
= -------------------------------------- = -------- (m2/s) (1.4)
Heat Stored per unit volume Cp

It can be seen from the definition of thermal diffusivity that the numerator represents the
ability of the material to conduct heat across its layers and the denominator represents the
ability of the material to store heat per unit volume. Hence we can conclude that larger the
value of the thermal diffusivity, faster will be the propagation of heat into the medium. A
small value of thermal diffusivity indicates that heat is mostly absorbed by
the material and only a small quantity of heat will be conducted across the material.

1.2.2. Convection:- Convection heat transfer is composed of two mechanisms. Apart from
energy transfer due to random molecular motion, energy is also transferred due to
macroscopic motion of the fluid. Such motion in presence of the temperature gradient
contributes to heat transfer. Thus in convection the total heat transfer is due to random
motion of the fluid molecules together with the bulk motion of the fluid, the major
contribution coming from the latter mechanism. Therefore bulk motion of the fluid is a
necessary condition for convection heat transfer to take place in addition to the temperature
gradient in the fluid. Depending on the force responsible for the bulk motion of the fluid,
convective heat transfer is classified into forced convection and natural or free
convection. In the case of forced convection, the fluid flow is caused by an external agency
like a pump or a blower where as in the case of natural or free convection the force
responsible for the fluid flow (normally referred to as the buoyancy force) is generated within
the fluid itself due to density differences which are caused due to temperature gradient within
the flow field. Regardless of the particular nature of convection, the rate equation for
convective heat transfer is given by

q = h T .. (1.5)

7
where q is the heat flux, T is the temperature difference between the bulk fluid and the
surface which is in contact with the fluid, and h is called the convective heat
transfercoefficient or surface film coefficient. Eq.(1.5) is generally referred to as the
Newtons law of cooling.If Ts is the surface temperature , Tf is the temperature of the bulk
fluid and if Ts> Tf, then Eq. (1.5) in the direction of heat transfer can be written as

q = h [Ts Tf] ...(1.6a)

and if Ts< Tf, the equation reduces to

q = h [Tf Ts] ...(1.6b)

The heat transfer coefficient h depends on (i) the type of flow (i.e. whether the flow
is laminar or turbulent), (ii) the geometry of the body and flow passage area, (iii) the thermo-
physical properties of the fluid namely the density , viscosity , specific heat at constant
pressure Cp and the thermal conductivity of the fluid k and (iv) whether the mechanism of
convection is forced convection or free convection. The heat transfer coefficient for free
convection will be generally lower than that for forced convection as the fluid velocities in
free convection are much lower than those in forced convection. The heat transfer
coefficients for some typical applications are given in table 1.2.

Table 1.2: Typical values of the convective heat transfer coefficient h


------------------------------------------------------------------------------------------------------------
Type of flow h ,W / (m2 K)
Free convection
Gases 2 25
Liquids 50 1000
Forced Convection
Gases 25 250
Liquids 50 20,000
Convection with change of phase
Boiling or condensation 2500 100,000

1.2.3. Thermal Radiation:- Thermal radiation is the energy emitted by matter (solid, liquid
or gas) by virtue of its temperature. This energy is transported by electromagnetic waves (or
alternatively, photons).While the transfer of energy by conduction and convection requires
the presence of a material medium, radiation does not require.Infact radiation transfer occurs
most effectively in vacuum.
Consider radiation transfer process for the surface shown in Fig.1.2a.Radiation that
is emitted by the surface originates from the thermal energy of matter bounded by the
surface, and the rate at which this energy is released per unit area is called as the surface
emissive power E.An ideal surface is one which emits maximum emissive power and is
called an ideal radiator or a black body.Stefan-Boltzmans law of radiation states that the
emissive power of a black body is proportional to the fourth power of the absolute

8
temperature of the body. Therefore if Eb is the emissive power of a black body at temperature
T 0K, then

G G E
q
s q
surr

Surface of emissivity , absorptivity , and


Surface of emissivity ,
temperature T
s area A, and temperature
T

Surroundings (black) at T
surr
(a) (b)

Fig.1.2: Radiation exchange: (a) at a surface and (b) between a surface


and large surroundings

Eb T 4

Or Eb = T 4 .(1.7)

is the Stefan-Boltzmanconstant ( = 5.67 * 10 8 W / (m2 K4) ). For a non black surface


the emissive power is given by

E = T 4(1.8)

where is called the emissivity of the surface (0 1).The emissivity provides a measure
of how efficiently a surface emits radiation relative to a black body. The emissivity strongly
depends on the surface material and finish.
Radiation may also incident on a surface from its surroundings. The rate at which the
radiation is incident on a surface per unit area of the surface is calle the irradiation of the
surface and is denoted by G. The fraction of this energy absorbed by the surface is called
absorptivity of the surface and is denoted by the symbol . The fraction of the

incident energy is reflected and is called the reflectivity of the surface denoted by and
the remaining fraction of the incident energy is transmitted through the surface and
is called the transmissivity of the surface denoted by . It follows from the definitions of ,
, and that

9
+ + = 1 .(1.9)

Therefore the energy absorbed by a surface due to any radiation falling on it is given by

Gabs = G (1.10)

The absorptivity of a body is generally different from its emissivity. However in many
practical applications, to simplify the analysis is assumed to be equal to its emissivity .

Radiation Exchange:- When two bodies at different temperatures see each other, heat is
exchanged between them by radiation. If the intervening medium is filled with a substance
like air which is transparent to radiation, the radiation emitted from one body travels through
the intervening medium without any attenuation and reaches the other body, and vice versa.
Then the hot body experiences a net heat loss, and the cold body a net heat gain due to
radiation heat exchange between the two. The analysis of radiation heat exchange among
surfaces is quite complex which will be discussed in chapter 10. Here we shall consider two
simple examples to illustrate the method of calculating the radiation heat exchange between
surfaces.
As the first example let us consider a small opaque plate (for an opaque surface = 0)
of area A, emissivity and maintained at a uniform temperature Ts. Let this plate is exposed
to a large surroundings of area Asu (Asu>> A) whish is at a uniform temperature Tsur as shown
in Fig. 1.2b.The space between them contains air which is transparent to thermal radiation.

The radiation energy emitted by the plate is given by

Qem = A Ts4

The large surroundings can be approximated as a black body in relation to the small plate.
Then the radiation flux emitted by the surroundings is Tsur4 which is also the radiaton flux
incident on the plate. Therefore the radiation energy absorbed by the plate due to emission
from the surroundings is given by

Qab = A Tsur4.

The net radiation loss from the plate to the surroundings is therefore given by

Qrad = A Ts4 A Tsur4.

Assuming = for the plate the above expression for Qnet reduces to

Qrad = A [Ts4 Tsur4 ] .(1.11)

The above expression can be used to calculate the net radiation heat exchange between a
small area and a large surroundings.
As the second example, consider two finite surfaces A1 and A2 as shown in Fig. 1.3.

10
Surroundings

A2, 2, T2
A1, 1, T1

Fig.1.3: Radiation exchange between surfaces A1 and A2

The surfaces are maintained at absolute temperatures T1 and T2 respectively, and have
emissivities 1 and 2. Only part of the radiation leaving A1 reaches A2. Similar
considerations apply for the radiation leaving A2.If it is assumed that the radiation from the
surroundings is negligible when compared to the radiation from the surfaces A1 and A2 then
we can write the expression for the radiation emitted by A1 and reaching A2 as

Q12 = F1 2 A11 T14(1.12)

where F1 2 is defined as the fraction of radiation energy emitted by A1 and reaching A2.
Similarly the radiation energy emitted by A2 and reaching A1 is given by

Q21 = F2 1 A2 2 T24 ..(1.13)

where F2 1 is the fraction of radiation energy leaving A2 and reaching A1. Hence the net
radiation energy transfer from A1 to A2 is given by

Q1 2 = Q12 Q21

= [F1 2 A11 T14] [F2 1 A2 2 T24]

F1-2 is called the view factor (or geometric shape factor or configuration factor) of A2 with
respect to A1 and F2 - 1 is the view factor of A1 with respect to A2.It will be shown in chapter
10 that the view factor is purely a geometric property which depends on the relative
orientations of A1 and A2 satisfying the reciprocity relation, A1 F1 2 = A2 F2 1.

Therefore Q1 2 = A1F1 2 [1 T14 2 T24].(1.14)

11
Radiation Heat Transfer Coefficient:- Under certain restrictive conditions it is possible to
simplify the radiation heat transfer calculations by defining a radiation heat transfer
coefficient hr analogous to convective heat transfer coefficient as

Qr = hrA T

For the example of radiation exchange between a surface and the surroundings
[Eq. (1. 11)] using the concept of radiation heat transfer coefficient we can write

Qr = hrA[Ts Tsur] = A [Ts4 Tsur4 ]

[Ts4 Tsur4 ] [Ts2 + Tsur2 ][Ts + Tsur][Ts Tsur]


Or hr = --------------------- = -----------------------------------------------
[Ts Tsur] [Ts Tsur]

Or hr = [Ts2 + Tsur2 ][Ts + Tsur] (1.15)

1.3.First Law of Thermodynamics (Law of conservation of energy) as applied to


Heat Transfer Problems:The first law of thermodynamics is an essential tool for solving
many heat transfer problems. Hence it is necessary to know the general formulation of the
first law of thermodynamics.
First law equation for a control volume:- A control volume is a region in space bounded by
a control surface through which energy and matter may pass.There are two options of
formulating the first law for a control volume. One option is formulating the law on a rate
basis. That is, at any instant, there must be a balance between all energy rates. Alternatively,
the first law must also be satisfied over any time interval t. For such an interval, there must
be a balance between the amounts of all energy changes.

First Law on rate basis :-The rate at which thermal and mechanical energy enters a control
volume, plus the rate at which thermal energy is generated within the control volume, minus
the rate at which thermal and mechanical energy leaves the control volume must be equal to
the rate of increase of stored energy within the control volume.
Consider a control volume shown in Fig. 1.4 which shows that thermal and
mechanical energy are entering the control volume at a rate denoted by Ein, thermal and

.
Eg . .

.
Est Eout
Ein

Fig. 1.4: Conservation of energy for a control volume on rate basis

12
.
mechanical energy are leaving the control volume at a rate denoted by Eout. The rate at
.
which energy is generated within the control volume is denoted by Egand the rate at
.
which energy is stored within the control volume is denoted by Est. The general form of the
energy balance equation for the control volume can be written as follows:
. . . .
Ein + Eg Eout = Est (1.16)
.


Est is nothing but the rate of increase of energy within the control volume and hence can be


written as equal to

First Law over a Time Interval t :- Over a time interval t, the amount of thermal and
mechanical energy that enters a control volume, plus the amount of thermal energy
generated within the control volume minus the amount of thermal energy that leaves the
control volume is equal to the increase in the amount of energy stored within the control
volume.
The above statement can be written symbolically as

Ein + Eg Eout = Est ..(1.16)

The inflow and outflow energy terms are surface phenomena. That is they are associated
exclusively with the processes occurring at the boundary surface and are proportional to the
surface area.
The energy generation term is associated with conversion from some other form
(chemical, electrical, electromagnetic, or nuclear) to thermal energy. It is a volumetric
phenomenon.That is, it occurs within the control volume and is proportional to the magnitude
of this volume. For example, exothermic chemical reaction may be taking place within the
control volume. This reaction converts chemical energy to thermal energy and we say that
energy is generated within the control volume. Conversion of electrical energy to thermal
energy due to resistance heating when electric current is passed through an electrical
conductor is another example of thermal energy generation
Energy storage is also a volumetric phenomenon and energy change within the
control volume is due to the changes in kinetic, potential and internal energy of matter within
the control volume.

1.4. Illustrative Examples:

A. Conduction

Example 1.1:- Heat flux through a wood slab 50 mm thick, whose inner and outer surface
temperatures are 40 0 C and 20 0 C respectively, has been determined to be 40 W/m2. What is
the thermal conductivity of the wood slab?

13
Sketch for the example:

Known: T1 = 40 0 C; T2 = 20 0 C; L = 0.05 m;
T1 q = 40 W / m2.
T1
q Find: Thermal conductivity,k
T2
T2 Assumptions: (i) steady state conduction

k=? (ii) one dimensional conduction

(iii) solid is of constant thermal conductivity


L

Solution: For steady state conduction across the thickness of the slab and noting that the slab
is not generating any thermal energy, the first law equation for the slab can be written as :
Rate at which thermal energy (conduction) is entering the slab at the surface x = 0is equal to
the rate at which thermal energy is leaving the slab at the surface x = L.
That is

Qx|x = 0 = Qx|x = L = Qx = constant

By Fouriers law we have Qx = kA



Separating the variables and integrating both sides w.r.t. x we have


L T2


Qx dx = kA dT . Or Qx =
0 T1



Heat flux = q = =

.

Hence k= = = 0.1 W / (m K)

Example 1.2:- A concrete wall, which has a surface area of 20 m2 and thickness 30 cm,
separates conditioned room air from ambient air.The temperature of the inner surface of the
wall is 25 0 C and the thermal conductivity of the wall is 1.5 W / (m-K).Determine the heat
loss through the wall for ambient temperature varying from 15 0 C to 38 0 C which
correspond to winter and summer conditions and display your results graphically.

14
Sketch for the example:

Known:T1 = 25 0 C ; A = 20 m2; L = 0.3 m :


T1
k = 1.5 W /(m-K) ;
Q
Find: Q for different values of T2 and plot Q
T2 vs T2 for different values of T2

Assumptions: (i) steady state conduction


(ii) one dimensional conduction
L (iii) solid is of constant thermal conductivity

Solution: By Fouriers law of conduction we have

.
.
For steady state Q = =

Or Q = 2500 100 T2 ...(1)

Heat loss Q for different values of T2 ranging from 15 0 C to + 38 0 C are obtained from Eq.
(1) and the
results are
Q 1.2 equation: Q= 2500-100T(2)
plotted as
shown: Scale
5000
x-axis : 1cm=
4000 5 C ; y-axis :
1cm =1000W
3000
Q ,watts

2000
Series2
1000

0
1 2 3 4 5 6 7 8 9 10 11 12
-1000

-2000
T(2) , celsius

15
Example 1.3:-What is the thickness required of a masonry wall having a thermal
conductivity of 0.75 W/(m-K), if the heat transfer rate is to be 80 % of the rate through
another wall having thermal conductivity of 0.25 W/(m-K) and a thickness of 100 mm? Both
walls are subjected to the same temperature difference.

Sketch for the example:


Let subscript 1 refers to masonry wall and subscript 2 refers to the other wall.

Known:Q1/Q2 = 0.8 ; ; L2 = 100 mm :


T1
k 1= 0.75 W /(m-K) ;k 2= 0.25 W /(m-K) ;
Q1
Find: L1
T2
Assumptions: (i) steady state conduction
(ii) one dimensional conduction
(iii) solid is of constant thermal conductivity
L1

Solution: By Fouriers law for steady state conduction we have

For masonary wall Q1 = k1A(T1 T2)|masonary wall / L1

and for other wall Q2 = k2A(T1 T2)|other wall / L2

Given that (T1 T2)|masonry wall = (T1 T2)|other wall , we have

Q1 k1 L2
---- = ----------
Q2 k2 L1

Q2 k1
Or L1 = ----------- L2 = (1 / 0.80) * (0.75/0.25) x 100 = 375 mm
Q1 k2

B. Convection:

Example 1.4:- Air at 40 0 C flows over a long circular cylinder of 25 mm diameter with
an embedded electrical heater. In a series of tests, measurements were made of power
per unit length, P required to maintain the surface temperature of the cylinder at 300 0 C
for different stream velocities V of the air. The results are as follows:

16
Air velocity, V (m/s) : 1 2 4 8 12

Power, P (W/m) : 450 658 983 1507 1963

(a) Determine the convective heat transfer coefficient for each velocity and display your
results graphically. (h = P / 20.43)
(b)Assuming the dependence of the heat transfer coefficient on velocity to be of the
form h = CV n , determine the parameters C and n from the results of part (a).

Sketch for the example:


Ts

V,T
D

Known: D = 0.025 m : Ts = 300 0 C ; T = 40 0 C;

Find: (a) Heat transfer coefficient as a function of velocity of the fluid (b) To find the values
of C and n in the equation h = CVn

Assumptions: (i) Heat transfer from the surface of the solid to the flowing fluid is by
convection only.(i.e. radiation heat transfer between the surface of the solid and the fluid is
negligible)

Solution: If h is the surface heat transfer coefficient then the power dissipated by the cylinder
by convection is given by

P = hAs (Ts - T)

where As is the area of contact between the fluid and the surface of the cylinder.

Therefore P = h DL (Ts - T)

Or h = P / [DL(Ts - T)] = P / [ * 0.025 * 1 * (300 40)]

Or h = P / 20.42 W/m2-k ..(1)

Values of h for different flow velocities are obtained and tabulated as follows:

Air Velocity, V (m/s) : 1 2 4 8 12

17
Power,P (W/m) : 450 658 983 1507 1963

h, (W / (m2 K) ) : 22.04 32.22 48.14 73.8 96.13

(a) A graph of h versus V can now be plotted as shown in Fig. P 1.4 (a).

Scale: X axis 1cm= 1m/s ; Y axis 1cm= 10 W/m2k

Q 1.4a

120

100

80

60
h

40

20

0
1 2 3 4 5 6 7 8 9 10 11 12 13
velocity, m/s

(b) h = CVn

Therefore ln h = ln C + n ln V (2)

If ln h is plotted against ln V it will be straight line and the slope of which will give the value
of n. Also the intercept of this line w.r.t the axis on which ln V is plotted will give the value
of ln C from which C can be determined. The log log plot is as shown in Fig. P 1.4(b).
Scale X axis 1cm=0.25; Y axis 1cm=0.5

18
1.4b Slope: 0.571

3
ln h

0
1 2 3 4 5 6 7 8 9 10 11 12
ln v

ln C = 3.1 or C = 22.2

(ln h ln C) (4.55 3.10)


and n = ----------------------- = ------------------- = 0.571
ln V 2.5

Therefore h = 22.2V0.571 is the empirical relation between h and V.

Example 1.5:- A large surface at 50 0 C is exposed to air at 20 0 C. If the heat transfer


coefficient between the surface and the air is 15 W/(m2-K), determine the heat transferred
from 5 m2 of the surface area in 7 hours.

Sketch for the example:

Ts
h

Known: Ts =50 0C; T = 20 0 C;h = 15 W/(m2 K) ;A = 5 m2 : time = t = 7 h ;

Find: Qtotal

Assumption (i) Steady state heat transfer; (ii)Heat transfer from the surface of the solid to
the surroundings is by convection only.(i.e. radiation heat transfer between the surface of the
solid and the surroundings is negligible)

Solution: Using Newtons law of cooling we have

Q total = Q* t = hA(Ts - T) t = 15 * 5 * (50 20) x 7 *3600 = 56.7 * 10 6 J = 56.7 MJ

Example 1.6:- A 25 cm diameter sphere at 120 0 C is suspended in air at 20 0 C. If the


convective heat transfer coefficient between the surface and air is 15 W/(m2-K),

19
determine the heat loss from the sphere.

Sketch for the eample:

h
Ts

D
T

Known: Ts = 120 0C;T = 20 0C; h = 15 W/(m2-K);D = 0.25 m

Find: Q

Assumptions: (i) Heat transfer from the surface of the solid to the surrounding fluid is by
convection only.(i.e. radiation heat transfer between the surface of the solid and the fluid is
negligible)(ii) Steady state heat transfer

Solution: Q = hAs(Ts - T) = h 4R2 (Ts - T) = 15 * 4 * (0.25/2)2 * (120 20) = 294.52 W

C. Radiation:

Example 1.7:- A sphere 10 cm in diameter is suspended inside a large evacuated


chamber whose walls are kept at 300 K. If the surface of the sphere is black and
maintained at 500 K what would be the radiation heat loss from the sphere to the walls
of the chamber?. What would be the heat loss if the surface of the sphere has an
emissivity of 0.8?
Sketch for the example:

Known:T1 = 500 K ; T2 = 300 K ; d1 = 0.10 m


T2
Find: (i) Q assuming the surface to be a black
surface; (ii) Q assuming the surface to have an
emissivity of 0.8
T1 .
Assumptions: (i) Steady state Heat Transfer

d1

Solution: Surface area of the sphere = As= 4R12 = 4* (0.1/2)2 = 0.0314 m2


If the surface of the sphere is black then

20
Qblack = As (T14 T24)

= 5.67 * 10 8* 0.0314 * (5004 3004)

= 96.85 W
If the surface is having an emissivity of 0.8 then

Q = 0.8 Qblack = 0.8 * 96.85 = 77.48 W

Example 1.8:- A vacuum system as used in sputtering conducting thin films on micro
circuits, consists of a base plate maintained at a temperature of 300 K by an
electric heater and a shroud within the enclosure maintained at 77 K by circulating
liquid nitrogen. The base plate insulated on the lower side is 0.3 m in diameter and has
an emissivity of 0.25.
(a) How much electrical power must be provided to the base plate heater?

(b) At what rate must liquid nitrogen be supplied to the shroud if its latent heat of
vaporization is 125 kJ/kg?

Sketch:

Known: T1 = 300 K ; T2 = 77 K ; d = 0.3 m ; 1 = 0.25

Find: (a) Qr ; (b) Mass of nitrogen vapourized per unit time, mN2

Assumptions:

Solution: Surface area of the top surface of the base plate = As = ( / 4)d12 = ( / 4) * 0.32

= 0.0707 m2
(a) Qr = 1 As (T14 T24)

= 0.25 * 5.67 * 10 8 * 0.0707 * (3004 774) = 8.08 W


.
(b) If mN2 = mass flow rate of nitrogen that is vapourised then

. 8.08

21
mN2 = Qr / hfg = ---------------- = 6.464 * 10-5 kg/s or 0.233 kg/s
125 x 1000

Example 1.9:- A flat plate has one surface insulated and the other surface exposed to the
sun. The exposed surface absorbs the solar radiation at a rate of 800 W/m2 and dissipates
heat by both convection and radiation into the ambient at 300 K. If the emissivity of the
surface is 0.9 and the surface heat transfer coefficient is 12 W/(m2-K), determine the
surface temperature of the plate.

Sketch for the example:


Qsolar
T
Qr
Qconv Ts , , h

Insulated

Known: T = 300 K ; qsolar = 800 W / m2 ; = 0.9 ; h = 12 W / (m2 K)

Find: Ts

Assumptions: (i) steady state heat transfer;(ii) the emissivity and the surface heat transfer
coefficient is uniform throughout the surface.

Solution: Energy balance equation for the top surface of the plate is given by

Qsolar = Qr + Qconv

qsolar As = As (Ts4 - T4) + h As (Ts - T)

Therefore 800 = 0.9 * 5.67 * 10 8* (Ts4 3004) + 12 * (Ts 300)

On simplifying the above equation we get

(Ts / 100)4 + 2.35 Ts = 943 (1)

Equation (1) has to be solved by trial and error.

22
Trial 1:- Assume Ts = 350 K. Then LHS of Eq. (1) = 972.6 which is more than RHS of
Eq.(1). Hence Ts< 350 K.

Trial 2 :- Assume Ts = 340 K. Then LHS of Eq. (1) = 932.6 which is slightly less than

RHS. Therefore Ts should lie between 340K and 350 K but closer to 340 K.

Trial 3:- Assume Ts = 342.5 K. Then LHS of Eq.(1) = 942.5 = RHS of Eq. (1).

Therefore Ts = 342.5 K

Example 1.10:- The solar radiation incident on the outside surface of an aluminum
shading device is 1000 W/m2. Aluminum absorbs 12 % of the incident solar energy and
dissipates it by convection from the back surface and by combined convection and
radiation from the outer surface. The emissivity of aluminum is 0.10 and the convective
heat transfer coefficient for both the surfaces is 15 W/(m2 K). The ambient temperature
of air may be taken as 20 0 C. Determine the temperature of the shading device.

Sketch for the example:

q solar qr
qc1

qc2

Known: qsolar = 1000 W / m2 ; absorptivity of aluminum = = 0.12 ; emissivity

of aluminum = = 0.10 ; h = 15 W /(m2 K) ; T = 20 + 273 = 293 K ;

Find: Temperature of the shading device, Ts

Assumptions: (i) Steady state heat transfer; (ii) the emissivity and the surface heat transfer
coefficient are uniform throughout the surface

Solution: Solar radiation flux absorbed by aluminum = qa = qsolar = 0.12 * 1000 = 120 W /
m2 .

23
Under steady state conditions,the energy absorbed by aluminum is dissipated by convection
from the back surface and by combined convection and radiation from the outer surface.
Hence the energy balance equation can be written as

qa = qr + qc1 + qc2

Therefore, qa = (Ts4 T4) + h1(Ts - T) + h2 (Ts - T)

Or 120 = 5.67 * 10 8* 0.10 (Ts4 0.12 * 2934) + (Ts 293) * (15 + 15)

On simplifying we get, (Ts / 100)4 + 53 Ts = 15873 (1)

Eq.(1) has to be solved by trial and error.

Trail 1:- Assume Ts = 300 K. Then LHS = 15981 which is > RHS.

Trail 2 :- Assume Ts = 295 K. Then LHS = 15710.73 which is < RHS. Hence Ts should lie
between 300K and 295 K.

Trial 3 :- Assume Ts = 297 K . Then LHS = 15819 which is almost equal to RHS (Within
0.34 %)

Therefore Ts = 297 K.

24
CHAPTER 2

GOVERNING EQUATIONS OF CONDUCTION


2.1.Introduction:In this chapter, the governing basic equations for conduction in Cartesian
coordinate system is derived. The corresponding equations in cylindrical and spherical
coordinate systems are also mentioned. Mathematical representations of different types of
boundary conditions and the initial condition required to solve conduction problems are also
discussed. After studying this chapter, the student will be able to write down the governing
equation and the required boundary conditions and initial condition if required for any
conduction problem.

2.2. One Dimensional Conduction Equation : In order to derive the one-dimensional


conduction equation, let us consider a volume element of the solid of thickness x along x
direction at a distance x from the origin as shown in Fig. 2.1.Qx represents the rate

25
A(x) q

Qx Qx + x

O
x x

Fig. 2.1: Nomenclature for one dimensional conduction equation

of heat transfer in x direction entering into the volume element at x, A(x) area of heat flow
at the section x ,q is the thermal energy generation within the element per unit volume and
Qx+x is the rate of conduction out of the element at the section x + x. The energy balance
equation per unit time for the element can be written as follows:

[ Rate of heat conduction into the element at x + Rate of thermal energy generation within
the element Rate of heat conduction out of the element at x + x ]

= Rate of increase of internal energy of the element.



i.e., Qx + Qg Qx+x =

or Qx + q Ax x {Qx + x + (x)2 / 2! + .}


= [A(x)xCpT]

Neglecting higher order terms and noting that and Cp are constants the above equation
simplifies to

Qx + q Ax x {Qx + x } = AxxCp

+ q Ax = Ax Cp

Or

Using Fouriers law of conduction , Qx = k Ax , the above equation simplifies to


26
{ k Ax } + q Ax = Ax Cp

Or {1/Ax} { k Ax }+ q = Cp (2.1)

Eq. (2.1) is the most general form of conduction equation for one-dimensional unsteady state
conduction.

2.2.1.Equation for one-dimensional conduction in plane walls :- For plane walls, the area
of heat flow Ax is a constant. Hence Eq. (2.1) reduces to the form

{ k } + q = Cp ..(2.2)

(i) If the thermal conductivity of the solid is constant then the above equation reduces to

+ (q / k) = (1/) (2.3)

(ii) For steady state conduction problems in solids of constant thermal conductivity
temperature within the solid will be independent of time (i.e.(T/ t) = 0)
and hence Eq. (2.3) reduces to

+ (q / k) = 0

.(2.4)

(iii) For a solid of constant thermal conductivity for which there is no thermal energy
generation within the solid q = 0 and the governing equation for steady state conduction is
obtained by putting q = 0 in Eq. (2.4) as

= 0

(2.4)

2.2.2.Equation for one-dimensional radial conduction in cylinders:-

27
R
Qr
r

Qr

For radial conduction in cylinders, by convention the radial coordinate is denoted by r


instead of x and the area of heat flow through the cylinder of length L,at any radius r is
given by Ax = A(r) = 2rL. Hence substituting this expression for Ax and replacing x by r in
Eq. (2.1) we have

{ {k 2rL } + q = Cp

{ {k r } + q = Cp ...(2.5)


Or

(i) For cylinders of constant thermal conductivity the above equation reduces to

{ {r } + q / k = ..(2.6)

where = k / Cp.
(ii) For steady state radial conduction (i.e. (T/ t) = 0 ) in cylinders of constant k, the above
equation reduces to

{ r } + q / k = 0 .(2.7)

(iii) For steady state radial conduction in cylinders of constant k and having no thermal
energy generation (i.e. q = 0) the above equation reduces to

{ r } = 0


(2.8)

2.2.3.Equation for one-dimensional radial conduction in spheres:- For one-dimensional


radial conduction in spheres, the area of heat flow at any radius r is given by Ar = 4r2.
Hence Eq.(2.1) for a sphere reduces to

28
{k 4 r2 } +q = Cp


{ }

{k r2 } + q = Cp .(2.9)


Or

(i) For spheres of constant thermal conductivity the above equation reduce to

{ r2 } + q / k = ..(2.10)

(ii) For steady state conduction in spheres of constant k the above equation further reduce to

{ r2 } + q/ k = 0


(2.11)

(iii) For steady state conduction in spheres of constant k and without any thermal energy
generation the above equation further reduces to

{ r2 } = 0


(2.12)

Equation in compact form:- The general form of one dimensional conduction equations
for plane walls, cylinders and spheres {equations (2..2), (2.5) and (2.9)} can be written in a
compact form as follows:

{k rn } + q = Cp ...(2.13)

Where n = 0 for plane walls,


n = 1 for radial conduction in cylinders
n = 2 for radial conduction in spheres,
and for plane walls it is customary to replace the r variable by x variable.

2.3.Three dimensional conduction equations: While deriving the one dimensional


conduction equation, we assumed that conduction heat transfer is taking place only along one
direction. By allowing conduction along the remaining two directions and following the same
procedure we obtain the governing equation for conduction in three dimenions.

2.3.1. Three dimensional conduction equation in Cartesian coordinate system: Let us


consider a volume element of dimensions x, y and z in x y and z directions respectively.
The conduction heat transfer across the six surfaces of the element is shown in Fig. 2.3.

29
z Qz + z Qy + y
y

z
x

Qx + x
Qx

Qy Qz
Fig. 2.3: Conduction heat transfer across the six faces of a volume element

Net Rate of conduction into the element in x-direction = Qx Qx + x

= Qx [Qx + (Qx/x) x + (2Qx/x2)(x)2 / 2! + .]

= (Qx/x) x by neglecting higher order terms.

= / x [ kx y z(T / x)] x

= / x[kx (T / x)] x y z

Similarly the net rate of conduction into the element


in y direction = / y[ky (T / y)] x y z

and in z direction = / z[kz (T / z)] x y z.

Hence the net rate of conduction into the element from all the three directions

Qin = { / x[kx (T / x)] + / y[ky (T / y)] + / z[kz (T / z)] } x y z

Rate of heat thermal energy generation in the element = Qg = q x y z

Rate of increase of internal energy within the element = E / t = x y z Cp (T / t)

Applying I law of thermodynamics for the volume element we have

30
Qin + Qg = E / t

Substituting the expressions for Qin, Qg and E / t and simplifying we get

{ / x[kx (T / x)] + / y[ky (T / y)] + / z[kz (T / z)] } + q= Cp (T / t)

(2.14)

Equation (2.14) is the most general form of conduction equation in Cartesian coordinate
system. This equation reduces to much simpler form for many special cases as indicated
below.
Special cases:- (i) For isotropic solids, thermal conductivity is independent of direction; i.e.,
kx = ky = kz = k. Hence Eq. (2.14) reduces to

{ / x[k (T / x)] + / y[k (T / y)] + / z[k (T / z)] } + q = Cp (T / t)

..(2.15)
(ii) For isotropic solids with constant thermal conductivity the above equation further reduces
to

2T / x2+ 2T / y2 + 2T / z2 + q/ k = (1 / ) (T / t).(2.16)

Eq.(2.16) is called as the Fourier Biot equation and it reduces to the following forms
under specified conditions as mentioned below:

(iii) Steady state conduction [i.e., (T / t) = 0]

2T / x2+ 2T / y2 + 2T / z2 + q/ k = 0 .(2.17)

Eq. (2.17) is called the Poisson equation.

(iv) No thermal energy generation [i.e. q = 0]:

2T / x2+ 2T / y2 + 2T / z2= (1 / ) (T / t)..(2.18)

Eq. (2.18) is called the diffusion equation.

(v) Steady state conduction without heat generation [i.e., (T / t) = 0 and q = 0]:

2T / x2+ 2T / y2 + 2T / z2= 0 (2.19)

Eq. (2.19) is called the Laplace equation.

31
2.3.2. Three dimensional conduction equation in cylindrical coordinate system:

It is convenient to express the governing conduction equation in cylindrical coordinate


system when we want to analyse conduction in cylinders. Any point P in space can be
located by using the cylindrical coordinate system r, and z and its relation to the Cartesian
coordinate system (See Fig. 2.4) can be written as follows:
z

P(x,y,z)

x

r

Fig.2.4: Cylindrical coordinate system


y

x = r cos ; y = r sin ; z = z. Using these transformations and after laborious simplifications


Eq. (2.15) reduces to

+ + + q = Cp


..(2.20)

The above equation is valid for only for isotropic solids.

2.3.2. Three dimensional conduction equation in Spherical coordinate system:

For spherical solids,it is convenient to express the governing conduction equation in


spherical coordinate system. Any point P on the surface of a sphere of radius r can be
located by using the spherical coordinate system r, and and its relation to the Cartesian
coordinate system (See Fig. 2.5) can be written as follows:

32
z
OP = r sin .Hence

x = r sin cos ;
P(x,y,z)
y = r sin sin ;

z = r cos
r

O
x

P
Fig: 2.5: Spherical coordinate system
y

Using the relation between x, y ,z and r, and , the conduction equation (2.15) can be
transformed into the equation in terms of r, and as follows.

+ + + q = Cp

.(2.21).
2.4.Boundary and Initial Conditions:

The temperature distribution within any solid is obtained by integrating the above conduction
equation with respect to the space variable and with respect to time.The solution thus
obtained is called the general solution involving arbitrary constants of integration. The
solution to a particular conduction problem is arrived by obtaining these constantswhich
depends on the conditions at the bounding surfaces of the solid as well as the initial
condition. The thermal conditions at the boundary surfaces are called the boundary
conditions . Boundary conditions normally encountered in practice are:
(i) Specified temperature (also called as boundary condition of the first kind),
(ii) Specified heat flux (also known as boundary condition of the second kind),
(iii) Convective boundary condition (also known as boundary condition of the third kind) and
(iv) radiation boundary condition. The mathematical representations of these boundary
conditions are illustrated by means of a few examples below.

2.4.1. Specified Temperatures at the Boundary:- Consider a plane wall of thickness L


whose outer surfaces are maintained at temperatures T0 and TL as shown in Fig.2.6.For one-
dimensional unsteady state conduction the boundary conditions can be written as

33
y
T(x,t)
T0 TL T(x,y)
T = (x)

L T2
b
(y)
a

x
T1
x
Fig. 2.6: Boundary condition Fig.2.7: Boundary conditions of
of first kind for a plane wall first kind for a rectangular plate

(i) at x = 0, T(0,t) = T0 ; (ii) at x = L, T(L,t) = TL.

Consider another example of a rectangular plate as shown in Fig. 2.7. The boundary
conditions for the four surfaces to determine two-dimensional steady state temperature
distribution T(x,y) can be written as follows.

(i) at x = 0, T(0,y) = (y) ; (ii) at y = 0, T(x,0) = T1 for all values of y

(iii) at x = a, T(a,y) = T2 for all values of y; (iv) at y = b, T(x,b) = (x)

2.4.2. Specified heat flux at the boundary:- Consider a rectangular plate as shown in Fig.
2.8 and whose boundaries are subjected to the prescribed heat flux conditions as shown in the
figure. Then the boundary conditions can be mathematically expressed as follows.

34
qb T(x,y)
y

q0 qa
b

a
x
insulated

Fig.2.8: Prescribed heat flux boundary conditions

(i) at x = 0, k (T / x)|x = 0 = q 0 for 0 y b ;

(ii) at y = 0 , (T / y)|y = 0 = 0 for 0 x a ;

(iii) at x = a, k (T / x)|x = a = q a for 0 y b ;

(iv) at y = b, k (T / y)|y = b = qb for 0 x a ;

2.4.3. Boundary surface subjected to convective heat transfer:- Fig. 2.9 shows a plane
wall whose outer surfaces are subjected to convective boundary conditions.The surface at x =
0 is in contact with a fluid which is at a uniform temperature Ti and the surface heat transfer
coefficient is hi. Similarly the other surface at x = L is in contact with another fluid at a
uniform temperature T0 with a surface heat transfer coefficient h0. This type of boundary
condition is encountered in heat exchanger wherein heat is transferred from hot fluid to the
cold fluid with a metallic wall separating the two fluids. This type of boundary condition is
normally referred to as the boundary condition of third kind. The mathematical
representation of the boundary conditions for the two surfaces of the plane wall can be
written as follows.

(i) at x = 0, qconvection = q conduction; i.e., hi[Ti T|x = 0 ] = k(dT / dx)|x = 0

(ii) at x = L, k(dT / dx)|x = L = h0 [T|x = L T0]

35
T(x)
Surface in contact with
fluid at T0 with surface
heat transfer coefficient h0

L
Surface in contact with fluid
at Ti with surface heat
transfer coefficient h i

x
Fig. 2.9: Boundaries subjected to convective heat transfer for a plane wall

2.4.4.Radiation Boundary Condition:Fig. 2.10 shows a plane wall whose surface at x =L is


having an emissivity and is radiating heat to the surroundings at a uniform temperature Ts.
The mathematical expression for the boundary condition at x = L can be written as follows:

T(x,t)
Surface with emissivity is
radiating heat to the
surroundings at Ts0K

x
Fig. 2.10: Boundary surface at x = L subjected to radiation heat transfer for a plane wall

at x = L, qconduction = qradiation ; i.e., k (dT / dx)| x = L = [( T| x = L)4 Ts4]

In the above equation both T| x = L and Ts should be expressed in degrees Kelvin.

36
2.4.5. General form of boundary condition (combined conduction, convection and
radiation boundary condition): There are situations where the boundary surface is
subjected to combined conduction, convection and radiation conditions as illustrated in Fig.
2.11.It is a south wall of a house and the outer surface of the wall is exposed to solar
radiation. The interior of the room is at a uniform temperature Ti. The outer air is at uniform
temperature T0 . The sky, the ground and the surfaces of the surrounding structures at this
location is modeled as a surface at an effective temperature of Tsky.
x
L

qradiation

qconduction qsolar

qconvection

Fig. 2.11: Schematic for general form of boundary condition

Energy balance for the outer surface [x = L] is given by the equation

qconduction + qsolar = qradiation + qconvection

k (dT / dx)|x = L + qsolar = [(T|x = L)4 Tsky4] + h0[T|x = L T0]

37
2.5. Illustrative Examples:

A. Derivation of conduction Equations:

Example2.1.By writing an energy balance for a differential cylindrical volume element in the
rvariable (r is any radius), derive the one-dimensional time dependent heat conduction
equation with internal heat generation and variable thermal conductivity in the cylindrical
coordinate system.

Sketch :

r
Qr
dr
Qr

Qg
Qr+dr
Qr+dr

Find: One dimensional time dependent heat conduction equation

Assumptions: One dimensional time dependent radial heat conduction

Solution:
A cylindrical element of thickness dr in the radial direction at a radius r is shown in the figure
above. For unsteady state one dimensional radial conduction with heat generation is given by
Qr + Qg Qr+dr = (E / t)

Or Qr + Qg [Qr + (Qr r)dr] = (E / t)

Or (Qr r)dr + Qg = (E / t) ..(1)

where Qr is the rate of conduction into the element at radius r = k 2rL (T /r)

Qg is the rate of heat generation within the element = 2 rL dr q

(E / t) is the rate of increase of the energy of the element. = 2 rL dr Cp (T / t)


where dV=2rLdr-------- volume

38
Substituting these expressions in Eq.(1) we get

[ /r ( 2 rLk (T /r) )]dr + 2 rL dr q = 2 rL dr Cp (T / t)


Simplifying we get

(1 / r) /r [kr(T/r)] +q = Cp (T / t)

Example2.2.By writing an energy balance for a differential spherical volume element in the
variable (r is any radius), derive the one-dimensional time dependent heat conduction
equation with internal heat generation and variable thermal conductivity in the spherical
coordinate system.

Sketch:

dr

Qr+dr
Qr

Qg

Find: One dimensional time dependent heat conduction equation

Assumptions: One dimensional time dependent radial heat conduction

Solution: Consider a spherical element of thickness dr at any radius r as shown in the figure
above. The energy balance equation for one dimensional radial unsteady state conduction
with heat generation is given by

Qr + Qg Qr+dr = (E / t)

Or Qr + Qg [Qr + (Qr / r) dr] = (E / t)

Or - (Qr / r) dr + Qg = (E / t) (1)

Where Qr = rate of heat conducted in to the element at radius r = - k 4 r2 (T / r),

Qg = rate of heat generation within the element = (4/3) [(r + dr)3 r3 ] q

39
(E / t) = rate of increase of energy of the element = (4/3) [ (r + dr) 3 r3 ](T/t)

Now (r + dr)3 r3 = r3 + 3r2dr + 3r(dr)2 + (dr)3 r3

= 3r2dr + 3r(dr)2 + (dr)3

Neglecting higher order terms like (dr)3 and (dr)2 we have

(r + dr)3 r3 = 3 r2 dr.

Therefore Qg = 4 r2 dr q

and (E / t) = 4 r2 dr Cp(T/t).

Substituting the expressions for Qr, Qg and (E / t) in Eq. (1) we have

[ /r{- k 4 r2 (T / r)}]dr + 4 r2 dr q = 4 r2 dr Cp(T/t)

Simplifying the above equation and noting that if k is given to be constant we have

/r{ r2 (T / r)} + r2 (q / k) = ( r2 Cp / k)(T/t)

Or (1 / r2) /r{ r2 (T / r)} + (q / k) = (1 / ) (T/t); where = k / ( Cp)

Example2.3.By simplifying the three-dimensional heat conduction equation, obtain one


dimensional steady-state conduction equation with heat generation and constant
thermal conductivity for the following coordinate systems:
(a) Rectangular coordinate in the x variable; (b) Cylindrical coordinate in the r variable.
(c) Spherical coordinates in the r variable

Find: Differential equation forone dimensional time dependent heat conduction with heat
generation

Assumptions: One dimensional time dependent radial heat conduction with heat generation

Solution: (a) The general form of conduction equation for an isotropic solid in rectangular
coordinate system is given by

/ x (kT / x) + / y (kT / y) + / z (kT / z) +q = ( Cp) (T / t)


..(1)
For steady state conduction (T / t) = 0 ; For one dimensional conduction in x direction
we have
T / y = T / z = 0 . Therefore T / x = dT / dx .
Therefore Eq. (1) reduces to

40
d / dx (k dT / dx) + q = 0.

For constant thermal conductivity the above equation reduces to

d2 T / dx2 + q / k = 0.
(b) The general form of conduction equation in cylindrical coordinate system is given by

(1 / r) / r (kr T / r) + (1 / r2) / (k T / ) + / z (kT / z) + q = Cp(T / t)

For steady state conduction, (T / t) = 0 ; For one-dimensional radial conduction we have


T / = 0 and T / z = 0. Therefore T / r = dT / dr. With these simplifications the
general form of conduction equation reduces to

(1 / r) d / dr (kr dT/dr) + q = 0

For constant thermal conductivity the above equation reduces to

(1 / r) d / dr (r dT/dr) + q / k = 0.

(a) The general form of conduction equation in spherical coordinate system is given by

(1/r2) / r(kr2 T / r) + {1/(r2 sin 2 )} / (k T/)

+ {1/(r2 sin )} / (k sin T/) + q = Cp (T/ t) ..(1)

For steady state conduction (T t) = 0 ; For one dimensional radial conduction we have

T/ = 0 and T/ = 0. Therefore T / r = dT / dr. Substituting these conditions in


Eq. (1) we have (1/r2) d / dr (kr2 dT / dr) + q = 0.

For constant thermal conductivity the above equation reduces to

(1/r2) d / dr (r2 dT / dr) + q / k = 0.

B. Mathematical Formulation of Boundary conditions:

Example2.4.A plane wall of thickness L is subjected to a heat supply at a rate of q0 W/m2 at


oneboundary surface and dissipates heat from the surface by convection to the ambient
which is at a uniform temperature of Twith a surface heat transfer coefficient of
h.Write the mathematical formulation of the boundary conditions for the plane wall.

Sketch :

41
h,T

q0
Boundary conditions are :
L (i)at x = 0; k (dT / dx)x = 0 = q0

(ii) at x = L; k(dT / dx)x = L = h(T|x=L - T)

Example2.5.Consider a solid cylinder of radius R and height Z. The outer curved surface of
thecylinder is subjected to a uniform heating electrically at a rate of q0 W / m2.Both the
circular surfaces of the cylinder are exposed to an environment at a uniform
temeperature T with a surface heat transfer coefficient h.Write the mathematical
formulation of the boundary conditions for the solid cylinder.

Sketch : Known: (i) Solid cylinder of radius R; (ii)Surface


heat flux at radiyus R = q0 W/m2; (iii) The two circular faces are exposed to ambient at T
with surface heat transfer coefficient h
Solution:
h,T Boundary conditions are:
z (i) at r = 0; (T/r) = 0 (axis of symmetry)
R
(ii) at r = R; k(T/r) = q0

(iii) at z = 0; h[T|z=0 - T] + k(T/z)z=0 =0

q0 (iv) at z = Z; k(T/r) = h [ T|z = z T]


q0
Z

h , T

Example2.6.A hollow cylinder of inner radius ri, outer radius r0 and height H is subjected
to thefollowing boundary conditions.
(a) The inner curved surface is heated uniformly with an electric heater at a constant rate of
q0 W/m2,

42
(b) the outer curved surface dissipates heat by convection into an ambient at a uniform
temperature, T with a convective heat transfer coefficient,h
(c) the lower flat surface of the cylinder is insulated, and
(d) the upper flat surface of the cylinder dissipates heat by convection into the ambient at T
with surface heat transfer coefficient h. Write the mathematical formulation of the boundary
conditions for the hollow cylinder.

Sketch for example 2.6:


Solution:
hz,T
hr,T Boundary conditions are:
(i) at r = ri, k(T/r) = q0 for all z;

(ii) at r = ro, k(T/r) = hr[T|r=r2 - T]


q0 for all z
H
q0
(iii) at z = 0, (T/z) = 0 for all r.

(iv) at z = H,

k(T/z)z =H = hz[T|z=H - T]
for all r
ri from the problem: hz=hr=h
Insulated ro

C. Formulation of Heat Conduction Problems:

Example2.7.A plane wall of thickness L and with constant thermal properties is initially
at auniform temperature Ti. Suddenly one of the surfaces of the wall is subjected to
heatingby the flow of a hot gas at temperature T and the other surface is kept
insulated. Theheat transfer coefficient between the hot gas and the surface exposed to it
is h. There isno heat generation in the wall. Write the mathematical formulation of the
problem todetermine the one-dimensional unsteady state temperature within the wall.

Sketch for example 2.7:

43
T = Ti at t = 0

L
Insulated h,T
T=
T(x,t)

Solution:
Governing differential equation to determine T(x,t) is given by

( 2T / x2) = (1 / ) (T / t)

where is the thermal diffusivity of the wall.

Initial condition is at time t = 0 T = Ti for all x.

The boundary conditions are : (i) at x = 0, (T / x)x=0 = 0. (Insulated) for all t >0

(ii) at x = L, k ( T / x)x=L = h [T|x=L T] for all t>0

Example2.8.A copper bar of radius R is initially at a uniform temperature Ti. Suddenly the
heating of the rod begins at time t=0 by the passage of electric current, which generates
heat at a uniform rate of q W/m3. The outer surface of the dissipates heat into an
ambient at a uniform temperature T with a convective heat transfer coefficient h.
Assuming that thermal conductivity of the bar to be constant, write the mathematical
formulation of the heat conduction problem to determine the one-dimensional radial
unsteady state temperature distribution in the rod.

Sketch for example 2.8:

h,T

T = Ti at t 0
q for t 0
R

Solution:

44
The governing differential equation to determine T(r,t) is given by

(1/r) / r (r T / r) + q0 / k = (1/) (T / t).

Boundary conditions are: (i) at r = 0, (T / r) = 0 ( Axis of symmetry)

(ii) at r = R, k (T / r)|r=R = h [T |r=R T]

Initial condition is : At t = 0, T = Ti for all r

Example2.9.Consider a solid cylinder of radius R and height H. Heat is generated in the


solid at auniform rate of q W/m3. One of the circular faces of the cylinder is insulated and
theother circular face dissipates heat by convection into a medium at a uniform
temperature of T with a surface heat transfer coefficient of h. The outer curved surface
of the cylinder is maintained at a uniform temperature of T0. Write the mathematical
formulation to determine the two-dimensional steady state temperature distribution
T(r,z) in the cylinder.

Sketch for example 2.9:


z Solution: The governing
h, T differential equation to determine
T(r,z) is given by
T0
(1/r) /r(rT/r) + 2T/z2

+ q/k = 0
H Boundary conditions are:
R (i) at r = 0, T/r|r=0 = 0, for all z
(axis
of symmetry).
(ii) at r = R, T = T0 for all z.
r
(iii) at z = 0 , T/z|z=0 = 0 for all r.

Insulated (iv) at z = H,
k (T/z)z=H = h (T |z=H T)
for all x.

Example2.10.Consider a rectangular plate as shown in Fig. 2.10. The plate is generating


heatat a uniform rate of q W/m3. Write the mathematical formulation to determine two
dimensional steady state temperature distribution in the plate.

45
h,T

T0

bb
a
a
x

Insulated
Fig. 2.10 : Schematic for example 2.10

Solution:The governing differential equation to determine T(x,y) is given by

2T / x2 + 2T / y2 + q/k = 0

Boundary conditions are:

(i) at x=0, k(T / x)|x=0 = q0 for all y ; (ii) at x = a, T = T0 for all y

(iii) at y = 0, T / y = 0 for all x ; (iv) at y = b, k(T / y)|y=b = h[T |y=b T].

Example2.11.Consider a medium in which the heat conduction equation is given in its simple
form as

2T / x2 = (1/) (T / t)

(a) Is heat transfer in this medium steady or transient?


(b) Is heat transfer one-, two- or three-dimensional?
(c) Is there heat generation in the medium?
(d) Is thermal conductivity of the medium constant or variable?

Solution:The given differential equation is

2T / x2 = (1/) (T / t)

It can be seen from this equation that T depends on one space variable x and the time variable
t. Hence the problem is one dimensional transient conduction problem. No heat generation
term appears in the equation indicating that the medium is not generating any heat.The
thermal conductivity of the medium does not appear within the differential symbol indicating
that the conductivity of the medium is constant.

46
Example2.12. Consider a medium in which the heat conduction equation is given in its
simple form as
(1/r) d / dr(r k dT/dr) + q = 0.

(a) Is heat transfer steady or unsteady?


(b) Is heat transfer one-, two- or three-dimensional?
(c) Is there heat generation in the medium?
(d) Is the thermal conductivity of the medium constant or variable?

Solution:The given differential equation is

(1/r) d / dr(r k dT/dr) + q = 0.

It can be seen from this equation that the temperature T depends only on one space variable
r and it does not depend on time t. Also the heat generation term q appears in the
differential equation.Hence the problem is a one-dimensional steady state conduction
problem with heat generation. Since the thermal conductivity appears within the differential
symbol, it follows that the thermal conductivity of the medium is not a constant but varies
with temperature.

Example2.13.Consider a medium in which heat the heat conduction equation in its simplest
form is given as
(1/r2) /r (r2 T /r) = (1/) (T/t)

(a) Is heat transfer steady state or unsteady state?


(b) Is heat transfer one-, two- or three-dimension?
(c) Is there heat generation in the medium?
(d) Is the thermal conductivity constant or variable?

Solution:The given differential equation is

(1/r) /r (k r T /r) + /z (k T /z) + q = 0

It can be seen from the above equation that the temperature T depends on two space variables
r and z and does not depend on time. There is the heat generation term appearing in the
equation and the thermal conductivity k appears within the differential symbol /r and
/z. Hence the problem is two-dimensional steady state conduction with heat generation in
a medium of variable thermal conductivity.

Example2.14.Consider a medium in which the heat conduction equation is given in its


simplest form as
(1/r) /r (k r T /r) + /z (k T /z) + q = 0

(a) Is heat transfer steady state or unsteady state?


(b) Is heat transfer one-, two- or three-dimension?
(c)Is there heat generation in the medium?

47
(d)Is the thermal conductivity constant or variable?

Solution:The given differential equation is

1 2T
2 2
(1/r ) /r (r T /r) + ---------- [ -------] = (1/) (T/t)
r2 sin 2 2

It can be seen from the given equation that the temperature T depends two space variables r
and and it also depends on the time variable t. There is no heat generation term appearing in
the given equation . Also the thermal conductivity k do not appear within the differential
symbol. Hence the given equation represents two-dimensional, steady state conduction in a
medium of constant thermal conductivity and the medium is not generating any heat.

Example2.15.Consider the north wall of a house of thickness L. The outer surface of the wall
exchanges heat by both convection and radiation.The interior of the house is maintained at a
uniform temperature of Ti, while the exterior of the house is at a
uniform temperature T0. The sky, the ground, and the surfaces of the surrounding structures
at this location can be modeled as a surface at an effective temperature of Tsky for radiation
heat exchange on the outer surface.The radiation heat exchange between the inner surface of
the wall and the surfaces of the other walls, floor and ceiling are negligible.The convective
heat transfer coefficient for the inner and outer surfaces of the wall under consideration are
hi and h0 respectively.The thermal conductivity of the wall material is K and the emissivity of
the outer surface of the wall is 0. Assuming the heat transfer through the wall is steady and
one dimensional, express the mathematical formulation (differential equation and boundary
conditions) of the heat conduction problem

Sketch for example 2.15:

Surface in
contact with
fluid at Ti x
and surface L T (x)
heat transfer
coefficient hi qradiation
surface in contact with fluid
at T0 and surface heat
qconduction transfer coefficient h0

qconvection
0

48
Solution: The problem is one-dimensional steady state conduction without any heat
generation and the wall is of constant thermal conductivity. Hence the governing differential
equation is
d2T / dx2 = 0.

The boundary conditions are:

(i) at x = 0, hi [Ti T |x = 0 ] = k (dT/dx)|x =0 ;

(ii) at x = L, qconduction = qconvection + qradiation

Or k (dT/dx)|x =L = h0[T|x = L T0] + 0 [{T|x = L}4 Tsky4}

49
CHAPTER 3

ONE DIMENSIONAL STEADY STATE CONDUCTION


3.1. Introduction:- In this chapter the problems of one-dimensional steady state conduction
without and with thermal energy generation in slabs, cylinders and spheres and subjected to
different types of boundary conditions are analyzed to determine the temperature distribution
and rate of heat flow. The concept of thermal resistance is introduced and the use of this
concept, for solving conduction in composite layers is illustrated. The problem of critical
thickness of insulation for cylinder and sphere are also analyzed. The effects of variable
thermal conductivity on temperature distribution and rate of heat transfer are also studied.
Finally the problems of one dimensional heat conduction in extended surfaces (fins)
subjected to different types of boundary conditions are examined.

3.2. Conduction Without Heat Generation

3.2.1. The Plane Wall (The Infinite Slab):- The statement of the problem is to determine the
temperature distribution and rate of heat transfer for one dimensional steady state conduction
in a plane wall without heat generation subjected to specified boundary conditions.

T = T(x)

T1 T2
Qx
R = L /(Ak)

L
Fig. 3.1: One dimensional steady state conduction in a slab

The governing equation for one dimensional steady state conduction without heat
generation is given by

=0

..(3.1)

Integrating Eq.(3.1) twice with respect to x we get

50
T = C1x + C2 (3.2)

where C1 and C2 are constants which can be evaluated by knowing the boundary conditions.

Plane wall with specified boundary surface temperatures:- If the surface at x = 0 is


maintained at a uniform temperature T1 and the surface at x = L is maintained at another
uniform temperature T2, then the boundary conditions can be written as follows:
(i) at x = 0, T(x) = T1 ; (ii) at x = L, T(x) = T2.

Condition (i) in Eq.(3.2) givesT1 = C2.

Condition (ii) in Eq. (3.2) gives T2 = C 1 L + T1



Or C1 =

Substituting for C1 and C2 in Eq. (3.2), we get the temperature distribution in the plane wall
as


T(x) = (T2 T1) + T1

Or T(x) T1 x
------------ = -------- ..(3.3)
(T2 T1) L

Expression for Rate of Heat Transfer:

The rate of heat transfer at any section x is given by Fouriers law as

Qx = k A(x) (dT / dx)

For a plane wall A(x) = constant = A. From Eq. (3.3), dT/dx = (T2 T1) / L.

Hence kA(T1 T2)


Qx = ---------------- ..(3.4)
L

Concept of thermal resistance for heat flow:


It can be seen from the above equation that Qx is independent of x and is a constant.
Eq.(3.4) can be written as
(T1 T2) (T1 T2)
Qx = -------------- = ------------------ ..(3.5)
{L /(kA)} R
where R = L / (Ak).

51
Eq. (3.5) is analogous to Ohms law for flow of electric current. In this equation (T1 T2) can
be thought of as thermal potential, R can be thought of as thermal resistance,so that the
plane wall can be represented by an equivalent thermal circuit as shown in
Fig.3.1.The units of thermal resistance R are 0 K / W.

Plane wall whose boundary surfaces subjected to convective boundary conditions:

Surface in contact with a fluid


at To with heat transfer
coefficient ho

Surface in contact with a fluid


at Ti with heat transfer
x coefficient hi
L
T1 T2
Rci R Rco
Ti
To
Qx Qx Qx Qx

Fig.3.2: Thermal Circuit for a plane wall with convective boundary conditions

Let T1 be the surface temperature at x = 0 and T2 be the surface temperature at x = L.


If we assume that Ti> To, then for steady state conduction heat will transfer by convection
from the fluid at Ti to the surface at x = 0, then it is conducted across the
plane wall and finally heat is transferred by convection from the surface at x = L to the fluid
at To.
The expression for rate of heat transfer Qx can be written as follows:

Qx = hi A [Ti T1]

(Ti T1) (Ti T1)


or Qx = --------------- = ---------------- (3.6a)
1 / (hi A) Rci

Rci = 1 / (hiA) is called thermal resistance for convection at the surface at x = 0


(T1 T2)

Similarly Qx = --------------- (3.6b)


R

where R = L /(Ak) is the thermal resistance offered by the wall for conduction and

52
(T2 To)
Qx = --------------- ..(3.6c)
Rco

Where Rco = 1 / (hoA) is the thermal resistance offered by the fluid at the surface at x = L for
convection. It follows from Equations (3.6a), (3.6b) and (3.6c) that

(Ti T1) (T1 T2) (T2 T0)


Qx = --------------- = ------------------ = --------------
Rci R Rco

(Ti To)
Or Qx = ------------------- (3.7)
[Rci + R+ Rco]

3.2.2. Radial Conduction in a Hollow Cylinder:

The governing differential equation for one-dimensional steady state radial conduction in a
hollow cylinder of constant thermal conductivity and without thermal energy generation is
given byEq.(2.10b) with n = 1: i.e.,
d
--- [r (dT / dr)] = 0 .(3.8)
dr

Integrating the above equation once with respect to r we get

r (dT / dr) = C1

or (dT / dr) = C1/ r


3.5
Integrating once again with respect to r we get

T(r) = C1 ln r + C2 ..(3.9)

where C1 and C2 are constants of integration which can be determined by knowing the
boundary conditions of the problem.

Hollow cylinder with prescribed surface temperatures: Let the inner surface at r = r1 be
maintained at a uniform temperature T1 and the outer surface at r = r2 be maintained at
another uniform temperature T2 as shown in Fig. 3.3.

Substituting the condition at r1 in Eq.(3.9) we get

T1 = C1 ln r1 + C2 .(3.10a)

and the condition at r2 in Eq. (3.9) we get

53
T2 = C1 ln r2 + C2.(3.10b)

Solving for C1 and C2 from the above two equations we get

(T1 T2) (T1 T2)


C1 = ---------------- = -------------------
[ln r1 ln r2] ln (r1 / r2)

(T1 T2)
and C2 = T1 ------------------ ln r1
ln (r1 / r2)

Substituting these expressions for C1 and C2 in Eq. (3.9) we have

(T1 T2) (T1 T2)


T(r) = -------------- ln r + T1 ---------------- ln r1
ln (r1 / r2) ln (r1 / r2)

or [T(r) T1] ln (r / r1)


--------------- = ------------------- (3.11)
[ T2 T1] ln (r2 / r1)

T2
T1

r2
r1

Fig.3.3: Hollow cylinder with prescribed surface temperatures

Eq. (3.11) gives the temperature distribution with respect to the radial direction in a hollow
cylinder. The plot of Eq. (3.11) is shown in Fig. 3.4.
Expression for rate of heat transfer:- For radial steady state heat conduction in a hollow
cylinder without heat generation energy balance equation gives

Qr = Qr|r = r1 = Qr|r = r2

54
Hence Qr = k [A(r) (dT / dr)] |r = r1 .(3.12)

Now A(r) |r = r1 = 2 r1 L .From Eq. (3.11) we have

(dT / dr) = {[ T2 T1] / ln (r2 / r1) }(1/r)

Hence (dT / dr)|r = r1 = {[ T2 T1] / ln (r2 / r1) }(1/r1).

Substituting the expressions for A(r)|r = r1 and (dT / dr)|r = r1 in Eq. (3.12) we get the expression
for rate of heat transfer as

2 L k (T1 T2)
Qr = --------------------------.(3.13)
ln (r2 / r1)

Thermal resistance for a hollow cylinder: Eq. 3.13 can be written as :

Qr = (T1 T2) / R .(3.14a)


3.7

(T T1)
(T2 T1)

0 r / r1
1.0 r2 / r1

Fig. 3.4:Radial temperature distribution for a hollow cylinder with


prescribed surface temperatures at inner and outer radii

ln (r2 / r1) 1
where R = ----------------- = --------- ..(3.14b)
2Lk k Am

Where Am = (A2 A1) / ln (A2 / A1), when A2 = 2 r2 L = Area of the outer surface of the

55
cylinder and A1 = 2 r1 L = Area of the inner surface of the cylinder, and Am is logarithmic
mean area.

Hollow cylinder with convective boundary conditions at the surfaces:- Let for the hollow
cylinder, the surface at r = r1 is in contact with a fluid at temperature Ti with a surface heat
transfer coefficient hi and the surface at r = r2 is in contact with another fluid at a
temperature To as shown in Fig.3.5.By drawing the thermal circuit for this problem and using
the concept of thermal resistance it is easy and straight forward to write down the expression
for the rate of heat transfer as shown.
(Ti T1)
Now Qr = hiAi(Ti T1) = 2 r1L hi (Ti T1) = -------------- ..(3.15a)
Rci

where Rci = 1 / (2 r1Lhi)..(3.15b)

(T1 T2)
Also Qr = -------------- ..(3.15c)
R

where R = ln (r2 / r1) / (2Lk).(3.15d)

Surface in
contact with
fluid at To and Surface in contact
heat transfer With fluid at Ti and
coefficient ho Surface heat transfer
Coefficient hi

r1
r2

Qr
Rco R Rci

(T2 To)
And Qr = --------------- .(3.15e)
Rco

1
Where Rco = --------------..(3.15f)
(2r2Lho)

56
From Eqs.(3.15a), (3.15c) and (3.15e) we have

(Ti T1) (T1 T2) (T2 To)


Qr = ------------- = -------------- = ----------------
RciR Rco

(Ti To)
Or Qr = ---------------------- .(3.16)
Rci + R + Rco

where Rci, R and Rco are given by Eqs.(3.15b), (3.15d) and (3.15f) respectively.

3.2.3. Radial Conduction in a Hollow Sphere:

The governing differential equation for one-dimensional steady state radial conduction in a
hollow sphere without thermal energy generation is given byEq.(2.10b) with n = 1: i.e.,

d
--- [r2(dT / dr)] = 0 .(3.17)
dr

Integrating the above equation once with respect to r we get

r2 (dT / dr) = C1

or (dT / dr) = C1/ r2

Integrating once again with respect to r we get

T(r) = C1 / r + C2 ..(3.18)

where C1 and C2 are constants of integration which can be determined by knowing the
boundary conditions of the problem.

Hollow sphere with prescribed surface temperatures:

(i) Expression for temperature distribution:-Let the inner surface at r = r1 be maintained at a


uniform temperature T1 and the outer surface at r = r2 be maintained at another uniform
temperature T2 as shown in Fig. 3.6.

The boundary conditions for this problem can be written as follows:

(i) at r = r1, T(r) = T1 and (ii) at r = r2, T(r) = T2.

57
Condition (i) in Eq. (3.18) gives T1 = C1 / r1 + C2 .(3.19a)

Condition (ii) in Eq. (3.18) gives T2 = C1 / r2 + C2 .(3.19b)

Solving for C1 and C2 from Eqs. (3.19a) and (3.19b) we have

(T1 T2) (T1 T2)


C1 = ------------------- and C2 = T1 + --------------------------
[1 / r2 1 / r1] r1[1 / r2 1 / r1]

Substituting these expressions for C1 and C2 in Eq. (3.18) we get

(T1 T2) / r (T1 T2) / r1


T(r) = ----------------------- + T1 + ----------------------
[1 / r2 1 / r1] [1 / r2 1 / r1]

Surface at temperature T2

Surface at temperature T1
r2

r1

Fig. 3.6: Radial conduction in a hollow sphere with prescribed surface


temperatures

Or T(r) T1[1 / r2 1 / r]
-----------------= ---------------------- (3.20)
[T1 T2] [1 / r2 1 / r1]

(ii) Expression for Rate of Heat Transfer:- The rate of heat transfer for the hollow sphere is
given by

Qr = k A(r)(dT / dr)..(3.21)

Now at any radius for a sphere A(r) = 4 r2 and from Eq. (3.20)

58
1
dT / dr = [T1 T2]------------------ (1 / r2)
[1 / r2 1 / r1]
Substituting these expressions in Eq. (3.21) and simplifying we get

4 k r1 r2 [T1 T2]
Qr = -------------------------- ...(3.22)
[r2 r1]

Eq.(3.22) can be written as Qr = [T1 T2] / R ..(3.23a)

Where R is the thermal resistance for the hollow sphere and is given by

R = (r2 r1) / {4 k r1 r2} .(3.23b)

Hollow sphere with convective conditions at the surfaces:- Fig. 3.7 shows a hollow sphere
whose boundary surfaces at radii r1 and r2 are in contact with fluids at temperatures Ti and T0
with surface heat transfer coefficients hi and h0 respectively.

Surface in contact with


fluid at T0 and surface
heat transfer coefficient h0
r2
Surface in contact with
fluid at Ti and surface heat
transfer coefficient hi
r1

Fig. 3.7: Radial conduction in a hollow sphere with convective


conditions at the two boundary surfaces

The thermal resistance network for the above problem is shown in Fig.3.8

Qci = Qr= Qco (3.24)

Where Qci = heat transfer by convection from the fluid at Ti to the inner surface of the hollow
sphere and is given by
[Ti T1]
Qci = hi Ai [Ti T1] = --------------- ..(3.25)
Rci

59
Qr Qco
TiQci To
Rci R Rco

.
Fig. 3.8: Thermal circuit for a hollow sphere with convective boundary conditions

When T1 = the inside surface temperature of the sphere and

Rci = 1 / (hiAi) = the thermal resistance for convection for the inside surface

Or Rci = 1 / (4 r12 hi).(3.25b)

Qr = Rate of heat transfer by conduction through the hollow sphere

= [T1 T2] / R with R = (r2 r1) / {4 k r1 r2}

And Qco = Rate of heat transfer by convection from the outer surface of the sphere to the
outer fluid and is given by
[T2 T0]
Qco = ho Ao [T2 To] = --------------- (3.26a)
Rco

Where T2 = outside surface temperature of the sphere and

Ao = outside surface area of the sphere = 4 r22 so that

Rco = 1 / {4 r22 ho}.(3.26b)

Now Eq.(3.24) can be written as

[Ti T1] [T1 T2] [T2 To]


Qr = hi Ai [Ti T1] = -------------- = ---------------- = ----------------
Rci R Rco

60
[Ti To]
Qr = ---------------------- (3.27)
[Rci + R + Rco]

3.2.4. Steady State conduction in composite solids:

There are many engineering applications in which heat transfer takes placethrough a medium
composed of several different layers, each having different thermal conductivity. These
layers may be arranged in series or in parallel or they may be arranged with combined
series-parallel arrangements. Such problems can be conveniently solved using electrical
analogy as illustrated in the following sections.

Composite Plane wall:- (i) Layers in series: Consider a plane wall consisting of three
layers in series with perfect thermal contact as shown in Fig. 3.10.The equivalent thermal
resistance network is also shown. If Q is the rate of heat transfer through an area A of the
composite wall then we can write the expression for Q as follows:

L1 L2 L3

Surface in
contact Surface in contact with a fluid
with fluid at T0 and surface heat
at Ti and k1 k2 k3 transfer coefficient ho
surface
heat
transfer
coefficient
hi
T1 T2 T3 T4

Rci R1 R2 R3 Rco

Q Q

Ti T1 T2 T3 T4 To

Fig. 3.10: A composite plane wall with three layers in series and the equivalent thermal
resistance network

(Ti T1) (T1 T2) (T2 T3) (T3 T4) (T4 Tco)
Q = -------------- = --------------- = ------------- = ------------ = ----------------
Rci R1 R2 R3 Rco

61
(Ti T0) (Ti T0)
Or Q = --------------------------------- = ------------.(3.28)
Rci + R1 + R2 + R3 + Rco Rtotal

Overall heat transfer coefficient for a composite wall:- It is sometimes convenient to


express the rate of heat transfer through a medium in a manner which is analogous to the
Newtons law of cooling as follows:

If U is the overall heat transfer coefficient for the composite wall shown in Fig. (3.10) then
Q = U A (Ti To) ...(3.29)

Comparing Eq. (3.28) with Eq. (3.29) we have the expression for U as
1
U = --------------- ..(3.30)
A Rtotal
1 1
Or U = --------------------------------= ----------------------------------------------------------------
A [ Rci + R1 + R2 + R3 ] A[1/(hiA) + L1/(Ak1) + L2/(Ak2) + L3/(Ak3)+ 1/(hoA)]

1
Or U = --------------------------------------------(3.31)
[ 1/hi + L1 / k1 + L2 / k2 + L3 / k3 + 1/ho]

(ii) Layers in Parallel:- Fig.3.11 shows a composite plane wall in which three layers are
arranged in parallel. Let b be the dimension of these layers measured normal to the plane of
the paper. Let one surface of the composite wall be in contact with a fluid at temperature Ti
and surface heat transfer coefficient hi and the other surface of the wall be in contact with
another fluid at temperature To with surface heat transfer coefficient ho. The equivalent
thermal circuit for the composite wall is also shown in Fig. 3.11. The rate of heat transfer
through the composite wall is given by

Q = Q1 + Q2 + Q3 .(3.32)

where Q1 = Rate of heat transfer through layer 1,

Q2 = Rate of heat transfer through layer 2, and

Q3 = Rate of heat transfer through layer 3.


F
(T1 T2)
Now Q1 = -------------- ...(3.33a)
R1

Where R1 = {L / (H1bk1)}
F

62
L H1

Surface in
contact k1 H2
with fluid
at Ti with
heat k2
transfer H3
coefficient
hi
k3
b Suface in contact
with fluid at To
and surface heat
Q1 transfer
R1 coefficient ho
Ti
T1 Q2 T2 To
Rci R2 Rco
Q
Q3
R3

FFig. 3.11: Schematic and equivalent thermal circuit for a composite wall with
layers in parallel

(T1 T2)
Similarly Q2 = -------------- (3.33b)
R2

Where R2 = {L / (H2bk2)}

and
(T1 T2)
and Q3 = -------------- = .. .(3.33c)
R3

Where R3 = {L / (H3bk3)}

Substituting these expressions in Eq. (3.32) and simplifying we get

(T1 T2) (T1 T2) (T1 T2) (T1 T2)


Q = ------------- + ---------------- + ----------------- = -------------------- .(3.34)
R1 R2 R3 Re

63
Where 1 / Re = 1/R1 + 1/R2 + 1/R3

(Ti T1) (T1 T2) (T2 To) (Ti To)


Hence Q = ----------- = ------------ = ------------- = -------------------- (3.35)
Rci Re Rco [Rci + Re + Rco]

Composite Coaxial Cylinders :- Fig. 3.12. shows a composite cylinder having two layers in
series. The equivalent thermal circuit is also shown in the figure.The rate of heat

transfer through the composite layer is given


k2

k1
r2 r3
r1

Surface in contact with


Surface in contact with fluid at Ti and surface
fluid at To and surface heat transfer coefficient
heat transfer coefficient h0 hi

Ti T1 T2 T3 To

Q Rci R1 R2 Rco

Fig. 3.12: Schematic and thermal circuit diagrams for a composite


cylinder
by

(Ti T1) (T1 T2) (T2 T3) (T3 To) (Ti T0)
Now Q = ------------- = ------------ = ----------- = ------------- = -----------------------------
Rci R1 R2 Rco [Rci + R1 + R2 + Rco]

..(3.36)

64
1 1
Where Rci = 1 / [hiAi] = -------------- ; R1 = ---------- ln (r2 / r1)
2 r1L hi 2 L k1

1 1
Rco = 1 / [hoAo] = -------------- ; R2 = ---------- ln (r3 / r2)
2 r3L ho 2 L k2

The above expression for Q can be extended to any number of layers.

Overall Heat Transfer Coefficient for a Composite Cylinder:- For a cylinder the area of
heat flow in radial direction depends on the radius r we can define the overall heat transfer
coefficient either based on inside surface area or based on outside surface area of the
composite cylinder. Thus if Ui is the overall heat transfer coefficient based on inside surface
area Ai and Uo is the overall heat transfer coefficient based on outside surface area Ao then

Q = UiAi (Ti To) .(3.37)

From equations (3.36) and (3.37) we have

(Ti T0)
Now UiAi (Ti To) = -----------------------------
[Rci + R1 + R2 + Rco]

Substituting the expressions for Ai, Rci,R1,R2 and Rco in the above equation we have

1
2 r1L Ui = --------------------------------------------------------------------------------------------
[1 /(2r1Lhi) + {1/(2Lk1)}ln (r2 / r1) + {1/(2Lk2)}ln (r3 / r2) + 1/(2r3Lho)]

1
Or Ui = ------------------------------------------------------------------------- ..(3.38)
[ 1/hi + (r1 / k1) ln (r2/r1) + (r1/k2) ln (r3/r2) + (r1/r3) (1/ho) ]

Similarly it can be shown that

1
Uo = ------------------------------------------------------------------------------ ..(3.39)
[(r3/ r2) (1/hi) + (r3 / k1) ln (r2/r1) + (r3/k2) ln (r3/r2) + (1/ho) ]

65
Composite Concentric Spheres:- Fig.3.13 shows a composite sphere having two layers with
the inner surface of the composite sphere in contact with fluid at a uniform temperature Ti
and surface heat transfer coefficient hi and the outer surface in contact with another fluid at a
uniform temperature To and surface heat transfer coefficient ho. The corresponding thermal
circuit diagram is also shown in the figure.

k2

k1

r1 r2 r1

r3

Surface in contact with


fluid at Ti and surface
Surface in contact with heat transfer coefficient
fluid at To and surface hi
heat transfer coefficient h0

Ti T1 T2 T3 To

Q Rci R1 R2 Rco

Fig. 3.13: Schematic and thermal circuit diagrams for a composite


sphere

Eq. (3.36) is also applicable for the composite sphere of Fig. 3.13 except that the expression
for individual resistance will be different. Thus



Q= .(3.40)



where Rci = = ; R1 = ; Rco = = ;



R1 =

66
Example 3.1:Consider a plane wall 100 mm thick and of thermal conductivity 100
W/(m-K). Steady state conditions are known to exist with T1 = 400 K and T2 = 600 K.
Determine the heat flux (magnitude and direction) and the temperature gradient dT/dx
for the coordinate system shown in Fig. P3.1.

T(x) T(x)
T(x)

T2 T2
T2
T1 T1
T1
L L L
x
x x
(a) (b) (c)

Fig. P3.1 : Schematic for problem 3.1


Sketch for example 3.1(a):
T(x),
Solution:
(a) It can be seen from the figure that the temperature is
L increasing with increase in x: i.e., dT/dx is +ve.

Therefore dT/dx = (T2 T1) / L = (600 400)/0.10


T2 = 2000 0C / m.
T1
Heat flux = qx = k dT/dx = 100 * 2000 = 200,000 W/m2
x ve sign for qx indicates that heat transfer is taking place in
the ve direction of x.

(b)Sketch for example 3.1(b):

67
T(x) Solution:It can be seen from the figure
shown that temperature is decreasing
with increase in x or in other words
dT/dx is ve.
Therefore dT/dx = (T1 T2)/L
= (400 600) / 0.1 = 2000 0 C / m.
qx = k dT/dx = 100 * ( 2000)
= + 200,000 W / m2.
+ ve sign for qx indicates that heat
T2
T1 transfer is taking place in the + ve
L direction of x.

(c)Sketch for example 3.1(c):


Solution: It can be seen from the figure
T(x) shown that the temperature increasing with
increase in x: i.e., dT/dx is + ve.

Therefore dT/dx = (600 400) / 0.1

= 2000 0C / m.

Heat flux = qx = k dT/dx


T2
T1 = 100 * 2000 = 200,000 W/m2.
x
Negative sign in qx indicates that heat
transfer takes place in a direction opposite to
the + ve direction of x.

Example 3.2.Fig. P3.2 shows a frustum of a cone (k = 3.46 W/m-K). It is of circular cross
section with the diameter at any x is given by D = ax, where a = 0.25. The smaller cross
section is at x1 = 50 mm and the larger cross section is at x2 = 250 mm. The corresponding
surface temperatures are T1 = 400 K and T2 = 600 K. The lateral surface of the cone is
completely insulated so that conduction can be assumed to take place in x-direction only.
(i) Derive an expression for steady state temperature distribution, T(x) in the solid and
(ii) calculate the rate of heat transfer through the solid.

Sketch for example 3.2:

68
T2 T1
.Known: X1 = 50 mm;X2 =
250 mm; D= ax with a =
0.25;T1 = 400 K; T2 = 600 K;
k = 3.46 W/m-K

D2 D1 Find: (i) T(x) ; (ii) Q


D
Assumptions: (i) Steady state
X1 conduction (ii) One
dimensional conduction
x through the cone; (iii) thermal
X2 conductivity is constant.

Solution: By Fouriers law, the rate of heat transfer in x-direction across any plane at a
distance x from the origin o is given by

Qx = k Ax (dT/dx).

For steady state conduction without heat generation Qx will be a constant. Also at any x,
D = ax. Therefore

Qx = k (D2/4) (dT/dx) = k [(ax)2/4] (dT/dx).

Separating the variables we get,

dT = (4/a2k) Qx (dx/x2)

Integrating the above equation we have


T x
dT = ( 4Qx / a2 k) (dx /x2)
T X
1 1

Or T T1 = (4Qx / a2 k) [(1 / x) (1 / X1)]

( 4 Qx)
Or T = T1 --------------- [(1 / x ) (1 / X1)] (1)
( a2 k)

At x = X2, T = T2. Substituting this condition in Eq.(1) and solving for Qx we get

69
( a2 k) (T2 T1)
Qx = ----------------------------- .(2)
4 (1/X2 1/X1)

Substituting this expression for Qx in Eq. (1) we get the temperature distribution in the cone
as follows:

(T2 T1) (1/x 1/X1)


T(x) = T1 + --------------------------------- ..(3)
(1/X2 1/X1)

Substituting the given numerical values for X1, X2, T1 and T2 in Eq.(3) we get the
temperature distribution as follows:
(600 400) [ 1/ x 1/0.05]
T(x) = 400 + ------------------------------------
[ 1/0.25 1/0.05}

Or T(x) = 400 + 12.5 [ 20 1/x]  Temperature distribution

* (0.25)2 * 3.46 * [ 600 400 ]


And Qx = -------------------------------------------- = 2.123 W
4 * [ 1/0.25 1/0.05 ]

Example 3.3.A plane composite wall consists of three different layers in perfect thermal
contact. The first layer is 5 cm thick with k = 20 W/(m-K),the second layer is10 cm thick
with k = 50 W/(m-K) and the third layer is 15 cm thick with k = 100 W/(m-K).The outer
surface of the first layer is in contact with a fluid at 400 0C with a surface heat transfer
coefficient of 25 W/(m2K),while the outer surface of the third layer is exposed to an
ambient at 30 0C with a surface heat transfer coefficient of 15 W/(m2-K).Draw the
equivalent thermal circuit indicating the numerical values of the thermal resistances and
calculate the heat flux through the composite wall. Also calculatetheoverall heat transfer
coefficient for the composite wall.

70
Sketch for example 3.3:

L1 L2 L3

hi h0

k1 k2 k3

Ti
To
Q

Rci R1 R2 R3 Rc0

Known: L1 = 0.05 m ; L2 = 0.10 m ; L3 = 0.15 m ; k1 = 20 W /(m-K) ;

k2 = 50 W /(m-K) ; k3 = 100 W/(m-K) ; hi = 25 W /(m2 K) ; h0 = 15 W/(m2 K) ;

Ti = 400 0 C ; T0 = 30 0 C.

Find: (i) heat flux q; (ii) U

Assumptions: (i) Steady state conduction (ii) One dimensional conduction through the
composite wall; (iii) thermal conducvity of each layer is constant.

Solution:

1
Rci = 1 / (hiA1) = ---------------- = 0.04 m2 K / W (A1 = A2 = A3 = A4 = 1 m2)
25 * 1

0.05
R1 = L1 /(k1A1) = --------------- = 0.0025 m2 K / W.
20 * 1

0.10
R2 = L2 / (k2A2) = ---------------- = 0.002 m2 K / W.
50 * 1

0.15
R3 = L3/ (k3A3) = ------------------ = 0.0015 m2 K / W.
100 * 1

71
1
Rco = 1 / (h0A4) = ---------------- = 0.067 m2 K / W.
15 * 1

R = Rci + R1 + R2 + R3 + Rco = 0.04 + 0.0025 + 0.002 + 0.0015 + 0.067

Or R = 0.113 m2-K/W.

(Ti T0) (400 30)


Heat Flux through the composite slab = q = --------------- = ------------------
R 0.113

= 3274.34 W / m2.

If U is the overall heat transfer coefficient for the given system then

Q 1 1
U = ---------------- = ------------- = -------------- = 8.85 W / (m2 K).
(Ti T0) R 0.113

Example 3.4.A composite wall consisting of four different materials is shown in Fig
P3.10. Using the thermal resistance concept determine the heat transfer rate per m2 of
the exposed surface for a temperature difference of 300 0 C between the two outer
surfaces. Also draw the thermal circuit for the composite wall.

Sketchfor example 3.4:

T1

k1 k2 k2 k4

2m

k3
k3 1m

4cm 10 cm 5 cm

72
Known: k1 = 100 W/(m-K) ; L1 =0.04 m;k2 = 0.04 W/(m-K) ; L2 = 0.1 m;

k3 = 20 W/(m-K) ; L3 = 0.1 m ;k4 = 70 W/(m-K) ; L4 = 0.05 m;

W = Width of the wall perpendicular to the plane of paper = 1 m (assumed).

T1 T4 = 300 0 C; A1 = A4 = 1 x 2 = 2 m2; A2 = A3 = 1 x 1 = 1 m2.

Find: (i) q; (ii) Thermal circuit

Assumptions: (i) Steady state conduction (ii) One dimensional conduction through the
composite wall; (iii) thermal conducvity of each layer is constant.

Solution:
0.04
0
R1 = L1 / (A1k1) = ---------------- = 0.0002 C / W.
2 * 100

0.10
0
R2 = L2 / (A2k2) = ---------------- = 2.50 C / W.
1 * 0.04

0.10
0
R3 = L3 / (A3k3) = --------------- = 0.005 C / W.
1 * 20

0.05
0
R4 = L4 / (A4k4) = ------------------ = 0.00036 C / W.
2 * 70

Q R2
R1 R4
R3

Thermal potential = T1 T4

Fig. Thermal Circuit for the given Example

R2 and R3 are resistances in parallel and they can be replaced by a single equivalent
resistance Re, where

73
R2 R3 2.5 * 0.005
1 / Re = 1 / R2 + 1 / R3 or Re = --------------- = ----------------------- = 0.00499 0C/W
(R2 + R3) (2.5 + 0.005)

Now R1, Re and R4 are resistances in series so that

(T1 T4) 300


Q = --------------------- = ------------------------------------ = 40.82 x 103 W
(R1 + Re + R4) [0.002 + 0.00499 + 0.00036]

Heat transfer per unit area of the exposed surface is given by

q = Q / A1 = 40.82 / 2.0 = 20.41 kW.

Example 3.5.A composite cylindrical wall is composed of two materials of thermal


conductivity kA and kB. A thin electric resistance heater for which the interfacial contact
resistances are negligible separates the two materials. Liquid pumped through the inner
tube is at temperature Ti with the inside surface heat transfer coefficient hi. The outer
surface of the Composite wall isexposed to an ambient at a uniform temperature of To
with the outside surface heat transfer coefficient ho. Under steady state conditions a
uniform heat flux of qh is dissipated by the heater.
(a) Sketch the equivalent thermal circuit for the composite wall and express all thermal
resistances in terms of the relevant variables
(b)Obtain an expression that may be used to determine the temperature of theheater,Th.
(c)Obtain an expression for the ratio of heat flows to the outer and inner fluid, qo/qi.

Sketch for example 3.5:

74
ro heater
r1
ri
hi,Ti
Qi kA

Q0 ho,To

kB
Ri Rci Qi
Qtotal
Ro Rco Qo
Find: (b) Expression to determine Temperature of heater Th; (ii) q0 / qi

Assumptions:(i) Steady state conduction (ii) One dimensional radial conduction through the
composite wall; (iii) thermal conductivity of each layer is constant; (iii)No contact resistance
between the surfaces of the two layers

Solution: Refer the thermal circuit for the given example

1 1 1
Ri = ------------- ln (r1/ri) ; Rci = ------------------ = ---------------------
(2 LkA) hi Ai (2 ri L hi)

1 1 1
Ro = ------------- ln (ro/r1) ; Rco = ------------------ = ---------------------
(2 LkB) ho Ao (2 ro L ho)

Qi = (Th Ti) / [Ri + Rci ]; Qo = (Th To) / [Ro + Rco] ;

Qtotal = Qi + Qo = (Th Ti) / (Ri + Rci) + (Th To)/ (Ro + Rco)

(Th Ti) (Th To)


2 r1L q h = -------------------------- + -----------------------
ln (r1/ri) 1 ln (ro/r1) 1
---------- + ------------- ---------- + -------------
2 L kA 2 L ri hi 2 L kB 2 L ro ho

75
(Th Ti) (Th To)
Therefore q h = ------------------------------- + --------------------------------
[ (r1/kA) ln (r1/ri) + r1/(rihi)] [(r1/kB) ln (ro/r1) + r1/(roho)

The temperature Th of the heater can be obtained from the above equation.

Qo (Th To) / (Ro + Rco) (Th To) x (Ri + Rci)


Now ------- = --------------------------- = --------------------------
Qi (Th Ti) / (Ri + Rci) (Th Ti) x (Ro + Rco)

(Th To) [ 1 / (rihi) + (1 / kA) ln (r1/ri) ]


= ----------- x -------------------------------------
(Th Ti) [ 1 / (roho) + (1 / kB) ln (ro/r1) ]

Example 3.6.A hollow aluminum sphere with an electrical heater in the centre is used to
determine the thermal conductivity of insulating materials. The inner and outer radii of the
sphere are 15 cm and 18 cm respectively and testing is done under steady state conditions
with the inner surface of the aluminum maintained at 2500 C. In a particular test, a spherical
shell of insulation is cast on the outer surface of the aluminum sphere to a thikness of 12 cm.
The system is in a room where the air temperature is 20 0 C and the convection coefficient is
30 W/(m2 K). If 80 W are dissipated by the heater under steady state conditions, what is the
thermal conductivity of the insulating material?

Sketch for example 3.6:

ho,To
r3 Known: r1 = 0.15 m ; r2 =
r2 0.18 m ;r3 = 0.18 + 0.12 =
r1 0.3 m ;k1 = 204 W/(m-K) from
tables; k2 = 0.30 W/(m-K);ho
= 30 W/(m2-K);Q = 60 W; T1
T1
= 250 0 C ; To = 20 0 C.
k1 Find: k2

k2

Assumptions:(i) Steady state conduction (ii) One dimensional radial conduction through the
composite wall; (iii) thermal conductivity of each layer is constant; (iii)No contact resistance
between the surfaces of the two layers

Solution:
(r2 r1) (0.18 0.15)
R1 = ---------------- = ------------------------------ = 4.335 x 10 40 C / W.
4 k1 r1 r2 4 * 204 * 0.18 * 0.15

76
(r3 r2) (0.30 0.18)
R2 = ---------------- = ------------------------------ = 0.177 / k20 C / W.
4 k2 r2 r3 4 * 0.30 * 0.30 * 0.18

1 1
0
Rco = 1 / (hoAo) = ------------------- = --------------------- = 0.0295 C / W.
4 r32 ho 4 * (0.3)2 * 30

(T1 To)
Q = -------------------- or R2 = (T1 To) / Q (R1 + Rco)
R1 + R2 + Rco

Or R2 = (250 20) / 80 (4.335 * 10 4 + 0.0295) = 2.412

Therefore 0.177 / k2 = 2.412

Or k2 = 0.177 / 2.412 = 0.0734 W / (m-K)

Example 3.7.In a hollow sphere of inner radius 10 cm and outer radius 20, the inner surface
is subjected to a uniform heat flux of 1.6 x 10 5 W/m2 and the outer surface is maintained at a
uniform temperature of 0 0C.The thermal conductivity of the material of the sphere is 40 W
/(m K).Assuming one-dimensional radial steady state conduction determine the
temperature of the inner surface of the hollow sphere.

Sketch for example 3.7:

T0

R2
q0
R1

Find: T/r= R1

Assumption: (i)Steady state one dimensional radial conduction without heat generation; (ii)
thermal conductivity of the sphere is constant.

77
Solution: The governing equation for one-dimensional steady-state radial conduction in a
sphere without heat generation is given by

d/dr ( r2 dT / dr ) = 0 ..(1)

The boundary conditions are : (i) at r = R1, k (dT/dr)|r=R1 = q0

(ii) at r = R2 T(r) = 0.

Integrating Eq. (1) w.r.t. r once, we get

r2 (dT/dr) = C1

or dT / dr = C1 / r2 (2)

Integrating once again w.r.t. r we get

T(r) = C1 / r + C2 .. (3)

From (2) (dT/dr)r = R1 = C1 / R12

Hence condition (i) gives

kC1 / R12 = q0

Or C1 = q0 R12 / k

Condition (ii) in Eq.(3) gives 0 = C1 / R2 + C2

Or C2 = C1 / R2 = (q0R12) / (kR2)

Substituting the expressions for C1 and C2 in Eq. (3) we have

q0 R12 q0 R12
T(r) = -------------- -------------------
kr k R2

Substituting the numerical values for q0, k, R1 and R2 we have

1.6 * 105 * 0.12 1.6 * 105 * 0.12


T(r) = -------------------- / r --------------------
40 40 * 0.2

78
Or T(r) = (40 / r) 200

Therefore T(r) |r= R1 = (40 / 0.1) 200 = 200 0 C.

3.2.5. Thermal Contact Resistance: In the analysis of heat transfer problems for composite
medium it was assumed that there is perfect thermal contact at the interface of two layers.
This assumption is valid only the two surfaces are smooth and they produce a perfect contact
at each point.But in reality, even flat surfaces that appear smooth to the naked eye would be
rather rough when examined under a microscope as shown in Fig. 3.14 with numerous . peaks
and valleys.

T2
T1
Rcont

LA LB

Gap between solids

T1 Enlarged view of the contact surface


Tc1

Tc2 T2
Fig.3.14: Temperature drop across
a contact resistance

The physical significance of thermal contact resistance is that the peaks will form good
thermal contact, but the valleys will form voids filled with air.As a result the air gaps act as
insulation because of poor thermal conductivity of air.Thus the interface offers some
resistance to heat conduction and this resistance is called the thermal contact
resistance,Rcont. The value of Rcont is determined experimentally and is taken into account
while analyzing the heat conduction problems involving multi-layer medium.The procedure
is illustrated by means of a few examples below.

Example 3.8. A composite wall consists of two different materials A [k = 0.1 W/(m-k)] of
thickness 2 cm and B[ k = 0.05 W/(m-K)] of the thickness 4 cm. The outer surface of layer A
is in contact with a fluid at 2000C with a surface heat transfer coefficient of 15 W/(m2-K) and
the outer surface of layer B is in contact with another fluid at 50 0 C with a surface heat
transfer coefficient of 25 W/(m2-K). The contact resistance between layer A and layer B is
0.33 (m2-K) /W. Determine the heat transfer rate through the composite wall per unit area of
the surface. Also calculate the interfacial temperatures and the inner and outer surface
temperature.

79
Sketch for example 3.8:
h0,T0 Known: Ti = 200 0 C ; T0 = 50 0 C ;
hi,Ti
Rcont
hi = 15 W/(m2 K) ; h0 = 25W/(m2 K)
kA kB
kA = 0.1 W/(m-K) ; kB = 0.05 W/(m-K)
LA LB
Rcont = 0.33 (m2 K) /W.

Find: (i) q; (ii) Tc1, Tc2, T1, T2

T1 Assumptions:(i) Steady state


Tc1
conduction (ii) One dimensional
conduction through the composite wall;
Tc2 T2 (iii) thermal conductivity of each layer
is constant;

Solution: The equivalent thermal circuit is also shown in the figure.


1
Rci = 1/(hiAA) = ------- = 0.067 m2-K/W
(15 * 1)

R1 = LA/(kAAA) = 0.02 / (0.1 * 1) = 0.2 m2-K / W.

R2 = LB / (kBAB) = 0.04 / (0.05 * 1) = 0.8 m2 K / W.

Rco = 1 / (hoAB) = 1 / ( 25 * 1) = 0.04 m2 K / W.

R = Rci + R1 + Rcont + R2 + Rco = 0.067 + 0.2 + 0.33 + 0.8 + 0.04 = 1.437 m2 K / W.

(200 50)
Heat flux = q = (Ti To) / R = ------------------ = 104.4 W/m2
1.437

Now q = (Ti T1) / Rci or T1 = Ti q Rci = 200 (104.4 * 0.067) = 193 0 C.

Similarly Tc1 = T1 q R1 = 193 (104.4 * 0.2) = 172.12 0 C.

Tc2 = Tc1 q Rcont = 172.12 (104.4 * 0.33) = 137.67 0 C.

T2 = Tc2 q R2 = 137.67 (104.4 * 0.8) = 54.15 0 C.

Check : To = T2 q Rco = 54.15 (104.4 * 0.04) = 49.97 0 C

80
Example 3.9. A very thin electric heater is wrapped around the outer surface of a long
cylindrical tube whose inner surface is maintained at 5 0C. The tube wall has inner and
outer radii of 25 mm and 75 mm respectively and a thermal conductivity of 10 W/(m-K).
The thermal contact resistance between the heater and the outer surface of the tube
per unit length is 0.01 (m-K) / W. The outer surface of the heater is exposed to a fluid
with a temperature of 10 0C and a convection coefficient of 100 W/(m2-K). Determine
the heater power required per metre length of the tube to maintain a heater
temperature of 250C.

Sketch for example 3.9:

h,T
Heater
r2
r1
Qtube T1 T2

Qamb
Qamb
Rco
Qtotal
Rcont Rcond Qtube

Fig. P3.6: Schematic and Thermal circuit for Example 3.6

Known:r1 = 0.025 m ; r2 = 0.075 m ; k = 10 W /(m-K) ; T = 10 0 C ;

h = 100 W/(m2 K); T1 = 5 0 C ; Rcont = 0.01 m K / W ; T2 = 25 0 C.

Find: Q per metre length

Assumptions:(i) Steady state conduction (ii) One dimensional conduction through the
composite wall; (ii) thermal conductivity of each layer is constant ;(iii)No contact resistance
between the surfaces of the two layers

81
Solution:
1
Rco = 1 / (h Ao) = ----------------------- = 0.0212 (m- K) / W.
100 * 2 * 0.075

1 1
Rcond = ------------- ln (r2 / r1) = ------------------ ln (0.075 / 0.025) = 0.0175 (m-K) / W.
2 L k (2* * 1 * 10)

[25 ( 10)]
Qamb = (T2 - T) / Rco = ------------------- = 1651 W / m.
0.0212

(T2 T1) [25 5]


Qtube = ------------------ = --------------------- = 727.3 W / m
Rcont + Rcond (0.01 + 0.0175)

Power required = Qtotal = Qamb + Qtube = 1650 + 727.3 = 2378.3 W/m

3.3. One Dimensional Steady State Conduction With Heat Generation:The governing
equation for one dimensional steady state conduction in solids which are generating is
given as follows.

(i) Plane wall : (d2T / dx2 ) + q / k = 0 (3.41)

(ii) Radial conduction in cylinder: (1/r) d / dr {r dT/dr} + q / k = 0.(3.42)

(iii) Radial conduction in spheres: (1/r2 ) d / dr {r2 dT/dr} + q / k = 0.(3.43)

The following examples illustrate the method of analysis of steady state heat conduction
In solids generating heat.

Example 3.10. A plane wall of thickness L and thermal conductivity k has one of its
surfaces insulated and the other surface is kept at a uniform temperature T1. Heat is
generated in the wall at a rate q(x) where q(x) = q0 cos{(x) / (2L)} W / m3
where q0 is a constant.
(a) Develop an expression for one-dimensional steady state temperature distribution in the
solid and (b) develop an expression for the temperature of the insulated surface.

Sketch for example 3.10:

82
L
Insulated T1

T = T(x)
q= q0 cos (x/2L)

Known: (i) Expression for heat generation q = q0 cos W/m3;(ii) the surface at x = 0


is insulated i.e. (dT/dx)/x=0 = 0; (iii) T = T1 at x = L

Find: (a) expression for temperature distribution in the solid T(x); (ii) T/x = 0

Assumptions: (i) One dimensional steady state conduction ; (ii) solid is of constant thermal
conductivity

Solution: The governing equation to determine T(x) is given by

d2T / dx2 + q / k = 0

Substituting the given expression for q the above equation reduces to

d2T / dx2 + (q0 / k) cos (x / 2L) = 0 (1)

The boundary conditions are ; (i) at x = 0 (dT/dx) = 0 (Insulated)

(ii) at x = L, T = T1

Integrating Eq.(1) once w.r.t. x we get

dT/dx = -(/2L) (q0/k) sin (x/2L) + C1 (2)

Integrating once again w.r.t. x we get

T(x) = -(/2L)2 (q0/k) cos (x/2L) + C1x + C2 .(3)

Condition (i) in Eq. (2) gives 0 = 0 + C1 or C1 = 0.

Condition (ii) in Eq. (3) gives T1 = 0 + 0 + C2 or C2 = T1.

Substituting the values of C1 and C2 in Eq. (3) we get the temperature distribution as

83
T(x) = -(/2L)2 (q0/k) cos (x/2L) + T1 ..(4).

At the insulated surface (x = 0) the temperature therefore is given by

T(x) |x=0 = (/2L)2 (q0/k) + T1.

Example 3.11. A long cylindrical rod of radius 5 cm and k = 10 W/(m-K) contain


radioactive material which generates heat uniformly within the cylinder at a rate of
3x10 5 W/ m3. The rod is cooled by convection from its cylindrical surface by ambient air
at 50 0C with a heat transfer coefficient of 60 W/(m2-K). Assuming one-dimensional
radial conduction determine the temperature at the centre of the rod as well as at the
outer surface of the rod.(175 0 C)

Sketch for example 3.11:

T = T(r)
q
h, T

Known: Outer radius of cylinder = R = 5 cm; k = 10 W/m-K; q = 3*10 5 W/m3;

T = 500C; h = 60 W/(m2 K)

Find: (i) T/r = 0 ; (ii) T/r = R

Assumptions: (i) one dimensional radial conduction; (ii) steady state conduction; (iii)
thermal conductivity of the rosd is constant

Solution: The governing differential equation to determine one-dimensional steady state


radial conduction with heat generation is given by

(1/r) d / dr (r dT/dr) + q / k = 0..(1)

The boundary conditions are : (i) at r = 0, dT/dr = 0 (axis of symmetry)

(ii) at r= R, k (dT/dr)|r=R = h [T |r= T]

Eq. (1) can be written as


d / dr (r dT/dr) + q r / k = 0.

84
Integrating once w.r.t. r we get
r dT/dr + (q r2) / 2k = C1

or dT/dr + (q r) / 2k = C1 / r (2)

Integrating once again w.r.t. r we have q r2


T(r) = -------------- + C1 ln r + C2 (3)
4k
Condition (i) in Eq. (2) gives 0 + 0 = C1 / 0 or C1 = 0.

From Eq. (3) we have q R2


T(r)|r=R = -------------- + C2 ..(3)
4k

and from Eq. (2) we have (dT/dr) |r=R = (q R) / 2k

Therefore condition (ii) gives

k [ (q /R) / 2k] = h [ (q R2 /4k) + C2 T]

q R q R2
Or C2 = ----------- + -------------- + T
2h 4k

Substituting the expressions for C1 and C2 in Eq.(3) we get the temperature distribution in the
cylinder as
q R2
T(r) = T + ------------- [ 1 (r/R)2 ] + (qR) / 2h (4)
4k

3 * 10 5 * (0.05)2
Now q R2 / (4 k) = ----------------------- = 18.75 0 C.
4 * 10
3 x 10 5 x 0.05
q R / (2h) = ------------------------ = 125 0 C
2 x 60

Therefore T(r) = 50 + 18.75 [ 1 (r/R)2] + 125

= 175 + 18.75 [ 1 (r/R)2]

At the centre T(r) |r=0 = 175 + 18.75 = 193.75 0 C.

At the surface T(r) |r=R = 5 0+ 18.75 [ 1 1] + 125 = 175 0 C.

85
Example 3.12. In a cylindrical fuel element for a gas-cooled nuclear reactor, the heat
generation rate within the fuel element due to fission can be approximated by the
equation
q = qo [1 (r/R)2] W/m3,
where R is the outer radius of the fuel element and qo is a constant. The outer surface of the
cylinder is maintained at a uniform temperature To. Assuming one-dimensional
radialconduction obtain an expression for the temperature distribution in the element. If R =
2 cm, k = 10 W/(m-K) and qo = 1.16 x 10 5 W/m3, what would be the temperature difference
between centre temperature and the outer surface temperature.

Sketch for example 3.12:

T0

q = q0 [ 1 (r/R)2]

Known: R = 2 cm; k = 10 W/(m-K); qo = 1.16 * 10 5 W/m3.

Find: (T/r=0 T/r=R)

Assumptions: (i) one dimensional steady state conduction; (ii) Thermal conductivity of the
cylinder is constant

Solution:The governing equation to determine the one-dimensional steady-state radial


temperature distribution in a cylinder with heat generation is given by

(1/r) d/dr(r dT/dr) + q/k = 0.

Multiplying by r and substituting for q the given expression we have

q0 r [ 1 (r/R)2 ]
d/dr(r dT/dr) + ---------------------- = 0 ..(1)
k

86
Boundary conditions are: (i) at r = 0, dT / dr = 0 (axis of symmetry)

(ii) at r = R, T = T0

Integrating Eq. (1) w.r.t. r once we get

q0 r2 r4
(r dT/dr) + ---- [----- -----------] = C1
K 2 4R2

Or q0 r r3 C1
dT/dr + ---- [----- -----------] = ----------- ..(2)
K 2 4R2 r
Integrating once again w.r.t. r we have

q0 r2 r4
T(r) = ---------- [---------- --------- ] + C1 ln r + C2 .....................(3)
k 4 16R2
Condition(i) in Eq.(2) gives

0 + 0 = C1 / 0 or C1 = 0.

Condition (ii) in Eq.(3) gives

q0 R2 R4
T0 = ------ [ -------- ---------- ] + C2
k 4 16 R2

3 q0 R2
Or C2 = T0 + ----------------
16 k
Substituting the expressions for C1 and C2 in Eq. (3) we have

q0 r2 r4 3 q0 R2
T(r) = ---------- [---------- ---------] + T0 + ---------------
k 4 16R2 16 k

Or q0 r2 r4 3 q0 R 2
T(r) T0 = ---------- [---------- ---------] + ---------------
k 4 16R2 16 k

3 q0 R2 3 x 1.16 x 105 x (0.02)2


Therefore T(r) |r=0 T0 = ------------------ = ------------------------------- = 0.87 0 C.
16 k 16 x 10

87
Example3.13. Develop an expression for one-dimensional radial steady state
temperature distribution in a solid sphere of radius R in which heat is generated at a
rate given by

q = qo [ 1 (r/R)] W/m3.
Assume that the outer surface is maintained at a uniform temperature To.

Sketch for example 3.13:


Known:Outer surface temperature = T0;
Solid is generating heat according to the law
q = q0(1 r/R)

T0 Find: One dimensional radial temperature


q = q0(1 r/R) distribution T(r)

Assumptions: (i) steady state one


dimensional conduction; (ii) thermal
conductivity of the cylinder is constant

Solution: The governing differential equation to find the one-dimensional radial steady state
temperature distribution in a sphere with heat generation is given by

(1/r2) d/dr(r2 dT/dr) + q k = 0.

Multiplying throughout by r2 and substituting the given expression for q we have

d/dr (r2 dT/dr) + [q0 r2{1 (r/R)}]/k = 0.(1).

Boundary conditions are : (i) at r = 0, dT/dr = 0 (axis of symmetry)

(ii) at r = R, T = T0.

Integrating Eq. (1) w.r.t. r once we get

q0 r3 r4
(r2 dT/dr) + ----- [ ------ ---------- ] = C1
k 3 4R

q0 r r2
Or dT/dr = ----- [ ------ ---------- ] + C1 / r2 .(2)
k 3 4R
Integrating once again w.r.t. r we get

88
q0 r2 r3
Or T(r) = ----- [ ------ ---------- ] C1 / r + C2 .(3)
k 6 12R

Condition (i) in Eq. (2) gives 0 = 0 + C1 / 0 or C1 = 0.

Condition (ii) in Eq. (3) gives


q0 R2 R2
T0 = ------ [ ------ ---------- ] + C2
k 6 12

or C2 = T0 + q0 R2 / 12k

Substituting the expressions for C1 and C2 in Eq. (3) we have

q0 r2 r3 q0 R 2
T(r) = ---- [ ------ ---------- ] + T0 + -----------
k 6 12R 12 k

q0 R2
or T(r) = T0 + ---------- [ 1 2 (r/R)2 + (r/R)3 ]
12k

Example 3.14.A plane wall of thickness 2L is generating heat according to the law

q = q0 [1 (T Tw)]

where qo, , and Tw are constants and T is the temperature at any section x from the mid-
plane of the wall. The two outer surfaces of the wall are maintained at a uniform temperature
Tw. Determine the one-dimensional steady state temperature distribution, T(x) for the wall.

Sketch for example 3.14:


q = q0 [1 (T Tw)]

Tw
Tw

2L

89
Known: The two outer surfaces of the wall maintained at Tw; Solid is generating heat at a
rate given by q = q0 [1 (T Tw)].

Find: One dimensional steady state temperature T(x)

Assumptions: (i) One dimensional steady state conduction; (ii) Solid is of constant thermal
conductivity

Solution: Governing differential equation for one-dimensional steady state conduction in a


plane wall with constant thermal conductivity and which is generating heat is given by

d2T / dx2 + q / k = 0.

Substituting for q we have

d2T / dx2 + qo [ 1 (T Tw)] /k = 0

Defining a new variable = T Tw, the above equation can be written as

d2 / dx2 + qo [ 1 ] /k = 0

or d2 / dx2 qo /k = qo / k

or d2 / dx2 m 2 = qo / k ..(1a)

where m2 = q0 /k ...(1b)

Eq.(1a) is a second order linear non-homogeneous differential equation whose solution is


given by

(x) = h(x) + p(x) .(2)

where h(x) satisfies the differential equation

d2h / dx2 m 2h = 0 .(3)

Solution to Eq.(3) is given by

h(x) = A1 e mx + A2 e mx.(4)

p(x) satisfies the differential equation

d2p / dx2 m 2 p = qo / k..(5)

The term qo/k makes the governing differential equation non-homogeneous. Since this is a
constant p(x) is also assumed to be constant. Thus let p(x) = C, where C is a constant.

90
Substituting this solution in Eq. (5) we get

m 2 C = qo / k

Or C = qo /(km2)

Substituting for m2we get C = 1 / .

Hence p(x) = 1 / .(6)

The complete solution (x) is therefore given by

(x) = A1 e mx + A2 e mx + 1 / ..(7)

The constants A1 and A2 in Eq.(7) can be determined by using the two boundary conditions,
which are:

(i)at x = 0, dT / dx = 0 (axis of symmetry) i.e., d / dx = 0

(ii) at x = L, T = Tw ; i.e., = 0

From Eq.(7), d / dx = m[A1e mx A2 e mx ]

Substituting condition (i) we get m[A1 A2] = 0

Or A1 = A2.

Substituting condition (ii) in Eq.(7) we get A1[e mL + e mL] = 1 /

(1 / )
Or A1 = ----------------------------
[e mL + e mL]

Substituting the expressions for A1 and A2in Eq. (7) we get the temperature distribution in the
plane wall as
(1 / )
(x) = T(x) Tw = -------------------- [ e mx + e mx ] 1 /
[e mL + e mL]

1 e mx + e mx
Or T(x) Tw = ---- [ ----------------- 1]
e mL + e mL

[e m(L x) + e m(L x)] cosh m(L x)


or T(x) Tw = (1 / )---------------------------- = (1 / )----------------------------
[e mL + e mL] cosh mL

91
3.4. Critical Radius of Insulation:-For a plane wall adding more insulation will result in a
decrease in heat transfer as the area of heat flow remains constant .But adding insulation to a
cylindrical pipe or a conducting wire or a spherical shell will result in an increase in thermal
resistance for conduction at the same will result in a decrease in the convection resistance of
the outer surface because of increase in surface area for convection. Therefore the heat
transfer may either increase or decrease depending on the relative magnitude of these two
resistances.

3.4.1.Critical Radius of Insulation for Cylinder:- Let us consider a cylindrical pipe of outer
radius rs maintained at a constant temperature of Ts. Let the pipe now be insulated with a
material of thermal conductivity k and outer radius r. Let the outer surface of the insulation
be in contact with a fluid at a uniform temperature T with a surface heat transfer coefficient
h. Then the thermal circuit for this arrangement will be as shown in Fig.3.15.

r Surface in contact with a fluid at T and


surface heat transfer coefficient h
Ts
rs
T
Ts
Rins Rco Q

Fig.3.15: Schematic of a cylindrical pipe covered with an insulation and


exposed to an ambient and the corresponding thermal circuit

The rate of heat transfer from the pipe to the ambient is given by

(Ts - T) (Ts To)


Q = ------------------ = ----------------------------------- ...(3.44)
[Rins + Rco] ln (r / rs) 1
---------- + --------------
2Lk 2rLh

It can be seen from Eq. (3.44) that if Ts and h are assumed not to vary with r then Q
depends only on r and the nature of variation of Q with r will be as shown in Fig.3.16.
The value of r at which Q reaches a maximum can be determined as follows.

92
(Ts To)
Eq. (3.44) can be written as Q = -----------
F(r)

ln (r / rs) 1
where F(r) = ---------- + --------------
2Lk 2rLh

Hence for Q to be maximum, F(r) has to be minimum: i.e., dF(r) / dr = 0

Qmax

Qbare

r
rs rcr = k / h

Fig.3.16: Variation of Q with outer radius of insulation

Now dF / dr = (1 / 2Lk)(1/r) (1/2Lh)(1/r2) = 0

Or r = k / h.

This value of r is called critical radius of insulation, rcr.

Therefore rcr = k /h ...(3.45)

The above expression for critical radius of insulation has been arrived with the assumption
that the change in outer radius of insulation r will have no effect on the inside surface
temperature of insulation or on the outer surface heat transfer coefficient h. Sparrow
considered a heat transfer coefficient that varies as

93
h r mTs Tn (3.46a)

For this case, it is found that the heat transfer is maximized when



r = rcr = .(3.46b)



The quantity is a correction factor ( 1) accounting for the r and T dependencies of
h.As an example, consider forced convection flow across a cylinder. For Reynolds number
based on diameter between 4000 and 40,000, m = 0.382 and n=0. Correspondingly

Eq.(3.46b) gives thecorrection factor

= 0.618
As a second example for free convection about a horizontal cylinder for which m=n = 0.25,
the correction factor will be 0.6.
It can be seen from Fig.(3.16) that if the outer radius of the bare tube or bare wire is
greater than the critical radius then, any addition of insulation on the tube surface
decreases the heat loss to the ambient. But if the outer radius of the tube is less than the
critical radius , the heat loss will increase continuously with the addition of insulation
until the outer radius of insulation equals the critical radius. The heat loss becomes
maximum at the critical radius and begins to decrease with addition of insulation beyond
the critical radius.
The value of critical radius rcr will be the largest when k is large and h is
small. The lowest value of h encountered in practice is about 5 W/(m2 K) for free
convection in a gaseous medium and the thermal conductivity of common insulating
materials is about 0.05 W/(m K). Hence the largest value of rcr that we may likely to
encounter is given by
0.05
rcr = --------- = 0.01 m = 1 cm
5

The critical radius would be much less in forced convection (it may be as low as 1mm)
because of large values of h associated with forced convection. Hence we can insulate hot
water or steam pipes freely without worrying about the possibility of increasing the heat
loss to the surroundings by insulating the pipes.
The radius of electric wires may be smaller than the critical radius. Therefore, the
plastic electrical insulation may enhance the heat transfer from electric wires, there by
keeping their steady operating temperatures at lower and safer levels.

3.4.2.Critical Radius Insulation for a Sphere:- The analysis described above for cylindrical
pipes can be repeated for a sphere and it can be shown that for a sphere the critical radius
of insulation is given by
2k
rcr = ------- ..(3.47)
h

94
Example 3.15. A conductor with 8 mm diameter carrying an electric current passes
through an ambient at 30 0 C with a convection coefficient of 120 W/(m2 K). The
temperature of the conductor is to be maintained at 130 0 C. Calculate the rate of heat
loss per metre length of the conductor when (a) the conductor is bare and (b) conductor
is covered with bakelite insulation [k = 1.2 W/(m-K)] with radius corresponding to the
critical radius of insulation.

Sketch for example 3.15:


Ts
Dc ,k
D
Ts

h,T h, T
(a) Conductor without Insulation. (b) Conductor with critical thickness of
Insulation
Known: D = 0.008 m; Ts = 130 0C ; for insulation k = 1.2 W/(m-K); h = 120 W/(m2-K);
T = 30 0C

Find: (a) Qbare ;(b) Qwith insulation

Assumptions: (i) one dimensional steady radial conduction (ii) the thermal conductivity of
insulation is constant (iii) by adding the insulation the outer surface temperature of the wire
will not change; (iv) the surface heat transfer coefficient for wire with and without insulation
remain same

Solution:(a) When the conductor is bare the rate of heat loss to the ambient is given by

Qbare = h D L (Ts - T) = 120 * * 0.008 * 1 * (130 30) = 301.6 W/m.

(b) When the conductor is covered with critical thickness of insulation,

Dc = 2 rc = 2 (k/h) = 2 x ( 1.2 / 120) = 0.02 m.

1 1
R insulation = -------------- ln (Dc / D) = ----------------------- ln (0.02 / 0.008)
2 L k 2 * 1.0 * 1.2

= 0.1215 (m 0 C) / W.
1 1
Rco = 1 / (h Ac) = ---------------- = ------------------------ = 0.133 (m 0 C)/W.
Dc L h * 0.02 * 1 8 120

95
R = Rinsulation + Rco = 0.1215 + 0.133 = 0.2545 (m 0 C) / W.

(Ts - T) (130 30)


Qwithinsulation= ------------------- = --------------- = 392.93 W / m.
R 0.145

Example 3.16. An electrical current of 700 A flows through a stainless steel cable
having a diameter of 5 mm and an electrical resistance of 6x10 4 ohms per metre
length of the cable. The cable is in an environment at a uniform temperature of 30 0 C
and the surface heat transfer coefficient of 25 W/(m2 0C).
(a) What is the surface temperature of the cable when it is bare?
(b)What thickness of insulation of k = 0.5 W/(m K) will yield the lowest value of the
maximum insulation temperature? What is this temperature when this thickness is used?

Sketch for example 3.16:See sketch for example 3.15

Known: Current flow through the cable = I = 700 A; diameter of cable = D = 5 mm;
Electrical resistance of cable / metre length = = 6* 104 /m; h= 25 W/(m2-K);
T = 300C; for insulation k =0.5 W/(m-K)

Find: (a) Ts when the wire is bare; (ii) thickness of insulation for lowest value of maximum
insulation temperature; (iii) lowest value of maximum insulation temperature

Assumptions: i) one dimensional steady radial conduction (ii) the thermal conductivity of
insulation is constant (iii) by adding the insulation the outer surface temperature of the wire
will not change; (iv) the surface heat transfer coefficient for wire with and without insulation
remain same

Solution:(a) When the cable is Bare:Electrical Resistance = Re = 6 x 10 4 / m

Current through the cable = I = 700 A; D = 0.005 m ; h = 25 W/(m2-K) ; T = 30 0 C.

Power dissipated = Q = I2 Re = (700)2 * 6 * 10 4 = 294 W / m.

But Q = hA(Ts - T) or Ts = T + Q / [(D L) x h]

Or Ts = 30 + 294 / [( * 0.005 * 1) * 25] = 779 0 C.

(b) When the cable is covered with insulation: Lowest value of maximum insulatiuon
temperature occurs when the outer radius of insulation is equal to critical radius.

k = 0.5 W/(m-K) ; Hence critical radius = rc = k / h = 0.5 / 25 = 0.02 (m-K) / W.

Thickness of insulation = rc D/2 = 0.02 0.005 / 2 = 0.0175 m

1 1

96
Rinsulation = --------------- ln (rc / ro) = ------------------- ln (0.02 / 0.0025) = 0.662 (m-K)/W
(2 L k) (2 * 1 * 0.5)

1 1
Rco = 1 / (hAo) = ------------------- = ------------------------------ = 0.318 (m-K) / W.
(2 rcL h) (2 * * 0.02 * 1 * 25)

(Ts - T)
Q = --------------------- or Ts = T + Q (Rinsulation + Rco) = 30 + 294 x (0.662 +0.318)
Rinsulation + Rco

Or Ts = 318.12 0 C.

Example 3.17. A 2 mm-diameter and 10 m-long electric wire is tightly wrapped


With a 1 mm-thick plastic cover whose thermal conductivity is 0.15 W / (m-K).
Electrical measurements indicate that a current of 10 A passes through the wire and
there is a voltage drop of 8 V along the wire. If the insulated wire is exposed to a
medium at 30 0C with a heat transfer coefficient of 24 W / (m2 K), determine the
temperature at the interface of the wire and theplastic cover in steady operation.
Also determine if doubling the thickness of the plastic cover will increase or decrease
this interface temperature.

Sketch for example 3.17:

r Surface in contact with a fluid at T and


surface heat transfer coefficient h
Ts
rs
T
Ts
Rins Rco Q

Known:Outer radius of the bare wire = rs = 1 mm = 0.001 m ; Length of the wire = L = 10


m ; outer radius of plastic insulation = r = 1 + 1 = 2 mm = 0.002 m ;
Current through the wire = I = 10 A ; Voltage drop in the wire = V = 8 V ; Ambient
temperature = T = 30 0C ; Thermal conductivity of the plastic cover = k = 0.15 W /(m K) ;
Surface heat transfer coefficient = h = 24 W /(m2 K).

Find: (i) Interface temperature = Ts ; (ii) Whether Ts increases or decreases when the
thickness of insulation is doubled.

97
Assumptions: i) one dimensional steady radial conduction (ii) the thermal conductivity of
insulation is constant (iii) by adding the insulation the outer surface temperature of the wire
will not change; (iv) the surface heat transfer coefficient for wire with and without insulation
remain same

Solution:(i) Q = VI = 8 x 10 = 80 W.

ln (r / rs) ln ( 0.002 / 0.001)


Rins = ------------ = ----------------------------- = 0.0735 K / W
2Lk 2 * * 10 * 0.15
1 1 1
Rco = -------------- = -------------- = ------------------------------ = 0.3316 K / W
h Ao 2Lrh 2 * * 10 * 0.002 * 24

Hence Rtotal = Rins + Rco = 0.0735 + 0.3316 = 0.405 K / W.

Now Q = (Ts - T) / Rtotal.

Hence Ts = T + Q Rtotal = 30 + 80 x 0.405 = 62.4 0C

(ii) Critical radius of insulation = rcr = k / h = 0.15 / 24 = 0.00625.

Since rcr> r, increasing the thickness of plastic insulation will increase the heat transfer rate if
Ts is held constant or for a given heat transfer rate the interface temperature Ts will decrease
till the critical radius is reached. Now when the thickness is doubled then
r = 3 mm = 0.003 m . Therefore

ln ( 0.003 / 0.001)
Rins = ----------------------------- = 0.1166 K / W
2 * * 10 * 0.15

1
Rco = ------------------------------ = 0.221 K / W
2 x x 10 x 0.003 x 24

Therefore Rtotal = 0.1166 + 0.221 = 0.3376 K / W.

and Ts = 30 + 80 * 0.3376 = 57 0C

3.5. Extended Surfaces (Fins):

3.5.1 .Introduction: Convection: Heat transfer between a solid surface and a moving fluid is
governed by the Newtons cooling law: Q = hA(Ts-T), where Ts is the surface temperature
and Tis the fluid temperature. Therefore, to increase the convective heat transfer, one can

98
Increase the temperature difference (Ts-T) between the surface and the fluid.
Increase the convection coefficient h. This can be accomplished by increasing the fluid flow
over the surface since h is a function of the flow velocity and the higher the velocity, the
higher the h. Example: a cooling fan.
Increase the contact surface area A. Example: a heat sink with fins.
Many times, when the first option is not in our control and the second option (i.e. increasing
h) is already stretched to its limit, we are left with the only alternative of increasing the
effective surface area by using fins or extended surfaces. Fins are protrusions from the base
surface into the cooling fluid, so that the extra surface of the protrusions is also in contact
with the fluid. Most of you have encountered cooling fins on air-cooled engines
(motorcycles, portable generators, etc.), electronic equipment (CPUs), automobile radiators,
air conditioning equipment (condensers) and elsewhere.

3.5.2. Extended surface analysis:


In this section consideration will be limited to steady state analysis of rectangular or pin fins
of constant cross sectional area. Annular fins or fins involving a tapered cross section may be
analyzed by similar methods, but will involve solution of more complicated equations.
Numerical methods of integration or computer programs can be used to advantage in such
cases.
3.5.2.1.Fin equation:
The steady state Conduction Equation in three dimensionsfor solids of constant thermal
conducvity is given by Eq.(2.16) :

(2T/ x2) + (2T/ y2) + (2T/z2) + q / k = 0.(2.16)

After making the assumption of One-Dimensional Conduction, this equation reduces to the
form:

d2T/dx2 + q/k = 0 . (2.4)

This is a second order, ordinary differential equation and will require 2 boundary conditions
to evaluate the two constants of integration that will arise.
Consider the cooling fin shown in Fig. 4.1 :
Heat and Mass Transfer Prof. P
The fin is situated on the surface of a hot surface at Ts and surrounded by a coolant at
temperature T, which cools with convective coefficient, h. The fin has a uniform cross
sectional area, Ac, (This is the area through with heat is conducted.) and an overall length, L.
Note that as energy is conducted down the length of the fin, some portion is lost, by
convection, from the sides. Thus the heat flow varies along the length of the fin.We further
note that the arrows indicating the direction of heat flow point in both the x and y directions.
This is an indication that this is truly a two- or three-dimensional heat flow, depending on the
geometry of the fin. However, quite often, it is convenient to analyse a fin by examining an
equivalent onedimensional system. The equivalent system

99
y

x Fin attached to the primary surface


and exposed to surroundings at T
with a heat transfer coefficient h

Qfin

Primary surface at T0

Qc = hPx (T - T)

Fig.3.1. Heat dissipation from the fin to surroundings

will involve the introduction of heat sinks (negative heat sources), which remove an amount
of energy equivalent to what would be lost through the sides by convection.

Consider a differential length of the fin. Across this segment the heat loss will be h(Px)(T-
T), where P is the perimeter around the fin. The equivalent heat sink would be qAc x
Equating the heat source to the convective loss we get

qAc x = hPx (T - T)

Or q = hP (T - T)/Ac (3.48a)
Heat and Mass Transfer Pro
Indian Institute of Science Bangalore
Substituting this expression for q into Eq.(2.4.) we get

d2T/dx2hP (T - T)/kAc = 0 .(3.48b)

which is the equation for a fin with a constant cross sectional area. This is the Second Order
Differential Equation that we will solve for each fin analysis. Prior to solving, a couple of
simplifications should be noted. First, we see that h, P, k and Ac are all independent of x inthe

100
defined system (They may not be constant if a more general analysis is desired.). We replace
this ratio with a constant.

Let m2 = (hp)/ (kAc)(3.49)

Then Eq. (3.48) reduces to


2
d2T/dx2 m (T - T) = 0 .(3.50)

3.5.2.2.Alternative method of obtaining equation Eq.(3.50): Eq. (3.50) can also be obtained
from basic principles by writing the energy balance equation for an elemental length of the
fin as follows:
Qconv

Qx Qx+dx

Under steady state conditions, the energy balance equation for the fin element shown above
can be written as

Qx = Qx+dx + Qconv

= Qx + (dQx / dx) dx + hP dx (T - T)

Or (dQx / dx) + hP (T - T) = 0

Writing expression for Qx using Fouriers law we get

d/dx[ kAx dT/dx] + hP (T - T) = 0

For a fin of uniform cross sectional area Ax = Ac = constant. Also if the thermal conductivity
of the fin material is constant then the above equation reduces to

m2 (T - T) = 0 (3.50)

Where m2 = (hp)/ (kAc) (3.49)

3.5.2.3.Justification for treating the conduction in fins as one dimensional conduction


(Quasi One-dimensional Heat Transfer):

The quasi one dimensional heat transfer problems are those where heat is transferred in two
directions, but temperature changes are important only in one direction. We can examine
when temperature variation across the fin can be neglected in comparison to the temperature
variation along the length of the fin.Consider the heat transfer in y direction for the fin of

101
length L, width W and uniform thickness t. At any x ,if the rate of heat transfer in x direction
is of the same order of magnitude as the rate in y direction we have

k Ax k Ay

Using first order linear approximation, the above equation can be written as




k t W k L W





Or

For fins t is very much smaller than L; therefore, for the heat transfers to be of the same order

than the temperature variation in x direction: i.e. .



of magnitude ,it follows that the temperature variation in y direction must be much smaller

Solution to the fin equation: We notice that equation (3.50) is non-homogeneous (due to the
Tterm). Recall that non-homogeneous differential equations require both a homogeneous
and a particular solution. We can make this equation homogeneous by introducing the fin
temperature relative to the surroundings:

Let T - T .(3.51)

Then d2/dx2 = d2T/dx2 (3.52)

Substituting for T - T and d2T/dx2in Eq. (3.50) we get


2
d2/dx2 m = 0 .(3.53)

Eq. (3.53) is called the fin equation

Solution for Equation (3.53):Eq. (3.53) is a second order linear homogeneous differential
equation the general solution for which is given by

= Aemx + Bemx (3.54)

The solution for Eq. (3.53) is also given by

= C cosh mx + D sinh mx .(3.55)

Generally the exponential solution is used for very long fins, the hyperbolic solutions for
other cases.
Boundary Conditions:

102
Since the solution results in 2 constants of integration we require 2 boundary conditions. The
first one is obvious, as one end of the fin will be attached to a hot surface and will come into
thermal equilibrium with that surface.
Hence, at the fin base (x = 0),

=T0 - T = 0 ..(3.56a) e of Science


The second boundary condition depends on the condition imposed at the other end of the fin.
There are various possibilities, as described below.
Very long fins:
For very long fins, the end located a long distance from the heat source will approach the
temperature of the surroundings. Hence,

() = 0 (3.56b)
Substitute the second condition into the exponential solution of the fin equation:

()= 0 = Ae m + Be m

The first exponential term is infinite and the second is equal to zero. The only way that this
equation can be valid is if A = 0. Now apply the second boundary condition.

(0) = 0 =B

The general temperature profile for a very long fin is then:

(x) = 0 e mx..(3.57)

If we wish to find the heat flow through the fin, we may apply Fourier Law:
At any x, the rate of heat conduction is given by

Q = kAc(dT/dx) = kAc(d/dx)

All the heat dissipated by the fin to the surroundings is coming from the fin base. Hence the
rate of heat transfer from the fin to the surroundings is given by

Qfin = Q|x=0

= kAc(d/dx)x=0 (3.58)

From Eq.(3.57) (d/dx)x=0 = m0

Substituting this expression in Eq. (3.58) we get


_____
Qfin = kmAc0= hpkAc 0(3.59)

103
The fin with insulated tip:

Assume that the tip is insulated and hence there is no heat transfer at the fin tip.
Hence the boundary condition at the tip is given by

At x = L, d/dx = 0.

Substituting this condition and the boundary condition at x = 0 and simplifying we get the
temperature profile for this case as



(x) = 0 . (3.60)

We may find the heat flow at any value of x by differentiating the temperature profile and
substituting it into the Fourier Law:

Qfin = Q/x=0 = kAc (d/dx)x=0.

From Eq. (3.60) , (d/dx)x=0 = 0 sinh mL / cosh mL

Therefore Qfin =kmActanh mL (3.61)

If we compare this result with that for the very long fin, we see that the primary difference in
form is in the hyperbolic tangent term. That term, which always results in a number equal to
or less than one, represents the reduced heat loss due to the shortening of the fin
Indian Institute of Science Bangalore
Other tip conditions:
We have already seen two tip conditions, one being the long fin and the other being the
insulated tip. Two other possibilities are usually considered for fin analysis: (i) a tip
subjected to convective heat transfer, and (ii) a tip with a prescribed temperature. The
expressions for temperature distribution and fin heat transfer for all the four cases are
summarized in the table below.

Other tip conditions:


We have already seen two tip conditions, one being the long fin and the other being the
insulated tip. Two other possibilities are usually considered for fin analysis: (i) a tip
subjected to convective heat transfer, and (ii) a tip with a prescribed temperature. The
expressions for temperature distribution and fin heat transfer for all the four cases are
summarized in the table below.

104
Table 3.1: Expressions for Temperature distribution and rate of heat dissipation from a
fin for different tip conditions

Tip Expression for


Condition Expression for Temperature Profile Rate of Heat Transfer

Long Fin (x) = 0 e mxQfin = kmAc0



Tip Insulated (x) = 0 Qfin = kmAc0tanh mL

Tip with prescribed






Temp,TL (x) = Qfin= kmAc0

Tip dissipating heat [cosh m(L x) [sinh mL +


by convection with + (he/km)sinh m(L-x)] (he/km) cosh mL]
surface heat transfer (x) = ----------------------------------- Qfin = kmAc0 ---------------------
Coefficient,he [cosh mL +(he/km)sinh mL][cosh mL +
(he/km) cosh mL]
3.5.2.4.Performance of a fin:

Fin Efficiency: The performance of a fin is assessed by using a parameter called efficiency
of a fin, given by

Actual rate of heat transfer from the fin Qfin


= ------------------------------------------------------ = ------------ (3.62)
Maximum Possible heat transfer rate from fin (Qfin)max

The rate of heat transfer from the fin will be maximum when the entire fin surface is at the
fin base temperature (This means that the fin is having an infinite thermal conductivity): i.e.,

(Qfin)max = hPL(To - T) = hPL0 ....(3.63)

Fin Effectiveness: Fins are used to enhance the heat transfer, and the use of fins on a surface
cannot be recommended unless the enhancement in heat transfer justifies the added cost and
complexity associated with the fins. In fact, there is no assurance that by adding fins on a
surface, the heat transfer rate will enhance.The performance of the fin is judged on the basis
of enhancement in heat transfer relative to the no-fin case.To know how effective an

105
extended surface as compared to a bare primary surface is, another parameter called
effectiveness is used. It is given by

Heat Transfer with fin Qfin Qfin


= --------------------------------- = -------------- = -------------- .(3.64)
Heat Transfer without fin Qbare hAb0

Here Ab is the cross sectional area of the fin at the base.


It should be noted that both the fin efficiency and the fin effectiveness are related
to the performance of the fin, but they are different quantities.However they are related to
each other as shown below.

From Eq. (3.64) = = =




(3.65)

Hence, the fin effectiveness can be determined easily when the efficiency of the fin is known,
or vice-versa.

Considerations in the design and selection of fins:The rate of heat transfer from a very
longfin of uniform cross sectionunder steady state conditions is given by
Eq.(3.59).Substituting this expression in Eq.(3.65) we get

[Qfin]long kmA0 k [(hp) / (kA)]


long = -------------- = ----------------- = ---------------------
hAb0 hAb0 hAb

Noting that Ab = A, the above expression for long reduces to

long =

(3.66)

Several important conclusions can be drawn from Eq.(3.66) which helps in design and
selection of fins for various applications.

(i) The therml conductivity k of the fin material should be as high as possible.Hence
fins are made of materials like copper.aluminium are iron. The most widely used
material is aluminium because of its low cost and weight and its resistance to
corrosion.
(ii) The ratio of the perimeter to the cross-sectional area of the fin P/A should be as
high as possible. This criterion is satisfied by employing thin plate fins and
slender pin fins.
(iii) The use of fins is most efficient in applications involving low convection heat
transfer coefficients. Thus the use of fins is easily justified when the medium is
gas instead of a liquid and the heat transfer mechanism is by free convection
(iv) rather than by forced convection.Therfore in a liquid to gas heat exchangers like
automobile radiators, fins are placed on the gas side of the heat exchangers.

106
Example 3.18. A steel rod of diameter 2 cm and thermal conductivity 50 W/(m K) is
exposed to ambient air at 20 0C with a heat transfer coefficient 64 W/(m2 K). One end of
the rod is maintained at a uniform temperature of 120 0C. Determine the rate of heat from
the rod to the ambient and the temperature of the tip of the rod exposed to ambient if (i) the
rod is very long, (ii) rod is of length 10 cm with negligible heat loss from its tip, (ii) rod is of
length 25 cm with heat loss from its tip.

Sketch for example 3.18: k h,T


x

Known: D = 2 cm.; k = 50 W/(m-K);T = 200C; h = 64 W/(m2-K);T0 = 1200C

Find: (i) Qfin if the rod is long; (ii)Qfin if L = 10 cm and negligible heat loss from the tip;
(iii)Qfin if L = 25 cm and with heat loss from the tip

Assumptions: (i) one dimensional steady state conduction along the length of the fin; (ii)
thermal conductivity of the rod is constant

Solution:(i) Long rod

m = [(hP) / (kA) = [(hD / (D2/4)] = [(4h) / (kD)]

m = = 16 m1

.
or

For a very long fin the rate of heat transfer is given by

Qfin = kmA(T0 - T) = 50 * 16 * ( / 4) * 0.022 * [ 120 20] = 25.13 W

(ii) Rod of finite length with negligible heat loss from the tip:

L = 0.10 m. Hence mL = 16 * 0.1 = 1.6

For rod of finite length with negligible heat loss from the tip we have

Q fin= kmA(T0 - T) tanh mL = 50 * 16 * ( / 4) * 0.022 * (120 20) * tanh 1.6

= 23.16 W

(iii)Rod of finite length with heat loss from the tip:

107
When the heat loss from the rod tip is not negligible with he = h , then we can use the same
formula as in case (ii) with modified length Le given by

Le = L + A /P = L + (D2/4) /(D) = L + D / 4 = 0.1 + 0.02/4 = 0.105

Hence mLe = 16 * 0.105 = 1.68 and tanh mLe = tanh 1.68 = 0.9329

Hence Q fin = 25.13 x 0.933 = 23.44 W

Example 3.19. A thin rod of uniform cross section A, length L and thermal conductivity
k is thermally attached from its ends to two walls which are maintained at temperatures
T1 and T2. The rod is dissipating heat from its lateral surface to an ambient at
temperature T with a surface heat transfer coefficient h.
(a) Obtain an expression for the temperature distribution along the length of the rod
(b) Also obtain an expression for the heat dissipation from the rod to the ambient

Sketch for example 3.19:

T1 k h,T
x T2

Known: Temperatures at the two ends of the rod, rod cross sectional area A,thermal
conductivity of the rod k, ambient temperature T and surface heat transfer coefficient h.

Find: (a) Temperature distribution T(x) along the length of the rod; (b)Rate of heat
dissipation from the rod to ambient

Assumptions: (i) one dimensional steady state conduction along the length of the fin; (ii)
thermal conductivity of the rod is constant

Solution:The general solution for the one-dimensional steady-state temperature distribution


along the length of a rod dissipating heat by convection from its lateral surface is given by

(x) = C1 cosh mx + C2 sinh mx ..(1)


___________
where (x) = T(x) - T ; m = (hP) / (kA) :

108
P = perimeter of the rod = D and A = Area of cross section of the rod = D2 / 4.

The boundary conditions are: (i) at x = 0, T = T1 or = T1 - T = o (say).

(ii) at x = L, T = T2 or = T2 - T = L (say).

Condition(i) in Eq. (1) gives o = C1.

Condition (ii) in Eq. (1) gives L = 0 cosh mL + C2 sinh mL

(L o cosh mL)
C2 = ------------------------ .
sinh mL

Substituting for C1 and C2 in Eq. (1) we have

(L o cosh mL)
(x) = o cosh mx + ------------------------ sinh mx
sinh mL

o cosh mx sinh mL + L sinh mx o cosh mL sinh mx


0r (x) = ------------------------------------------------------------------------
sinh mL

L sinh mx + o sinh m(L x)


0r (x) = -------------------------------------- ..(2)
sinh mL
Expression for the rate of heat dissipation from the rod:

Qamb
x

Q|x=0
Q |x=L

Energy balance for the rod is given by

Qamb = Q |x = 0 Q |x = L = kA (d / dx)|x = 0 + kA (d / dx)|x = L(3)

109


From Eq. (2) we have (d / dx) =



Therefore (d / dx)|x = 0 =



and (d / dx)|x = L =



Hence Qamb =

Example 3.20. Heat is generated at a constant rate of q W/m3 in a thin circular rod of
length L and diameter D by the passage of electric current. The two ends of the rod are
maintained at uniform temperatures with one end at temperature T0 and the other end
at 0 0 C, while heat is being dissipated from the lateral surface of the rod to an ambient
at 0 0C with a surface heat transfer coefficient h.
(a) Derive the one-dimensional steady state energy equation to determine the
temperature distribution along the length of the rod
(b) Solve the above equation and obtain the temperature distribution.

Sketch for example 3.20:


00 C h,T
q W/m3

T = T0

x
L
Qamb

Qx Qx+dx

dx Qg
x

110
Known: Thin rod of diameter D and length L is generating heat at a constant rate of q
W/m3;One end of the rod is maintained at 00C and the other end at T0; Lateral surface of the
rod is exposed to ambient at T with a surface heat transfer coefficient h.

Find: (a) One dimensional steady state energy equation


(b) Temperature distribution T(x) along the length of the rod

Assumptions: (i) one dimensional steady state conduction along the length of the rod; (ii)
thermal conductivity of the rod is constant

Solution: Since the rod is generating heat and dissipating heat to the ambient, the governing
differential equation to determine the one-dimensional steady state temperature distribution
has to be obtained from first principles as illustrated below.

Consider an elemental length dxof the rod as shown in the figure above. The various
energies crossing the boundaries of the rod as well as the energy generated are also shown in
the figure. For steady state condition the energy balance equation for the rod element can be
written as

Qx + Qg = Qx+dx + Qamb

Or Qx + Qg = Qx + (dQx/dx) dx + Qamb

Or (dQx/dx) dx + Qamb = Qg

d/dx( kA dT/dx) dx + hPdx (T - T) = Adx q

Or (d2T / dx2) (hP / kA) (T - T) = (q / k)

Let T - T = and (hP / kA) = m2. then the above equation reduces to

(d2 /d x2) m2 = (q / k) .(1)

Eq.(1) is a non-homogeneous linear second order ordinary differential equation whose


solution can be written as

(x) = h(x) + p(x) ------------------------------------(2)

where h(x) satisfies the homogeneous part of the differential equation namely

(d2 h /d x2) m2 h = 0 -------------------------------------(3)

and p(x) is the particular integral which satisfies Eq. (1). Solution to Eq.(3) is given by

(x) = C1 e mx + C2 e mx -----------------------------(4)

111
To find p(x) :Since the RHS of Eq.(1) is a constant let us assume p(x) = B, where B is a
constant. Substituting this solution in Eq.(1) we have

0 m2 B = (q / k)

Or B = (q / km2)

Therefore the complete solution for Eq. (1) can be written as

(x) = C1 e mx + C2 e mx + (q / km2)

Or T(x) = T + C1 e mx + C2 e mx + (q / km2)..(5)

Boundary conditions are: (i) at x = 0, T = 0

(ii) at x = L, T = T0

Condition (i) in Eq. (5) gives

0 = T + C1 + C2 + (q/ km2)

Or C1 + C2 = T (q/km2) ------------------------------- (a)

Condition(ii) in Eq.(5) gives T0 = T + C1 e mL + C2 e mL + (q/km2)

Or C1 e mL + C2 e mL = T0 T (q/km2) ----------------------------- (b)

From Eq.(a) C2 = C1 T (q/km2). Substituting this expression in Eq.(b) we have

C1 e mL [C1 + T + (q/km2)] e mL = T0 T (q/km2)

T0 {T + (q/km2)}{1 e mL }
Solving for C1 we get C1 = --------------------------------------------------
{ e mL e mL }

T0 {T + (q/km2)}{1 e mL }
C2 = { T + (q/km2) } --------------------------------------------------
{ e mL e mL }

{ T + (q/km2) }{ e mL e mL } T0 + {T + (q/km2)}{1 e mL }
C2 = ------------------------------------------------------------------------------------------------
{ e mL e mL }

112
{ T + (q/km2) }[ e mL + e mL + 1 e mL] T0
C2 = -----------------------------------------------------------------------------------------------
{ e mL e mL }

{ T + (q/km2) }[ 1 e mL] T0
C2 = ----------------------------------------------
{ e mL e mL }
Substituting the expressions for C1 and C2 in Eq. (5) and simplifying we get

[T0 {T + (q/km2)}{1 e mL }] e mx
T(x) = T + (q/km2) + -------------------------------------------------
{ e mL e mL }

[ { T + (q/km2) }[ 1 e mL] T0] e mx


+ ----------------------------------------------------
{ e mL e mL }

T0(e mx - e mx)
2
T(x) = T + (q /km ) + --------------------- +
{ e mL e mL }

[ {T + (q/km2)}{ 1 e mL }] e mx + { T + (q/km2) }[ 1 e mL] e mx]


----------------------------------------------------------------------------------------------
{ e mL e mL }

Example 3.21. Two very long slender rods of the same diameter are given. One rod is
of aluminum (k = 200 W/(m-K)). The thermal conductivity of the other
rod is not known. To determine this, one end of each rod is thermally attached to a metal
surface maintained at a uniform temperature T0. Both rods are losing heat to the ambient air
at T by convection with a surface heat transfer coefficient h. The surface temperature of
each rod is measured at various distances from hot base surface. The temperature of the
aluminum rod at 40 cm from the base is same as that of the rod of unknown thermal
conductivity at 20 cm from the base. Determine the unknown thermal conductivity.

Sketch for example 3.21:

Ta
xa
ka

Tb = Ta
xb
kb = ?

113
Known: Two slender rods A and B are of same diameter;Base temperature T0 is same for
both the rods;Conductivity of rod A = ka = 200 W/(m-K);Ambient temperature T same for
both the rods. Surface heat transfer coefficient h same for both the rods; Ta = Tb with xa =
0.4 m and xb = 0.2m

Find: Thermal conductivity of rod B , kb

Assumptions: (i) one dimensional steady state conduction along the length of the rods; (ii)
thermal conductivity of the two rods is constant (iii)rods are assumed to be of infinite length

Solution:For very long slender rods the steady-state one-dimensional temperature


distribution along the length of the rod is given by

(x) = 0 e mx ..(1)

where (x) = T(x) - T and 0 = T0- T.

For rod A Eq.(1) can be written as a(x) = 0 e ma xa ...(2)

And for rod B it can be written as b(x) = 0 e mb xb (3)

It is given that when xa = 0.4 m and xb = 0.2 m, a(xa) = b(xb)

Therefore we have 0 e 0.4 ma = 0 e 0.2 mb

Or mb = 2 ma

Or [(hPb) / (kbAb)] = 2[ (hPa) / (kaAa)]

Since Pa = Pb and Aa = Ab, we have ka = 2 kb or ka = 4 kb

Therefore kb = 200/4 = 50 W/(m-K).

Example 3.22. Show that for a finned surface the total heat transfer rate is given by

Qtotal = [ + (1 )] a h 0 = a h 0

Where = fin efficiency ; = af / a : af = surface area of the fin, a = total heat transfer area
(i.e. finned surface + unfinned surface) ; 0 = T0 - T, with T0 = fin base temperature and T
= ambient temperature, and = area weighted fin efficiency.

Solution:
Qtotal = Qfin + Qbare

114
Where Qtotal = Total heat transfer rate, Qfin = Heat transfer rate from the finned surface

and Qbare = Heat transfer rate from the bare surface.

Therefore Qtotal = h af 0 + h(a af) 0

= h a 0 [( af) / a + (1 af/a)]
here, =af/a
= ha 0 [+ (1 )]

= ha 0 , where = [+ (1 )]

Example 3.23. The handle of a ladle used for pouring molten lead at 327 0 C is 30 cm long
and is made of 2.5 cm x 1.5 cm mild steel bar stock (k = 43 W/(m-K)). In order to reduce the
grip temperature, it is proposed to make a hollow handle of mild steel plate 1.5 mm thick to
the same rectangular shape. If the surface heat transfer coefficient is 14.5 W/(m2-K) and the
ambient temperature is 27 0C, estimate the reduction in the temperature of the grip. Neglect
the heat transfer from the inner surface of the hollow shape.

Sketch for example 3.23 (a) : (a) When the handle is made of solid steel bar:

2.5 cm
Known: h = 14.5 W/(m2-K) ;

1.5 cm k = 43 W/(m-K)

0 = 327 27 = 300 0 C
Cross section of the handle

Area of cross section of the bar = A = 2.5 x 1.5 x 10 4 m2 = 3.75 x 10 4 m2

Perimeter of the bar = P = 2 [ 2.5 + 1.5] x 10 2 m = 8 x 10 2 m

( hP)(1/2) [14.5 * 8 * 10 2]
Therefore m = ------ = -------------------------- = 8.48 (1/m)
(kA)(1/2) [43 * 3.75 * 10 4]

Therefore mL = 8.48 * 0.3 = 2.54.

When the heat loss from the tip of the handle is neglected the temperature at any point along
the length of the handle is given by

cosh m(L x)
(x) = 0----------------------
cosh mL

115
Therefore (x)|x=L = 0 / cosh mL = 300 / cosh 2.54 = 47 0 C.

Or T(x)|x=L = 47 + 27 = 74 0 C.

(b) When the handle is hollow made out of a sheet:


Area of the cross section of the fin is
2.5 cm
A = [(2.5 * 1.5) (2.5 0.3) * (1.5 0.3)]

= 1.11 cm 2 = 1.11 * 10 4 m2
1.5 cm
P = 2 * [ 2.5 + 1.5 ] = 8 cm = 8 * 10 2 m

_________ (14.5 * 8 * 10 2)
1.5 mm thick
m = (hP) / (kA) = ----------------------------
( 43 * 1.11 * 10 4)

Or m = 15.59 1/m. Therefore mL = 15.59 * 0.3 = 4.68

0 (327 27)
(x)|x=L = -------------------- = ----------------- = 5.57 0 C.
cosh mL cosh 4.68

Therefore T(x)|x=L = 5.57 + 27 = 32.57 0 C.

Reduction in grip temperature = 74 32.57 = 41.43 0 C.

Example 3.24. Derive an expression for the overall heat transfer coefficient across a plane
wall of thickness b and thermal conductivity k having rectangular fins on both sides.
Given that over an overall area A of the wall, the bare area on both sides, not covered by the
fins are Au1 and Au2, the fin efficiencies are 1 and 2, and the heat transfer coefficients h1
and h2.

Solution:

Let Ti be the temperature of the fluid in contact with the surface 1, T0 be the temperature of
the fluid in contact with surface 2, T1 be the temperature of surface 1 and T2 be the
temperature of surface 2.Let Ti>T0. Then the rate of heat transfer from Ti to T0 is given by

Q = Qbare + Q fin
= hiAu1 (Ti T1) + hi 1Af1(Ti T1)

116
(Ti T1) (Ti T1)
Or Q = --------------- -------------------
(1 /h1Au1) (1/h11Af1)

(Ti T1)
Q = ------------------ ---------------- (1)
[(1 /h1Au1) + (1/h11Af1) ]

(T2 T0)
Q = ------------------ ---------------- (2)
[(1 /h2Au2) + (1/h22Af2) ]

Rate of heat transfer is also given by


(T1 T2)
Q = ---------------- (3)
(b/Ak)

Therefore as A/B = C/D = E/F = (A+C+E)/(B+D+F) 

(Ti T1) + (T1 T2) + (T2 T0)


Q = --------------------------------------------------------------------------
[(1 /h1Au1) + (1/h11Af1) + (1 /h2Au2) + (1/h22Af2) +(b/Ak)]

(Ti T0)
Q = -------------------------------------------------------------------------- (4)
[(1 /h1Au1) + (1/h11Af1) + (1 /h2Au2) + (1/h22Af2) +(b/Ak)]

If U = overall heat transfer coefficient for the plane wall then

Q = UA(Ti T0)

(Ti T0)
= ----------------- .(5)
(1/UA)
From Eqs. (4) and (5) we have

1
U = ------------------------------------------------------------------------------
A [(1 /h1Au1) + (1/h11Af1) + (1 /h2Au2) + (1/h22Af2) +(b/Ak)]

Example 3.24. Calculate the effectiveness of the composite pin fin shown in Fig.P3.22.
Assume k1 = 15 W/(m-K), k2 = 50 W/(m-K) and h = 12 W/(m2 K).

117
Solution:

k1 3 mm = d1
d2 = 10 mm

x k2

Qc K1=15 W/m-k , K2=50W/m-k,


h=12 W/(m2 K)

Qx Qx+dx
(b) Energy transfer across the
surfaces of the fin element
dx

Energy balance equation for the fin element is given by

Qx = Qx+dx + Qc= Qx + (dQx/dx) dx + Qc

Or dQx / dx + Qc = 0 ..(1)

Qx consists of two components namely the heat transfer Qx1 through the material of thermal
conductivity k1 and the rate of heat transfer Qx2 through the material of conductivity k2.

Therefore Qx = Qx1 + Qx2 = k1A1 (dT / dx) k2A2 (dT / dx)

= (k1A1 + k2A2) (dT / dx)

and Qc = (hP2 dx) (T - T).

Substituting these expressions for Qx and Qc in equation (1) we get

hP2
(d2T / dx2) ------------------- (T T) = 0
(k1A1 + k2A2)

Or (d2 / dx2) m2 = 0 (2)

Where = T T and m = [ hP2 / (k1A1 + k2A2) ].

118
When the heat loss from the fin tip is negligible , the solution to equation (2) is given by

cosh [m(L x)]


(x) = 0 ---------------------- ..(3)
cosh mL
The rate of heat transfer from the fin base is given by

Qx|x=0 = (k1A1 + k2A2) (d / dx)|x=0

(k1A1 + k2A2) sinh [m(L x)]x=0 (- m)0


= ----------------------------------------------------
cosh mL

= m0 (k1A1 + k2A2) tanh mL

Now = Qx|x=0 / Qmax

m0 (k1A1 + k2A2) tanh mL


= ----------------------------------------
hP2L 0

Noting that hP2 / (k1A1 + k2A2) = m2, the above expression for simplifiers to

tanh mL
= ------------------------ (4)
mL

In the given problem A1 = ( / 4) * (0.003)2 = 7.1 * 10 6 m2.

A2 = ( / 4) * [ (0.01)2 (0.003)2] = 7.15 * 10 5

P2 = x 0.01 0.0314 m.

[ 12 x 0.0314]
m = ----------------------------------------------------- = 10.12
[(15 x 7.1 x 10 6 ) + (50 x 7.15 x 10 5)]

Therefore mL = 10.12 * 0.1 = 1.012

tanh (1.012)
= ------------------ = 0.757
1.012

119
Examples on Conduction in Solids with Variable Thermal Conductivity:

Example 3.25. A plane wall 4 cm thick has one of its surfaces in contact with a
fluid at 130 0C with a surface heat transfer coefficient of 250 W/(m2 K) and the
other surface is in contact with another fluid at 30 0C with a surface heat transfer
coefficient of 500 W/(m2-K). The thermal conductivity of the wall varies with
temperature according to the law
k = 20 [ 1 + 0.001 T]
where T is the temperature. Determine the rate of heat transfer through the wall and
the surface temperatures of the wall.

Given:- L = 0.04 m; Ti = 130 0C; hi = 250 W/(m2-k); To = 30 0C; ho = 500

W/(m2-K); k = 20 [ 1 + 0.001 T].

To find:- (i) Qx (ii) T1 and T2

Solution:
Rci = Thermal resistance for convection at the surface at Ti = 1/(hiA) = 1 / (250 x 1)
= 0.004 m2 K /W

Rco = Thermal resistance for convection at the surface at To = 1/(hoA) = 1/(500 x 1)

Or Rco = 0.002 m2-K/W

Now Q = (Ti T1) / Rci, where T1 = Surface temperature in contact with fluid at Ti.

Hence T1 = Ti QRci = 130 0.004 Q .(1)

Similarly Q = (T2 To) / Rco

Or T2 = To + QRco = 30 + 0.002Q (2)

From equations (1) and (2) we have

T1 T2 = 100 0.006Q .(3)

And Tm = ( T1 + T2) / 2 = 80 0.001Q ....(4)

Hence km = ko [ 1 + Tm] = 20 * [1 + 0.001* {80 0.001Q}] = 21.6 2 * 10 5Q

Hence thermal resistance offered by the wall = R = L/(Akm)

0.04
Or R = ---------------------------
[21.6 2 * 10 5Q]

120
[T1 T2] [100 0.006Q] [21.6 2 x 10 5Q]
Q = --------------------- = ---------------------------* --------------------------
R 0.04

Cross multiplying we have

0.04Q = 2160 0.1316Q + 1.2 * 10 7 Q2

Or Q2 1.41 * 10 6 Q + 1.8 * 10 10 = 0.Hence Q = (1.41 * 10 6 1.39 * 10 6) / 2

For physically meaningful solution T1 should lie between Ti and To. This is possible only
If
Q = (1.41 * 10 6 1.39 * 10 6) / 2 = 10000 W.

Now T1 = Ti QRci = 130 10000 * 0.004 = 90 0C

and T2 = T0 + Q Rco = 30 + 10000 * 0.002 = 50 0C.

Example 3.26. The thermal conductivity of a plane wall varies with temperature
according to the equation
k(T) = k0 [ 1 + T2 ]
where k0 and are constants.
(a) Develop an expression for the heat transfer through the wall per unit area of the wall
if the two surfaces are maintained at temperatures T1 and T2 and the thickness of the
wall is L.
(b) Develop a relation for the thermal resistance of the wall if the heat transfer area is A.

Solution:

K = k0 [ 1 + T2]

T1
T2

L
L

121
For steady state conduction we have

Qx = kA(dT / dx) = constant.

Or Qx = k0[1 +T2]A(dT/dx)

Qxdx = k0[1 +T2]A dT

Integrating the above equation between x = 0 and x = L we have


L T2
Qxdx = k0A [1 +T2]dT
0 T1

Or QxL = k0A [(T2 T1) + (/3)(T23 T13)]

Or Qx = (k0A / L)(T1 T2) [1 + (/3)(T12 +T1T2 + T22)]

(T1 T2)
Qx = -------------------------------------------------------
1
---------------------------------------------------
(k0A/L)[1 + (/3)(T12 +T1T2 + T22)]

Therefore thermal resistance of the wall is given by


1
R = ---------------------------------------------------
(k0A/L)[1 + (/3)(T12 +T1T2 + T22)]

Example 3.27. Find the steady-state heat flux through the composite slab made up of two
materials as shown in Fig. P 3.26. Also find the interfacial temperature. The
thermalconductivities of the two materials vary linearly with the temperature in the following
manner:
k1 = 0.05 [1 + 0.008 T] W/m-K and k2 = 0.04 [1 + 0.0075 T] W/m-K

122
600 0 C 30 0 C
k1 k2

5 cm 10 cm

Fig. P 3.26: Schematic for problem 3.26

T3
Data:- T1 = 600 0 C ; T2 = 30 0 C ;
T2
L1 = 0.05 m ; L2 = 0.10 m ;
T1
k1 k2
k1 = 0.05 [ 1 + 0.008T ] ;

k2 = 0.04 [ 1 + 0.0075T ]

To find T3 and q

L1 L2

Mean thermal conductivity for the first layer is given by

km1 = 0.05 [ 1 + 0.008(T1 + T3) /2] = 0.05[1 + 0.004(600 +T3)]

= 0.17 + 2 * 10 4T3W/(m-K)

Similarly km2 = 0.04 [ 1 + 0.0075(T3 + T2) /2] = 0.04[1 + 0.00375(T3 + 30)]

= 0.0445 + 1.5 * 10 4 T3 W/(m-K)

Rate of heat transfer through the first layer is given by

123
km1A(T1 T3) [0.17 + 2 * 10 4 T3] (600 T3)
Q = ----------------------- = ---------------------------------------------
L1 0.05

Similarly for the second layer we have

km2A(T3 T2) [0.0445 + 1.5 * 10 4 T3] (T3 30)


Q = -------------------- = ----------------------------------------------
L2 0.10

Equating the two expressions for Q we have

[0.17+ 2 * 10 4 T3] (600 T3) [0.0445 + 1.5 * 10 4 T3] (T3 30)


--------------------------------------------- = ---------------------------------------------------
0.05 0.10

Cross multiplying and simplifying we get

T32 + 254.55 T3 373337 = 0


______________________
254.55 [ (254.55)2 4 * 373337]
Therefore T3 = --------------------------------------------------
2

Since T3 has to lie between T1 and T2 it follows that


________________________
254.55 + [ (254.55)2 4 * 373337]
T3 = ----------------------------------------------------
2

= 496.9 0 C.

Therefore km1 = 0.17 + 2 * 10 4 * 496.9 = 0.269 W / (m-K)

Heat flux = q = Q / A = km1(T1 T3) / L1

0.269 * [600 496.8]


= ----------------------------- = 555.22 W / m2.
0.05

Example 3.28. Consider a slab of thickness L in which heat is generated at a uniform rate of
q W/m3. The two boundary surfaces are maintained at temperatures T1 and T2. The thermal
conductivity of the slab varies with temperature according to the equation

124
k(T) = k0 [ 1 + T ]

where k0 and are constants. Develop an expression for the heat flux q(x) in the slab.
Measurements show that steady-state conduction through a plane wall without heat
generation produced a convex temperature distribution such that the mid-point temperature
was T0 higher than expected for a linear temperature distribution. Assuming that the
thermal conductivity has a linear dependence on temperature [k = k0(1 + T)], where k0 and
are constants,develop a relationship to evaluate in terms of T0, T1 and T2.

Solution:
L

k = k0 [ 1 + T ]

T0 k = k0

T1
T2

For constant thermal conductivity k0, the temperature distribution in the wall is linear and is
given by

T(x) = T1 (T1 T2) x / L

Therefore T(x)|x=L/2 = (T1 + T2) / 2 .(1)

When the thermal conductivity varies with temperature the temperature distribution in the
wall is determined as follows.

Qx = kA(dT/dx) = k0(1 + T)A (dT/dx)

Therefore Qx dx = k0A(1 + T) dT ..(2)

125
Integrating the above equation between x = 0 and any x at which the temperature is given by
T*(x) we have
x T*
Qxdx = k0A(1 + T) dT
0 T1

Or Qx x = k0A [(T* - T1) + (/2)(T*2 T12)] (3)

If equation is integrated between x = 0 and x = L we get

Qx L = k0A [(T2 - T1) + (/2)(T22 T12)]

Or Qx = (k0A)[1 + (T1 + T2) / 2] (T1 T2) / L

Or Qx = kmA (T1 T2) / L .(4)

Where km = k0 [1 + (T1 + T2) / 2].

Substituting this expression for Qx in equation (3) we get

km (T1 T2) (x / L) = k0[(T* - T1) + (/2)(T*2 T12)]

The above equation simplifies to

T*2 + (2/)T* + [ (2/){(km/k0)(T1 T2)(x/L) T1} T12 ] = 0.

Therefore
___________________________________________
T* = (1/) (1/2) (2km / k0)(x / L)(T1 T2) + T12 + (2 / )T1
_____________________________________
T*|x=L/2 = (1/) (1/2) (km / k0)(T1 T2) + T12 + (2 / )T1
__________________________________________________
= (1/) (1/2) [k0{1 + (T1 + T2)/2}/ k0](T1 T2) + T12 + (2 / )T1
_______________________________________________
= (1/) (1/2) [{1 + (T1 + T2)/2}/ ](T1 T2) + T12 + (2 / )T1

But T*|x=L/2 = T(x)|x=L/2 + T0 = (T1 + T2) / 2 + T0.

Hence we have
______________________________________________
(T1 + T2)/2 +T0 = (1/) (1/2) [{1 + (T1 + T2)/2}/ ](T1 T2) + T12 + (2 / )T1

Example 3.29. A slab of thickness L has its two surfaces at x=0 and x = L maintained at
uniform temperatures of T0 and TL respectively. The thermal conductivity of the slab has

126
spatial variation according to the law k = k0 [1 + x], where ko and are constants. Obtain
expressions for (i) temperature distribution in the slab, and (ii) rate of heat transfer through
the slab assuming one dimensional steady state conduction.

Solution:

k = k0 [1 + x]

T1 T2

Fig. E3.30 : Figure for Example 3.30

The governing differential equation for one dimensional steady state conduction without heat
generation is given by

k = 0

substituting for k we have

k 1 + x = 0..(1)

Let 1 + x = y ; Then dx = dy or dy/dx = . Then Eq.(1) can be written in


terms of the variable y as follows.

= 0

= 0

Or

127
Integrating w.r.t y once we get

= C1

dT =


Or

Integrating once we have,

T= ln y + C2

Substituting for y in terms of x we have

T= ln (1 + x) + C2 ..(2)

Eq.(2) is the general solution of Eq.(1). The values of C1 and C2 can be obtained from the
two boundary conditions at x = 0 and at x = L as follows.

(i) at x = 0, T = T1; and (ii) at x = L, T = T2 ;

Condition (i) in Eq. (2) gives T1 = C2;

Condition (ii) in Eq. (2) gives T2 = ln (1 + L) + T1





Or C1 = (T2 T1) .

Substituing the values of C1and C2 in Eq.(2) we get the temperature distribution as:



T(x) = (T2 T1) + T1

Expression for rate of heat transfer:

At any x Qx = - k A (dT/dx)

Or Qx = [k0 (1 + x)] A (dT/dx)

= [k0 (1 + x)] A d/dx{(T2 T1) [ ln (1 + x) / ln (1 + L)] + T1}

A (T2 T1) 1

128
= [k0 (1 + x)] ------------------- x -----------------
ln(1 + L)(1 + x)

k0 A (T1 T2)
Or Qx = -----------------------------
ln(1 + L)

Example 3.30.If in the above problem the thermal conductivity varies with distance
according to the law
K = k0 [1 + x2]

Obtain expressions for (i) the temperature distribution T(x) and (ii) the rate of heat transfer.

Solution:
L

k = k0 [1 + x 2]

T1 T2

Fig. E3.31 : Figure for Example 3.31

The governing differential equation for one dimensional steady state conduction without heat
generation is given by

d / dx [k dT/dx] = 0 substituting for k we have

d / dx [k0 (1 + x2 ) dT/dx] = 0 ..(1)

Eq. (1) can be solved by using the following substitution.

Let x = tan y. Differnatiating both sides w.r.t.x we have

= sec 2 y (dy/dx)

or (dy/dx) = / sec 2 y = ------------------- .

129
[ 1 + x2 ]

Or [ 1 + x2 ] (dy / dx) = .

Eq. (1) can be written in terms of variable y as

d / dy[k0 (1 + x2) (dy/dx) (dT/dy)] (dy/dx) = 0

Substituting for (1 + x2) (dy/dx) we get

d/ dy [k0 (dT/dy)] [ / sec 2 y] = 0.

Ord/ dy [k0 (dT/dy)] = 0.

Integrating w.r.t.y we have, k0 (dT / dy) = C1

Or dT = [C1 /( k0 )] dy.

Integrating once again we get T = [C1/ (k0 )] y + C2

Or T(x) = [C1/(k0 )] tan 1(x) + C2 .(3)

Boundary conditions are: (i) at x = 0 T (x) = T1 and (ii) at x = L ,T = T2.

Condition (i) in Eq. (3) gives C2 = T1.

Condition (ii) in Eq. (3) gives T2 = [C1/(k0 )] tan 1( L) + T1

[T2 T1]k0
Or C1 = ------------------
tan 1( L)

Substituting the values of C1 and C2 we have


tan 1 ( x)
T(x) = [T2 T1] -----------------
tan 1 ( L)

Expression for Rate of Heat Transfer:

Qx = k0 [1 + x2] A (dT/dx)


2
= k0 [1 + x ] A [T2 T1] ---------------------------
[1 + x2] tan 1 ( L)

130
k0 A [T1 T2]
Or Qx = -------------------------------
tan 1 ( L)

Example 3.31. A hollow cylinder has its internal surface at radius r1 maintained at a uniform
temperature T1 and external surface at radius r2 maintained at a uniform temperature T2. The
thermal conductivity of the material of the cylinder varies with radius according to the law k
= k0 [1 + r], where k0 and are constants. Derive expressions for (i) radial temperature
distribution in the cylinder and (ii) rate of heat transfer through the cylinder. Assume one-
dimensional radial steady state conduction in the cylinder.

Example 3.32.A hollow cylinder has its internal surface at radius r1 maintained at a uniform
temperature T1 and external surface at radius r2 maintained at a uniform temperature T2. The
thermal conductivity of the material of the cylinder varies with radius according to the law k
= k0 [1 + r2], where k0 and are constants. Derive expressions for (i) radial temperature
distribution in the cylinder and (ii) rate of heat transfer through the cylinder. Assume one-
dimensional radial steady state conduction in the cylinder.

Example 3.33.A hollow sphere has its internal surface at radius r1 maintained at a uniform
temperature T1 and external surface at radius r2 maintained at a uniform temperature T2. The
thermal conductivity of the material of the cylinder varies with radius according to the law k
= k0 [1 + r], where k0 and are constants. Derive expressions for (i) radial temperature
distribution in the cylinder and (ii) rate of heat transfer through the cylinder. Assume one-
dimensional radial steady state conduction in the cylinder.

Example 3.34.A hollow sphere has its internal surface at radius r1 maintained at a uniform
temperature T1 and external surface at radius r2 maintained at a uniform temperature T2. The
thermal conductivity of the material of the cylinder varies with radius according to the law k
= k0 [1 + r2], where k0 and are constants. Derive expressions for (i) radial temperature
distribution in the cylinder and (ii) rate of heat transfer through the cylinder. Assume one-
dimensional radial steady state conduction in the cylinder.

131
Chapter 4

Transient Conduction
4.1.Introduction:-In general, the temperature of a body varies with time as well as
position.In chapter 3 we have discussed conduction in solids under steady state conditions for
which the temperature at any location in the body do not vary with time. But there are many
practical situations where in the surface temperature of the body is suddenly altered or the
surface may be subjected to a prescribed heat flux all of a sudden. Under such circumstances
the temperature at any location within the body varies with time until steady state conditions
are reached. In this chapter, we take into account the variation of temperature with time as
well as with position.However there are many practical applications where in the temperature
variation with respect to the location in the body at any instant of time is negligible. The
analysis of such heat transfer problems is called the lumped system analysis. Therefore in
lumped system analysis we assume that the temperature of the body is a function of time
only.

4.2. Lumped system analysis:-Consider a solid of volume V, surface area A, density ,


Specific heat Cp and thermal conductivity k be initially at a uniform temperature Ti.Suddenly
let the body be immersed in a fluid which is maintained at a uniform temperature T, which
is different from Ti.The problem is illustrated in Fig.4.1.Now if

Surface in contact with fluid at


T with surface heat transfer
Coefficient h

V = volume
A=surface area
= density
Cp = specific heat
k = conductivity

Fig.4.1: Nomenclature for lumped system analysis of transient


Conduction heat transfer

132
T(t) is the temperature of the solid at any time t, then the energy balance equation for the
solid at time t can be written as

Rate of increase of energy of the solid = Rate of heat transfer from the fluid to the solid

VCp = hA[T - T(t)]




i.e.,

= [T T(t)]


Or



For convenience, a new temperature (t) = T(t) - T is defined and denoting m =
the above equation can be written as

= m

..(4.1)

Eq.(4.1) is a first order linear differential equation and can be solved by separating the
variables. Thus

= m dt

Integrating we get ln = mt + ln C, where ln C is a constant.

Or = C e mt ...(4.2)

At time t = 0 , T(t) = Ti or = Ti T = i (say). Substituting this condition in Eq. (4.2) we


get
C = i.
Substituting this value of C in eq. (4.2) we get the temperature (t) as follows.

(t) = i e mt



or = e mt (4.3)

Since LHS of Eq.(4.3) is dimensionless, it follows that 1/m has the dimension of time and is
called the time constant.Fig. 4.2 shows the plot of Eq.(4.3) for different values of m. Two
observations can be made from this figure and Eq. (4.3).

1.Eq. (4.3) can be used to determine the temperature T(t) of the solid at any time t or to
determine the time required by the solid to reach a specified temperature.

133
2.The plot shows that as the value of m increases the solid approaches the surroundings
temperature in a shorter time.That is any increase in m will cause the solid to
respond more quickly to approach the surroundings temperature.

t
1.0

t
Fig.4.2: Dimensionless temperature as a function of time for a
solid with negligible internal temperature gradients

The definition of m reveals that increasing the surface area for a given volume and the heat
transfer coefficient will increase m. Increasing the density, specific heat or volume decreases
m.

4.2.2.Criteria for Lumped System Analysis:-To establish a criterion to neglect internal


temperature gradient of the solid so that lumped system analysis becomes applicable, a
Characteristic length Lsis defined as

Ls = V /A (4.4)


and the Biot. number Bi as


Bi = ..(4.5)

For solids like slabs, infinite cylinder, and sphere, it has been found that the error by
neglecting internal temperature gradients is less than 5 %, if

Bi < 0.1 (4.6)


The physical significance of Biot number can be understood better by writing the expression
for Biot number as follows

h Ls (Ls / Ak) Thermal resistance for conduction

134
Bi = ------ = -------------- = -------------------------------------------
k ( 1 / hA) Thermal resistance for convection

Hence a very low value of Biot number indicates that resistance for heat transfer by
conduction within the solid is much less than that for heat transfer by convection and
therefore a small temperature gradient within the body could be neglected.

Illustrative examples on lumped system analysis

Example 4.1.A copper cylinder 10 cm diameter and 15 cm long is removed from a liquid
nitrogen bath at 196 0 C and exposed to room temperature at 30 0 C. Neglecting internal
temperature gradients find the time taken by the cylinder to attain a temperature of 0 0C,
with the following assumptions: Surface heat transfer coefficient = 30 W / m2 K.
Density of the copper cylinder = 8800 kg / m3.Specific heat of the cylinder = 0.38 kJ/(kg-K)
Thermal conductivity of the cylinder = 350 W / (m-K).

Sketch for example


4.1:

Ti Known:Ti = 196 0C; T = 30 0C

D = 10 cm or R = 0.05 m; L = 0.15 m
T D h = 30 W/(m2 K); k = 350 W / (m-K) ;

= 8800 kg / m3 ;cp = 0.38 kJ / (kg-K) ;

h T(t) = 0

Find: Time taken t for the solid to reach 00C.

Assumptions: (i)Internal temperature gradients are negligible; (ii) solid properties are
constant

Solution: Biot Number = hR / k = 30 x 0.05 / 350 = 0.0043 which is << 0.1. Hence internal

temperature gradients can be neglected. Let (t) = T(t) T


In that case we have
(t) = T(t) Ti = 0 e (hA/Vcp) t, where 0 = Ti T

2{R2 + RL)h 2{R+L}h 2 x {0.05 +0.15} x30


(hA/Vcp) = ------------------- = ----------- = ------------------------------------------
R2L cp cpRL 8800 x 0.38 x 1000 x 0.05x 0.15

= 4.785 x 10 4 1/s

135
T(t) T
Now ------------------ = e (hA/Vcp) t
Ti T

0 30
Hence ------------------- = exp ( 4.785 x 10 4 x t)
196 30

Solving for t we get t = 4226 s = 1 hr 10.43 mins.

Example 4.2:A thin copper wire having a diameter D and length L (insulated at
the ends) is initially at a uniform temperature of T0. Suddenly it is exposed to a gas
stream, the temperature of which changes with time according to the equation

T = Tf (1 e ct) + T0

where Tf, T0 and c are constants. The surface heat transfer coefficient is h. Obtain an
expression for the temperature of the wire as a function of time t.

Sketch for example 4.2:


h,T

T(0) = T0

L
Known: Rod of diameter D and length L, initially at T0, is suddenly exposed to en
environment whose temperature varies with time according to the law T = Tf (1 e ct) +
T0, where Tf, T0 and c are constant

Find: Temperature of rod as a function of time

Assumptions: (i) Internal temperature gradients for the rod is negligible; (ii) Thermal
properties of the rod are constant

Solution: Let T(t) be the temperature of the cylinder at any time t. Energy balance for the
cylinder for a time interval dt is given by

hA [T - T(t)] dt = VCp dT
where dT is the increase in temperature of the cylinder in time dt.

136


Or = (hA/VCp) [T - T(t)]

Putting m = (hA/VCp), the above equation reduces to

+ m T(t) = m T

Substituting the given expression for T we have

+ m T(t) = m [T0 + Tf (1 e - ct)]



or + m [T(t) T0] = mTf (1 e ct )

Let (t) = T(t) T0. Then the above equation reduces to

+ m (t) = mTf (1 e ct ) (1).



This equation is of the form + Py = Q, which is solved by multiplying throughout by an
integrating factor and then integrating. For equation (1) the integrating factor is

e mdt = e mt. Therefore multiplying equation (1) by e mt we get

d
[ e mt dt + m e mt (t)] = mTf [ e mt e (m c)t]



or (emt) = mTf [ e mt e (m c)t]

Integrating with respect to t we have

emt(t) = mTf [ (emt / m ) e(m c)t / (m c) ] + C1



or (t) = Tf Tf e ct + C1e mt (2)

When t = 0 , T(0) = T0 i.e., (0) = 0. Substituting this condition in equation (2) we get



or 0 = Tf Tf + C 1

137
c
Or C1 = m c Tf.

Substituting this expression for C1 in equation (2) we get the temperature of the cylinder as


(t) = Tf Tf e ct + Tf e mt



Or T(t) T0 = Tf [ 1 e ct + e mt]

Where m = hA / (VCp) = DLh / {(D2/4)LCp}= (4h) /(DCp).

Example 4.3:A solid sphere of radius R is initially at a uniform temperature T0. At a certain
instant of time (t >0), the sphere is suddenly exposed to the surroundings at a temperature Tf
and the surface heat transfer coefficient, h. In addition from the same instant of time, heat is
generated within the sphere at a uniform rate of q units per unit volume. Neglecting internal
temperature gradients, derive an expression for the temperature of sphere as a function of
time

Sketch for example 4.3:

T(0) = T0

qfor t > 0
R

h,Tf

Known: Solid sphere of radius R is initiallt at temperature T0; Suddenly at time t = o the
sphere is generating heat at q W/m3 and simultaneously the sphere is losing heat to the
ambient at Tf with a surface heat transfer coefficient h.

Find: Temperature of sphere as a function of time T(t)

Assumptions: (i) Internal temperature gradients for the rod is negligible; (ii) Thermal
properties of the rod are constant

138
Solution:Energy balance equation for the sphere at any time t can be written as

dT
(4/3)R3 q + 4R2h [Tf T(t)] = (4/3)R3 Cp
dt

dT
+ (3h/ RCp)[T(t) Tf] = (q /Cp)
dt
Or

Let (t) = T(t) Tf. Then the above equation reduces to

d
dt + m = q0 ..(1)

Where m = (3h/ RCp) and q0 = (q Cp)

Multiplying equation (1) by the integrating factor e mt we have

d
emt + emt m = q0 emt
dt



or (emt) = q0emt

Integrating throughout w.r.t. t we get

emt = (q0 / m) emt + C1

or = (q0/ m) + C1e mt ...(2)

At t = 0 , T = T0 or = T0 Tf = 0 (say). Substituting this condition in equation (2) we

get C1 = (T0 Tf) (q0 / m). Therefore the temperature in the sphere as a function of time is
given by

(t) = [(T0 Tf) (q0 / m)] e mt + (q0 / m)

or (t) = (q0 / m)[ 1 e mt ] + (T0 Tf) e mt



where (q0 / m) = * = (q R / 3h)

Example 4.4:A solid steel ball ( =8000 kg/m3 ; cp = 0.42 kJ/kg-K) 5 cm in diameter is at a
uniform temperature of 450 0 C. It is quenched in a controlled environment which is initially
at 90 0C and whose temperature increases linearly with time at the rate of 10 0C per minute.
If the surface heat transfer coefficient is 58 W/(m2-K), determine the variation of the
temperature of the ball with time neglecting internal temperature gradients. Find the value of
the minimum temperature to which the ball cools and the time taken to reach this minimum

139
temperature.

Sketch for example 4.4:

T(0) = Ti

h,Tf

Known: T(0) = Ti = 4500C ;h = 58 W / (m2 K) ;Cp = 0.42 kJ / (kg K) ; = 8000 kg / m3;

at t = 0, Tf = 90 0C; = 10 0C / min = (1/6) 0 C / s



Find: (i) Tminimum ; (ii) time taken to reach Tminimum

Assumptions: (i) Internal temperature gradients for the rod is negligible,i.e.Temperture of


the solid is a function of time only; (ii) Thermal properties of the ball are constant

Solution: Tf = a + bt, where a and b are constants;at t = 0, Tf = 90 0C;(dTf / dt) = (1/6) 0 C / s

Therefore a = 90 0 C and b = = (1/6) 0 C / s.



Or Tf = 90 + t / 6 , t in seconds .(1)

Energy balance equation for the sphere at any time t can be written as

VCp = hA [Tf(t) T(t)]



= [Tf(t) T(t)]


Or

Letting m = the above equation can be written as



+ mT(t) = m Tf(t)

Substituting for Tf(t) from equation (1) we have

140
+ mT(t) = m [90 + t / 6 ]

Multiplying the above equation with the integrating factor e mt we get

e mt + mT(t) emt = m [90 + t / 6 ] e mt





or (T e mt) = m [90 + t / 6 ] e mt

Integrating throughout w.r.t t we have

(T e mt) = m [90 + t / 6 ] e mt dt + C1

Or T(t) = m e mt [90 + t / 6 ] e mt dt + C1 e mt

= m e mt [(90e mt /m) + (temt/6m) (e mt/6m2)] + C1 e mt

Or T(t) = [ 90 + (t / 6 ) (1/6m)] + C1 e mt..(2)

When t = 0 , T(t) = Ti. Substituting this condition in the above equation and solving for C1
we get
C1 = [Ti 90 + 1 / 6m]

Therefore the temperature of sphere as a function of time is given by

T(t) = [ 90 + (t / 6 ) (1/6m)] + [Ti 90 + 1 / 6m] e mt .(3)

For T(t) to be extremum = 0.



Therefore we have = 1/6 + [Ti 90 + 1 / 6m] e mt ( m) = 0



Substituting Ti = 450 0 C and simplifying we get

(360 m + 1/6) e mt = 1/6


Or e mt = (2160 m + 1) -----------(4)

Now m = = =


..
= = 2.07 x 10 3. (5)

Using (4) & (5) in (3), we get

T(t)= 90+ (t/6) (1/(6*2.07*10-3)) + [ 240-90+(1/(6*2.07*10-3))] * exp{-2.07*10 3t}

141
T(t)=9.4857 + (230.5152)*(0.9979)t



Now from the above expression for T(t), > 0.

Hence value of t obtained from Eq.(4) will give Tminimum.

Therefore e mt = [ 2160 x 2.07 x 10 3 + 1 ] = 5.47

mt = 1.7
Or t = 1.7 / m = 1.7 / (2.07 x 10 3) = 821 s = 13.7 min

Substituting this value of t in equation (3) we get the minimum temperature as

Tminimum = [90 + (821/7) {1 / (6x 2.07 x 10 3) } ]

+ [ 450 90 + {1 / (6x 2.07 x 10 3) } ] e 1.7 = 226.7 0C.

Example 4.5: A house hold electric iron has a steel base [ =7840 kg/m3 ; cp = 450 J/(kg-K)
;k = 70 W/(m-K)] which weighs 1 kg. The base has an ironing surface area of 0.025 m2 and
is heated from the other surface with a 250 W heating element. Initially the iron is at a
uniform temperature of 20 0 C with a heat transfer coefficient of 50 W/(m2-K). (a) Determine
the temperature of the plate 300 seconds after the heater switch is on .(b) What would be the
equilibrium temperature of the iron if the control of the iron box did not switch of the
current?

Sketch for example 4.5:

Qc h ,T
A

Known: Q = 250 W;A = 0.025 m; T=20 0C;h = 50 W /(m2-K); = 7840 kg / m3 ;

Cp = 450 J / (kg K) ; k = 70 W /(m K) ; m = 1 kg ; t = 5 min = 300 s

Find: (a)T/t = 300 s. (b)T after steady state is reached

142
Assumptions: (i) Internal temperature gradients for the rod is negligible; (ii) Thermal
properties of the iron are constant

Solution: (a)V = m / = 1 / 7840 = 0.0001275 m3 = 1.275 x 10 4 m3.

1.275 x 10 4
L = V / A = ------------------ = 0.005 m
0.025

50 x 0.005
Bi = (hL / k) = -------------------- = 0.00364
70

Since Bi < 0.1, it can be assumed that temperature gradients within the plate are negligible.
Hence the temperature of the plate depends only on time till steady state condition is reached.
Energy balance at any time t for the plate can be written as

Q Qc = VCp

Q hA(T - T) = VCp


Or

+ m(T - T) = (Q / VCp) ..(1)



Or

Where m = (hA / VCp). Letting = T - T, equation (1) can be written as

+ m = (Q / VCp)

Multiplying the above equation by the integrating factor e mt,( emdt=emt) we get

e mt + m e mt = (Q / VCp) emt



Or (e mt) = (Q / VCp) emt

Or (e mt) = (Q / VCp) emt (1/m) + C1

Or = (Q / VCpm) + C1e mt .(2)

When t = 0, T = Ti or = Ti - T = 20 20 = 0 0 C.

Substituting this condition in equation (2) we get

0 = (Q / VCpm) + C1 or C1 = (Q / VCpm)

143
Therefore the temperature in the plate as a function of time is given by

= (Q / VCpm) [ 1 e mt ]

But VCpm = hA. Therefore

= (Q / hA) [ 1 e mt ] ..(3)

.
.
= = 200 ; m = = 2.8 * 10 3

Therefore = 200 [ 1 e 0.028t]

When t = 300 s, = T - T = 200 x [ 1 e 0.028 x 300] = 113.7

Or T = 113.7 + 20 = 133.7 0 C.

(b) When the control switch is not switched off and the iron is left in the ambient, steady
state condition will be attained as t tends to so that the heat transferred to the base plate
will be convected to the ambient. i.e., Q = Qconvection

Therefore 250 = 50 x 0.025 x [T 20 ]

Or T = 220 0 C.

This answer can also be obtained by putting t = in equation (3) and solving for T.

4.3 One-dimensional Transient Conduction ( Use of Heisslers Charts): There are many
situations where we cannot neglect internal temperature gradients in a solid while analyzing
transient conduction problems. Then we have to determine the temperature distribution
within the solid as a function of position and time and the analysis becomes more complex.
However the problem of one-dimensional transient conduction in solids without heat
generation can be solved readily using the method of separation of variables.The analysis is
illustrated for solids subjected to convective boundary conditions and the solutions were
presented in the form of transient temperature charts by Heissler. These charts are now
familiarly known as Heisslers charts.

4.3.1.One-dimensional transient conduction in a slab:- Let us consider a slab of thickness


2L, which is initially at a uniform temperature Ti. Suudenly let the solid be exposed to an
environment which is maintained at a uniform temperature of T with a surface heat transfer
coefficient of h for time t > 0.Fig.4.3 shows the geometry , the coordinates and the boundary
conditions for the problem. Because of symmetry in the problem with respect to the centre
of the slab the x coordinate is measured from the centre line of the slab as shown in the
figure.

144
2L
Surfaces
exposed to a T = Ti at t = 0
fluid at T
with heat
transfer T = T(x,t)
coefficient h
for time t 0

x
Fig.4.3: Geometry, coordinates and boundary conditions for one
dimensional transient conduction in a slab
conduction in a slab

The mathematical formulation of this transient conduction problem is given as follows:



Governing differential equation: = (1/) ..(4.7a)

Initial condition : at t = 0, T = Ti in 0 < x < L .(4.7b)

Boundary conditions are :

(i) at x = 0, T / x = 0 (axis of symmetry) for all t > 0..(4.7c)

(ii) at x = L, k |x = L = h(T|x = L T) for all t > 0.(4.7d)



It is more convenient to analyze the problem by using the variable (x,t), where

(x,t) = T(x,t) - T. Then equations (4.7a) to (4.7d) reduce to the following forms:



= (1/) ..(4.8a)

Initial condition : at t = 0, = Ti T = i in 0 < x < L .(4.8b)

Boundary conditions reduce to :

145


(i) at x = 0, = 0 for all t > 0 (axis of symmetry) ..(4.8c)

(ii) at x = L, k |x = L = h|x = L for all t > 0




...(4.8d)

Eq.(4.8a) can be solved by the method of separation of variables as shown below.

Let (x,t) = X(x) Y(t) ..(4.9)

Substituting this in Eq. (4.8a) we get

Y = (X/)

= .(4.10)

Or

LHS of Eq. (4.10) is a function of x only and the RHS of Eq. (4.10) is a function of t
only.They can be equal only to a constant say 2.(The reason to choose the negative sign is
to get a physically meaningful solution as explained later in this section).Hence we have two
equations namely

(1 / X) (d2X / dx2) = 2 and [1/(Y)] ((dY / dt) = 2

Or (d2X / dx2) + 2X = 0 .(4.11)

and (dY/dt) = 2 Y (4.12)

Solution to Eq. (4.11) is X(x) = C1 cos (x) + C2 sin (x) .(4.13)

and solution to Eq. (4.12) is Y(t) = D exp ( 2t) ..(4.14)

with C1, C2 and D are constants of integration. Substituting these solutions in Eq.(4.9) we
have
(x,t) = D exp ( 2t) [C1 cos (x) + C2 sin (x)]

or (x,t) = exp ( 2t) [A1 cos (x) + A2 sin (x)] ..(4.15)

Eq.(4.15) is the general solution involving the constants A1, A2 and which can be
determined using the two boundary conditions and the initial condition as illustrated below.



Now from Eq. (4.15), = exp ( 2t) [ A1 sin (x) + A2 cos (x)]

146
Substituting boundary condition (i) we have 0 = exp ( 2t) [0 + A2] for all t.

Hence A2 = 0. Therefore Eq. (4.15) reduce to

(x,t) = A1 exp ( 2t) cos (x) .(4.16)

Now (L,t) = A1 exp ( 2t) cos (L)



and = exp ( 2t) [ A1 sin (x) ]

Hence [ / x ] x = L = A1 exp ( 2t) sin (L)

Therefore boundary condition (ii) can be written as

k A1 exp ( 2t) sin (L) = h A1 exp ( 2t) cos (L)

or tan (L) = h / (k)

or L tan (L) = Bi (4.17)

where Bi = hL / k.
Equation (4.17) is called the characteristic equation and has infinite number of roots
namely 1, 2, 3, ..............Corresponding to each value of we have one solution and hence
there are infinite number of solutions. Sum of all these solutions will also be a solution as the
differential equation is linear. Therefore the solution (x,t) can be written as follows.

(x,t) = An exp ( n2t) cos (nx) .(4.18)

To find An:- The constants An in Eq. (4.18) can be found using the orthogonal property of
trigonometric functions as shown below.Substituting the initial condition we have

i = An cos (nx)

Multiplying both sides of Eq.(4.18) by cos mx and integrating w.r.t x between the limits 0
and L we have L L
i cos (mx) dx = An cos (mx) cos (nx) dx
1 1

Using the orthogonal; property


An cos (mx) cos (nx) dx = 0 for n m
The above equation reduce to
L L
i cos (nx) dx = An cos 2(nx) dx
0 0

L
i0 cos (nx) dx
Or An = ---- L-------------------

147
cos 2 (nx)dx
0

It is very convenient to express Eq. (4.18) in dimension less form as follows:

(x,t)
----- = (An* exp ( n*2 Fo) cos (n*x / L) (4.19)
i

where An* = An / i ; n* = nL ; Fo = Fourier Number = t / L2 ;

4.3.2.Heisslers Charts for transient conduction:- For values of Fo > 0.2 the above series
solution converges rapidly and the solution will be accurate within 5 % if only the first term
in the series is used to determine the temperature. In that case the solution reduces to
(x,t)
----- = A1* exp ( 1*2 Fo) cos (1*x /L) (4.20)
i

From the above equation the dimensionless temperature at the centre of the slab (x =0) can be
written as

(0,t)
----- = A1* exp ( 1*2 Fo) .(4.21)
i

The values of A1* and 1* for different values of Bi are presented in the form of a table (See
Table 4.1). These values are evaluated using one term approximation of the series solution.

Table 4.1: Coefficients used in one term approximate solution of transient one dimensional heat
conduction in plane walls, cylinders, and spheres (Bi = hL/k for a plane wall of thickness 2L, and Bi =
hro/k for a cylinder or sphere of outer radius r0)

Bi Plane Wall Infinite Cylinder Sphere


1* A 1* 1* A 1* 1* A 1*
0.01 0.0998 1.0017 0.1412 1.0025 0.1730 1.0030
0.02 0.1410 1.0033 0.1995 1.0050 0.2445 1.0060
0.04 0.1987 1.0066 0.2814 1.0099 0.3450 1.0120
0.06 0.2425 1.0098 0.3438 1.0148 0.4217 1.0179
0.08 0.2791 1.0130 0.3960 1.0197 0.4860 1.0239
0.10 0.3111 1.0161 0.4417 1.0246 0.5423 1.0298
0.20 0.4328 1.0311 0.6170 1.0483 0.7593 1.0592
0.30 0.5218 1.0450 0.7465 1.0712 0.9208 1.0880
0.40 0.5932 1.0580 0.8516 1.0931 1.0528 1.1164
0.50 0.6533 1.0701 0.9408 1.1143 1.1656 1.1441
0.60 0.7051 1.0814 1.0184 1.1345 1.2644 1.1713
0.70 0.7506 1.0918 1.0873 1.1539 1.3525 1.1978
0.80 0.7010 1.1016 1.1490 1.1724 1.4320 1.2236
0.90 0.8274 1.1107 1.2048 1.1902 1.5044 1.2488
1.00 0.8603 1.1191 1.2558 1.2071 1.5708 1.2732

148
2.00 1.0769 1.1785 1.5995 1.3384 2.0288 1.4793
3.00 1.1925 1.2102 1.7887 1.4191 2.2889 1.6227
4.00 1.2646 1.2287 1.9081 1.4698 2.4556 1.7202
5.00 1.3138 1.2403 1.9898 1.5029 2.5704 1.7870
6.00 1.3496 1.2479 2.0490 1.5253 2.6537 1.8338
7.00 1.3766 1.2532 2.0937 1.5411 2.7165 1.8673
8.00 1.3978 1.2570 2.1286 1.5526 2.7654 1.8920
9.00 1.4149 1.2598 2.1566 1.5611 2.8044 1.9106
10.0 1.4289 1.2620 2.1795 1.5677 2.8363 1.9249
20.0 1.4961 1.2699 2.2880 1.5919 2.9857 1.9781
30.0 1.5202 1.2717 2.3261 1.5973 3.0372 1.9898
40.0 1.5325 1.2723 2.3455 1.5993 3.0632 1.9942
50.0 1.5400 1.2727 2.3572 1.6002 3.0788 1.9962
100.0 1.5552 1.2731 2.3809 1.6015 3.1102 1.9990
1.5708 1.2732 2.4048 1.6021 3.1416 2.0000

It can also be concluded from Eq.(4.20) at any time t the ratio (x,t) / (0,t) will be
independent of temperature and is given by

(x,t)
------ = cos (1*x /L) (4.22)
(0,t)

Heissler has represented Eq. (4.21) and (4.22) in the form of charts and these charts are
normally referred to as Heisslers charts. Eq. (4.21) is plotted as Fourier number Fo versus
dimensionless centre temperature (0,t) / i using [Fig.4.4(b)].
reciprocal of Biot number1 / Bi as the parameter [Fig.4.4(a)], where as Eq. (4.22) is plotted
as (x,t) / (0,t) versus reciprocal of Biot number using the dimensionless distance x / L as
the parameter.In Fig.[4.4(a)], the curve for 1/Bi = 0 corresponds to the case
h , or the outer surfaces of the slab are maintained at the ambient temperature T. For
large values of 1 / Bi, the Biot number is small, or the internal conductance is large in
comparison with the surface heat transfer coefficient. This in turn, implies that the
temperature distribution within the solid is sufficiently uniform and hence lumped system
analysis becomes applicable.
Fig. (4.5) shows the dimensionless heat transferred Q / Q0 as a function of
dimensionless time for different values of the Biot number for a slab of thickness 2L. Here Q
represents the total amount of thermal energy which is lost by the slab up to any time t during
the transient conduction heat transfer. The quantity Q0, defined as

Q0 = V Cp[Ti - T] ..(4.23)

represents the initial thermal energy of the slab relative to the ambient temperature.

4.3.3.Transient-Temperature charts for Long cylinder and sphere: The dimensionless


transient-temperature distribution and the heat transfer results for infinite cylinder and
sphere can also be represented in the form of charts as in the case of slab. For infinite
cylinder and sphere the radius of the outer surface R is used as the characteristic length so

149
that the Biot number is defined as Bi = hR / k and the dimensionless distance from the centre
is r/R where r is any radius (0 r R).These charts are illustrated in Figs. (4.6) to (4.9).

4.3.4.Illustrative examples on the use of Transient Temperature Charts:-Use of the


transient temperature charts for slabs, infinite cylinders and spheres is illustrated in the
following examples.

Example 4.6: A brick wall ( = 0.5 x 10 6 m2/s, k = 0.69 W/(m-K) and = 2300 kg/m3 ) of
10 cm thick is initially at a uniform temperature of 230 0 C. The wall is suddenly exposd to a
convective environment at 30 0 C with a surface heat transfer coefficient of 60 W/(m2-K).
Using the transient-temperature charts, determine
(a) the centre temperature at hour and 2 hours after the exposure to the cooler
ambient,
(c) energy removed from the wall per m2 during hour and during 2 hours.
(d) What would be the time taken for the surface of the wall to reach a
temperature of 40 0C.

Sketch for example 4.6:

x
Known: = 0.5 x 10 6 m2 / s ; k = 0.69 W / (m
K)

= 2300 kg / m3 ; 2L = 0.1 m ; Ti = 230 0 C ;


Ti
T = 30 0C; h = 60 W / (m2 K) ;Ti = 2300C

Find: (a) T/x=0 at t = hour and at t = 2 hours

(b) Qremoved per m2 during hour and 2 hours

(c) t for T/x=L = 40 0C

2L

Assumptions: (i) one dimensional conduction; (ii) thermal conductivity of the solid is
constant

Solution: (a) (i) t = 0.5 h = 0.5 x 3600 = 1800 s.

60 x 0.05
Bi = hL / k = --------------- = 4.35.
0.69
Since Bi > 0.1, internal temperature gradients cannot be neglected. i.e. T = T(x,t)

150
Hence Heisslers transient temperature charts are to be used.

( 0.5 x 10 6 x 1800)
1/ Bi = 1 / 4.35 = 0.23 ; Fo = (t / L2) = -------------------------- = 0.36
0.052

From the Heisslers chart for a slab of thickness 2L,

(T0 - T)
0 = --------------- = 0.8 (T0 = Centre temperature of the slab)
(Ti - T)

Therefore T0 = T + 0.8 (Ti - T) = 30 + 0.8 x (230 30)

=190 0 C.
(0.5 x 10 6 x 7200)
(ii) when t = 2 h = 7200 s we have Fo = ---------------------------- = 1.44.
(0.052)
(T0 T)
From chart, -------------- = 0.125.
(Ti T)

Therefore T0 = T + 0.125 (T0 T) = 30 + 0.125 x (230 30)= 55 0 C.

(b) (i) t = 1800 s. At the surface x / L = 1.0 ; k / hL = 0.23 ;

(T|x=L T)
Hence from the chart ---------------- = 0.275
(T0 T)

Or T|x=L = T + 0.275 (T0 T) = 30 + 0.275 x (190 30)

= 74 0 C.

(ii) t = 7200 s. Hence T|x=L = T + 0.275 (T0 T) = 30 + 0.275 x (55 30)

= 36.9 0 C.

(c) (i) Bi2 Fo = 4.35 2 x 0.36 = 6.81; From chart Qremoved / Qmax = 0.50 .

0.69 x 1 x 0.1
Qmax = CpV(Ti T) = (k / )V(Ti T) = ------------------- x (230 30)
0.5 x 10 6

= 27.6 x 10 6 = 27.6 MJ / m

151
Therefore Qremoved = 0.5 x 27.6 = 13.8 MJ

(ii) when t = 7200 s , Bi2 Fo = 4.35 2 x 1.44 = 27.25

From chart, Q / Qmax = 0.9 ; Therefore Q = 0.9 x 27.6 = 24.8 MJ.

(d) It is given that T(L,t) = 30 0C.

Hence (L,t) ( 40 30)


------- = ---------------- = 0.05
i (230 30)

Now for x / L = 1.0 and 1 / Bi = 0.23 from the chart the ratio of surface temperature
difference to the centre temperature difference can be read as

(L,t) / (0,t) = 0.225

Hence (0,t) / i = 0.05 / 0.225 = 0.2222

From the chart corresponding to this value of (0,t) / i and 1 / Bi = 0.23, the Fourier
number can be read as

Fo = ( t) / L2 = 1.0. Therefore t = L2 / = 0.05 2 / 0.5 x 10 6 = 5000 s = 1.39 h

Example 4.7:- A long solid cylinder [ = 0.05 m2/h, k = 50 W/(m-K)] of 5 cm diameter is


initially at 200 0 C. Suddenly it is immersed in water at a temperature of 20 0 C. Assuming
the heat transfer coefficient to be 200 W/(m2-K), determine (a) the centre and the surface
temperatures after 10 minutes have elapsed, and (b) the energy removed from the cylinder
during this 10 minute period.

Sketch for example 4.7 :


T,h

R Ti

Known: = 0.05 m2 / h = 0.05 / 3600 = 1.39 x 10 5 m2 / s ; k = 50 W /(m K) ;

Ti = 200 0 C ; T = 20 0 C ; R = 0.025 m ; h = 200 W/(m2 K) ;t = 10 x 60 = 600 s

Find: (a) T/r=0 and T/r = R after t = 10 min; (b) Qremoved during 10 minute period

152
Assumptions:(i) one dimensional conduction; (ii) thrmal conductivity of the solid is constant

Solution:(a) (i) To find centre temperature T0 :

Bi = (hR / k) = (200 x 0.025) / 50 = 0.1.

Since Bi = 0.1, internal temperature gradients cannot be neglected.


(1.39 x 10 5 x 600)
2
1/Bi = 1 / 0.1 = 10 ; Fo = (t / R ) = ------------------------
0.0252

From chart for transient conduction in an infinite cylinder we have

(T0 T)
-------------------- = 0.08 ; Hence T0 = T + 0.08 (Ti - T)
(Ti T)

Or T0 = 20 + 0.08 x (200 20)

= 34.4 0 C.

(ii) To find the surface temperature, T|r=R:

(T|r=R T)
r / R = 1.0 ; 1 / Bi = 0.1 ; From chart -------------- = 0.13
(T0 T)

Therefore T|r=R = T + 0.13 (T0 T) = 20 + 0.13 x (34.4 20) = 21.9 0 C.

(b) Bi2 Fo = (0.12) x 13.34 = 1.33 x 10 1.

From energy chart for the infinite cylinder, Q / Qmax = 0.875

Qmax = VCp (Ti T) = ( x 0.0252 x 1) x (50 / 1.35x10 6) (200 20)

= 1.27 x 10 6 kJ / m = 1.27 MJ/m

Therefore Q = 0.875 x 1.27 = 1.11 MJ / m.

Example 4.8:-An orange of 10 cm diameter is initially at a uniform temperature of 30 0C.


Suddenly it is placed in a refrigerator in which the air temperature is 2 0 C. If the surface
heat transfer coefficient is 50 W/(m2-K), determine the time required for the centre of the
orange to reach 10 0 C. Assume for the orange = 1.4 x 10 7 m2/s and k = 0.59 W/(m-K).

153
Sketch for example
4.8:

Known: R = 0.05 m ; Ti = 30 0 C ; T = 2 0 C ; T0 = 10 0 C ;

h = 50 W/(m2 K);k = 0.59 W/(m K);


=1.4 x107 m2/s

Find: Time t required for the centre temperature, T/r=0 to reach 10 0C

Assumpttions: (i)Orange is assumed to be in the shape of a sphere.(ii) one dimensional


radial conduction; (iii) Orange properties are constant.

Solution:

(T0 - T) (10 2)
Now----------- = 0 = ---------- = 0.286
(Ti T) (30 2 )

1 / Bi = k / hR = 0.59 / (50 x 0.05 ) = 0.24 ; From chart for transient conduction in

Sphere we have Fo = (t / R2) = 0.3.

(0.3 x 0.05 2)
Therefore t = ------------------ = 5357 s = 1.5 h
(1.4 x 10 7)

Solution using Tables: For the given problem we have

Bi = 4.24. Therefore from Table 4 1 , by interpolation 1 = 2.4831 and A1 = 1.7362.

(T0 T)
Therefore -------------- = 0.286 = 1.7362 exp[ 12Fo]
(Ti T)

Solving for 12Fo we get 12Fo = 1.804 or Fo = 1.804 / (2.48312) = 0.2925.

Therefore t / (R2) = 0.2925

154
. . .
.
Or t= = = 5223 s = 1 h 27 min.

4.3.5. Transient conduction in semi-infinite solids:-A semi-infinite solid is an idealized


body that has a single plane surface and extends to infinity in all directions.The transient
conduction problems in semi-infinite solids have numerous practical applications in
engineering. Consider, for example, temperature transients in a slab of finite but large
thickness, initiated by a sudden change in the thermal condition at the boundary surface. In
the initial stages, the temperature transients near the boundary surface behave similar to those
of semi-infinite medium, because some time is required for the heat to penetrate the slab
before the other boundary condition begins to influence the transients.The earth for example,
can be considered as a semi-infinite solid in determining the variation of its temperature near
its surface
Consider a semi-infinite solid with constant thermophysical properties, without any
internal heat generation and uniform thermal conditions on its exposed surface.Let the solid
initially be at a uniform temperature Ti. Heat transfer in this case occurs only in the direction
normal to the surface (x-direction) and thus it is one dimensional. The depth of the solid is
very large (x ) compared to the depth upto which heat can penetrate and these
phenomena can be expressed mathematically as a boundary condition as T(x,t) = Ti. Heat
conductionin a semi-infinite solid is governed by the thermal condition imposed on the
exposed surface , and thus the solution depends strongly on the boundary condition at x = 0
We come across basically three types of boundary conditions while
analyzing the problem of one-dimensional transient conduction in semi-infinite solids.These
three problems are as follows:

Problem 1:- The solid is initially at a uniform temperature Ti and suddenly at time t>0
the boundary-surface temperature of the solid is changed to and maintained at a uniform
temperature T0 which may be greater or less than the initial temperature Ti.

Problem 2:- The solid is initially at a uniform temperature Ti and suddenly at time t>0 the
boundary surface of the solid is subjected to a uniform heat flux of q0 W/m2.

Problem 3:- The solid is initially at a uniform temperature Ti. Suddenly at time t>0 the
boundary surface is exposed to an ambience at a uniform temperature T with the surface
heat transfer coefficient h. T may be higher or lower than Ti.

Solution to Problem 1:- The schematic for problem 1 is shown in Fig. 4.10. The
mathematical formulation of the problem to determine the unsteady temperature distribution
in an infinite solid T(x,t) is as follows:
The governing differential equation [Eq. 4.7(a)] is



= (1/) 4.7(a)

155
The initial condition is at time t = 0, T(x,0) = Ti ..4.7(b)

and the boundary condition is at x = 0, T(0,t) = T0. ....................................... 4.24

It is convenient to solve the above problem in terms of the variable (x,t), where (x,t) is

,
defined as


(x,t) = ..4.25
The governing differential equation in terms of (x,t) will be

2 / x2 = (1/) ( /t) 4.26(a)

For t > 0, the surface at T0

Initially (t=0) solid at Ti

0
x

Fig. 4.10: Semi-infinite solid with specified surface temperature T0 for t > 0

The initial condition will be at time t = 0, (x,0) = Ti T .4.26(b)

And the boundary condition will be at x = 0, (0,t) = T0 T 4.26(c)

This problem has been solved analytically and the solution (x,t) is represented graphically
as (x,t) as a function of the dimensionless variable x / [2(t)] as shown in Fig. 4.11.
In engineering applications, the heat flux at the boundary surface x = 0 is also
of interest. The analytical expression for heat flux at the surface is given by

k(T0 Ti)
qs(t) = -------------- ..4.27
(t)

Solution to problem 2:- The schematic for this problem is shown in Fig. 4.11.

156
T(x,t) = Ti at t = 0

q0 W/m2

for t > 0 x

Fig. 4.11: An infinite solid subjected to a constant heat flux at x = 0 for t > 0

Governing differential equation in terms of T(x,t) and the initial condition are same that for
problem 1[i.e. equations 4.26(a) and 4.26(b)].

The boundary condition is : at x = 0, k ( / x)|x = 0 = q0.

The temperature distribution within the solid T(x,t) is given by

2q0
T(x, t) = Ti + ------ (t) [ (1 / ) exp ( 2) + erf () ] ..(4.28 a)
k
___ 2
where = x / (2 t ) and erf () = ------- exp ( y2) dy ...(4.28b)
0

Here erf () is called the error function of argument and its values for different values of
are tabulated.

Solution to Problem 3 :- The solid is initially at a uniform temperature Ti and suddenly for t
>0 the surface at x = 0 is brought in contact with a fluid at a uniform temperature T with a
surface heat transfer coefficient h [see Fig. 4.12]. For this problem the solution is represented
in the form of a plot where the dimensionless temperature [1 (x,t)] is plottedagainst
dimensionless distance x / (t), using h(t) / k as the parameter. It can be noted that the
case h is equivalent to the boundary surface ay x = 0 maintained at a constant
temperature T.

157
For t > 0, the surface is in contact with fluid at
T with heat transfer coefficient h

Initially (t=0), solid at Ti

0
x

Fig. 4.12: Semi-infinite solid with surface at x = 0 is subjected to convective


heat transfer coefficient h

Illustrative examples on Transient Conduction in Semi Infinite solids

Example 4.9:-A thick stainless steel slab [ = 1.6 x 10 5 m2/s and k = 61 W/(m-K)] is
initially at a uniform temperature of 150 0 C. Its surface temperature is suddenly lowered to
20 0 C. By treating this as a one-dimensional transient conduction problem in a semi-infinite
medium, determine the temperature at a depth 2 cm from the surface and the heat flux 1
minute after the surface temperature is lowered

Sketch for example 4.9:

158
For t > 0, the surface at T0

Initially (t=0) solid at Ti

0
x

Semi-infinite solid with specified surface temperature T0 for t > 0

Known: Ti = 150 0 C ; T0 = T|x=0 = 20 0 C ; = 1.6 x 10 5 m2 / s ; k = 61 W/(m K) ; x =


0.02 m ;t = 1 min

Find: (i) T/x= 0.02 m ; (ii) qs

Assumptions: (i) one dimensional conduction; (ii) thermal properties of the solid is constant

Solution: (i)

x 0.02
= -------------- = --------------------------------- = 0.323
2 (t) 2 x ( 1.6 x 10 5 x 60)

T(x,t) T0
From chart, --------------------- = 0.35
Ti T0

Therefore T(x,t) = T0 + 0.35 (Ti T0) = 20 + 0.35 x (150 20) = 65.5 0 C.

(ii)

k(T0 Ti) 61 x (20 - 150


qs(t) = --------------------- = ------------------------------ = 435.5 W / m2
(t) ( x 1.6 x 10 5 x 60)

Example 4.10:- A semi-infinite slab of copper ( = 1.1 x 10 4 m2/s and k = 380 W/(m-K) is
initially at a uniform temperature of 10 0 C. Suddenly the surface at x = 0 is raised to 100 0C.
Calculate the heat flux at the surface 5 minutes after rising of the surface temperature . How

159
long will it take for the temperature at a depth of 5 cm from the surface to reach 90 0 C?

Sketch for example 4.10:


For t > 0, the surface at T0

Initially (t=0), solid at Ti

0
x

Semi-infinite solid with specified surface temperature T0 for t > 0

Known: Ti = 10 0 C ; T0 = 100 0 C ; k = 380 W / (m K) ; = 1.1 x 10 4 m2 / s; t = 300 s ;

Find: Time t required for T/x = 5 cm = 90 0C

Assumptions: (i) one dimensional conduction; (ii) thermal conductivity of the solid is
constant.

Solution:

k(T0 Ti) 380 x (100 10)


qs(t) = ------------------- = -------------------------------- = 11012 W / m2 = 11.012 kW/m2
(t) ( x 1.1 x 10 4 x 300)

T(x,t) T0 90 100
(x,t) = ------------------ = --------------------- = 0.11 . From chart = 0.1
Ti T0 10 100

x x2 0.05 2
= -------------- or t = ------------------- = --------------------------------- = 586 s = 9.46 min
2 (t) 4 2 4 x 1.1 x 10 4 x (0.1) 2

Example 4.11:-A thick bronze [ = 0.86* 10 5 m2/s and k = 26 W/m-K] is initially at 2500C
Suddenly the surface is exposed to a coolant at 25 0 C. If the surface heat transfer coefficient

160
is 150 W/(m2-K), determine the temperature 5 cm from the surface 10 minutes after the
exposure.

Sketch for example 4.11:

Initially (t=0), solid at Ti

0
x

Semi-infinite solid with surface at x = 0 is subjected to convective heat


transfer coefficient h

Known: Ti = 250 0 C; T = 25 0 C; h = 150 W/(m2 K) ; k = 26 W /(m K) ;

= 0.86 x 10 5 m2/s ; t = 600 s ; x = 0.05 m ;

Find: T/x = 5 cm when t = 10 min

Assumptions: (i) one dimensional conduction; (ii) thermal conductivity of the solid is
constant.

Solution:
x 0.05
= ---------------- = ------------------------------------ = 0.35
2 ( t) 2 x ( 0.86 x 10 5 x 600)
____ __________________
h ( t) 150 x [ 0.86 x 10 5 x 600]
-------------- = -------------------------------------- = 0.414
K 26

[T(x,t) T]
Therefore from chart 1 ------------------------ = 0.15
(Ti T)

161
Solving for T(x,t) we have T(x,t) = T + (1 0.15)(Ti T)

= 25 + 0.85 x (250 25 ) = 216.25 0 C.

Example 4.12:- A thick wood [ = 0.82 x 10 7 m2/s and k = 0.15 W/(m-K)] is initially at 20
0
C. The wood may ignite at 400 0 C. Suddenly the surface of the wood is exposed to gases at
500 0 C. If the surface heat transfer coefficient is 45 W/(m2-K), how long will it take for the
surface of the wood to reach 400 0 C ?

Sketch for example 4.12

Initially (t=0), solid at Ti

0
x

Semi-infinite solid with surface at x = 0 is subjected to convective heat


transfer coefficient h

Known: Ti = 20 0 C ;T = 500 0C ;h = 45 W /(m 2 K) ;k = 0.15 W /(m K) ;


= 0.82 x 10 7 m2 / s.
Find: Time t for T/x = 400 C
Assumptions: (i) one dimensional conduction; (ii) thermal conductivity of the solid is
constant.

Solution:
____ _______________
h ( t) 45 x (0.82 x 10 7 x t)
---------- = ------------------------------- = 0.086 t
k 0.15
____ T(x,t) T (400 500)
x / 2( t) = 0. Also 1 -------------------- = 1 ------------------- = 0.9799
Ti T (20 5000)
___
Hence from chart h ( t) / k = 2.75.
_
Therefore 0.086 t = 2.75 or t = 1023 s = 17 min.

162
4.3.6. Use of Heisslers charts for Solving Multi-dimensional Transient Conduction
Problems (Product Solution):
The solution to the problem of multi-dimensional transient conduction in solids without heat
generation subjected to convective boundary conditions can be obtained using the Heisslers
charts developed for one dimensional transient conduction. The approach adopted is called
the Product Solution.

Product Solution for Transient Conduction in a Rectangular Bar:


Consider a rectangular bar of sides 2L1 and 2L2 confined to the region L1 x L1 and L2
y L2 as shown in Fig.4.13.Initaially the bar is at a uniform temperature Ti. Suddenly at

y
h1, T
L2

h2, T
L1o L1
x
Solid initially at Ti
h2,T
L2

L2

h1,T

Fig. 4.13: Product solution for transient conduction in a rectangular bar

t = 0 all boundary surfaces are subjected to convection to an ambient at a constant


temperature T.The mathematical formulation of this conduction problem is given by



+ = in L1 x L1 and L2 y L24.29a


The boundary conditions are: at x = 0, = 0 (axis of symmetry) .4.29b


at x= L1, k = h1 [T T] ..4.29c


at y = 0, = 0 (axis of symmetry) .4.29d


at y= L2, k = h2 [T T] ..4.29e
The initial condition is : at t = 0, T = Ti 4.29f

In terms of the dimensionless temperature

163
,,

(x,y,t) = 4.30

Eq.4.29a to 4.29f can be written as follows:



+ = in L1 x L1 and L2 y L24.31a


The boundary conditions are: at x = 0, = 0 (axis of symmetry) ... .4.31b


at x= L1, k = h1 ..4.31c


at y = 0, = 0 (axis of symmetry) .4.31d


at y= L2, k = h2 ..4.31e
The initial condition is : at t = 0, = 1 4.31f

It can be shown that the solution for the problem given by Eq. 4.31a to 4.31f can be
expressed as a product of solutions of two one dimensional problems 1(x,t) and 2(y,t) as
illustrated below.

Let (x,y,t) = 1 (x,t) * 2(y,t) .4.32

Substituting this in Eq. 4.31a to 4.31f and rearranging we have 1 (x,t) to be the solution of
the following one-dimensional problem:



= in L1 x L14.33a


The boundary conditions are: at x = 0, = 0 (axis of symmetry) ....4.33b


at x= L1, k = h1 1 ..4.33c
The initial condition is : at t = 0, 1 = 1 4.33d

and 2 (y,t) to be the solution of the following one-dimensional problem:



= in L2 y L24.34a


The boundary conditions are: at y = 0, = 0 (axis of symmetry) ....4.34b


at x= L2, k = h1 1 ..4.34c
The initial condition is : at t = 0, 2 = 1 4.34d

The above one dimensional problems for 1(x,t) and 2(y,t) are exactly the same as that
whose solution is given by the transient temperature chart for infinite slabs. Thus the solution
for two dimensional transient conduction problem for a rectangular region L1 x L1
and L2 y L2 can be constructed as the product of two one-dimensional transient

164
conduction problems for infinite slabs.This concept of product solution is also illustrated in
Fig.4.14(a)

2L2 x

2L1

Fig.4.14(a): Product Solution for Transient conduction ina Rectangular Bar

The basic idea developed here can be extended to other configurations. For example the
product solution ,(r,x,t)for two dimensional transient conduction in a finite cylinder of
radius R and height 2L will be the product of one dimensional conduction solution for an
infinite slab,1 (x,t) and one dimensional solution for an infinite cylinder,2(r,t).This is
illustrated in Fig.4.14(b).
x

Infinite cylinder of radius R

Infinite slab of thickness 2L

r
2L

Fig.4.14(b): Product Solution for Transient conduction in a Finite Cylinder

165
Illustrative examples on Product Solution:

Example 4.13: A rectangular iron bar ( = 1.6 * 10 5 m2/s, k= 60 W/(m-K)), 5 cm X 4 cm is


initially at a uniform temperature of 2250C. Suddenly the bar is immersed in a fluid which is
maintained at a uniformtemperature of 250C. The surface heat transfer coefficient for all the
outside surfaces of the bar is 500 W/(m2-K). Determine (i) the temperature at the centre of
the bar 2 minutes after the immersion of the bar into the fluid, (ii) the temperature at any one
corner of the bar 2 minutes after immersion

Sketch for the example:

h2,T
h1,T

2L2 x

2L1

Fig. Product Solution for Transient conduction in a Rectangular Bar

Known: 2L1 = 5 cm; 2L2 = 4 cm; = 1.6 * 105m2/s; h1= h2= 500 W/(m2-K); T =250C;
Ti = 2250C; t=2 min; k = 60 W/(m-K)

Find: (i)T(0,0,2min);(ii) T(L1,L2,2 min)

Assumptions:(i) Surface heat transfer coefficient is constant and is same for all
surfaces,i.e.h1= h2; (ii)Thermal properties of the solid are constant

Solution:
(i) (0,0,t) = 1 (0,t) * 2(0,t)

,, , ,

i.e. = *

. .
.
To find 1 (0,t): Bi = = = 0.21 ; Fo = = = 3.072

From Heisller chart we have 1 (0,t) = 0.58

166
. .

= .
To find 2 (0,t): Bi = = = 0.167 ; Fo = = 4.8

From Heisller chart we have 2 (0,t) = 0.50

,,

Hence (0,0,2 min) = = 0.58 * 0.50 = 0.29

Therefore T(0,0,2min) = 25 + 0.29* (225 25) = 83 0 C

, ,

(ii) = 1 (L1,t) * 2(L2,t)

1 (L1,t)
1 (L1,t) = ------------ * 1 (0,t)
1 (0,t)


For the corner = 1 and Bi = 0,21 as calculated before.
Hence from Heisllers chart

1 (L1,t)
--------------- = 0.9 . Hence 1 (L1,t) = 0.9 * 0.58 = 0.522
1 (0,t)

Similarly 2 (L1,t) = 0.92 * 0.5 = 0.46

, ,

Therefore = 0.522 * 0.46 = 0.24

and T(L1,L2, 2 min) = 0.24 *(225 25) + 25 = 730C

Example 4.14: A short cylinder made of Iron (k = 60 W/m-K; = 1.6 * 105m2/s) is 5 cm in


diameterand 4 cm in height. It is initially at a uniform temperature of 2250C and is suddenly
exposed to an environment at 250C with a surface heat transfer coefficient of 300 W/(m2-K).
Determine (i) the centre temperature and (ii)the temperature at a location 1 cm from the
outer surface of the cylinder and at a height of 2 cm from one of its ends 2 minutes after
exposure to the environment.

167
Sketch for example 4.14

Infinite cylinder of radius R

Infinite slab of thickness 2L

r
2L

Known: Short cylinder with R = 2.5 cm and 2L = 4 cm; = 1.6 * 105m2/s; k = 60 W/m-K;

h =300 W/(m2 K).

Find: (i) T(0,0,2 min) ; (ii) T(1.5cm,2cm,2min)

Assumption: (i) Two-demensional conduction; (ii) Thermal properties of the cylinder are
constant;(iii) the surface heat transfer coefficient is constant and is same for all boundary
surfaces; (iv) The solid is not generating any heat

Solution: (i)(r,x,t) = 1 (r,t) * 2(x,t)

(0,0,2 min) = 1 (0,2 min) * 2(0,2 min)

.

To find 1 (0,2 min) :Bi = = = 0.208or 1/Bi = 4.8

.
.
Fo = = = 3.1

From Heisllers chart for infinite cylinder we have 1 (0,2 min) = 0.31.

.

Similarly for infinite slab, Bi = = = 0.167 or 1 / Bi = 6.0

168
.
.
Fo = = = 4.8

Hence from chart, 2(0,2 min) = 0.45

,,

and (0,0,2 min) = = 0.31 * 0.45 = 0.14

Therefore T(0,0,2 min) = 0.14 * (225 25) + 25 = 530C

(ii) (1.5 cm,2 cm,2 min) = 1 (1.5 cm,2 min) * 2(2 cm,2 min)

.
To find 1 (1.5 cm,2 min): for r = 1.5 cm, r/R = . = 0.6 and 1/Bi = 4.8.

. ,
,
Hence from chart = 0.95 .

Or 1(1.5 cm, 2 min) = 0.95 * 0.31 = 0.2945.



To find 2 (2 cm,2 min):For infinite slab we have x/ L = = 0.5 and 1 / Bi = 6.0

,
,
Therefore from chart we have = 0.94

Or 2(2 cm, 2 min) = 0.94 * 0.45 = 0.423

Therefore . (1.5 cm,2 cm,2 min) = 1 (1.5 cm,2 min) * 2(2 cm,2 min)

= 0.2945 * 0.423 = 0.1246

.,,

Therefore = 0.1246

Or T(1.5cm,2cm,2min) = 0.124 * (225 25) + 25 = 48.8 0C

Example 4.15:An aluminium ingot (k = 200 W/m-K;= 2700 kg/m3; Cp = 890 J/kg-K) of
dimensions 6 cm X 5 cm X 3 cm is initially at a uniform temperature of 1750C. Suddenly the
ingot is immersed in a fluid which is maintained at a uniform temperature of 250C with the
surface heat transfer coefficient of 250 W/(m2-K). Determine (i) The temperature at the
centre of the ingot 2 minutes after immersion and (ii)temperature at one of the corners after
the same time interval.

169
Sketch for example 4.15:
z

2L2
2L3

2L1

Known: 2L1 = 6 cm ; 2L2 = 5 cm ;2L3 = 3 cm ;k = 200 W/m-K;= 2700 kg/m3;

Cp = 890 J/kg-K);Ti = 1750C; T = 250C; h = 250 W/(m2-K); t = 2 min.

Find: (i) T(0,0,0,2 min) ; (ii) T (L1,L2,L3,2 min)

Assumptions: (i)Properties of the ingot are constant; (ii) the surface heat transfer
coefficient is constant and is same for all boundary surfaces; (iii) The solid is not generating
any heat

Solution:

,,,

(i) (0,0,0,2 min) = = 1 (0,2 min) * 2(0,2 min) * 2(0,2 min)



To find 1 (0,2 min): = = = 8.323 * 10 5 m2/s

. .
.
Bi = = = 0.0375 ; Fo = = = 11.1

From chart for infinite slab, 1 (0,2 min) = 0.6

Similarly it can be found that 2 (0,2 min) = 0.4

and 3 (0,2 min) = 0.35

170
,,,

Therefore = 0.6 * 0.4 * 0.35 = 0.084

and T(0,0,0,2 min) = 0.084 * (175 25) + 25 = 37.6 0C

,,,

= 1 (L1,2 min) * 2(L2,2 min) * 2(L3,2 min)

To find 1 (L1,2 min): x/L1 = 1and 1/Bi = 1/0.0375 = 27

From chart 1 (L1,2 min)


---------------- = 0.975
1 (0,2 min)

Therefore 1 (L1,2 min) = 0.975 * 0.6 = 0.585

Similarly 2 (L2,2 min) = 0.975 * 0.4 = 0.39

and 3 (L3,2 min) = 0.985 * 0.35 = 0.34475

,,,

Therefore = .585 * 0.39 * .34475 = 0.07865

Or TL1, L2, L3,2 min = 0.07865 * (175 25) + 25 = 36.8 0C

171
CHAPTER 5
NUMERICAL METHODS IN CONDUCTION
5.1. Introduction: Analytical solutions that allow for the determination of the exact
temperature distribution in a solid are only available for limited ideal cases.Graphical
solutions have been used to gain an insight into complex conduction heat transfer problems,
where analytical solutions are not available, but they have limited accuracy and are primarily
used for two-dimensional problems. For solids having complicated geometries, boundary
conditions, and temperature dependent thermal properties, numerical method of solution
has to be used.Advances in numerical computing now allow for complex heat transfer
problems to be solved rapidly on computers using numerical techniques..The current
numerical techniques include: finite-difference analysis; finite element analysis (FEA); and
finite-volume analysis. In general, these techniques are routinely used to solve problems in
heat transfer, fluid dynamics, stress analysis, electrostatics and magnetics, etc.In this chapter
the use of finite-difference analysis to solve conduction heat transfer problems is illustrated.
In the finite difference method the governing partial differential equation of heat conduction
is approximated by a set of algebraic equations for temperature at a number of nodal points
over the region. Hence the first step in the analysis is the finite difference representation of
the governing partial differential equation into a set of algebraic equations, either by using
Taylors series expansion and neglecting higher order terms or by writing the energy balance
equation for each node in the region. Then the set of algebraic equations thus obtained are
solved for unknown temperatures.

5.2. Finite Difference Formulationfor One-dimensional Steady state Conduction in a


Plane wall:

5.2.1. Finite difference formulation from differential equation using Taylors Series
Expansion:

The governing differential equation for one dimensional steady-state conduction in a plane
wall of thickness L with heat generation is given by




+ =0 in 0 < x L 5.1

The region 0 < x L is divided into M equal sub-regions each of size



x = 5.2

172
There are M + 1 nodes for i = 0 to i = m as illustrated in Fig.5.1. Node i corresponds to a
location whose coordinate is ix. Let Ti be the temperature at node i;Then the region
contains M + 1 nodal temperatures for i = 0, 1,2,3,..,M nodes.

x x
L

0 1 2 i1 i i+1 M1 M

x
ix

i i+

Fig. 5.1 Nomenclaturefor finite difference representation of derivatives



The second derivative of temperature at a node i can be represented in finite


differences as shown below.The first derivative of temperature at locations i + and
i can be approximated using Taylors series expansion as

i + )


5.3(a)

(i )


5.3(b)



Now the second derivative of temperature at node i can be approximated as



= 5.4

When Eq. 5.4 is introduced into Eq. 5.1 we get

173



+ =0

Or Ti-1 2Ti + Ti+1 = q





5.5

Where q

represents the heat generation per unit volume at node i.Eq. 5.5 is known
as the finite difference form of the heat conduction equation 5.1 and is valid for all interior

generation q
nodes, i.e. i= 1, 2, 3,.M 1, of the region.In this equation the heat
, thermal conductivity k and mesh size x are known quantities. Then Eq. 5.5
gives
M 1 algebraic equations for the M + 1 unknown node temperatures Ti, i= 0,1,2,3M+1.
Two additional relations are needed to make the number of equations equal to the number of
unknown node temperatures Ti. These two relations are obtained from the finite difference
representation of the two boundary conditions at nodes i=0 and i=M.

5.2.2. Finite difference formulation using Energy Balance Approach:

Eq. 5.5 can also be obtained by writing the energy balance equation for the control volume
surrounding the node i (shown by hatched portion in Fig.5.1) as follows.For steady state
conduction the energy balance equation can be written as

Rate of conduction into the control volume from the left + Rate of heat generation within the
control volume Rate of heat conduction out of the control volume to the right = 0.

i.e. Q(i 1), i + Qg Qi,(i +1) = 05.6

Assuming linear variation of temperature between the adjacent nodes and using Fouriers of
conduction, Eq.5.6 can be written as

+ Ai xq



(kA)(i-1),i (kA)i,(i + 1) =0 5.7

+ V q
=0

, ,
Or + 5.8

WhereR , =

,
= Thermal Resistance between nodes i-1 and i 5.9a

R , =

,
= Thermal Resistance between nodes i+1 and i 5.9b

V = Aix = Volume of the element surrounding the node i 5.9c

174
Eq.5.8 is the finite difference formulation of the one-dimensional steady state conduction
problem with heat generation. This equation is more general than Eq.5.5, as it allows for the
variation of thermal conductivity and cross sectional area with position along the x-axis.If k
and A are constants, Eq. 5.7 reduces to Eq.5.5

5.2.3.Finite Difference formulation for Boundary Nodes:

(i)Boundaries with prescribed temperatures: Let the temperatures at the boundaries be


specified as
(i) at x = 0, T(x) = T0 5.10a

and (ii) at x = L, T(x) = TM. 5.10b

where T0 and TM are known temperatures.Then the temperatures at the nodes i = 0 and
i = M in Fig.5.1 are taken as
Ti=0 = T0 5.11a

And Ti = M = TM 5.11b

Eq.5.11a and 5.11b provide the two additional relations needed to make the number of
equations equal the number of unknown node temperatures.

(ii)Boundaries with prescribed heat flux: Let the boundary surface at x = 0 is subjected to
a prescribed heat flux of q0 and the boundary surface at x = L is subjected to a prescribed
heat flux of qM as shown in Fig. 5.2.

q0
0 12 M2 M1 M
qM


xx


Fig. 5.2 Nomenclature for finite difference formulation for prescribed surface heat flux
boundary conditions

The energy balance equation fora differential volume element surrounding the grid
pointocan be written asfollows:
q0 A0 + Qg Q 0,1 = 0 5.12

175


q0 A0 + A0(x/2)q0 kA0 =0

Or 2T1 2T0 + (q0 x2/k) + ( 2q0x / k) = 0 for i = 0 5.13

Similarly for the volume element surrounding the grid point M, the energy balance equation
after simplification reduces to

2TM 1 2TM + (qM x2/k) + ( 2qMx / k) = 0 for i = M 5.14

For the insulated boundary at x = 0 we have q0 = 0 and for the insulated boundary at x = L
we have qM = 0.Substituting these conditions in Eq. 5.13 and 5.14 we get the finite
difference equations for the boundary nodes as

2T1 2T0 + (q0 x2/k) = 0 5.15

and 2TM 1 2TM + (qM x2/k) = 0 5.16

(iii)Boundary surfaces subjects to convection:


Let the two boundary surfaces are exposed to ambient at a uniform temperature T with
surface heat transfer coefficient h as shown in Fig.5.3.

T,h
0 1 2 M2 M1 M
T, h



x x

Fig. 5.3 Nomenclature for finite difference formulation for convection boundary
conditions

The energy balance equation for the element surrounding node 0 can be written as



hA0[T T0] + A0(x/2)q0 kA0 =0

Or 2T1 2 + +


+ T = 0 for i = 05.17

176
and 2TM 1 2 + +




+ T = 0 for i = M 5.18

Example 5.1: A slab of thickness 1 cm is generating heat at a uniform rate of 7.2 * 10


7
W/m3.The boundary surface at x = 0 is maintained at a uniform temperature of 500C and the
boundary surface at x = L is in contact with a fluid at a temperature of 100 0C with a surface
heat transfer coefficient of 200 W/(m2- K). The thermal conductivity of the slab is 18 W/(m
K).Dividing the region into five equal sub regions write the finite difference formulation of
this heat conduction problem and express the equations fro determination of unknown
temperatures in matrix form.

Sketch for example 5.1:

L
0 1 2 3 4 5
T, h

= 2* 103m; = 7.2*107 W/m3


Known: L = 0.01m; k = 18 W/(m-K);T0=500C; T = 1000C; h = 200 W/(m2-K);x = 0.01/5

Find: Finite difference equations to determine the unknown temperatures; (ii) Express these
equations in matrix form.

Assumptions: (i) One dimensional steady state conduction: (ii) Thermal conductivity of the
slab is constant

Solution: For intermediate nodes 1 to 5 the finite difference equations are given by Eq.5.5:

Ti-1 2Ti + Ti+1 =





i = 1,2,.,5

For 1 = 1 we therefore have

T0 2T1 + T2 =


177
50 2T1 + T2 =
. .

Or

Or 2T1+ T2 = 66 (1)

For i = 2 Eq.5.5 gives

T1 2T2 + T3 =
. .

Or T1 2T2 + T3 = 16 (2)

Similarly for i = 3 and I = 4 we have

T2 2T3 + T4 = 16 (3)

and T3 2T4 + T5 = 16 (4)

For the boundary nod 5 the finite difference equation is given by Eq.5.18:


2T4 2 + +


+ T = 0

. . .
2T4 2 + +
.

+ * 100 = 0

Or 2T42.044 T5 = 20.44 (5)

Eqs.(1) to (5) can be written in matrix form as:

2 1 0 0 0 66
1 2 1
0 0 16

0 1 2 1 0 = 16
0 0 1 2 1 16

(6)

0 0 0 2 2.044 20.44
The above system of equations are solved using ONLINE solutions using MATLAB and the
unknown temperatures are found to be as shown in the Table 5.1 below. In the same table the
exact solution is also given for comparison. The exact solution for this example is given by

T(x) = 50 + 5* + 200 * 19 *


(7)

178
Table 5.1: Comparison betweenfinite difference solution and exact solution for one
dimensional steady state conduction in a slab
x/L 0.2 0.4 0.6 0.8 1.0
0
Temperature in C T1 T2 T3 T4 T5
Finite Difference Solution 119.05 172.09 209.13 230.18 235.22
Exact Solution 119.00 172.00 209.00 230.00 235.00

5.3. Finite Difference Formulation for One-dimensional Steady state Conduction in a


Fin of Uniform Cross Section:
Consider a fin of uniform cross section whose base is maintained at a uniform temperature T0
and is exposed to an ambient at a uniform temperature T..Let L be the length of the fin and
A be the area of cross section of the fin.Let h be the surface heat transfer coefficient for the
lateral surface of the fin and he be the surface heat transfer coefficient for the fin tip. The fin
is subdivided into M sub regions as shown in Fig.5.4(a).

T, h

L he

0 1 2 i-1 i i+1 M-1 M

Fig. 5.4(a) Nomenclature for finite difference formulation for a fin of uniform cross
section

Finite difference equations for intermediate nodes 1 to M -1: For the sub region surrounding
an intermediate node i the difference equation is obtained by writing the energy balance
equation for the sub region shown in Fig.5.4(b)

Qconvection

Nodei-1Nodei+1

iii

Node i

Q i-1,i Qi,i+1
Fig5.4(b) Thermal Energy crossing the boundaries of the volume element surrounding
the node i

179
Rate of conduction into the sub region from the node i-1 to node i

Rate of conduction out of the region from node i to node i+1 Rate of convective heat

transfer from the boundary surface of the region into the ambient = 0

i.e. Q i-1,i Qi,i+1 Qconvection = 0

kA kA
Ti Ti+1
x


or hPx [Ti T] = 0

Substituting = T - T in the above equations and simplifying we get

i - 1 [ 2 + N2x2] i + i + 1 = 0 i = 1 ,2,4 (5.19)

Where N2 = hP/(kA) (5.19a)

Finite difference equations for the outer boundary node M: The sub region surrounding the
outer boundary node M is shown Fig.5.4(c).

Qc1

Node M - 1 Node M
Qc2

he, T
QM-1,M

Fig5.4(c) Thermal Energy crossing the boundaries of the volume element surrounding
the boundary node M

Energy balance equation for the sub region surrounding the node 5 can be written as

Q4-5 Qconvection = 0

kA hP

i.e.
[TM T] heA T T = 0

In terms of the above equation reduces to

2 M-1 [2 + (Nx)2+ 2 ] M = 0


(5.20)

If the heat loss from the tip is negligible or if the tip is insulated then he = 0. In that case
Eq.(5.20) reduces to

180
2 M-1 [2 + (Nx)2 ] M = 0 (5.21)

Example 5.2:An iron rod (k = 50 W/m-K) of length 5 cm and diameter 2 cm protrudes from a
wall and is exposed to an ambient at 200C with a surface heat transfer coefficient of 100
W/(m2-K).The base of the rod is at 3200C and the heat transfer from the tip of the rod is
negligible. Determine the temperature distribution along the length of the rod using finite
difference method and compare the solution with the exact solution.

Sketch for example 5.2:

k h,T
x Insulated

Known: D = 2 cm.; k = 50 W/(m-K);T = 200C; h = 100 W/(m2-K);T0 = 3200C

Find: Temperature distribution along the length using finite difference method and compare
this with the exact solution

Assumptions: (i) One dimensional steady state conduction along the length of the rod
(ii)Thermal conductivity of the rod is constant

Solution: The rod is divided into 5 equal subdivisions as shown below. Then x = L/5 = 5/5
= 1 cm = 0.01 m

T, h

L
Insulated
0 1 2 3 4 5

For 5 sub regions, the difference equation for intermediate nodes are given by Eq.5.20

i-1 [2 + (Nx)2] i + i+1 = 0 i = 1 ,2,4 (1)

Finite difference equations for the boundary node 5:Since the heat loss from the tip is
negligible the difference equation for node 5 is giv en by Eq.(5.21). i.e.

181
2 4 [2 + (Nx)2] 5 = 0 (2)

.
Now (Nx)2 = = 0.04
.

Substituting this numerical value in Eq. (1) and (2) we get the finite difference equations for
temperatures of the nodes as

i-1 2.04i + i+1 = 0 i= 1.2.3.4 (3)

and 2 4 2.045 = 0 (4)

Therefore we have

For i = 1, i-1 = 0 = 320 20 = 300

2.041 + 2 = 300 (5a)

for i = 2, 1 2.042 + 3 = 0 (5b)

for i = 3, 2 2.043 + 4 = 0 (5c)

for i = 4, 3 2.044 + 5 = 0 (5d)

and for i = 5 2 4 2.045 = 0 (5e)

Eqs. (5a) to (5e) can be arranged in matrix form as

The above system of equations is solved using ONLINE Linear Algebra Tool Kit and the
results are compared with exact solution in the table 5.2 given below

Table 5.2: Comparison betweenfinite difference solution and exact solution for one
dimensional steady state conduction in a fin of uniform cross section

x/L 0.2 0.4 0.6 0.8 1.0


0
Temperature in C 1 2 3 4 5
Finite Difference Solution 260.10 230.6 210.4 198.6 194.7
Exact Solution 260.00 230.5 210.2 198.3 194.4

182
5.4. Finite Difference Formulation for One-dimensional Steady state Radial Conduction
in cylinders and spheres:

5.4.1. Cylindrical Geometry:The finite difference formulation for the problem of one
dimensional radial steady state conduction in a solid cylinder is illustrated in this section.Let


the solid cylinder is of radius R and is generatingthermal energy at rate of q(r) W/m3.The


region 0 r R is divided into M cylindrical subregions, each of thickness r = as shown
in Fig.5.5.

TM
TM-1

Ti+1
Ti
Ti-1

T1

T0

0 1 i1 i i+1 M-1 M

r
ir

Fig.5.5 Nomenclature for finite difference formulation for one dimensional radial
conduction in a solid cylinder

5.4.1.1Finite difference equation for an intermediate node i: Energy balance equation for
the sub region surrounding the grid point i can be written per unit length of the cylinder as

Ti1 Ti
k 2 1 + 2 1

r

Ti Ti+1
k 2 + 1

r
=0 i = 1,2,3M-1

The above equation simplifies to


1 Ti-1 2Ti + 1 + Ti+1 =


i= 1, 2,3,. M 1 (5.22a)

5.4.1.2.Finite difference equation for the innermost node 0:The energy balance equation for
the sub region surrounding the node 0 can be written as

183
*1 k 2 1 = 0

q


Or T0 + T1 = (5.22b)

5.4.1.3. Finite difference equation for the outermost node M:

(i)Boundary surface is subjected to convection boundary condition: Let the boundary


surface is exposed to an environment at a uniform temperature T with a surface heat transfer
coefficient h.The various energies crossing the boundary surface of the volume element
surrounding node M is shown in Fig.5.6

Qconv

QM-1.M

oM 1 M

Fig. 5.6 Energy balance for outer most node M

Energy balance equation for the volume element surrounding the node M can be written as

QM-1, M + Qgen Qconv = 0

TM1 Tm
Or k * 2 1 + 1 + q 2Mr 1

r

h *2Mr [TM - T] = 0

After simplifying we get

1 TM-1 1 + TM =




T (5.22c)

Eqs.(5.22a) to (5.22c) form a set of M+1 simultaneous equations which can be solved for
unknown temperatures T0 to TM+1.

184
(ii)Boundary surface is maintained at a specified temperature,TM: For this case , since the
temperature at node M is specified, the number of unknown temperatures will be M and
Eqs.(5.22a) and (5.22b) will form a set of M equations which can be solved for unknown
temperatures T0,T1, ,TM-1.

5.4.2. Spherical Geometry: Following the procedure illustrated in section 5.4.1for radial
steady state conduction in a solid cylinder, the finite difference formulation for radial
conduction in a solid sphere can be written as follows:


1 + 1 + +


=0 i= 1,2,M-1


(5.23)
Making the approximation that 1 1 and 1 + 1 +

Eq. (5.23) reduces to


1 Ti-1 2Ti + 1 + Ti+1 =


i= 1, 2,3,. M 1 (5.24a)

Eq. (5.24a) is similar to Eq.(5.23a) for a solid cylinder except that 1/2i is replaced by 1/i.

The difference equation for the innermost node iis given by




6T1 6T0 = (5.24b)

The difference equation for the outermost node M is given by

hr


1 TM-1 1 + K TM =


Making the approximation that 1 1 and 1 + 1 +

We get

TM-1 1 + TM =



1


(5.24c)

Eqs. (5.24a) to (5.24c) form a set of M+1 simultaneous equations which can be solved for
unknown temperaturatures T0 to TM.

185
Example 5.3: A long cylindrical fuel element of radius 1 cm and thermal conductivity of 25
W/(m-K) generates energy at a constant rate of 5* 10 8 W/m3. The boundary surface is
maintained at a uniform temperature of 1000C.Assuming one-dimensional radial conduction,
determine the radial temperature distribution in the fuel element using finite difference
method and compare the values with exact solution for the problem.

Sketch for Example 5.3:Refer Fig.5.5 and Fig. 5.6

Known: R = 0.01m; k = 25 W/(m-K); = 5* 10 8 W/m3 for all the nodes;TM = 1000C

Find: Radial steady state temperature distribution in the cylinder by finite difference method
and compare with exact solution

Assumptions: (i)One dimensional radial steady state conduction; (ii)Thermal conductivity is


constant

Solution: Divide the region 0 r R into 5 cylindrical sub regions so that


.

r = = 0.002 m

The finite difference equation for the innermost node 0 is obtained from Eq.(5.22b) as



.

T0 T1 = =

Or 4T0 +4T1 = 80 (1)

The finite difference equations for intermediate nodes i = 1 to 4 is given by Eq.(5.22a) as


1 Ti-1 2Ti + 1 + Ti+1 =


i= 1, 2,3,. M 1

= 80

Hence for i = 1 we have 1 Ti-1 2Ti + 1 + Ti+1 =

0.5T0 2T1 + 1.5T2 = 80 (2)

For i = 2: 0.75T1 2T2 + 1.25T3 = 80 (3)

For i = 3; 0.83T2 2T3 + 1.17T4 = 80 (4)

For i = 4 0.88T3 2T4 + 1.13 * 100 = 80

Or 0.88T3 2T4 = 193 (5)

186
Eqs. (1) to (5) are solved using Online Linear Algebra Tool Kit and tabulated as shown in
the Table below. This table also gives the exact solution at the 5 nodes for comparison. The


exact solution is given by
T(r) = 100 + 500* 1

(6)

Table 5.3:: Comparison betweenfinite difference solution and exact solution for one
dimensional steady state radial conduction in a cylinder

r/R 0.0 0.2 0.4 0.6 0.8


0
Temperature in C T0 T1 T2 T3 T4
Finite Difference Solution 600.3 580.3 520.3 420.3 280.4
with 5 sub regions
Finite Difference Solution 600.27 580.0 520.0 420.0 280.0
with 10 sub regions
Exact Solution 600.0 580.0 520.0 420.0 280.0

The numerical solution agrees very well with the exact solution (the maximum deviation
being 0.143 %) and hence there is no need to divide the region 0 r R into more number of
sub regions to improve the accuracy. It can also be seen from the above table that by dividing
the given region into 10 sub regions the numerical predictions exactly matches with the exact
solution

Example 5.4.A spherical fuel element of radius 1 cm and thermal conductivity of 25 W/(m-K)
generates energy at a constant rate of 7.5* 10 8 W/m3. The boundary surface is maintained at
a uniform temperature of 1000C.Assuming one-dimensional radial conduction, determine the
radial temperature distribution in the fuel element using finite difference method and
compare the values with exact solution for the problem.

Sketch for example 5.4: Refer Fig.5.5 and 5.6

Known: R = 0.01m; k = 25 W/(m-K); = 7.5* 10 8 W/m3 for all the nodes; TM = 1000C

Find: Radial steady state temperature distribution in the sphere by finite difference method
and compare with exact solution

Assumptions: (i)One dimensional radial steady state conduction; (ii)Thermal conductivity is


constant

Solution: Divide the region 0 r R into 5 cylindrical sub regions so that


.
r = = 0.002 m
Finite difference equation for the inner most node 0 is obtained from Eq.(24b) as

187

. .

6T1 6T0 = =

Or 6T0 + 6T1 = 120 (1)

For intermediate nodes form i = 1 to 4 Eq.(5.24a) is applicable:


1 Ti-1 2Ti + 1 + Ti+1 =


i= 1, 2,3,4

For i =1 we have: 2T1+ 2T2 = 120 (2)


(2)

For i = 2: 0.5T1 2T2 + 1.5 T3 = 120 (3)

For i = 3: 0.667T2 2T3 + 1.333T4 = 120 (4)

For i = 4 0.75T3 2T4 + 1.25 * 100 = 120

Or 0.75T3 2T4 = 245 (5)

Eqs. (1) to (5) are solved using online Linear Algebra Tool Kit and the solution is shown in
Table below.The table also gives the exact solution for comparison. The exact solution is
given by


T(r) = 100 + 500 1 +

Table 5.3:: Comparison betweenfinite difference solution and exact solution for one
dimensional steady state radial conduction in a sphere

r/R 0.0 0.2 0.4 0.6 0.8


0
Temperature in C T0 T1 T2 T3 T4
Finite Difference Solution 600 580 520 420 280
with 5 sub regions
Exact Solution 600 580 520 420 280

Example 5.5: A long triangular fin attached to a surface is made of Aluminium (k = 180
W/(m2-K).The fin is 5 cm long having a base thickness of 1 cm.The base is maintained at a
uniform temperature of 200 0C. The fin is exposed to an ambient at 250C and the convective
heat transfer coefficient of 15 W/(m2-K). Using finite difference method with six equally
spaced nodes determine(a) the temperature at the nodes, (b) the rate of heat dissipation from
the fin to the ambient and (c) the fin efficiency

188
Sketch for example 5.5h,T
1 2 3 4

T0
0 5
b

Known: L = 0.05 cm; b = 0.01 cm; k = 180 W/(m-K); T0= 200 0C; T = 250C; h = 15
W/(m2-K) ; x = L/5 = 0.05/5 = 0.01m ;
Find: (a) Temperatures at nodes 1 to 5; (b) Qfin; (c) fin efficiency

Assumptions: (i) one dimensional steady state conduction; (ii) thermal conductivity of the fin
is constant; (iii) heat dissipation to the ambient is by convection only

Solution: Volume element surround ing an intermediate node i is shown below with adjacent
nodes.



A i- 1/2Length =
Fin BaseNode i
Ai+1/2

Node (i 1) Node (i + 1)
x

ix

Energy balance equation for the volume element surrounding node i can be written as

Q(i-1) -i Qi (i+1) Qconvection = 0

k A i- k A i+ h , = 0

i = 1,2,3 and 4 (1)

where A i- = 2w[L (i )x] tan ; A i+ = 2w[L (i + )x] tan

Substituting these expressions in Eq.(1) we get

189
2kw tan 2kw + tan

, = 0


h


Or 1 Ti 1 [2 2i ] Ti+ 1 + Ti + 1




= T i = 1,2,3,and 4 (2)

.
.
Now tan = b/2L = = 0.1 Hence sin = 0.0995

For i =1 Eq.(2) gives 0.9 * 200 + 0.7T2 1.6 T1 = 0.0017 * 25

Or 1.6 T1+ 0.7 T2 = 180.042 (a)

For i = 2 ; 0.7T1 1.198 T2 + 0.5T3 = 0.042 (b)

For i = 3; 0.5T20.802 T3 + 0.3T4 = 0.042 (c)

For i = 4; 0.3T3 0.402 T4 + 0.1T5 = 0.042 (d)

The finite difference equation for node i = 5 is obtained by writing the energy balance
equation for the volume element having a length of x / 2 surrounding node 5 as shown in
Fig. below

x/(2cos )
4 5

x/2 x/2

Energy balance equation for node 5 can be written as

tan h *2w * (T5 - T) = 0


/

k *2w


or T4 [1 + ]T5 = T

Hence for node 5 we have T4 1.008 T5 = 0.2 (e)

190
Solving Eqs. (a) to (e) using ONLINE Linear Algebra Tool Kit we get the unknown
temperatures as follows:

Node 1 2 3 4 5
Temp (0 C) 199.76 198.99 197.31 195.70 194.30

(b) Qfin = Sum of heat dissipated from all the volume elements to the surroundings

/

= h* 2w (T0 - T) + h*2W [(T1 T) + (T2 T) + (T3 T)
/

+ (T4 T)] + h* 2w (T5 T)


= h [T0 + 2(T1 + T2 + T3 + T4) + T5 10 T]

.
.
= * [200 + 2* (199.76 + 198.99 + 197.31 + 195.7) + 194.3 10 *25]

= 260.46 W
.
(c) Qmax = h Afin,total (T0 T) = h (T0 T) = .
* (200 25)= 263.8 W

.
.
Fin efficiency = = = 0.987 = 98.7%

Example 5.6: A pin fin of variable circular cross section has a diameter of 2 cm at its base
and 1 cm at its tip and is 12 cm long. It is made of a material of thermal conductivity of 1
W/(m-K). The fin base temperature is 1250C and the fin is exposed to an ambient at 250C
with a surface heat transfer coefficient of 10 W/(m2-K).Determine the steady state
temperature distribution along the length of the fin using finite difference method

Sketch for example 5.6 showing the nomenclature adopted

x/2

x/2
D2

0 1 2 3 4 5 6

D1

x
L

191
Known: D1 = 0.02 m; D2 = 0.01 m; L = 0.12 m; k = 1 W/(m-K); h = 10 W/(m2-K);

T0 = 1250C; T = 250C

Find: (a) Steady state temperature distribution along the length using finite difference
method; (b)Qfin

Assumptions:(i) one dimensional steady state conduction; (ii) thermal conductivity of the fin
is constant; (iii) heat dissipation to the ambient is by convection only

Solution: The fin is divided equally into 6 subregions each of length x so that

. .
.
x = L/6 = 0.12/6 = 0.02 m. Also tan = = = 0.0417 or = 2.388 0

Difference equations for intermediate nodes from 1 to 5: The volume element surrounding
an intermediate node i along with corresponding nomenclature is shown in Fig. E5.6(a).

x / cos
X

i1 i i +1

i x

Ai Ai +1/2

Fig. E5.6(a): Nomenclature for the volume element surrounding an intermediate


node i

Energy balance equation for the volume element shown can be written as

kAi kAi + h Asi[Ti T] = 0i = 1 to 5




(1)



Let = T T. and Hi = Asi

Then Eq.(1) can be written as

Ai i-1 [Ai + Ai + + H i] i+ Ai + i+1 = 0 i = 1 to 5 (2)



where Ai = [D2/2 + (L ix + ) tan ]2 ; Ai + = [D2/2 + (L ix ) tan ]2

192
Asi = 2 [(D2/2 + (L ix) tan ]

Now Ai = [0.005 + (0.12 0.02i +0.01) 0.0417]2 = (0.0104 8.34*104i) 2

Ai += [0.005 + (0.12 0.02i 0.01) 0.0417]2 = (9.587 * 103 8.34*104i)2

2 + 0.12 0.02 0.0417


. .
.
Asi =

= 1.258* 10 3 1.049 * 10 6 i

Numerical values of Ai ,Ai + and Hi are found for different values of i from 1 to 5 are
tabulated as shown below.

Ai = (0.0104 8.34*104 i) 2 Ai + = (9.587 * 103 8.34*104 i)2



Node
Hi = [1.258* 10 3 1.049 * 10 6 i]

1 2.875 * 10 4 2.407* 10 4 3.771 * 10 4


2 2.395* 10 4 1.970* 10 4 3.768* 10 4
3 1.960*10 4 1.577* 10 4 3.765* 10 4
4 1.568*10 4 1.228* 10 4 3.761* 10 4
5 1.219*10 4 0.922* 10 4 3.758* 10 4

Substituting the calculated values of Ai ,Ai + and Hi for different nodes the final form of
Eqs.(3) for nodes 1 to 5 c an be written as

For i = 1; 2.875 * 10 4125 25 9.053*10 4 + 2.407 * 10 4 = 0

Or 9.053 + 2.407 = 287.5 (a)

Similarly for i =2: 2.395 8.133 + 1.970 = 0 (b)

i = 3: 1.960 7.302 + 1.577 = 0 (c)

i = 4; 1.568 6.557 + 1.228 = 0 (d)

i = 5; 1.219 5.899 + 0.922 = 0 (e)

Difference equations for node 6: The volume element surrounding node 6 is shown in Fig.
E5.6(b).

193

A5+1/2

5 6
D2

x/2 x/2

Fig. E5.6(b): Nomenclature for volume element surrounding node 6

Energy balance equation for the volume element shown can be written as follows:

Q5-6 Qconvection = 0

i.e. k A5+1/2 h As6 [T6 T] = 0




Where A5+1/2 = + tan and As6 = + 2 + tan

Sustituting = T - T and simplifying we get

6 = 0


A5+1/2 5 [A5+1/2 +


Now A5+1/2 = + 0.0417 = 9.219 * 10 5
. .

+ . 2 + 0.0417 = 4.192 * 10 4
. . . .

and As6 =

.

Or 9.219 * 10 55 [9.219 * 10 5 + * 4.192 * 10 4] 6 = 0

Or 5 2.364 6 = 0 (f)

Eqs. (a) to (f) can be solved using any standard technique of solving a system of linear
equations.

194
5.5. Finite Difference Formulation for Two-dimensional Steady State conduction with
Energy Generation

5.5.1. Difference Equation in terms of Cartesian Coordinates: The two dimensional


steady-state heat conduction equation with heat generation in a solid with constant thermal
conductivity is given by

+
,

+ =0 in region R (5.25)

The region R is replaced by a number of small rectangular meshes each of x by .y as


shown in Fig.5.7.Symbols i,j are used to denote the location of a nodal point whose
coordinates are x = ix, y = jy. Then the temperature at a node (i,j) is denoted by Ti,j.

y
(i 1, j) (i, j+1) (i, j) (i+1, j)
x
(i, j-1)
(j +1)y

jy
y
(j 1)y

x
(i 1)x ix (i +1)x

Fig.5.7: Rectangular mesh of size x, y and intermediate node (i,j) surrounded by 4


neighboring nodes

Difference equation for all intermediate nodes: Control volume surrounding the nodal point
i,j is shown in Fig.5.7.The energy balance equation for this control volume under steady state
conditions can be written as follows:

Q(i-1,j) (i,j) + Q(i,j+1) (i,j) +Q(i+1,j) (i,j) +Q(i,j+1) (i,j) + Qg = 0 (5.26)

Using Fouriers law Eq.(5.26) can be written as

, , , , , ,

k (y *1) + k (x *1) + k (y *1) +

195
, ,
+ (x *y*1) q
, = 0

k (x *1)

Choosing x = y, the above equation reduces to


Ti 1,j + Ti ,j+1 + Ti + 1,j + Ti ,j 1 4Ti ,j + =0


(5.27)
Eq.(5.27) is the finite difference form of the heat conduction equation for any interior nodal
point (i,j).

Difference equations for boundary nodal points:

(i) Boundary subjected to prescribed heat flux:

(i , j+1)

(i +1, j)
q0( i , j)

yFig.5.9 Nomenclature for a boundary node


subjected to prescribed heat flux
(i, j 1)
x

Energy ba;ance equation for the control volume surrounding the nodal point (i,j) can be
written as:

q0*(y*1) + Q(i , j+1) , (i,j) + Q(i , j+1) , (i,j) + Q(i , j+1) , (i,j) + Qg= 0 (5.28)

Using Fouriers law for heat conduction the above equation reduces to

, , , ,

q0*(y*1) + k (x/2 *1) + + k (y *1)

, ,
+ y 1 q

+ k (x/2 *1)
, = 0

Substituting x = y and simplifying, the above equation reduces to


Ti , j+1 + 2Ti+1 , j + Ti, j1 4Ti , j = (5.29)

196
If the boundary containing the node (i,j) then q0 = 0. In that case Eq. (5.29) reduces to


Ti , j+1 + 2Ti+1 , j + Ti, j1 4Ti , j = (5.29a)

(ii) Boundary subjected to convection:

(i , j+1)

(i +1, j)
( i , j)

h,Ty Fig.5.9 Nomenclature for a boundary node


subjected to convection
(i, j 1)
x

Energy balance equation for the control volume surrounding the nodal point (i,j) can be
written as:

Qconvection + Q(i , j+1) , (i,j) + Q(i , j+1) , (i,j) + Q(i , j+1) , (i,j) + Qg= 0

, , , ,

h(y*1)[T Ti j] + k (x/2 *1) + + k (y *1)

, ,
+ y 1 q

+ k (x/2 *1)
, = 0

Substituting x = y and simplifying, the above equation reduces to


,
Ti , j+1 + 2Ti+1 , j + Ti, j1 + Ti , j =



(5.30)

If the boundary containing the node (i,j) is insulated then h = 0 and there fore Eq.(5.30)
reduces to Eq. (29a).

Difference equations for corner nodal point:

(i) Difference equation for exterior corner nodal point subjected to convection with ambient
temperature at Tand surface heat transfer coefficient h.

The nomenclature for an external nodal point is shown in Fig.5.10

197
Q2-convection

(i , j)(i +1, j)

Q1-convection
yFig.5.10 Nomenclature for an exterior nodal point

(i, j 1)x

Q1-convection + Q2-convection + Q(i +1, j)- (i,j) + Q(i , j 1)- (i,j) + Qg= 0

1 , + h 1 , + k 1
, ,

h

+k 1 + 1 q
, ,
, = 0

Substituting x = y and simplifying we get


2Ti+1, j + 2Ti, j1 + , =
,

T (5.31)

(ii) Difference equation for nodal point at the intersection of two convection boundarieswith
ambient temperature at Tand surface heat transfer coefficient h.

The nomenclature for the nodal point (i,j) at the intersection of two convection boundaries is
shown in Fig. 5.11.

(i, j+1)

(i-1 j) (i, j) (i+1,j)

h, T Fig.5.11 Nomenclature for a node at the


intersection of two convection boundaries
(i,j-1)

Energy balance equation for the control volume surrounding the node (i,j) can be written as
follows:

198
Q(i-1,j) (i,j) + Q(i,j+1) (i,j) + Q(i+1,j) (i,j) + Q(i ,j-1) (i,j) + Qconvection + Qg= 0

k (y *1) + k (x *1) + k (y/2*1)


, , , , , ,

+ k (x/2 *1) + h +
, ,
[T T,j] + 3 1 ,


=0

Substituting x = y and simplifying we get

Ti,j-1 + 2Ti-1,j + 2Ti,j+1 + Ti+1,j + Ti,j =




, = 0

T (5.32)

Difference equation for a node next to an irregular boundary

Consider a node (i,j) located next to a curved surface as shown in Fig.5.12.Let the two stirngs
from the node (i,j) intersect the curved boundary at say points A and B as shown in Fig.5.12.

Node (i-1,J)
A
Node (i,j)
y
B
y Fig. 5.12 Nomenclature for a node adjacent to a
x x curved boundary

Node (i,j-1)

Assuming that the temperatures at A and Bi.e. TA and TB are known the energy balance
equation for the control volume surrounding the node (i,j) can be written as follows

Q(i-1,j) (i,j) + QA (i,j) + QB (i,j) + Q(i, j-1) (i,j) + Qg = 0

, , , ,
k + 1 + k + 1 + k + 1

, ,
+ k + 1 + + + q

, = 0

Assuming x = y and simplifying we get


+ + Ti,j =
, , ,

(5.33)

199
Example 5.7: A solid body of square cross section of side 2 cm. is generating heat at a
uniform rate of 2*10 6W/m3.The thermal conductivity of the body is 15 W/(m-K).The left
surface of the body is insulated and the bottom surface is maintained at a uniform
temperature of 1000C.The right surface is subjected to a uniform heat flux of 5000 W/m2 and
the top surface is in contact with a fluid at a temperature of 250C with a surface heat
transfer coefficient of 75 W/(m2-K).Assuming x = y = 1 cm obtain the finite difference
equations for the unknown temperatures assuming steady state conduction at the six nodes
and solve for these temperatures.

Sketch for example 5.7


Node 2
xh = 75 W/(m2-K), T = 25 0C

1 2 3

Insulated46 qs = 5000 W/m2


Node 5
y

7 8 9

T = 100 0C

Known: x = y = 0.01 m; k = 15 W/(m-K); =2*10 6W/m3 for all i;qs = 5000 W/m2;

h = 75 W/(m2-K); T7 = T8 = T9 = 1000C; T = 250C

Find: (i) Finite difference equations to find unknown temperatures T1 to T6 (ii) Numerical
values of T1 to T6 by solving the finite difference equations

Assumptions: (i) steady state two dimensional conduction heat transfer; (ii) thermal
conductivity and heat generation in the solid are constant

Solution:Finite difference equation for node 1:

h, T
1 2

Insulated y/2

x/2Fig.E5.7(1) : Volume element surrounding


y/2 node 1
4

200
Energy balance equation for the volume element surrounding node 1per unit width measured
normal to the paper can be written as:

Q4-1 + Q2-1 + Qconvection + Qg = 0


T4 T1
1 *1T T + 1 q

y =0

k +k +h

Substituting y = x in the above equation and simplifying we get

T4 + T2 2 + T1 = q1

Substituting the given data we get

T4 + T2 2 + T1 = 2 106
. . .

Or 2.05T1 + T2 + T4= 6.717 (a)

Finite difference equation for node 2:Volume element surrounding node 2 with the
nomenclature is shown in Fig. E5.7(2).

xh, T

1 2 3
y/2

x/2 y/2
5

Fig.E5.7(2): Volume elementsurrounding node 2

Energy balance equation for the volume element surrounding node 2per unit width measured
normal to the paper can be written as:

Q1-2 + Q3-2 + Q5-2 + Qconvection + Qg = 0

k 1 + k 1 + k x 1

+ h (x *1)(T - T2)

+ x 1 q

= 0
Substituting y = x and simplifying we get

201
T2 + T3 +2T5 = q

T1 [4 +

T
=0

Substituting the given data in the above equation we have

T2 + T3 +2T5 = 2 10 = 0
. . .

T1 [4 + *25

Or T1 4.1T2 + T3 +2T5 = 15.83 (b)

Finite difference equation for node 3:Volume element surrounding node 3 with the
nomenclature is shown in Fig. E5.7(3).

h, T
2 3

y/2 qs

y x/2

x 6

Fig. E5.6(3) Volume element surrounding node 3

Energy balance equation for the volume element surrounding node 3per unit width measured
normal to the paper can be written as:

Q2-3 + Q6-3+ Qconvection + Qs + Qg = 0

Or k 1 + k 1 + h 1(T T3)

+ 1qs + 1 q

=0

Substituting y = x and simplifying we get

q

T2 [2 +

] T3 + T6 =

T

qs
=0

Substituting the given data in the above equation we have

. . . .

T2 [ 2 + ] T3 + T6 = *25 * 5000 * 2 * 10 6

Or T2 2.05T3 + T6 = 11.25 (c)

202
Finite difference equation for node 4:Volume element surrounding node 4 with the
nomenclature is shown in Fig. E5.7(4).

h, T

1
Insulated

4 5

y Fig.E5.6(4) Volume element surrounding node 4

7
x

Energy balance equation for the volume element surrounding node 4per unit width measured
normal to the paper can be written as:

Q1-4 + Q5-4+ Q7-4 + Qg = 0

k 1 + k(y*1) + k 1


Or

+ y 1 q

=0

Substituting y = x and simplifying we get

q

T1 4T4 + 2T5 = T7

Substituting the given data in the above equation we have

.

T1 4T4 + 2T5 = 100 * 2 * 10 6

Or T1 4T4 + 2T5 = 113.33 (d)

Finite difference equation for node 5:Volume element surrounding node 5 with the
nomenclature is shown in Fig. E5.7(5).

203
5

4 5 6

x y
8

Fig. E5.6(5) Volume element surrounding node 5

Energy balance equation for the volume element surrounding node 3per unit width measured
normal to the paper can be written as:

Q4-5 + Q2-5+ Q6-5 + Q8-5 + Qg = 0

T T T T T T T T
Or k y 4x 5 + k x 2y 5 + k y 6x 5 + k x 8y 5 + (x*y*1)q
=0

Substituting y = x and simplifying we get


T2 + T4 + T6 4 T5 = T8 q


Substituting the given data in the above equation we have
.

T2 + T4 + T6 4 T5 = 100 * 2 * 10 6

Or T2 + T4 + T6 4 T5 = 113.33 (e)

Finite difference equation for node 6:Volume element surrounding node 6 with the
nomenclature is shown in Fig. E5.7(6).

x
y

5 6
qs

Fig. E5.6(6) Volume element surrounding node 6

204
Energy balance equation for the volume element surrounding node 6per unit width measured
normal to the paper can be written as:

Q5-6 + Q3-6+ Q9-6 + Qs + Qg = 0

Or k y 5x 6 + k 1 + k 1 + (y*1)qs
T T

+ y 1 q

=0
Substituting y = x and simplifying we get

q

T3 + 2T5 4T6 = T9

Substituting the given data in the above equation we have

. .

T3 + 2T5 4T6 = * 2* 10 6

Or T3 + 2T5 4T6 = 120 (f)

Eqs. (a) to (f) are solved using Online Linear Algebra Tool Kit and the nodal temperatures
are found to be as follows:

T1 = 118.47 0C ; T2 = 119.630C; T3 = 121.930C; T4 = 116.510C; T5 = 117.130C ;

T6 = 119.030C

Example 5.8:Hot combustion gases of a furnace are flowing through a concrete chimney
[(k = 1.4 W/(m-K)] of rectangular cross section as shown in Fig. E5.8.The flow section of
the chimney is 20 cm X 40 cm, and the thickness of the wall of the chimney is 10 cm. The
average temperature of the hot gases in the chimney is Ti = 280 0C, and the average
convective heat transfer coefficient inside the chimney is hi = 75 W/(m2-K). The chimney is
losing heat from its outer surface to the ambient air at 15 0C by convection with a surface
heat transfer coefficient of ho = 18 W/(m2-K). Using the finite difference method with x = y
= 10 cm and taking full advantage of the symmetry,determine (a) the finite difference
formulation of the problem to find steady two-dimensional temperatures at the nodal points,
(b) the temperatures at these nodal points, and (c) the rate of heat loss for a 1-m-long section
of the chimney.

h0, T0

205
k

10 cm

20 cm

10 cm
H

10 cm 40 cm 10 cm

hi , Ti

Fig.E5.8: Figure for example 5.

Sketch for example 5.8 with nodes:Because of symmetry only one fourth of the cross section
of the chimney is considered for analysis as shown below.

Axis of symmetry
(Insulated boundary)
1 2

hi,Ti

x
3 4 5 6
4
y/2

y/2

7 8 9 10
x/2 h0,T0
Known: k = 1.4 W/(m-K); x = y = 0.1 m ; hi =75 W/(m2-K); Ti = 280 0C;

206
h0 = 18 W/(m2-K); T0 = 15 0C

Find: (i) finite difference equations to determine steady two dimensional temperatures at
different nodes in the chimney T1 to T10 (ii) temperatures at the nodes and (iii) rate of heat
transfer for 1 m height of the chimney

Assumptions: (i) Two dimensional steady state conduction; (ii) Thermal conductivity of the
chimney is constant; (iii) No radiation losses from the outer surface of the chimney to the
surroundings

Solution: Finite difference equation for node 1: The volume element surrounding node 1 is
shown in Fig. E5.8(1). Energy balance equation for the volume element can be written as

Axis of Symmetry (Insulated)


1 2

y/2
hi,Ti

y
x/2

Fig. E5.8(1): Volume element surrounding node 1

Q5-1 + Q2-1 + Qconvection = 0

Or k 1 + k 1 + hi 1(Ti T1) = 0

Substituting y = x and simplifying we get

T1 + T2 + T5 = [ Ti


[2 +

. .
. .
[2+ ]* T1 + T2 + T5 = [ ]*280

Or 7.357T1 + T2 + T5 = 1500 (a)

Finite difference equation for node 1: The volume element surrounding node 2 is shown in
Fig. E5.8(2). Energy balance equation for the volume element can be written as

207
xAxis of Symmetry (Insulated)
1 2

y/2
ho,To

y
x/2

Fig. E5.8(1): Volume element surrounding node 2

Q1-2 + Q6-2 + Qconvection = 0

k 1 + k 1 + ho 1(To T2) = 0


Or

T1 [2 + T2 + T6 =
Substituting y = x and simplifying we get


To
Substituting for the given quantities we have

. .
. .
T1 [ 2 + ]* T2 + T6 = [ ]*15

Or T1 7.357T2 + T6 = 110.355 (b)

Finite difference equation for node 3: The volume element surrounding node 3 is shown in
Fig. E5.8(3). Energy balance equation for the volume element can be written as follows.

hi,Ti
3 4

y/2

y
x/2

7
Axis of Symmetry (Insulated)

Fig. E5.8(3): Volume element surrounding node 3

Q7-3 + Q4-3 + Qconvection = 0

208
k 1 + k 1 + hi 1 =0


Or

Substituting y = x and simplifying we get

T7 [2 + T3 + T4 =


Ti

Substituting for the given quantities we have

. .
. .
T7 [ 2 + ]* T3 + T4 = [ ]*15

Or 7.357T3 + T4 + T7 = 110.355 (c)

Finite difference equation for node 4: The volume element surrounding node 4 is shown in
Fig. E5.8(4). Energy balance equation for the volume element can be written as follows.

x hi , Ti
4
3 5
y/2

x/2 y/2
8

Fig.E5.8(4): Volume elementsurrounding node 4

Q3-4 + Q8-4 + Q5-4 + Qconvection = 0

Or k 1 + k (x *1) + k 1

+ hi(x*1)(Ti T4) = 0

Substituting y = x and simplifying we get

T3 [4 + 2hi ] T4 + 2T8 = 2hi Ti


Substituting for the given quantities we have


. .
. .
T3 [ 4 + ]* T4 + 2T8 = * 280

Or T3 14.714T4 + 2T8 = 3000 (d)

209
Finite difference equation for node 5: The volume element surrounding node 5 is shown in
Fig. E5.8(5). Energy balance equation for the volume element can be written as follows.

hi,Ti

y/2
4 5 6

y/2

x/2 x/2

Fig.5.8(5): Volume elementsurrounding node 5

Q4-5 + Q1-5 + Q6-5 + Q9-5 + Qconvection = 0

k 1 + k 1 + k 1


Or

+ k 1 + hi + 1 = 0


Substituting y = x and simplifying we get

T4 [6 + 2hi (x/k)]T5 + T1 +2T6 + 2T9 = 2hi (x/k)Ti

Substituting for the given quantities we have

T4 [ 6 + 2*75* 0.1/1.4]T5+ T1+ 2T6 + 2T9 = 2*75* (0.1/1.4)* 280

Or T1 + T4 10.714T5 + 2T6 + 2T9 = 3000 (e)

Finite difference equation for node 6:Volume element surrounding node 6 with the

210
nomenclature is shown in Fig. E5.8(6).Energy balance equation for the volume element can
be written as follows:

x
y ho, To

5 6

10

Fig. E5.8(6) Volume element surrounding node 6

Q5-6 + Q3-6 + Q10-6 + Qconvection = 0

Or k (y*1) + k 1 + k 1

+ ho(y *1) (To T6) = 0

Substituting y = x and simplifying we get

T3 + 2T5 [4 + 2ho(x/k)]T6 + T10 = 2 ho To


Substituting for the given quantities we have

T3 + 2T5 [4 + 2* 18*(0.1/1.4)]T6 + T10 = 2* 18*(0.1/1.4)* 15

Or T3 + 2T5 6.571T6 + T10 = 38.571 (f)

Finite difference equation for node 7:Volume element surrounding node 7 with the
nomenclature is shown in Fig. E5.8(7).Energy balance equation for the volume element can
be written as follows:

Q3-7 + Q8-7 + Qconvection = 0

k 1 + k 1 + ho 1(To T7) = 0


Or

211
Axis of symmetry
3
x
y

7 8

ho, To

Fig. E5.8(7) Volume element surrounding node 7

Substituting y = x and simplifying we get

T3 [2 + hox / k] T7 + T8 = ho(x / k) To

Substituting for the given quantities we have

T3 [2 + 18* 0.1 / 1.4] T7 + T8 = (18* 0.1 / 1.4) * 15

Or T3 3.286T7 + T8 = 19.286 (g)

Finite difference equation for node 8:Volume element surrounding node 8 with the
nomenclature is shown in Fig. E5.8(8).Energy balance equation for the volume element can
be written as follows:

4
x

7 8 9
ho,To

Fig. E5.8(8) Volume element surrounding node 8

Or k 1 + k (x *1) + k 1

Q7-8 + Q4-8 + Q9-8 + Qconvection = 0

212
+ ho (x *1)(To T8) = 0

Substituting y = x and simplifying we get

T7 [4+ 2ho (x/k)]T8 + 2T4 = 2ho (x/k)To

Substituting for the given quantities we have

T7 [4 + 2*18* 0.1 / 1.4] T8 + 2T4 = 2*(18*0.1 / 1.4) * 15

Or T7 6.571T8 + 2T4 = 38.571 (h)

Finite difference equation for node 9:Volume element surrounding node 9 with the
nomenclature is shown in Fig. E5.8(9).Energy balance equation for the volume element can
be written as follows:

5
x

8 9 10

ho, To

Fig. E5.8(9) Volume element surrounding node 9

Q8-9 + Q5-9 + Q10-9 + Qconvection = 0

k 1 + k (x *1) + k 1


Or

+ ho (x *1)(To T9) = 0

Substituting y = x and simplifying we get

2T5 + T8 [4 + 2ho (x/k)]T9 + T10 = 2ho (x/k)To

Substituting for the given quantities we have

213
2T5 + T8 [4 + 2*18* 0.1 / 1.4] T9 + T10 = 2*(18*0.1 / 1.4) * 15

Or 2T5 + T8 6.571T9 + T10 = 38.571 (i)

Finite difference equation for node 10:Volume element surrounding node 10 with the
nomenclature is shown in Fig. E5.8(10).Energy balance equation for the volume element can
be written as follows:

9 10

ho, To

Fig. E5.8(10) Volume element surrounding node 10

Q9-10 + Q6-10 + Qconvection = 0

k 1 + k 1 + ho +

1[To T10] = 0


Or

Substituting y = x and simplifying we get

T9 [2 +2 hox /k] T10 + T6 = 2hox /k T0

Substituting for the given quantities we have

T6 + T8 [2 + 2*18* 0.1 / 1.4] T10 + T9 = 2*(18*0.1 / 1.4) * 15

Or T6 4.571T10 + T9 = 38.571 (j)

Equations (a) to (j) form a system of 10 linear equations and are listed below:

7.357T1 + T2 + T5 = 1500 (a)


T1 7.357T2 + T6 = 110.355 (b)
7.357T3 + T4 + T7 = 110.355 (c)

214
T3 14.714T4 + 2T8 = 3000 (d)
T1 + T4 10.714T5 + 2T6 + 2T9 = 3000 (e)
T3 + 2T5 6.571T6 + T10 = 38.571 (f)
T3 3.286T7 + T8 = 19.286 (g)
T7 6.571T8 + 2T4 = 38.571 (h)
2T5 + T8 6.571T9 + T10 = 38.571 (i)
T6 4.571T10 + T9 = 38.571 (j)

The above system of equations are written in matrix form as follows:

[ai,j]{Ti} = {ci}

5.6. Finite Difference Formulation For Two-dimensional Steady State Conduction in r


and z directions in Cylinders with Energy Generation

215
5.6.1.Finite difference equations for intermediate nodes: The cylinder is divided into small
sub regions as shown in Fig. 5.13 (a).The volume element surrounding the node i,j is shown

z ir

jz

i,j+1
i+1,j
i,j
i-1,j

i, j-1

Fig.5.13(a): Nomenclature for finite difference formulation for a cylinder

separately in Fig.5.13(b). The volume element is in the form of a circular ring of thickness z
in the z-direction and of radial thickness r in the radial direction as shown in the
figure.Energy balance equation for this volume element can be written as follows:

Q(i,j-1)-(i,j) + Q(i+1,j)-(i,j) + Q(i,j+1)-(i,j) + Q(i-1,j)-(i,j) + Qg = 0

Or k*2 ir*r + k*2 (ir + r/2)z


, , , ,

+ k*2 ir*r + k*2 (ir r/2)z + 2ir*r*z ,


, , , ,

=0

for I = 1,2,3 .M-1 and j = 1,2,3N-1

Assuming z = r and simplifying we get

Ti,j-1 + Ti,j+1 + Ti-1,j + + Ti+1,j 4Ti,j = ,




(5.34)

216
for i = 1,2,3 .M-1 and j = 1,2,3N-1

(j+1)z

jz
(j-1)z (i,j)
(i+1,j)

(i,j-1) (i,j+1) r

(i+1)rz

(i-1,j)
ir

Axis of the cylinder


(i-1)r

Fig.5.13(b): Nomenclature for the volume element surrounding node i,j

5.6.2.Finite difference equations for intermediate nodes on the axis of the


cylinder:Fig.5.13(c) shows a volume element surrounding the node (0,j) on the axis of the
cylinder.

(1,j)

z
r
Axis of cylinder

(0, j-1) (0,j) (0,j+1)


z

Fig.5.13(c): Nomenclature for the volume element surrounding an


intermediate node (0,j) on the axis of the cylinder
Energy balance equation for the volume element can be written as follows:

217
Q(0,j-1) (0,j) + Q(1,j) (0,j) + Q(0,j+1) (0,j)+ Qg = 0 for j = 1,2,3,,N-1

k (r/2)2 + k* 2 (r/2) z + k (r/2)2


, , , , , ,

+ (r/2)2z ,

=0

Assuming z = r and simplifying we get

, + , + 4, 6, = , j = 1,2,3N-1


(5.35)

5.6.3.Finite difference equations for intermediate nodes on the outer curved surface of the
cylinder subjected to convection boundary condition:Fig.5.13(d) shows a volume element
surrounding the node (M,j) on the outer surface of the cylinder subjected to the convection
boundary condition.Energy balance equation for this volume element can be written as
follows:

hM, T
jz

M,j-1 M,j M,j+1

r/2

z r

M-1,j

Mr Outer surface of the cylinder

Axis of the cylinder

Fig.5.13(d): Nomenclature for the volume element surrounding node (M,j)

Q(M,j-1) (M,j) + Q(M,j+ 1) (M,j) + Q(M-1,j) (M,j) + Qg + Qconvection = 0 for j = 1,2,3,N-1

218

k + k
, , , ,


+ k*2 z + z,
, ,

+ 2 M r z hM (T TM,j) = 0

Assuming z = r and simplifying we get

+ TM,j =

,
TM,j-1 + TM,j+1[4

j = 1,2,3,N-1(5.36)

5.6.4.Finite difference equations for intermediate nodes on the outer curved surface of the
cylinder subjected to prescribed heat flux condition: Fig.5.13(e) shows a volume element
surrounding the node (M,j) on the outer surface of the cylinder subjected to the convection
boundary condition.Energy balance equation for this volume element can be written as
follows:
(qs)M,j

jz

M,j-1 M,j M,j+1

r/2

z r

M-1,j

Mr Outer surface of the cylinder

Axis of the cylinder

Fig.5.13(e): Nomenclature for the volume element surrounding node (M,j)


Q(M,j-1) (M,j) + Q(M,j+ 1) (M,j) + Q(M-1,j) (M,j) + Qg + Qs = 0 for j = 1,2,3,N-1

219

k + k
, , , ,


+ k*2 z + z,
, ,

+ 2 M r z (qs)M,j = 0

Assuming z = r and simplifying we get

,


TM,j =

,
TM,j-1 + TM,j+1 4

j = 1,2,3,N-1 (5.37)

5.6.5.Finite difference equations for intermediate nodes on the outer flat surface of the
cylinder subjected to convection boundary condition: Fig.5.13(f) shows a volume element
surrounding the node (i,0) on the flat surface at z = 0 of the cylinder subjected to the
convection boundary condition. Energy balance equation for this volume element can be
written as follows

i+1,0

h0,T r

i,0 i,1

i-1,0 ir

Fig. 5.13(f): Volume element surrounding the node i-1,0

Q(i+1, 0) (i,0) + Q(i,1) (i,0) + Q(i-1,0) - (i,0) + Qconvection + Qg = 0 for i = 1,2,3,,M-1

220

) + k +
, , , ,

k*2 (ir +


) + h0 + (T Ti,0)
, ,

+ k*2 (ir


+ + ,


=0

Assuming z = r and simplifying we get

+ Ti+1,0 + Ti1,0 + 2 Ti,1 [2 + Ti,0 = ,




T

i = 1,2,3,M-1 (5.38)

Similarly theenergy balance equation for the volume element surrounding the node i,N [See
Fig.5.13(g)] can be written as follows:

+ Ti+1,N + Ti1,N + 2 Ti,N-1 [2 + Ti,N = q




T
,

i = 1,2,3,M-1 (5.39)

i+1,N

hN,T
i,N-1 i,N

i-1,N ir

Fig. 5.13(g): Volume element surrounding the node i-1,N

221
5.6.6.Finite difference equations for volume elements surrounding the nodes (0,0) and
(0,N): The nomenclature for the volume elements surrounding the nodes (0,0) and (0,N) are
shown in Fig. 5.13(h). The energy balance equation for the volume element surrounding the
node (0,0) can be written as:

z h0,T hN,T z
(1,0) (1,N)

r r

(0,0) (0,1) (0,N-1) (0,N)

Axis of symmetry

Fig.5.13(k): Nomenclature for volume elements surrounding nodes (0,0) and (0,N)

Q(1,0) (0,0) + Q(0,1) (0,0) + Qconvection + Qg = 0

k*2 (r/2)(z/2) + k* (r/2)2 + h0(r/2)2[T T0,0]


, , , ,


+ (r/2)2(z/2), =0

Assuming z = r and simplifying we get

2T1,0 + T0,1 [4 + (h0r/k)] T0,0 = (h0r/k)] T (r2/2k)


, (5.40)

Similarly it can be shown that the energy balance equation for the volume element
surrounding the node (0,N) is given by

2T1,N + T0,N-1 [4 + (hNr/k)]T0,N = (hNr/k)] T (r2/2k)


, (5.41)

5.6.7.Finite difference equations for volume elements surrounding the nodes (M,0) and
(M,N):
The nomenclature for the volume elements surrounding the nodes (M,0) and (M,N) are
shown in Fig. 5.13(k). The energy balance equation for the volume element surrounding the
node (M,0) can be written as:

222
hM,T

(M,0) (M,1) (M,N-1) (M,N)

r r

zz

(M-1,0) Mr (M-1,N)

Fig.5.13(k): Nomenclature for volume elements surrounding nodes (M,0) and (M,N)

Q(M,1) (M,0) + Q(M-1,0) (M,0) + Qconvection + Qg = 0

k [(Mr)2 (Mr r/2)2] + k 2 (Mr r/2)(z/2)


, , , ,

+ hM {[(Mr)2 (Mr r/2)2] + 2Mr(z/2)}(T TM,0)



+ [(Mr)2 (Mr r/2)2] (z/2), =0

Assuming z = r and simplifying we get

2 M(r/2) [TM,1 TM,0] + 1 Mr [TM-1,0 TM,0]



+ hMMr(r/k) [2 ] [T TM,0] + 2 Mr ,



=0

Or TM,1 + TM-1,0 + TM,0



= + T


, (5.42)

223
Similarly for the volume element surrounding the node (M,N) the energy balance equation
reduces to

TM,N-1 + TM-1,N + TM,N



= + T


, (5.43)

5.7. Finite Difference Formulation For Two-dimensional Steady State Conduction in r


and directions in Cylinders with Energy Generation

5.7.1.Finite difference equations for intermediate nodes: The cylinder is divided into small
sub regions and the volume element surrounding the node i,j will be as shown in Fig.5.14.

(i,j+1)

(i+1,j)

(i,j)
(i-1,j)

(i, j-1)

ir r

Fig.5.14: Nomenclature for the volume element surrounding an intermediate node (i,j)

Energy balance equation for the volume element surrounding the node (i,j) can be written as
follows:

Q(i-1,j) (i,j) + Q(i+1,j) (i,j) + Q(i,j+1) (i,j) + Q(i,j-1) (i,j) +


,
=0

for i = 1,2,3. M and j = 1,2,3,.N

224
+ k (ir + ) + k [r*1]
, , , , , ,

Or k [(ir )*1]

+ (ir*r*1)
, ,
,
+ k [r*1] =0


] Ti+1,j + Ti,j-1 + Ti,j+1

Or [1 ] Ti1,j + [1 +




[2 + ] Ti,j =
,

for i = 1,2,3. M and j = 1,2,3,.N (5.44)

5.7.2 Finite difference equations for innermost node (0,0): The volume element
surrounding the innermost node (0,0) will be as shown in Fig.5.15.Energy balance equation


(1,j)

(0,0)

Fig.5.15: Nomenclature for volume element surrounding the node (0,0)

for the volume element surrounding the node (0,0) can be written as follows:

Q(1,j) -(1,j) + Q
,
=0 for j = 1,2,3, ..N-1

k 1 + 1 q

, ,
,
Or = 0 for j = 1,2,3, ..N-1

225



,
Or T1,j T0,0 = for j = 1,2,3, ..N-1 (5.45)
5.7.3.Finite difference equations for nodes on the outer surface with convective boundary
condition:The volume element surrounding a node (M,j) on the outer surface of the cylinder
will be as shown in Fig.5.16.Energy balance equation for this element can be written as
follows:

(M,j+1)
h,T

(M,j)
(M-1,j)

(M,j-1)
Mr

Fig.5.16: Nomenclature for volume element surrounding the node (M,j)

Q(M-1,j) (M,j) + Q(M,j+1) (M,j) + Q(M,j1) (M,j) + Qconvection + (Qg)M,j = 0for j = 1,2,3,.N

+ k ( * 1) +k ( * 1)
, , , , , ,

Or k { (Mr )*1}

)* *1 q


,
+ h (Mr*1)(T TM,j) + (Mr = 0 for j =1,2,3,.N

Simplifying we get

TM-1,j + + + + TM,j
, ,

= T


,
for j=1,2,3,..N (5.46)

Example 5.9.A a heating element in the form of a short solid cylinder of radius 2 cm and
length 6 cm is generating heat at a uniform rate of 2*106 W/m3. The thermal conductivity of
the heating element may be taken as 50 W/(m-K). The element is exposed to the surroundings
at a uniform temperature of 300C with a surface heat transfer coefficient of 200 W/(m2-
k).Assuming two dimensional steady state conduction in r and z direction (i) obtain the finite

226
difference equations to determine the steady state temperature in the cylinder assuming r =
z = 1 cm (ii) solve the system of equations thus obtained for temperature distribution in the
cylinder.Assume that the circular faces of the cylinder are insulated.

Sketch for example5.9.


r
Symmetry lines
8 9 10 11
h,T

4 5 6 7 R Insulated
r
0123z

,
Known: L = 6 cm; R = 2 cm; k = 50 W/(m-K) ; h = 200 W/(m2-K); r = L = 1 cm

= 2*106 W/m3; T = 30 0C.

Find: (i) Finite difference equations to determine the temperatures at discrete points in the
cylinder;(ii Temperatures from the finite difference equations thus formed

Assumptions: (i) two dimensional steady state conduction in r and z directions only
(ii)Thermal conductivity and the heat generation are constants.(iii) Heat transfer coefficients
from all the outer surfaces (the curved surface and the flat surface)are same.

Solution: Since there are two symmetry lines as shown in the figure only one fourth of the
cylinder need be considered for finite difference formulation .Since r = z = 1 cmthere will
be 12 nodes as shown in the figure above.

Finite difference Equations for node 0:

Lines of symmetry (Insulated surfaces)


4

0 1
z

The nomenclature for the volume element surrounding node 0 is shown in the figure
above.Energy balance equation for this volume element can be written as

227
Q4-0 + Q1-0 + Qg = 0

+ k (r/2)2 + (r/2)2)(z/2) = 0


Or k 2 (r/2)(z/2)

Putting z = r and simplifying we get





4(T4 T0) + 2(T1 T0) =

Substituting for r,k and

we get

.

6T0 + 2T1 + 4T4 = *2*10 6

Or 3T0 + T1 + 2T4 = 2 (1)

Finite difference Equations for node 3: The volume element surrounding the node 3 is
shown in figure below.Energy balance equation for the volume element surrounding node 3
can be written as

7 Insulated
Axis of symmetry
r

2 3

Q7-3 + Q2-3 + Qg = 0

Or k 2 (r/2)(z/2) + k (r/2)2 + (r/2)2)(z/2)


= 0

Putting z = r and simplifying we get

T23T3 + 2T7 = 2 (2)

Finite Difference Equations for nodes 1 and 2: The volume elements surrounding nodes 1
and 3 are shown below.

5 6
z
r

0 1 2 1 2 3
Axis of symmetry

228
Energy balance equation surrounding the node 1 can be written as

Q0-1 + Q5-1 + Q2-1 + Qg = 0



k (r/2)2 + k 2 (r/2)z + k (r/2)2 + (r/2)2z =0

Substituting z = r and simplifying we have





T0 6T1+ T2 + 4T5 =

Substituting the values for r,k and

we get

.

T0 6T1 + T2 + 4T5 = * 2* 106

Or T0 6T1 + T2 + 4T5 = 4 (3)

Energy equation for the element surrounding node 2 is

Q1-2 + Q6-2 + Q3-2 + Qg = 0

k (r/2)2 + + k 2 (r/2)z + k (r/2)2




+ (r/2)2z

=0

Substituting z = r and simplifying we have





T1 + 4T6 + T3 6 T2 =

Substituting the values for r,k, and

we get

.

T1 + 4T6 + T3 6T2 = * 2* 106

Or T1 6T2 + T3+ 4T6 = 4 (4)

Finite Difference Equations for node 4:The volume element surrounding the node 4 is
shown in the figure below.Energy balance equation for the element is

Q0-4 + Q8-4 + Q5-4 + Qg = 0

229
Or k 2 (r/2) (z/2) + k 2 (r +r/2) (z/2) +


k +


+ +


(z/2)

=0

Substituting z = r and simplifying we have

Symmetry line (Insulated boundary)

4 5

z
0





T0 8T4 + 3T8 =


Substituting the values for r,k, and

we get

T0 8T4 + 3T8 = 8 (5)

Finite Difference Equations for nodes 5 and 6:Both these nodes are internal nodes and
hence the heat transfer across the boundary surfaces of these nodes are only by
conduction.The volume elements surrounding node 5 is shown below.

9
5

4 6

r
z
1
Fig.: Volume element surrounding node 5

230
Energy balance equation for the volume element surrounding node 5 can be written as

Q4-5 + Q9-5 + Q6-5 + Q1-5 = 0


Or k + + k 2 + z


+ k + + k 2 z


+ +

=0

Substituting z = r and simplifying we get





T1 + 2T4 8T5 + 2T6 + 3T9 =


Substituting the values for r,k, and

and simplifying we get

T1 + 2T4 8T5 + 2T6 + 3T9 = 8 (6)

Similarly the energy balance equation for node 6 can be obtained as

T2 + 2T5 8T6 + 2T7 + 3T10 = 8 (7)

Finite Difference Equations for node 7: The volume element surrounding node 7 is shown
below.The energy balance equation is

11

r Insulated

6 7

Fig. Volume element surrounding node 7

231
Q6-7 + Q11-7 + Q3-7+ (Qg)7 = 0


k + + k 2 (r +r/2) (z/2) +


Or

+


k 2 (r/2) (z/2)

+ +
(z/2)

=0

Substituting z = r and simplifying we get





T3 + 4T68 T7 + 3T11 =


Substituting the values for r,k,h,T and and simplifying we get

T3 + 4T6 8T7 + 3T11 = 8 (8)

Finite Difference Equations for node 8:The volume element surrounding the node 8 is
shown in the figure below. Energy balance equation for the element is given by

8 h,T 9

z
Symmetry line

Fig. Volume element surrounding node 8

Q4-8 + Q9-8 + Qconvection + (Qg)8 = 0


Or
+ k 2 +


k 2 (r+ r/2)z/2)


h{2 2r (z/2)}(T T8)+ 2
z/2

=0

232
Substituting z = r and simplifying we get





6 T4 [13 + 8 (hr/k)]T8 + 7T9 = 8 (hr/k)T


Substituting the values for r,k,h,T and

and simplifying we get

6T4 13.32 T8 + 7 T9 = 37.6 (9)

Finite Difference Equations for nodes 9 and 10 : The volume element surrounding node 9
is shown in figure below.Energy balance equation for the volume element shown is

h,T

8 9 10

r
z
5
Axis of cylinder

Q8-9 + Q5-9 + Q10-9 + Qconvection + (Qg)9 = 0

+ k 2(r + r/2)z +

Or


k {(2r)2 (r + r/2)2}

+ h2(r + r/2)z (T T9)




k {(2r)2 (r + r/2)2}


+ {(2r)2 (r + r/2)2}z

=0

Substituting z = r and simplifying we get



12 T5 + 7T8 [26 + (12hr/k)]T9 + 7T10 = (12hr/k)T 7 (r2/k)


Substituting the values for r,k,h,T and

and simplifying we get

12 T5 + 7T8 26.48 T9 + 7T10 = 42.4 (10)

The volume element surrounding node 10 is shown in the figure below.

233
h,T

9 10 11

r
z
6
Axis of cylinder

It can be shown that the energy balance equation for this volume element will be

12 T6 + 7T9 26.48 T10 + 7T11 = 42.4 (11)

Finite Difference Equations for node 11: The volume element surrounding node 11 is
shown in the figure below. The energy balance equation for this element is as follows
h,T
10 11 Insulated

7
Fig. Volume element surrounding node 11

Q10-11 + Q7-11 + Qconvection + (Qg)11 = 0

Or

k [(r + r/2)2 (r/2)2] + k 2 (r + r/2)z



+ h 2(2r)(z/2)(T T11)

234

+ [(r + r/2)2 (r/2)2](z/2)

=0

Substituting z = r and simplifying we get



3T7 + 2T10 [5 + 2hr/k]T11 = 2(hr/k)T (r2/k)


Substituting the values for r,k,h,T and

and simplifying we get

3T7 + 2T10 5.24T11 = 6.4 (12)

Equations (1) to (12) form a system of linear equations which can be solved by using any of
the standard techniques.

5.8. Finite Difference Formulation for One Dimensional Transient Conduction

In steady state conduction problems the solution obtained is valid for any time since under
steady state conditions, the temperatures in the solid do not change with time. But in transient
conduction problems the temperatures change with time as well as with position. Therefore
the finite difference solution of transient problems requires discretization in time in addition
to discretization in space as shown in Fig.5.17 for one dimensional transient conduction
problem.The discretization in time domain is done by selecting a suitable time step t and
solving for unknown nodal temperatures for each time step t until the solution at the desired
time is obtained.In a transient conduction problem a superscript (say k) is used to indicate
the index or counter of time steps with k = 0 corresponding to the specified initial condition.
Thus Tik represents the temperature at the node i after time steps kt have elapsed.The
energy balance equation for transient conduction involves one additional term representing
the change in the energy content of the solid with time, Hence this equation for a volume
element for time duration of t can be written as follows:

{Net amount of heat transfer into the volume element across all its surfaces during time
interval t} + {Heat generated within the element during time interval t}

= {The amount of increase of the energy content of the element during the time
interval t}.

235
k +1 k+1 k+1
T Ti-1 Ti Ti+1

k +1
t
k

k 1k
Ti+1
k
Tik
Ti-1

tx x
x
0 1 2 i1 i i +1

Fig. 5.17: Finite difference formulation of transient one dimensional conduction


problem

Or t * Q + t *[Qg]element = Eelement (5.44)


All sides

Where the rate of heat transfer Q normally consists of conduction terms for interior nodes,
but may involve convection, surface heat flux and radiation for boundary nodes.
Noting that Eelement = VCp T, where is the density,V is the volume and Cp is the
specific heat of the element and dividing Eq. (5.44) by t we have

Q + [Qg]element = V Cp (5.45)
All sides

Eq.(5.45) is applicable to volume element surrounding any node i in the domain.T


represents the change in the temperature of the volume element surrounding the node i for a
time step of t and this can be written as [Tik+1 Tik]. Hence Eq. (5.45) reduces to



Q + [Qg]element = V Cp (5.45)
All sides

The term Q involves temperatures at adjacent nodes i 1 and i +1 and question arises as
All sides

236
whether to use these temperatures at previous time step k or the new time step k+1. Both
these approaches are used in practice. The approach using the previous time step is called the
explicit method where as the method using the new time step is called the implicit method
Therefore Eq.(5.45) takes the following forms for the two methods:



ExplicitMethod: Qi + [Qg]ielement = V Cp (5.46)
All sides



Implicit Method: Qi+1 + [Qg]i+1element = V Cp (5.47)
All sides

The explicit and implicit formulations given in Eqs.(5.46) and (5.47) are quite general and
can be used in any coordinate system and is applicable to multi dimensional transient
conduction problems also except that the volume element in multi dimensional problems will
have more surfaces and thus involve more terms in the summation.
The explicit method is easy to implement but imposes a limit on the allowable time step
in order to avoid instabilities in the solution and the implicit method requires the nodal
temperatures to be solved simultaneously for each time step but imposes no limit on the
magnitude of the .time step

5.8.1.Explicit Method for Transient conduction:

(a) Explicit Method for One-dimensional Transient conduction in a Plane Wall:

generation per unit volume equal to (x,t). In generalthe heat generation may wary with
Consider unsteady one dimensional conduction in a plane wall of thickness L with heat

time and position. Let the conductivity of the wall be a constant equal to k. The wall is
subdivided into M small regions so that the thickness of each such small region is x =
L/M.For any internal small regions shown in Fig.5 18 the energy balance equation per unit
time can be written as follows:

Qi-1,I Qi+1,i

x
0 1 2 3 i-1 i i+1 M-1 M

237
Fig. 5.18: Nomenclature for the volume element surrounding an internal
node i


Qi-1,i+ Qi+1,i + (Qg)i =

k Ax + k Ax + Ax = AxCp






Or

The above equation simplifies to

= + + +



; i = 1,2,3,..M-1 (5.48)


Where = (5.49)

is the Fourier number for the element under consideration with characteristic length x.

Eq. (5.48) is called the explicit scheme because the temperature at any node i at the present
time step (k+1) is explicitly expressed in terms of the temperatures at the previous time step
k.

Restriction on (Stability Criterion):The explicit method, though easy to use has an


undesirable feature which restricts its utility. The method is not unconditionally stable and

restriction is that in Eq.(5.48) 0 i.e.


there is a restriction on the value of the parameter if the scheme has to be stable.The

0< (5.50)

This restriction implies that for given values of and x, the time step t cannot exceed the
limit imposed on it by Eq. (5.50). This is called the stability criterion. It can be shown

satisfied if the coefficients of all in the expressions for are greater than or equal to
mathematically or by invoking second law of thermodynamics that the stability criterion is

zero for all nodes.If this stability criterion is violated then the solution becomes unstable as
shown in Fig.5.19.It is instructive to determine this stability criterion by the following

Let at any time step k the temperatures


and

physical argument.

but less than at node i between them. Then if the value of exceeds , the
at nodes i-1 and i+1 are equal

coefficient1 2 becomes negative. Then, according to Eq.(5.48),for negative value of


1 2, the temperature at node i at the next time step will be less than that at the
neighbouring two nodes. This is not possible according to the second law of

238
thermodynamics, since we had assumed that higher than that at neighboring nodes. Hence
to obtain meaningful solution it is necessary that should not be negative.

Finite difference solution with >

Finite difference solution with

Exact solution

Fig. 5.19: Effects of the parameter on stability of


finite difference solution

Finite Difference Equations for Nodes on Boundaries: Eq. (5.48) can be used for all
internal nodes (i = 1,2,3,M-1).If the boundary temperatures are specified then we
have M-1 equations to solve for the temperatures at M-1 nodes.If the boundaries are
subjected to convection or prescribed surface heat flux then we have to develop the finite
difference equations for the boundary nodes also to determine the boundary temperatures.The
time dependent energy balance equation for the volume element surrounding the node 0 (see
Fig.5.20) subjected to convection can be written as follows:

h0, T
0 1
x/2

Fig.5.20: Nomenclature for the volume element surrounding boundary node 0


Q1-0 + Qconvection+ =

239

kA + h0A + A(x/2) = A(x/2) Cp


Or

Solving for we get


= + 2 + 2



T + (5.51)

Similarly for the other boundary node M subjected to convection boundary condition the
finite difference equation can be written as


= + 2



+ 2 T + (5.52)

For stability of the scheme the coefficients of T and T



should be 0. i.e.

0 0


and


/
Or (5.53a)


/
and (5.53b)

Example 5.10.A marble slab[k = 2W/(m 0C) and = 1*10 6 m2/s],2 cm thick is initially at
a uniform temperature of 200 0C. Suddenly one of its surfaces is lowered to 00C and is
maintained at that temperature, while the other surface is kept insulated. Develop an explicit
finite difference scheme to determine the temperature distribution in the slab as a function of
position after 80 seconds have elapsed by dividing the slab into 5 sub-regions and also find
the heat flux at the boundary surface which is maintained at 00C after 80 seconds.

Sketch for example 5.10x

Insulated T5

0
0 1 2 3 4 5 x

Known: L = 2=0.02 m; k = 2 W/(m-K); = 1* 10 6 m 2/s; T5 = 00C; M = 5;T5 = 00C

time steps; (ii)


Find: (i) Finite difference equations to determine temperatures at nodes 0 to 4at different

240
Assumptions: (i)solid is having constant thermal properties;(ii) one dimensional conduction
across the thickness of the slab

(5.48) and (5.49) with = 0 and M = 5. With these conditions we have


Solution: The finite difference equations for internal nodes from 1 to 4 are given by Eq.

=

+

+ 1 2 i = 1,2,3,and 4.

Therefore for node 1 we have = + + 1 2 (a)

For i = 2 = + + 1 2 (b)

For i = 3 = + + 1 2 (c)

For i = 4 = + + 1 2 (d)

Since the surface at x = 0 is insulated then

= 0 for all k.

Therfore we have = (e)

Now x = L/M = 0.02/5 = 0.004 m ;

For stability it follows that 1 2 0.


Choosing 1 2 = 0 we have = =

.

Or t = = = 8 s.

With = Eqs. (a) to (e) reduce to

= 0.5* + (i)

= 0.5* + (ii)

= 0.5* + (iii)

= 0.5* + (iv)

T= T(v)

241
Eqs. (i) to (v) are used to calculate the temperatures at different nodes for each time step as
shown in the table below :

Table E5.10 Finite difference calculations of temperatures for example 5.10

k t (s) i=0 1 2 3 4 5
x(m) = 0 00.004 0.008 0.012 0.016 0.020
0 0 200 200 200 200 200 200
1 8 200 200 200 200 200 0.00
2 16 200 200 200 200 100 0.00
3 24 200 200 200 150 100 0.00
4 32 200 200 175 150 75 0.00
5 40 187.5 187.5 175 125 75 0.00
6 48 181.2 181.2 156.25 125 62.5 0.00
7 56 168.7 168.7 153.1 109.4 62.5 0.00
8 64 160.9 160.9 139.1 107.8 54.7 0.00
9 72 150 150 134.4 96.9 53.9 0.00
10 80 142.2 142.2 123.5 94.2 48.5 0.00

Heat flux at the boundary surface at x = L after t = 80 s i.e after 10 time steps is given by

=k =2* = 24,250W/m 2
. .
.

Example 5.11. A large uranium plate of thickness 4 cm, thermal conductivity 28 W/(m-K)
and thermal diffusivity of 12.5* 10 6 m2/s is initially at a uniform temperature of 200
0
C.Heat is generated at a constant rate of 5*10 6 W/m3.Suddenly at time t = 0, one side of
the plate is brought into contact with iced water and is maintained at 0 0C at all times, while
the other side is subjected to convection to an environment at 30 0C with a heat transfer
coefficient of 45 W/(m2-K). Considering a total of three equally spaced nodes, two at the
boundaries and one at the middle of the plate, estimate the temperature distribution in the
plate 150 s after the start of the cooling process using explicit method.

Sketch for example 5.11

Node 1
L

T0 h,T

0 2 x

x/2
x

242
T = 300C;h2 = 45 W/(m 2-K); = 5 * 10 6 W/m3;
Known: L = 4 cm =0.04 m; k = 28 W/(m-K); = 12.5 * 10 6 m 2/s; T0 = 00C; M = 3;

Find: (i) Finite difference equations to determine temperatures at nodes 1 and 2 after time
150 s

Assumptions: (i)the plate is having constant thermal properties and constant rate of heat
generation (ii) one dimensional conduction across the thickness of the plate

Solution: The finite difference equations for internal node 1 is given by Eq. (5.48) and
(5.49) with i = 1 and M = 2. With these conditions we have

= + + 1 2 +


(a)


Where =

The finite difference equation for boundary node 2 is given by Eq.(5.52) with M = 2.


T= 1 2 2 T + 2T + 2


i.e. T + (b)

.

x = L / M = 0.04/2 = 0.02 m ; = = 0.032

From Eq.(a) for stability of the difference scheme 1 2 0 or 0.5 (c)

And from Eq.(b) for stability 1 2 2 0




.
Or (1 - 2 2*0.032 ) 0 i.e. i.e. 0.484 (d)

Therefore we have to choose a value of which satisfies both Eqs. (c) and (d)

. .
.
Let = 0.484. Then t = = 15.5 s.

Any time step 15.5 s can be used. Since we want the temperature after 150 s it will be
convenient to choose t = 15 s so that we get the required temperatures at nodes 1 and 2 after
10 time steps.Therefore

.
.
= = 0.46875

Substituting this value of in Eqs. (a) and (b) we get

= 0.46875* + + 1 2 0.46875 + 0.46875*


.

243
Or T = 0.0625T + 0.46875 T + 33,482 (i)

and T = 0.9375T + 0.032366 T + 34,386 (ii)

The temperatures at nodes 1 and 2 at each time step are calculated using the above two
equations and substituting the temperatures at the previous time step. The results are shown
in Table E5.11

Table E5.11 Finite difference calculations of temperatures in 0C for example 5.11

k t (s) i=0 1 2
x(m) = 0 0.02 0.04
0 0 0 200 200
1 15 0 139.7 228.4
2 30 0 149.3 172.8
3 45 0 123.8 179.9
4 60 0 125.6 156.3
5 75 0 114.6 157.1
6 90 0 114.3 146.9
7 105 0 109.5 146.3
8 120 0 108.9 141.8
9 135 0 106.7 141.1
10 150 0 106.3 139.0

244
5.8.1(b).Explicit Method for One-dimensional Transient Radial conduction in a cylinder

(i) Finite difference Equations for internal nodes i (i = 1,2,3, M-1):

The energy balance equation for the volume element surrounding node i (see Fig. 5.5) for a
time step t can be written as follows.


Qi-1,i+ Qi+1,i + (Qg)i =

Or
Tki1 Tki
k 2 1 + 2 1

r

k
Tki+1 Ti
+ k 2 + 1 = *2ir2*1*Cp


r


i = 1,2,3M-1

After simplification the above equation reduces to

= + + + +


i = 1,2,3M-1
where = and = k/(Cp ) (5.54)

For stability of numerical solution (1 - 2) 0 or (5.55)

(ii) Finite difference Equations for innermost node 0:

Referring to Fig. 5.5, the energy balance equation for node 0 can be written as follows:


k 2 1 + q
*1 = Cp 1



After simplification the above equation reduces to

= 4 +
+


(5.56)

For stability of numerical solution (1 - 4) 0 or (5.57)

(iii) Finite difference Equation for outermost node M when outer surface is subjected to
prescribed heat flux: The nomenclature for the volume element surrounding the outer most node M
is shown in Fig. 5.21.Energy balance equation for the volume element can be written as follows:

245
M qs
M-1

Mr

Fig.5.21: Nomenclature for volume element surrounding the node M with


prescribed heat flux






k *2 [ Mr r/2] *1
+ 2 Mr*1 qs + [(Mr)2 (Mr r/2)2]*1*

= [(Mr)2 (Mr r/2)2]*1Cp[ / t

After simplification the above equation reduces to




= + +



+ (5.58)



For stability,
0

(5.59)

(iii) Finite difference Equation for outermost node M when outer surface is in contact with a
fluid at a uniform temperature T with a surface heat transfer coefficient h (convective
boundary condition):

The nomenclature for the volume element surrounding the outer most node M is shown in Fig.
5.22.Energy balance equation for the volume element can be written as follows:

+ 2 Mr*1*h [T T






k *2 [ Mr r/2] *1 ]


+ [(Mr)2 (Mr r/2)2]*1*

= [(Mr)2 (Mr r/2)2]*1Cp[ / t

246
After simplification the above equation reduces to

M h,T
M-1

Mr

Fig.5.21: Nomenclature for volume element surrounding the node M subjected


to convective condition


= + +



+

(5.60)


For stability,


0

(5.61)

Example 5.12.A long chrome steel cylinder[k= 40 W/(m-K); = 1.1 * 10 5m2/s] 4 cm in


diameter is initially at a uniform temperature of 1000C. It is suddenly subjected to a
convection environment at 500C with a surface heat transfer coefficient of 400 W/(m2- K).
Assuming one dimensional radial unsteady state conduction determine the radial
temperature distribution in the cylinder 9 seconds after the exposure to the environment
using explicit finite difference scheme.

Schematic :The cylinder is divided equally as shown in the figure with r = R/2, where

r2 h,T

r/2 1

0 R
.

247
R is the radius of the cylinder.

R = 0.02 m; r = 0.02/2 = 0.01 m; M = 2; Initial temperatures at the three nodes, =


Known: k = 40 W/(m-K); = 1.1 * 10 5 m2/s;

= = 100 0C; T = 50 0C;h = 400 W/(m2-K) ; Length of cylinder = 1 m

Find: Temperatures at the nodes after 9 seconds.

Assumptions: (i) Constant physical properties for aluminium; (ii) Radial unsteady state
conduction

Solution:Finite difference equation for the internal node 1:

The finite difference equation for the internal node 1 is given by Eq.(5.54) with i = 1.

i.e., T = 0.5 T + 1.5 T + 1 2T .(1)


where = .

For stability (2)

Finite difference equation for the innermost node 0:

The finite difference equation for the internal node 0 is given by Eq.(5.54) with = 0.

i.e. T = 4T+ 1 4T(3)

For stability (4)

Finite difference equation for the outermost node 2:

The finite difference equation for the outermost node 2 is given by Eq.(5.60) with = 0.


i.e., T = 4 T + 1 4 T



+

Or T = 1.714 T + 1 1.714 T +
. .
. .

Or T = 1.714 T + [ 1 1.943]T + .(5)



For stability 1 1.943 0 Or . .(6)

248
The numerical value of has to be selected such that all the three stability conditions viz.,
Eqs.(2),(4) and (6) are satisfied. The condition will satisfy all the three stability
condtions.Hence choose : i.e. 0.25.

. .
.
Then 0.25 or t

Or t 2.27 s

=

.
Selecting t = 2.25 s, = 0.247 no of time steps required = =4

With the value of = 0.247 the nodal equations (1), (3) and (5) reduce to

T = 0.012 T +0.988T (a)

T = 0.123T + 0.506T + 0.37T (b)

And T = 0.423T + 0.52T + 2.82 (c)

The temperatures at the three nodes at different time steps are calculated using Eqs. (a), (b)
and (c) and are shown in the table below.

Time step (k) t (s) t = kt (s) Temperature at different nodes (0C)


T0 T1 T2
0 2.25 0 100 100 100
1 2.25 2.25 100 99.9 97.10
2 2.25 4.50 99.9 98.78 95.57
3 2.25 6.75 98.79 97.63 94.30
4 2.25 9.00 97.64 96.44 93.15

249
5.8.2.Implicit Method for Transient conduction

In the explicit finite difference scheme, the temperature at any node at time (t + t) is
calculated from knowledge of temperatures at the same and neighboring nodes for the
preceding time t. Hence determination of temperature at a node at some time is independent
of temperatures at other nodes for the same time. This method, though offers computational
convenience, it suffers from the limitations on the time step t. For a given x, the time step
is limited by the stability requirements as illustrated in examples 5.10 and 5.11. This dictates
the use of smaller t resulting in large number of time steps to determine the transient
tenmpeartures of the nodes after certain time.By employing an implicit scheme the restriction
on t can be completely eliminated.In the implicit scheme the space derivatives in finite
difference form are expressed using temperatures of the all the nodes at time t+t, while the
backward finite difference form of the derivative of temeperature iwthrespect to time is
used.

Refering to Fig 5.18, for a plane wall the energy balance equation in implicit form for any
internal node can be written as

k Ax + k Ax + Ax = AxCp





i = 1,2,3.. M 1

The above equation simplifies to

+
= +




i = 1,2,3.. M 1

Similar equations for the boundary nodes 0 and M can be obtained depending on the
boundary conditions specified.

250
Chapter 6

Basic Concepts of Convective Heat Transfer


6.1. Definition of Convective Heat Transfer:- When a fluid flows over a body or inside a
channel and if the temperatures of the fluid and the solid surfaceare different, heat transfer
will take place between the solid surface and the fluid due to the macroscopic motion of the
fluid relative to the surface. This mechanism of heat transfer is called as convective heat
transfer.If the fluid motion is due to an external force (by using a pump or a compressor)
the heat transfer is referred to as forced convection. If the fluid motion is due to a force
generated in the fluid due to buoyancy effects resulting from density difference (density
difference may be caused due to temperature difference in the fluid) then the mechanism of
heat transfer is called as natural or free convection.For example, a hot plate suspended
vertically in quiescent air causes a motion of air layer adjacent to the plate surface because
the temperature gradient in the air gives rise to a density gradient which in turn sets up the air
motion.

6.2. Heat Transfer Coefficient:- In engineering application, to simplify the heat transfer
calculations between a hot surface say at temperature Tw and a cold fluid flowing over it at a
bulk temperature T as shown in Fig. 6.1 a term called heat transfer coefficient,h is
defined by the equation

q = h(Tw T)..6.1(a)

where q is the heat flux (expressed in W / m2) from the surface to the flowing fluid.
Alternatively if the surface temperature is lower than the flowing fluid then the heat transfer
takes place from the hot fluid to the cold surface and the heat flux is given by

q = h(T Tw)..6.1(b)

The heat flux in this case takes place from the fluid to the cold surface.If in equations 6.1(a)
and 6.1(b) the heat flux is expressed in W / m2, then the units of heat transfer coefficient will
be W /(m 2 K) or W / (m 2 0 C).

The heat transfer coefficient is found to vary with (i) the geometry of
the body, (ii) the type of flow (laminar or turbulent), (iii) the transport properties of the fluid
(density, viscosity,specific heat and thermal conductivity),(iv) the difference in temperature
between the average temperature of the fluid and the surface with which the fluid is in
contact , (v) the position along the surface of the body, and (vi) whether the heat transfer is
by forced convection or free convection. For convection problems involving simple
geometries like flow over a flat plate or flow inside a circular tube, the heat transfer
coefficient can be

251
u, T T

Fluid Temperature Profile


y

x Tw

Fig. 6.1: Temperature distribution of the fluid at any x for Tw> T

determined analytically. But for flow over complex configurations, experimental / numerical
approach is used to determine h.There is a wide difference in the range of values of h for
various applications.Typical values of heat transfer coefficients encountered in some
applications are given in Table 6.1.

Table 6.1: Typical Values of heat transfer coefficients

Type of flow h [W /(m2 K) ]

Free convection air 5 15

-----do --------- oil 25 60

-----do--------- water 400 800

Forced Convection air 15 300

-------do------------ oil 50 1700

-------do----------- water 300 12000

Boiling water 3000 55000

Condensing steam 5500 120000

6.3. Basic concepts for flow over a body:- When a fluid flows over a body, the velocity and
temperature distribution at the vicinity of the surface of the body strongly influence the heat
transfer by convection.By introducing the concept of boundary layers (velocity boundary
layer and thermal boundary layer) the analysis of convective heat transfer can be simplified.

252
6.3.1.Velocity Boundary Layer:-Consider the flow of a fluid over a flat plate as shown in
Fig. 6.2 The fluid just before it approaches the leading edge of the plate has a velocity u
which is parallel to the plate surface.As the fluid moves in x-direction along the plate,

u
y

u(x, y)
(x)
u(x, y)

x
xcr Turbulent Region
Laminar Region Transition

Fig. 6.2: Velocity boundary layer for flow over a flat plate

those fluid particles that makes contact with the plate surface will have the same velocity as
that of the plate. Therefore if the plate is stationary, then the fluid layer sticking to the plate
surface will have zero velocity (This is called no-slip condition).But far away from the plate
(y = ) the fluid will have the velocity u.Therefore starting from the plate surface (y = 0)
there will be retardation of the fluid in x-direction component of velocity u(x,y).This
retardation effect is reduced as we move away from the plate surface.At distances sufficiently
long from the plate(y = ) the retardation effect is completely reduced: i.e. u u as y .
This means that there is a region surrounding the plate surface where the fluid velocity
changes from zero at the surface to the velocity u at the outer edge of the region. This region
is called the velocity boundary layer. The variation of the x-component of velocity u(x,y)
with respect to y at a particular location along the plate is shown in Fig. 6.2.The distance
measured normal to the surface from the plate surface to the point at which the fluid attains
99% of u is called velocity boundary layer thickness and denoted by (x)
Thus for flow over a flat plate, the flow field can be divided into two distinct regions,
namely, (i) the boundary layer region in which the axial component of velocity u(x,y) varies
rapidly with y with the result the velocity gradient (u /y) and hence the shear stress are
very large and (ii) the potential flow region which is outside the boundary layer region,
where the velocity gradients and shear stresses are negligible.

The flow in the boundary layer, starting from the leading edge of the
plate will be initially laminar in which the fluid particles move along a stream line in an
orderly manner. In the laminar region the retardation effect is due to the viscosity of the fluid
and therefore the shear stress can be evaluated using Newtons law of viscocity.The laminar

253
flow continues along the plate until a critical distance xcr is reached. After this the small
disturbances in the flow begin to grow and fluid fluctuations begin to develop. This
characterizes the end of the laminar flow region and the beginning of transition from laminar
to turbulent boundary layer. A dimensionless parameter called Reynolds number is used to
characterize the flow as laminar or turbulent. For flow over a flat plate the Reynolds number
is defined as


Rex = .6.2

where u = free-stream velocity of the fluid, x = distance from the leading edge of the plate
and = kinematic viscosity of the fluid.

For flow over a flat plate it has been found that the transition from
laminar flow to turbulent flow takes place when the Reynolds number is 5 x 10 5.This
number is called as the critical Reynolds number Recr for flow over a flat plate.Therefore

= 5 x 105 .6.3

Recr=

The critical Reynolds number is strongly dependent on the surface roughness and the
turbulence level of the free stream fluid. For example, with very large disturbances in the free
stream, the transition from laminar flow to turbulent flow may begin at Rex as low as 1 x 10 5
and for flows which are free from disturbances and if the plate surface is smooth transition
may not take place until a Reynolds number of 1 x 10 6 is reached.But it has been found that
for flow over a flat plate the boundary layer is always turbulent for Rex 4 x 10 6.In the
turbulent boundary layer next to the wall there is a very thin layer called the viscous sub-
layer, where the flow retains its viscous flow character. Next to the viscous sub-layer is a
region called buffer layer in which the effect of fluid viscosity is of the same order of
magnitude as that of turbulence and the mean velocity rapidly increases with the distance
from the plate surface.Next to the buffer layer is the turbulent layer in which there is large
scale turbulence and the velocity changes relatively little with distance.

Boundary Layer Concept for flow over a Curved Body: Fig. 6.3 illustrates the boundary
layer growth for flow over a curved body.Here x coordinate is measured along the curved
surface of the body. Starting from the stagnation point, and at each location along x, the y
coordinate is measured normal to the surface of the body.The free stream velocity U, in this
case is not a constant as in the case of flow over a flat plate but is a function of x. The

Fig. 6.3: Boundary layer growth for flow over a curved body

254
boundary layer thickness (x) increases with the distance x along the surface. But, because of
the curvature of the body, after some distance, the velocity profile u(x,y) exhibits a point of
inflexion; that is at this location (u / y) becomes zero at the wall surface. Beyond the point
of inflexion, reversal of flow takes place, and the boundary layer is said to be detached from
the surface.Beyond the point of flow reversal the flow become very complicated and the boundary
layer analysis is no longer valid.

6.3.2. Drag coefficient and Drag force:-If the velocity distribution u(x,y) in the boundary
layer at any x is known then the viscous shear stress at the wall can be determined using
Newtons law of viscosity. Thus the wall-shear stress,w(x) at any location x can be written
as :

w(x) =


.6.4

where is the absolute viscosity of the fluid.The drag coefficient is dimensionless wall shear
stress. Therefore the local drag coefficient, Cx at any x is defined as


Cx =

.6.5

Substituting for w(x) in the above equation from Eq. 6.4 and simplifying we get

2 (u / y)y = 0
Cx = ---------------------- .6.6
u 2

Therefore if the velocity profile u(x,y) at any x is known then the local drag coefficient Cx at
that location can be determined from Eq. 6.6.The average value of Cx for a total length L of
the plate can be determined from the equation
L
Cav = (1/L) Cx dx 6.7
0

Substituting for Cx from Eq. 5.5 we have


L
w(x)dx
Cav = ------------------------
L (1/2) u2

_
w
Or Cav = -------------------- .6.8
(1/2) u2

255
_
Where w is the average wall-shear stress for total length L of the plate.

The total drag force experienced by the fluid due to the presence of the plate can be written
as
_
FD = As w .6.9

Where As is the total area of contact between the fluid and the plate. If W is the width of the
plate then As = LW if the flow is taking place on one side of the plate and As = 2LW if the
flow is on both sides of the plate.

6.3.3.Thermal boundary layer:-Similar to the velocity boundary layer one can visualize the
development of a thermal boundary layer when a fluid flows over a flat plate with the
temperature of the plate being different from that of the free stream fluid.Consider that a fluid
at a uniform temperature T flows over a flat plate which is maintained at a uniform
temperature Tw.Let T(x,y) is the temperature of the fluid at any location in the flow field.Let
the dimensionless temperature of the fluid (x,y) be defined as
T(x,y) Tw
(x,y) = ------------------- .6.10
T Tw

The fluid layer sticking to the plate surface will have the same temperature as the plate
surface [T(x,y)y = 0 = Tw] and therefore (x,y) = 0 at y = 0.Far away from the plate the fluid
temperature is T and hence (x,y) 1 as y . Therefore at each location x along the
plate one can visualize a location y = t(x) in the flow field at which (x,y) = 0.99.t(x) is
called the thermal boundary layer thickness as shown in Fig. 6.3. The locus of such
points at which (x,y) = 0.99 is called the edge of the thermal boundary layer. The relative
thickness of the thermal boundary layer t(x) and the velocity

256
boundary layer (x) depends on a dimensionless number called Prandtl number of the
fluid.It is denoted by Pr and is defined as
Cp (/)
Pr = --------- = ---------- = -------- ..6.11
k (k/Cp)

Where is the absolute viscosity of the fluid, Cp is the specific heat at constant pressure

k is the thermal conductivity, is the kinematic viscosityand is the thermal diffusivity of


the fluid.The Prandtl number for fluids range from 0.01 for liquid metals to more than
100,000 for heavy oils. For fluids with Pr = 1 such as gases t(x) = (x), for fluids with Pr
<< 1,such as liquid metals, t(x) >> (x) and for fluids with Pr >> 1, like oils t(x) << (x).

6.3.4. General expression for heat transfer coefficient:-Let us assume that Tw> T.Then
heat is transferred from the plate to the fluid flowing over the plate.Therefore at any x the
heat flux is given by
q = k (T /y)y=0 ..6.12(a)

In terms of the local heat transfer coefficient hx, the heat flux can also be written as

q = hx (Tw T) ..6.12(b)

From equations 5.12(a) and 5.12(b) it follows that


k (T /y)y=0
hx = ------------------ 6.13
(Tw T)

From equation 6.10 we have (T /y)y=0 = [T Tw] ( /y)y=0. Substituting this expression
in Eq.6.13 and simplifying we get the general expression for hx as

hx = k ( /y)y=0 .6.14

The same expression for hx could be obtained even when Tw < T. Equation 6.14 can be used
to determine the local heat transfer coefficient for flow over a flat plate if the dimensionless
temperature profile (x,y) is known.

Average heat transfer coefficient:- For a total length L of the plate the average heat transfer
coefficient is given by
L
hav = (1 /L) hxdx ..6.15a
0

Substituting for hx from Eq. 6.14 we get


L
hav = (1 /L) k ( /y)y=0 dx .6.15b

257
0

Since ( /y)y=0 at any x depends on whether the flow at that section is laminar or turbulent
the expression for hav can be written as

xcr L
hav = (1 /L) { k [( /y)y=0]laminar dx + k [( /y)y=0]turbulent dx }5.16
0
xcr

Relation Between Fluid Friction and Heat Transfer (Reynolds Colburn Analogy):
For laminar flow over a flat plate an analogy between momentum transfer and heat transfer
exists; that is, it is possible to establish a relation between the local drag coefficient, Cx and
the local Nusselt number Nux as shown below.
The excat expression for the local drag coefficient is given by

(Cx / 2 ) = 0.332 Rex .6.17a

The exact expression for the local Nusselt number, Nux is given by

Nux = 0.332 Pr 1/3 Rex ..6.17b

Another dimensionless number called STANTON NUMBER is defined as


hx
Stx = -------------- .
C p u
The expression for Stx can be written as
hx x /k Nux
Stx = -------------------------- = ---------------
( x u /) ( Cp/k) Rex Pr

Substituting for Nux from Eq. 6.17b and rearranging we get

Stx Pr 2/3 = 0.332 Rex 1/2

Or Stx Pr 2/3 = Cx / 2 ..6.18

Eq. 6.18 is also applicable to turbulent flow over a flat plate, but it does not apply to flow
through tubes.This analogy is referred to as Reynolds Colburn Analogy.

Illustrative examples on flow over a flat plate:

Example 6.1:- Assuming the transition from laminar to turbulent flow takes place at a
Reynolds number of 5 x 10 5, determine the distance from the leading edge of a flat plate at
which transition occurs for the flow of each of the following fluids with a velocity of 2 cm/s at
40 0 C.(i) Air at atmospheric pressure;(ii)Hydrogen at atmospheric pressure;(iii) water;(iv)

258
Engine oil;(v) mercury.Comment on the type of flow for the 5 fluids if the total length of the
plate is 1 m.

Solution: Data:- Recr = 5 x 10 5; u = 2 m/s ; T = 40 0 C

(i)Air at atmospheric pressure :- At 40 0 C, = 17 x 10 6 m2/s.

u xcr Recr 5 x 10 5 x 17 x 10 6
Recr = ----------- or xcr = -------------------- = ---------------------------- = 4.25 m.
u 2

(ii) Hydrogen :- For hydrogen at 40 0 C, = 117.9 x 10 6 m2/s.

5 x 10 5 x 117.9 x 10 6
Therefore xcr = ------------------------------- = 29.5 m
2

(iii) Water:- For water at 40 0 C, = 0.658 x 10 6 m2/s.

5 x 10 5 x 0.658 x 10 6
Therefore xcr = ------------------------------- = 0.1645 m
2

(iv) Engine oil:- For engine oil at 40 0 C, = 0.24 x 10 3 m2/s.

5 x 10 5 x 0.24 x 10 3
Therefore xcr = ------------------------------- = 60 m
2

(v) Mercury:- For mercury at 40 0 C, = 0.107 x 10 6 m2/s.

5 x 10 5 x 0.107 x 10 6
Therefore xcr = ------------------------------- = 0.027 m
2

Comments on the type of flow

Sl.No Type of fluid xcr xcr vs L Type of Flow

1 Air 4.25 xcr> L Flow is Laminar for entire length

2 Hydrogen 29.5 xcr>> L Flow is laminar for entire length

259
3 Water 0.1645 xcr < L Flow is partly Laminar & Partly Turbulent

4 Engine oil 60 xcr>> L Flow is laminar for entire length

5 Mercury 0.027 xcr<< L Flow is turbulent for almost entire length

Example 6.2:- An approximate expression for the velocity profile u(x,y) for laminar
boundary layer flow along a flat plate is given by

u(x, y)/u =2[y / (x)] 2[y / (x)] 3+ [y / (x)] 4

where (x) is the velocity boundary layer thickness given by the expression

(x) / x = 5.83 / (Rex)1/2

(a) Develop an expression for the local drag coefficient.


(b) Develop an expression for the average drag coefficient for a length L of the plate.
(c) Determine the drag force acting on the plate 2 m x 2 m for flow of air with a free
stream velocity of 4 m /s and a temperature of 80 0C.

Solution:- (a) The velocity profile u(x,y) is given as

u(x, y) =u {2[y / (x)] 2[y / (x)] 3+ [y / (x)] 4}

Therefore (u / y)y=0 = 2u / (x)

w(x) = (u / y)y=0 = (2u) / (x)

(2u) Rex (2u) [(ux) / ]1/2


= ----------------- = ------------------------ = 0.343(u) [u /(x )]1/2 .(1)
5.83 x 5.83 x

The local drag coefficient at any x is given by

w(x) 0.343 (u) [u /(x )]1/2


Cx = ----------------- = -------------------------------------------
(1/2) u2(1/2) u2

0.686 0.686
= ---------------------- = -----------------
{(ux) / }1/2 (Rex)

260
(b) The average drag coefficient is given by
L L
Cav = (1/L) Cx dx = (1/L) 0.686 (Rex) dx
0 0

{ 0.686 (u/) } L
= ----------------------- x dx
0
L

Or 2 x 0.686 1.372
Cav = --------------- = ------------------
(uL / ) (ReL)

(c) At 80 0 C for air = 20.76 x 10 6 m2 / s ; = 1.00 kg / m 3

u L 4x2
ReL = ----------------- = ------------------- = 3.793 x 10 5
21.09 x 10 6

1.372 1.372
Average drag coefficient = Cav = ------------------- = ------------------------ = 2.228 x 10 3
ReL0.5 (3.793 x 10 5)0.5

Drag force assuming that the flow takes place on one side of the plate is given by
_
FD = w LW = (1/2) u2 Cav LW for flow over one side of the plate

= (1/2) x 1.00 x 42 x 2.2228 x 10 3 x 2 x 2

= 0.071 N

Example 6.3:- An approximate expression for temperature profile (x,y) in the thermal
boundary layer region for flow over a flat plate is given by

(x,y) = 2y / t [y / t] 2

where the thermal boundary layer thickness t is given by


5.5
t / x = ----------------- ; Rex is the Reynolds number based on x and
Rex0.5Pr 1/3
Pr is the Prandtl number of the fluid. Develop an expression for (i) the local heat transfer
coefficient hx and (ii) the average heat transfer coefficient for total length L of the plate.

261
Solution: (i) The local heat transfer coefficient hx is given by

hx = k ( / y)|y = 0.

Now (x,y) = 2(y / t) (y / t)2

2 Rex0.5Pr 1/3
Hence ( / y)|y = 0. = 2 / t = ---------------------
5.5.x
2 k Rex0.5Pr 1/3
Or hx = ------------------------ = 0.364 (k / x) Rex0.5 Pr 1/3
5.5.x
hx x
----- = 0.364 Rex0.5Pr 1/3
K
hx x /k is a dimensionless number involving local heat transfer coefficient and is called

local Nusselt number.

(ii) The average heat transfer coefficient for a total length L of the plate is given by

L
hav = (1 / L) hx dx = (1 / L) 0.364 (k / x) Rex0.5 Pr1/3 dx
0

= (1 / L) (L 0.5 /0.5) 0.364 Pr1/3k (U / )0.5 = 0.728 (k / L) (UL / ) 0.5Pr1/3

Or hav L / k = 0.728 ReL0.5 Pr1/3

hav L / k is a dimensionless number involving the average heat transfer coefficient and is
called the average Nusselt number.

Example 6.4:- The heat transfer rate per unit width from a longitudinal section x2 x1 of a
flat plate can be expressed as q12 = h12 (x2 x1)(Ts - T), where h12 is the average heat
transfer coefficient for the section length of (x2 x1). Consider laminar flow over a flat plate
with a uniform temperature Ts. The spatial variation of the local heat transfer coefficient is of
the form hx = C x 0.5, where C is a constant.
(a) Derive an expression for h12 in terms of C,x1 and x2.
(b) Derive an expression for h12 in terms of x1, x2, and the average coefficients h1 and h2
corresponding to lengths x1 and x2 respectively.

262
Solution:
u, T q12
Ts

x1

x2

Fig. P6.4: Schematic for problem 6.4

(a) hx = C x 0.5
_ 1 x2
Therefore h12 = -------------- hx dx
(x2 x1) x1
1 x2
= ------------------- C x 0.5 dx
0
(x2 x1)

2C
= ------------------ [ x2 0.5 x1 0.5]
(x2 x1)

_ x1
(b) h1 = (1/x1) C x 0.5 dx
0

= 2C / x1
_ ___
Similarly h2 = 2C / x2

x1 __ _ 1 x2 x1
Since hxdx = x1h1, h12 = -------------- [ hxdx - hxdx ]
0 00
( x2 x1 )
_ _
_ h2x2 - h1x1{2C / x2 } {2C / x1 }
h12 = ----------------- = ----------------------------------- = 2C [ x2 x1]
x2 x1(x2 x1)

263
6.4. Basic Concepts For Flow Through Ducts :- The basic concepts developed on the
development of velocity and thermal boundary layers for flow over surfaces are also
applicable to flows at the entrance region of the ducts.

6.4.1.Velocity Boundary Layer:- Consider the flow inside a circular tube as shown in
Fig.6.5. Lat uo be the uniform velocity with which the fluid approaches the tube. As the fluid
enters the tube, a velocity boundary layer starts to develop along the wall-surface. The
velocity of the fluid layer sticking to the tube-surface will have zero velocity and the fluid
layer slightly away from the wall is retarded. As a result the velocity in the central portion of
the tube increases to satisfy the continuity equation (law of conservation of mass).The
thickness of the velocity boundary layer (z) continuously grows along the tube-surface until
it fills the entire tube. The region from the tube inlet up to little beyond the hypothetical
location where the boundary layer reaches the tube centre is called hydrodynamic entrance
region or hydrodynamically developing region and the
corresponding length is called hydrodynamic entrance length Lh. In the hydrodynamically
developing region the shape of the velocity profile changes both in axial and radial direction,
i.e., u = u(r,z). The region beyond the hydrodynamic entry length is called
Hydrodynamically developed region, because in this region the velocity profile is invariant
with distance along the tube,i.e., u = u(r).
If the boundary layer remains laminar until it fills the tube, then laminar flow will prevail
in the developed region. However if the boundary layer changes to turbulent before its
thickness reaches the tube centre, fully developed turbulent flow will prevail in the
hydrodynamically developed region. The velocity profile in the turbulent region is flatter
than the parabolic profile of laminar flow. The Reynolds number, defined as

Red = (um Dh) / (6.19)

is used as a criterion for change from laminar flow to turbulent flow. In this definition, um is
the average velocity of the fluid in the tube, Dh is the hydraulic diameter of the tube and is
the kinematic viscosity of the fluid. The hydraulic diameter is defined as

264
Hydrodynamic Entrance Hydrodynamically developed
uo Region Region
(z) r
R

Lh
z Fully developed profile
u = u(r)
Fig. 6.5: development of velocity boundary layer at entrance region of a tube

4 x Area of flow
Dh = ------------------------------ (5.20)
Wetted Perimeter

For flows through ducts it has been observed that turbulent flow prevails for

Red 2300 ..(6.21)

But this critical value is strongly dependent on the surface roughness, the inlet conditions and
the fluctuations in the flow. In general, transition may occur in the range 2000 < Red< 4000.
It is a common practice to assume a value of 2300 fro transition from laminar flow to
turbulent flow.

6.4.2. Friction Factor and Pressure Drop Relations For Hydrodynamically Developed
Laminar Flow

In engineering applications, the pressure gradient (dp / dz) associated with the flow is a
quantity of interest, because this decides the pumping power required to overcome thr
frictional losses in the pipe of a given length.
Consider a differential length dz of the tube at a distance z from the entrance and
let this length be in the fully developed region. The various forces acting on the fluid
element in the direction of flow are shown in Fig.6.6.

265
(PA)z
(pA)z + d/dz(pA) dz

w Sdz

Fig. 6.6: Forces acting on a fluid element in fully developed floe region

Resultant force in the direction of motion = Fz = (pA)z (pA)z+dz w Sdz

where S is the perimeter of the duct.


Using Taylors series expansion and neglecting higher order terms we can write

(pA)z+dz = (pA)z + d/dz(pA) dz

Therefore Fz = d/dz(pA) dz w Sdz

Rate of change of momentum in the direction of flow = 0 because the velocity u does not
vary with respect to z in the fully developed region.

Hence d/dz(pA) dz w Sdz = 0

For duct of uniform cross section A is constant. Therefore the above equation reduces to

dp/dz = w S /A .(6.22)

For laminar flow w = (du / dr)|wall. Hence Eq. (5.22) reduces to

dp S
------ = -------- (du/dr)|wall .(6.23)
dz A

Eq.(6.23) is not practical for the determination of (dp/dz), because it requires the evaluation
of the velocity gradient at the wall. Hence for engineering applications a parameter called
friction factor, f is defined as follows:

(dp/dz) Dh
f = ------------------- .(6.24a)
(um2)

266
Substituting for (dp/dz) from Eq. (6.23) we have

(S/A)(du/dr)|wall Dh
f = ---------------------------- ..(6.24b)
(um2)

For a circular tube S = Di, and A = Di2 / 4. Hence Dh = Di

Hence for a circular tube Eq. (6.24b) reduces to

8
f = ------- (du/dr)|wall (6.24c)
(um2)

Also from Eq. (5.24a) we have


( ) (um2) f
dp = ----------------- dz
Dh

Integrating the above equation over a total length L of the tube we have

p2 ( ) (um2) f L
dp = ------------------- dz
p1 Dh 0

or pressure drop = p = (p1 p2) = ( ) (L/Dh) f um2 (6.25)


.
Pumping power is given by P = V p ...(6.26)
.
where V = volume flow rate of the fluid.

6.4.3. Thermal Boundary Layer for Flow Through Tubes: In the case of temperature
distribution in flow inside a tube, it is more difficult to visualize the development of thermal
boundary layer and the existence of thermally developed region. However under certain
heating or cooling conditions such as constant wall-heat flux or constant wall-temperature it
is possible to have thermally developed region.
Consider a laminar flow inside a circular tube subjected to uniform heat flux
at the wall. Let r and z be the radial and axial coordinates respectively and T(r,z) be the
local fluid temperature. A dimensionlesstemperature (r,z) is defined as

T(r,z) Tw(z)
(r,z) = ------------------- ..(6.27a)
Tm(z) Tw(z)

where Tw(r,z) = Tube wall-temperature and Tm(z) = Bulk mean temperature of the fluid.

267
The bulk mean temperature at any cross section z is defined as follows:

(2rdr) u(r,z) Cp T(r,z) rdr u(r,z)T(r,z)


Tm(z) = --------------------------------- = ---------------------- ..(6.27b)
(2rdr) u(r,z) Cp rdr) u(r,z)

At the tube wall it is clear that (r,z) = 0 and attains some finite value at the centre of the
tube. Thus we can visualize the development of thermal boundary layer along the tube
surface as shown in Fig. 6.7.The thickness of the thermal boundary layer t continuously
grows along the tube surface until it fills the entire tube. The region from the tube inlet to the
hypothetical location where the thermal boundary layer thickness reaches the tube centre is
called the thermal entry section. In this region the shape of the dimensionless temperature
profile (r,z) changes both in axial and in radial directions. The region beyond the thermal
entry section is called as the thermally developed region, because in this region the
dimensionless temperature profile remains invariant with respect to z. That is in this region
= (r).It is difficult to explain qualitatively why should be independent of z even though
the temperature of the fluid T depends both on r and z. However it can be shown
mathematically that ,for both constant wall-heat flux and constant wall-temperature
conditions, depends only on r for large values of z.For constant wall-heat flux condition
the wall-temperature Tw(z) increases with z.

Tw(z) Tw(z) Tw(z)


Tfi

Thermally Developing Region Thermaly


Developed Region
Thermal Entrance Length Lth
= (r)

Fig. 6.7: Development of Thermal Boundary Layer In a Flow Through


A Tube Subjected to Constant Wall-Heat Flux Condition

The variation of wall-temperature and the bulk fluid temperature as we proceed along the
length of the tube for constant wall-heat flux conditions is shown in Fig. 6.8.It can be shown
that for constant wall-heat flux condition the temperature difference T between the tube
wall and the bulk fluid remains constant along the length of the tube in the thermally
developed region

268
The growth of the thermal boundary layer for constant wall-temperature conditions
is similar to that for constant wall-heat flux condition except that the wall temperature does
not vary with respect to z. Therefore thetemperature profile T(r,z) becomes flatter and flatter
as shown in Fig. 6.9 as we proceed along the length of the tube and eventually the fluid
temperature becomes equal to the wall temperature. Since the wall-temperature remains
constant and the bulk fluid temperature varies along the length the temperature difference
between the tube wall and the bulk fluid varies along the length of the tube as shown in Fig.
5.10.

Tw(z)|z=L

T = Tw(z) Tm(z) = Const


Tm(z)|z=L = Tfo

0 L

Lth Thermally developed region


z = (r)

Fig. 6.8: Variation of tube wall-temperature and bulk fluid


temperature along the length of the tube

269
Tw Tw Tw
Tw
Tfi

z
Thermally developing region Thermally developed
Thermal entrance length Lth region
Fig.6.9: Growth of thermal boundary layer for flow through a tube with
constant wall-temperature

Tw
To
Tfo

Ti

Tfi

L
0
z
Fig. 6.10: Variation of bulk fluid temperature along the length of the tube
for tube with constant wall-temperature

6.4.4. Mean Temperature Difference, Tm:If Q is the total heat transfer rate between the
fluid and the tube surface , As is the area of contact between the fluid and the surface, hm is
the average heat transfer coefficient for the total length of the tube then we can write

Q = hm As Tm .(6.28)

270
Where Tm = mean temperature difference between the tube wall and the bulk fluid.An
expression for Tm can be obtained in terms of Ti and Towhere Ti is the temperature
difference between the tube-wall and the bulk fluid at inlet and To is the temperature
difference between the tube-wall and the bulk fluid at exit as shown below.

dQ

mC
mmp Tm mCp(Tm+dTm)

dz
z

Consider a control volume consisting an elemental length dz of the tube at a distance z from
the inlet as shown in Fig.5.11. Let Tm be the bulk temperature of the fluid entering the
control volume and Tm + dTm be the bulk temperature of the fluid leaving the control volume.
Let dQ be the heat transfer rate from the tube wall to the fluid over the length dz.

Energy balance for the control volume neglecting the changes in kinetic and potential
energies in comparison with change in enthalpy gives

mCpTm + dQ = mCp(Tm+dTm) (6.29 a)

and dQ = h S dz [Tw(z) Tm(z)] (6.29b)

where S is the perimeter of the tube.From the above two equations wer have

dTmhSdz
-------------------- = -------------- (6.29c)
[Tw(z) Tm(z)] m Cp

For a tube with constant wall-heat flux condition, since the temperature difference between
the fluid and the tube surface remains constant along the length of the tube it follows that
Tm = [Tw(z)|z=0 Tfi] = [Tw(z)|z=L Tfo] (6.29d)

For a tube with constant wall-temperature condition the mean temperature difference is given
by

Tm = (Ti To) / ln (Ti / To) (6.29e)

271
6.4.5. General expression for heat transfer coefficient: Let the fluid be heated as it flows
through the tube. Then at any z the heat flux from the tube surface to the fluid is given by
Fouriers law as
qw(z) = k (T / r)|wall (6.30)

[Note that when the fluid is heated Tw> Tm so that (T / r)|wall will be positive).
If hz is the heat transfer coefficient then

qw(z) = hz [Tw(z) Tm(z)] .(6.31)

Therefore from Eq. (6.30) and (6.31) we have

k (T / r)|wall
hz = ------------------- (6.32)
[Tw(z) Tm(z)]

[T(r,z) Tw(z)]
Now (r,z) = --------------------
[Tm(z) Tw(z)]

Therefore (T / r)|wall = [Tm(z) Tw(z)] ( / r)|wall

Substituting this expression in Eq. (6.32) and simplifying we get

hz = k ( / r)|wall .(6.33)

In the thermally developed region depends only on r. Hence

hz = k (d / dr)|wall (6.34)

Since (d / dr)|wall is independent of z it follows that the heat transfer coefficient hz is


independent of z. This is true both for constant wall-temperature and constant wall-heat flux
conditions.

Illustrative Examples on Flow Through Ducts:

Example 6.5:- The velocity profile for hydrodynamically developed laminar flow inside a
circular tube of radius R is given by

u(r) = 2um[ 1 (r/R)2]

272
where um is the average velocity of the fluid in the tube.Develop an expression for the friction
factor f and express it in terms of the Reynolds number Red where Red is defined as Red =
(umD) / .

Solution:
u(r) = 2um[ 1 (r/R)2]

Therefore (du / dr) = 2um [ 0 2r /R2]

Or (du / dr)|wall = (du / dr)|r = R = 4um / R = 8um / D

Therefore (dp/dz) = (S/A)w = [(D) /(D2/4)] {(du / dr)|wall}

= (4/D)(8um / D ) = 32 um /D2

(dp/dz) D 32 um /D 64
Friction factor = f = ----------------- = --------------- = --------------
(um2) (um2) (umD / )

Or f = 64 / Red

Example 6.6:- The velocity profile u(y) for hydro dynamically developed laminar
flow between two parallel plates a distance 2L apart is given by u(y) / um = (3 / 2)
[1 (y /L) 2]where um is the mean flow velocity and the coordinate axis y is as shown
in Fig. P6.6.

2L
o x

H
Fig. P 6.6 : Schematic for problem 5.6

(a)Develop an expression for the friction factor f.


(b) Write the expression for calculating the pressure drop p over a length H of the channel.

273
y
dz

z w1

p
z p+ (dp/dz)dz
2L
w2
H

Fig. P6.6a : Schematic for problem 6.6

Solution:

Consider a fluid element of length dz at a distance z from the origin as shown in the
figure. Resultant force acting on the fluid element is given by

F = p ( 2L x 1) [p + (dp/dz) dz] (2L x 1) w1 (dz x 1) w2(dz x 1)

= 2L (dp/dz)dz (w1 + w2) dz

For fully developed flow there is no change in the momentum of the fluid in z-direction.
Hence by Newtons second law F = 0.

Therefore we have 2L (dp/dz)dz (w1 + w2) dz = 0

Or (dp/dz) = (w1 + w2)/ 2L .(1).

It is given that,
u = (3/2) um [ 1 (y/L)2 ]

Therefore (du / dy) = (3/2)(2y / L2)

By Newtons law of viscosity w1 = (du/dy)|y=L = [ 3um / L]

= (3um)/L

Similarly w2 = + (du/dy)|y=L = + [ + 3um / L] = (3um)/L

Substituting these expressions for w1 and w2 we have

(dp/dz) = [(3um)/L + (3um)/L ] / 2L

= [(3um)/ L2]

274
(a) The friction factor f is given by
(dp/dz) dh [(3um)/ L2] dh
f = --------------------- = ------------------------
(1/2) um2 (1/2) um2

4 x 2L
as ( dp/dz) = (1/2) um2 f (1/dh), dh = hyd. Diameter = -------------- = 4L
2
12 x 2 24
= ------------------- = ----------------
(umL)/ ReL

b) The total pressure drop for length H of the plate is given by

p2 H
p = p1 p2 = dp = [(3um) / L2]dz = 3 (H/L)(um / L)
p10

3 (H/L) (um2)
= --------------------- = 3 (H/L) (um2) / ReL
(umL/)

Example 6.7:-The friction factor for hydro dynamically developed laminar flow
through a circular tube is given by

f = 64 / Red ; Red = (umd) / .

Water at a mean temperature of 60 0 C and a mean velocity of 10 cm/s flows inside a tube of
1 cm ID. Calculate the pressure drop for a length of 10 m of the tube and also the
corresponding pumping power required.

Solution:
Properties of water at 60 0 C are : = 985 kg/m3 ; = 0.78 x 10 3 kg / (m s);
Mean velocity of water = um = 0.1 m/s ; Di = 0.01 m ; L = 10 m.

985 x 0.1 x 0.01


Red = (umDi ) / = ------------------------- = 2060.7 or 2061
0.78 x 10 3

Friction factor = f = 64/Red = 64 / 2061 = 0.031

0.031 x 10 x 985x (0.1)2


Pressure drop = p = f (L/Di) (1/2)um2 = ----------------------------------
2 x 0.01

275
= 152.68 N / m2.

. x (0.01)2 x 0.1
Volume flow rate = V = (Di2/4) um = ------------------------ = 7.85 x 10 6 m3/s
4
.
Pumping power = p V = 152.68 x 7.85 x 10 6 = 1198.5 x 10 6 J / s.

Example 6.8:- Engine oil [ = 0.8 x 10 4 m2/s and k = 0.14 W /(m-K)] is in


laminar flow between two parallel plates a distance 3 cm apart and subjected to a
constant heat flux of 2500 W/m2. The average heat transfer coefficient for the hydro
dynamically and thermally developed flow is given by

(hm 4L)/k = 8.235,

where 2L is the distance between the plates. Calculate the temperature difference between
the plate surface and the mean fluid temperature.

Solution:

2L = 0.03 m ; = 0.8 x 10 4 m2 / s ; k = 0.14 W /(m-K) ; q = 2500 W / m2;

(hm4L) / k = 8.235

Therefore hm = (8.235 k) / (2 x 2L)

= (8.235 x 0.14)/(2 x 0.03) = 19.215 W / (m2 K)

Temperature difference = T = q / hm = 2500 / 19.215 = 130.11 0 C.

6.5. Concepts of Turbulent Flow:

6.5.1.Introduction: Osborne Reynolds conducted experiments on flow through a transperent


circular tube and showed that at low flow rates the flow was streamlined; but as the flow rate
was increased, the streamlines become unstable and the laminar motion changed to turbulent
flow: i.e. the fluid motion become chaotic in nature and involves crosswise mixing or
eddying superimposed on the motion of the main stream.
The crosswise mixing in turbulent flow is advantageous in the sense that it assists
greatly in improvig the heat transfer, but it has the disadvantage of increasing the resistance
to flow. The flow pattern in turbulent flow is quite complex as they consist of a spectrum of
coexisting vortices which are varying in size from the larger ones to those of microscopic
dimensions. For example, in the case of turbulent flow through a tube , the size of the

276
vortices varies from a significant fraction of the tube radius at the central region of the tube
to microscopic sizes near the wall where the velocity approaches zero.
In turbulent flow, the fluid properties such as velocity, pressure, and temperature are
subject to fluctuations both with the location in the flow field and with time. Hence the
instantaneous values of these properties can be represented as a sum of a time averaged
mean part and a fluctuating part as follows:
_ _ _ _
u = u + u; v = v + v ; T = T + T ; p = p + p ..6.35
_ _ _ _
where u,v,T and p are the instantaneous values; u,v,T and p are the time-averaged values and
u,v,T and p are the fluctuations. If the temperature of a flowing fluid is recorded at a
location in the flow field and if the flow is turbulent at that location, then the plot of
temperature versus time will be as illustrated in Fig.6.11. In turbulent flow, the fluctuations
are considered superimposed on local average flow.The time averaged value of temperature,
for example, can be defined as

__ 1 t + t
T = ------ T dt6.36
t
t

T
T = Temperature Fluctuation

__
T = Time Averaged Temperature

Time,t
Fig: 6.11. Temperature fluctuations with time in turbulent flow

where t is a very small time interval which is large enough to record the turbulent
fluctuations but small for the temperature to be unaffected by the external disturbances on the
system.Similar averaging process can be defined for velocity and pressure (according to
Eq.5.36, the time average of fluctuating quantities are all equal to zero).The effects of these
fluctuations occurring in velocity and temperature in turbulent flow are to increase heat
transfer and resistance to fluid flow.

277
Consider turbulent flow along a flat plate as shown in Fig. 6.12.The y coordinate
is measured normal to the plate surface. Let u, and v be the instantaneous

u
y

u(x, y)
u(x, y)

x
xcr Turbulent Region
Turbulent Layer
Laminar Region Transition
Buffer Layer

Viscous sublayer
Fig. 6.12: Turbulent boundary layer for flow over a flat plate

values of the components of of velocities along x and y directions and T be the instantaneous
value of the temperature. Let u,v and T be the corresponding fluctuations respectively.Let
be the shear stress in x direction at a location y from the plate surface.This shear stress can
be considered as to be composed of two parts:
_
viscous shear stress resulting from the mean flow velocity u and turbulent shear stress
resulting from the velocity fluctuations u and v. Then we can write

= viscous + turbulent ..6.37a


_
where viscous = (u / y) .6.37b
___
turbulent = uv .6.37c

Here the viscous shear stress term given by Eq. 6.37b is the laminar flow contribution.
The physical significance of the turbulent shear stress term given by Eq. 6.37c is not quite
apparent,but it can be derived by introducing the instantaneous velocities u and v in the x
momentum equation (See Ref.1) and by applying the rules of averaging to terms involving
the cross products of fluctuations.
Now if q is the heat flux in the y direction, then q can be considered as composed of
two parts: the diffusive or conductive heat flux due to gradient of mean temperature and the
turbulent heat flux resulting from the temperature fluctuations T and the velocity
fluctuations v in the y direction. With this consideration, we can write

278
q = qdiffusive + qturbulent .6.38a
_
Where qdiffusive = k (T / y)...6.38b
____
qturbulent = cp vT ...6.38c.

Here the diffusive heat flux given by Eq. 6.38bis similar to that encountered in conduction
heat transfer, but the significance of turbulent heat flux given by Eq. 6.38c is not quite
apparent.It is derived by introducing the instantaneous velovities u and v and the
instantaneous temperature T in the energy equation (See Ref.1) for forced convection and
time averaging these equations.The relations for turbulent shear stress and turbulent heat flux
given by equations 6.37c and 6.38c are not useful for computational purposes unless they are
related to the mean quantities and T. A number of models heve been proposed to develop
such relations by semi-empirical hypothesis. For example the mixing length model
proposed by Prandtl and von Karman have been used extensively and successively in relating
turbulent shear stress and turbulent heat flux to the gradients of average velocity and average
temperature.

6.5.2. Prandtls Mixing Length Model: The basic idea behind this model is analogus to the
concept ofmean free path for the motion of molecules in kinetic theory of gases. The
main difference is that in turbulent flow macroscopic lumps are envisioned.That is, in
turbulent flow in the x-direction along the surface, Prandtl postulated that fluid particles at a
distance y from the wall surface coalesce into macroscopic lumps and then travel, on the
average a distance lin the direction normal to the main flow while retaining their x-direction
momentum before they are dispersed.Thus , if the slow moving lumps enter the fast-moving
layer, they act as drag on the fast moving layer, and the momentum is transferred between
layers as a result of transeverse mixing. It is to be noted that l is an unknown quantity, and in
reality there is no such clear defined distance. Though the concept lacks generality, it has
been found useful in most engineering applications.According to Prandtls theory, the
velocity fluctuations can be related to
( / y) by the following equations.

u l1( / y) ; v l2( / y) 6.39a,b

where u and v are of opposite sign and l1 and l2 are the mixing lengths for momentum
transport.Similarly the temperature fluctuation is related to the gradient of mean
tempaerature as _
T = l3 (T / y) ..6.39c

Where l3 is the mixing length for energy transport. Now the turbulent shear stress can be
written as:

___
turbulent = uv = l1l2 ( / y)2 = lm2 ( / y)2

279
or turbulent = m ( / y) ..6.40a

where l1l2 = lm2 and m = lm2 ( / y) .6.40b,c

Similarly the turbulent heat flux can be written as:


___ __
qturbulent = Tv = l3l2 ( / y)(T / y) = lh2( / y)(T / y)
__
qturbulent = h(T / y) .5.41a

where l2l3 = lh2 and h = lm2( / y) ..5.41b,c

m and h are called as eddy diffusivity for momentum transfer and eddy difussivity for
heat transfer respectively. Therefore for turbulent flow the total shear stress and the total
heat heat flux can be written as follows:

( / ) = ( + m)( / y) .6.42a
__
(q / cp) = ( + h)(T / y) ..6.42b

Where is the kinematic viscosity and is the thermal diffusivity of the fluid. Equations
6.42a and 6.42b clearly demonstrates the effects of turbulent flow in enhancing the drag as
well as the heat transfer. Depending on the level of turbulence in the flow field, the turbulent
transport properties m and h may be an order of magnitude larger than the diffusive
properties and . In general m and h are not the same. Their ratio is defined as turbulent
Prandtl number, Prt. Thus

Prt = m / h .6.43.

This definition is analogus to the definition of Prandtl number Pr:

Pr = / ..6.44.

The physical significance of Pr and Prt should be distinguished. The Prandtl number is a
physical property of the fluid.It varies from a value much less than 1 for liquid metals to
values much greater than 1 for liquds and ois. It is about 1 for gases. The turbulent Prandtl
number is a property of the flow field and various models have been proposed for its
determination. The simplest model is due to Reynolds who assumed Prt = 1, which implies
that heat and momentum transfer in turbulent flow takes place exactly by the same process.
The numerical value of Prt may vary between 1 and 2.

6.5.3.Velocity Distribution in Turbulent Flow: Velocity distribution in turbulent flow has


been investigated extensively due to its importance in practice.But no fundamental theory is
yet available to determine this distribution by purely theoretical approaches. Therefore,
empirical and semi-empirical relations are used to correlate the velocity in turbulent flow.

280
One of the velocity distribution which has been used widely is based on the concept
of dividing the turbulent flow field into three distinct layers as illustrated in Fig.5.10.A very
thin layer adjacent ot the wall where laminar or viscous shear stress dominate is called the
viscous sublayer. Adjacent to this layer is the buffer layer, where the viscous and
turbulent shear stresses are of the same order of magnitude and the third layer next to the
buffer layer is called turbulent layer, where the turbulent shear stress dominates.For the
steady, turbulent flow of an incompressible, constant property fluid over a smooth surfacethe
velocity distribution is expressed using the following dimensionless parameters:

u + = ------------ = Dimensionless velocity


(w / )

y
+
y = ------ (w / ) = Dimensionless distance

where is the density, w is the shear stress at the wall, is the kinematic viscosity and
component of mean velocity parallelto the wall surface.
Experiments have shown that for region where y + < 5, turbulent shear stress
contribution is negligible and the region is viscous sublayer.Therefore the shear stress takes
the form w = ( / y). Integration of this expression for constant w with = 0 at y = 0
gives the following distribution for viscous sublayer:

u += y + for 0 < y +< 5 6.45.

The buffer layer is considered to be in the region between y + = 5 and y + = 30. In this region
the velocity distribution is assumed to follow the logarithmic law of the form
u + = A ln y + + B. The constants A and B are determined from requirement that at
y + = 5 ,velocity u + be equal to that of viscous sublayer and be equal to that of turbulent layer
at y + = 30.Hence for the buffer region the velocity distribution will be of the form

u + = 5.0 ln y + 3.05 for 5 y + 30 . 6.46

The region y +> 30 is considered to be the turbulent layer where the laminar
shear stress is negligible in comparison to the turbulent shear stress. By using the mixing
length concept and assuming that the mixing length varies linearly with the distance from the
wall in the form l = y, it can be shown that the velocity distribution in the turbulent layer
has a logarithmic profile in the form

u + = (1/ ) ln y + + C .6.47

where is called the universal constant. Experiments have shown that = 0.4 and the
constant C has been determined by the correlation of Eq. 6.47 with the measured velocity
profile. For turbulent flow inside a smooth pipe, C = 5.5. Then the velocity distribution in the
turbulent layer is given as

281
u + = 2.5 ln y + + 5.5 for turbulent layer y + > 30 ..6.48

Though the above velocity distribution [Eq.6.45, Eq. 6.46 and Eq. 6.48] by dividing the
turbulent flow field into three distinct regions appears to be in reasonably good agreement
with the experimental data, the transition from a viscous to turbulent flow regime takes place
gradually.Therefore , he representation of of velocity distribution by three different curves
having discontinuous slopes at locations where they join is not realistis.A more serious
inconsistency of the logarithmic velocity distribution given by Eq.5.48 is that it does not give
zero velocity gradient at the tube centre. For this reason the average velocity for flow inside a
pipe as determined using the above equations overestimates the velocity.But these equations
have been used extensively in literature to study the relation between momentum and heat
transfer in turbulent flow.

6.5.4.Effects of Surface Roughness on Velocity Distribution: The velocity distribution


discussed above is applicable to turbulent flow over hydrodynamically smooth surfaces. A
surface is considered as hydrodynamically smooth if the heights of the protrusions are
much smaller than the thickness of the viscous sublayer. Surfaces encountered in engineering
applications generally are not smooth and since viscous sublayer is very thin, the protrusions
may penetrate it.Nikuradse made extensive experiments with turbulent flow inside artificially
roughned pipe over a wide range of relative roughness, /Difrom about 1 / 1000 to 1 / 30.
The sand grain roughness used in these experiments has been adopted as standard for the
effects of roughness. The experiments showed that to study the effects of roughness, it is
desirable to introduce a roughness Reynolds number + defined as


+
= ---- (w / ) .6.49

If +< 5, the roughness has no effect on the friction factor.With this consideration three
distinct situations are envisioned for the effects of roughness:

(i) Hydrodynamically smooth surface: 0 + 5 ..6.50a

(ii) Transitional: 5 + 70 .6.50b

(iii) Fully rough: +> 70 6.50c

For the hydrodynamically smooth case, the heights of roughness are so small that all
protrusions are covered by the viscous sublayer and hence roughness has no effect on friction
factor. For the transitional case, the protrusions are partly outside the viscous sublayer and
cause some additional rsistance to flow. For the fully rough case, the heights of protrusions
are so large that all protrusions penetrate the viscous sublayer; hence the viscous sublayer no
longer exists and protrusions influence the turbulent mixing.
For the fully turbulent region (y + > 30),the logarithmic velocity profile guven by Eq.
6.47 is applicable if y + is replaced by y / . In that acse the velocity distribution is given by

282
u + = (1/ ) ln (y / )+ C .6.51

where = 0.4 as in the case of smooth wall but the constant C is different. A correlation of
this relation with Nikuradses experiments has shown that C = 8.5. Then Eq. 5.51 takes the
form

u + = (1/ ) ln (y / )+ 8.5 6.52

Eq. 6.52. is called logarithmic velocity distribution for turbulent flow in rough pipes in the
fully rough region.
A significant difference between laminar and turbulent flow lies in the fact that, in
turbulent flow the velocity profile is affected by the surface roughness, where as in laminar
flow, the velocity profile is not affected by the surface roughness. Since the velocity
distribution affects the shear stress at the wall and hence the friction factor, it is expected that
in turbulent flow the friction factor will depend not only on the Reynplds number but also on
the relative roughness, where as in laminar flow the friction factor is a function of Reynolds
number only.

Illustrative Examples on Concepts of Turbulence:


Example 6.11.Starting with the definition of the total shear stress in the turbulent flow given
by ___ ___
= (u / y) uv , uv = lm2 ( / y)2 and lm = y

derive the logarithmic velocity distribution, u + = (1/ ) ln y + + C for the fully turbulent
region.
y
+ +
Where u = ------------ and y = ------ (w / )
(w / )

Example 6.12. Show that the velocity distribution in the viscous sublayer is given by

u+ = y +
+ +
with u and y are as defined in example 5.11.

283
Chapter 7

Forced Convection Heat Transfer For Flow


Through Ducts
7.1. Introduction:

Heat transfer and pressure drop calculations in flow through ducts are required in many
industrial applications. Laminar flow through pipe is encountered generally in compact heat
exchangers, cryogenic systems, heating or cooling of heavy fluids like oils.For such flows
analytic expressions are available for the prediction of friction factor and heat transfer
coefficient.But most of the available correlations to determine heat transfer and friction
factor for turbulent flows are either empirical or semi-empirical in nature.

7.2. Analysis of Hydrodynamically and Thermally Developed Laminar Flow

7.2.1. Couette Flow:Consider a special case of parallel flow of an incompressible fluid


between two parallel plates where one plate is stationary and the other plate is
moving with an uniform velocity U.A distance L separates the two plates (refer Fig.
7.1). The stationary plate is maintained at temperature To and the moving plate at
temperature TL. This type of flow is referred to as COUETTE flow and occurs, for
example, in journal bearing. The continuity, momentum and energy equations for
such a flow are given as follows:
For parallel flow along the plate the continuity equation (6.54b) reduces to

du / dx = 0 (7.1)
The momentum equation (6.59a) reduces to

d2u / dy 2 = 0 (7.2)

And the energy equation reduces to

k (d2T / dy 2) + (du / dy) 2 = 0 (7.3)

284
y
TL
y
u(y) = U

To, u(y) = 0
Fig. 7.1: Schematic for Couette Flow

The boundary conditions for the momentum equation are :

at y = 0, u(y) = 0 (7.4)

and at y = L, u(y) = U. (7.5)

Integrating equation (7.2 ) twice, we have

u(y) = C1y + C2 (7.6).

Condition (7.4) in equation (7.6) gives C2 = 0.

Condition (7.5) in equation (7.6) gives U = C1L or C1= U / L.

Substituting these values of C1 and C2 in equation (2) we get the velocity distribution as

u(y) = (Uy) / L (7.7)

From equation (7.7) (u / y) = U / L substituting this in the energy equation (7.3)

We have k(d2T / dy2) = (U/L)2

Or (d2T / dy2) = (/k)(U/L)2

Integrating twice with respect to y we get

T(y) = (/k)(U/L)2 (y2/2) + C1y + C2 .(7.8)

The boundary conditions for the enrgy equation are :

285
at y = 0 T = T0 (7.9)

and at y = L, T = TL. (7.10)

Condition (7.9) in equation (7.8) gives C2 = T0.

Condition (7.10) in equation (7..8) gives TL = (1/2)(U2/k) + C1L + T0

Or C1 = (1/L)[(TL T0) + (1/2)(U2/k)]

Substituting the expressions for C1 and C2 in Equation (7.8 ) we get the temperature as

T(y) = (/k)(U/L)2 (y2/2) + (y /L) [(TL T0) + (1/2)(U2/k)] + T0

T(y) T0 = + 1


Or (7.11)

= 1 + 1


(7.12)



Now the term can be written as follows:



= = Pr * E where Pr is the Prndtl number and E is

Eckert number. Hence Wq.(7.11) can be written as follows:

= 1 + Pr E 1


(7.13)



Fig. 7.2 shows the plot of as a function of for different values of the parameter
Pr*E.The case when Pr*E = 0 corresponds to no flow condition. Hence heat transfer from the
hot plate to cold plate is by conduction in y direction. Therefore the temperature distribution
is linear.The physical significance for other values of Pr*E can be understood better if heat
transfer at the wall (either at the upper wall or at the lower wall) is considered.The heat flux
at the upper wall is given by

q(y)| y = L = k (dT /dy)| y = L

1 Pr E


From Eq.(7.12) we have (dT /dy)| y = L =

1 Pr E


Therefore q(y)| y = L = k (7.14)

286
The above result shows that the direction of heat flow at the upper wall for the case TL >
T0 depends on the value of the parameter Pr*E.

1.0

0.8

0.6

0.4

0.2

0.2 0.4 0.6 0.8 1.0 1.2 1.4 1.6

For Pr * E > 2, the right side of Eq.(7.14) is positive. Therfore the heat flows in the
positive y direction, i.e., from the fluid into the wall though the upper wall is at higher
temperature than the lower wall.
For Pr * E < 2, the right side of Eq. (7.14) is negative. Hence the heat flows in the
negative y direction,i.e., from the upper wall to the fluid.
For Pr * E = 2, the right hand side of Eq.(7.14) reduces to zero. Hence there is no heat
flow at the upper wall.

When TL = T0, both plates are at the same temperature, Therfore from Eq. (7.11) the
temperature distribution reduces to

(7.15)

287
It can be seen from the aboe equation that the maximum temperature occurs at y = L/2
and the maximum temperature will be



Tmax T0 = (7.16)

Combining Eq. (7.15) and Eq. (7.16) the the temperature distribution can be expressed in

= 4 1

terms Tmax as


(7.17)

Illustrative examples on Couette flow:

Example7.1:Aheavy lubricating oil[ = 0.25 kg/(m-s), k = 0.125 W/(m-K)] at room


temperature flows in the clearance between a journal andits bearing. Assuming that both the
journal and the bearing are at the same temperature, determine the maximum temperature
rise in the fluid for velocity U = 6 m/s.

Schematic: See Fig.7.1

Known: For the oil, = 0.25 kg/(m-s), k = 0.125 W/(m-K); U = 6 m/s; T0 = TL

Find: Tmax T0

Assumptions: (i)The oil properties are constant; (ii) the flow of oil in the space between the
journal and its bearing is considered as flow between two parallel plates; i.e. the flow is
coquette flow.

Solution:For Couette flow when T0 = TL, the maximum temperature rise is given by
Eq.(7.16): i.e.

.
.
Tmax = Tmax T0 = = = 9 0C

Example 7.2.A lubricating oil of viscosity and thermal conductivity k fills the clearance L
between two rotating cylinders, which can be regarded as two parallel plates in motion for
the purpose of analysis. The velocity of the inner cylinder is U1 and that of the outer cylinder
is U2.Find the relation for the velocity distribution in the oil layer for (a) the inner and outer
cylinders are rotating in the same direction but U2> U1 and (b) the inner and outer cylinders
are rotatibg in the opposite directions. Also find an expression for the shear stress in the
fluid resulting from the rotation.

Schematic: Case (a) :- U2> U1 :See Fig.E7.2a

Known: Viscosity of the oil = ; Thermal conductivity of oil = k ; Clearnace between the two
rotating cylinders = L;velocity of lower plate=U1; Velocity of upper plate = U2

288
Find: (a) u(y) when both U1 and U2 are in same direction with U2> U1; (b) u(y) and shear
stress when U2 is in opposite direction with respect to U2.

Assumptions: (i) Fluid properties are constant; (ii) The flow between the two rotating
cylinders is considered as flow between two parallel plates (Couette flow)

Solution: The momentum equation for Couette flow is

d2u / dy 2 = 0 (1)

y
y
u(y) = U2

L
u(y) = U1

Fig. E7.2a: Schematic for Example 7.2,case(a)


Integrating Eq.(1) twice we have

u(y) = C1y + C2 (2)

Substituting the condition that at y = 0, u = U1 we get

C2 = U1 (3)

Substituting the condition that at y = L, u = U2 in Eq. (2) we get

U2 = C1L + U1



Or C1 = (3)

Substituting the expressions for C1 and C2 in eq. (2) we get the velocity distribution as

u(y) = U U + U1



or = (4)

= =


Shear stress is given by

289
Schematic for Case (b) :- U2 = U1 :See Fig.E7.2b

y
y
u(y) = U2

L
u(y) = U1

Fig. E7.2b: Schematic for Example 7.2,case(b)

The general solution Eq.(7.2) holds good for this case also.

For this case at y = 0, u(y) = U1, Hence from Eq.(7.2) C2 = U1

At y = L, u(y) = U2. Substituting this in Eq.(7.2) we have

U2 = C1L + U1



Or C1 =

Hence the velocity distribution is given by

u(y) = + + U1



= (5)

= =


Shear stress is given by (6)

Example 7.3.Consider Couette flow with heat transfer for which the lower plate moves with
a velocity of U = 15 m/s and is perfectly insulated (see Fig. E7.3). The upper plate is
stationary and is made of material with kup = 1.5 W/(m-K) and thickness Lup = 3 mm. Its

290
outer surface is maintained at Tup = 40 0 C. The plates are separated by a distance of L0 = 5
mm which is filled with an engine oil of viscosity = 0.8N-s/m2 and thermal conductivity k0
=0.145W/(m- K).
(a) On T(y) y coordinates, sketch the temperature distribution in the oil film and in the
moving plate.
(b) Obtain an expression for the temperature at the lower surface of the film T0 in terms of
the plate speed U, the stationary plate parameters Tup,kup,Lup and the oil parameters , k0, L0.
Calculate this temperature for the prescribed conditions.

Schematic for the example:

y Tup
Stationary Plate

Lup

oil L0

Moving plate
Fig. E7.3: Schematic for example 7.3

Solution:

For Couette flow the momentum equation is :

d2u /dy2 = 0.

Integrating twice with respect to y we have

u(y) = C1y+ C2 (1)

The boundary conditions are (i) at y = 0, u(y) = U ;

And (ii) at y = L0, u(y) = 0.

Condition (i) in equation (1) gives C2 = U and condition (ii) in equation (1) gives

C1 = U / L0.

291
Substituting the expressions for C1 and C2 in equation (1) we get the velocity distribution as

u(y) = U [1 (y/L0)] ..(2)

The energy equation for Couette flow is

k(d2T / dy2) = (du/dy)2 (3)

From equation (2) we have (du/dy) = (U / L0).

Therefore equation (3) reduces to


k0(d2T / dy2) = (U / L0)2

or (d2T / dy2) = (U / L0)2 / k0

Integrating once with respect to y we have

dT / dy = (/k0) (U / L0)2 y + C1 (4)

Integrating once again with respect to y we have

T = (/k0) (U / L0)2 (y2/2) + C1y+ C2 ...(5)

At y = 0 the surface is insulated i.e. (dT/dy) = 0. Substituting this condition in equation (4)
we have
C1= 0.
At y = L0, the condition is

k0 (dT/dy)|y=L0 = kup[ T|y = L0 Tup] /Lup(6)

From equation (4) we have (dT / dy )|y = L0 = (/k0) (U / L0)2 L0 = U2/(L0k0).

From equation (5) we have T|y = L0 = (/k0)(U2/2) + C2.

Therefore equation (6) reduces to U2/L0 = kup [(/k0)(U2/2) + C2 Tup] / Lup

Or C2 = U2 / k0 [ ( ) + ( k0Lup/ kup L0) ] + Tup

Therefore T|y = L0 = Tup + { U2 / k0 [ ( ) + ( k0Lup/ kup L0) ] } (/k0)(U2/2)

= Tup + ( k0Lup/ kup L0)

Or Temperature distribution is given by :

292
T = -(/k0) ( U/ L0)2 (y2/2) + { U2 / k0 [ ( ) + ( k0Lup/ kup L0) ] } + Tup

At lower surface y = 0

Therefore T|y =0 = Tup + { U2 / k0 [ ( ) + ( k0Lup/ kup L0) ] }

7.2.2.Hydrodynamically and thermally developed laminar flow through a circular pipe:

7.1. Determine the friction factor, the pressure drop and pumping power for fully
developed laminar flow of water at 21 0 C [ = 9.8 kg/(m-s) ; = 997.4
kg/m3]through a 2.5 cm diameter, 100 m long tube for a mass flow rate of 0.015
kg/s. What are the mean and maximum velocities of flow?

7.2. Determine the friction factor, the pressure drop and pumping power required for
the flow of water at 0.5 kg/s and 40 0 C through a tube of square cross section of
2 cm x 2 cm and 12 m long. What would be the corresponding values if the pipe
is of equilateral-triangular cross section of side 2 cm and length 5 m ?

7.3. Water at 30 0 C with a mass flow rate of 2 kg/s enters a 2.5 cm-ID tube whose
wall is maintained at a uniform temperature of 90 0 C. Calculate the length of the
tube required to heat the water to 70 0 C.

7.4. Water at 20 0 C with a mass flow rate of 5 kg/s enters a circular tube of 5 cm-ID
and 10 m long. If the tube surface is maintained at 80 0 C, determine the exit
temperature of water.

7.5. Air at 27 0 C with a flow rate of 0.01 kg/s enters a rectangular tube 0.6 cm x 1.0
cm in cross section and 2 m long. The duct wall is subjected to a uniform heat
flux of 5 kW/m2. Determine the outlet temperature of air and the duct surface
temperature at the exit assuming that the flow is hydro dynamically and
thermally developed.

7.6. Three kg/min of liquid sodium is heated from a bulk mean temperature of 400 0 C
to 500 0 C, as it flows through a stainless steel tube of 5 cm-ID and 2 mm thick.
The sodium is heated by a constant wall-heat flux, which maintains the tube-wall
temperature at 30 0 C above the bulk mean temperature of sodium all along the
length of the tube. Calculate the length of the tube required. Assume the
following properties for liquid sodium.
= 846.7 kg/m3 ; k = 68.34 W/(m-K) ; Cp = 1.274 kJ/(kg-K) ; Pr = 0.00468 ;

293
6
=0.2937 x 10 m2 /s.

7.7. Consider hydro dynamically and thermally developed turbulent flow of water with
a mass flow rate of M kg/s inside a circular tube of inside diameter D. The
Dittus-Boelter equation can be used to determine the heat transfer coefficient. If
the tubes inside diameter is changed from D to D/2 while the mass flow remains
same, determine the resulting change in the heat transfer coefficient.

7.8. Mercury at a temperature of 100 0 C and with a velocity of 1 m/s enters a 1.25
cm ID tube, which is maintained at a uniform temperature of 250 0 C. Determine
the length of the tube required to heat mercury to a temperature of 2000C.

B. Hydrodynamic and thermal entry lengths:

7.9. Determine the hydro dynamic entry lengths for flow at 60 0 C and at a rate of
0.015 kg/s of water, ethylene glycol and engine oil through a circular tube of 2.5
cm ID.

7.10. Determine the hydro dynamic entry length, thermal entry length and the heat
transfer coefficient for fully developed flow for engine oil at 60 0 C flowing at a
rate of 0.01 kg/s through a square duct 1 cm x 1cm cross section and subjected
to a uniform wall-temperature. Assume the following physical properties for the
engine oil:
= 864 kg/m3 ; Cp = 2047 J/(kg-K) ; k = 0.14 W/(m-K) ; = 0.0725 kg/(m-s) ;
Pr = 1050

C. Flow over a flat plate:

7.11. Atmospheric air at 25 0 C flows over both the surfaces of a flat plate 1 m long
with a velocity of 5 m/s. The plate is maintained at a uniform temperature of
750C.
(a) Determine the velocity boundary layer thickness, the surface shear stress and
the heat flux at the trailing edge of the plate.
(b) Determine the drag force on the plate and the total heat transfer from the plate
to air.

7.12. Air at 30 0 C flows with a velocity of 10 m/s along a flat plate 4 m long. The plate
is maintained at a uniform temperature of 130 0 C. Assuming a critical Reynolds
number of 2 x 10 5 and width of plate to be 1 m determine (a) the heat flux at
the trailing edge of the plate, (b) the heat transfer from the laminar portion of
the plate, (c) the total heat transfer from the plate and (d) the heat transfer from
the turbulent portion of the plate.

7.13. A highly conducting thin wall 2 m long separates the hot and cold air streams
flowing on both sides parallel to the plate surface. The hot stream at 250 0 C is
flowing with a velocity of 50 m/s while the cold stream at 50 0 C is flowing with a
velocity of 15 m/s. Calculate (a) the average heat transfer coefficients for both
the air streams and the heat transfer between the two streams per metre width
of the plate and (b) the local heat flux at the mid point of the plate. Assume that

294
the wall is at the arithmetic mean of the temperature of the two streams for the
purposes of calculating the physical properties of the two streams and the critical
Reynolds number to be 2 x 10 5.

7.14. A flat plate of width 1 metre is maintained at a uniform temperature of 150 0 C


by using independently controlled heat generating rectangular modules of
thickness 10 mm and length 50 mm. Each module is insulated from its
neighbours, as well as its back side.(see Fig. P 6.14). Atmospheric air at 25 0 C
flows over the plate at a velocity of 30 m/s. The thermo-physical properties of
the module are : k = 5.2 W/(m-K) ; Cp = 320 kJ/(kg-K) ; = 2300 kg / m3.

150 0 C
T = 25 0 C
u = 30 m/s

10 mm

700 mm

50 mm

Fig. P 7.14 : Schematic for problem 7.14

(a) Find the required power generation in W/m2 in a module positioned at a distance
of 700 mm from the leading edge of the plate.
(b) Find the maximum temperature in the heat generating module.

D. Flow across a cylinder:

7.15. A circular pipe of 25 mm OD is placed in an air stream at 25 0 C and 1 atm


pressure. The air moves in cross flow over the pipe at 15 /s, while the outer
surface of the pipe is maintained at 115 0 C. What is the drag force exerted on
the pipe per unit length of the pipe? What is the rate of heat transfer per unit
length of the pipe?

7.16. A long cylindrical heating element [k = 240 W/(m-K), = 2700 kg/m3 and Cp =
900 kJ/(kg-K)] of diameter 10 mm is installed in a duct in which air moves in
cross flow over the heating element at a temperature of 27 0 C with a velocity of
10 m/s.
(a) Estimate the steady state surface temperature of the heater when electrical
energy is being generated at a rate of 1000 W per metre length of the cylinder.
(b) If the heater is activated from an initial temperature of 27 0 C, estimate the time
required for the surface temperature to come to within 10 0 C of its steady state
value.

295
7.17. Air at 40 0 C flows over a long 25 mm diameter cylinder with an embedded
electrical heater. Measurements of the effect of the free stream velocity V on the
power per unit length P, required to maintain the cylinder surface temperature at
300 0 C yielded the following results:

V (m/s) : 1 2 4 8 12

P (W/m) : 450 658 983 1507 1963

(a) Determine the convection coefficient for each of the above test conditions.
Display your results graphically.
(b) For the corresponding Reynolds number range, determine the suitable constants
C and m for use with an empirical correlation of the form Num = C Redm Pr1/3.

7.18. A thermocouple is inserted into a hot air duct to measure the air temperature.
The thermocouple (T1) is soldered to the tip of a steel thermocouple well of
length 15 cm and inner and outer diameters of 5 mm and 10 mm respectively. A
second thermocouple (T2) is used to measure the duct wall temperature (see Fig.
P 6.18).

T2
Air
at T, u L Steel well
Di

Do

Fig. P 7.18 : Schematic for problem 7.18

Consider the conditions for which the air velocity in the duct u = 3 m/s and the
two thermocouples register temperatures of T1 = 450 K and T2 = 375 K.
Neglecting radiation determine the air temperature T. Assume that for steel k =
35 W/(m-K), and for air = 0.774 kg / m3, = 251 x 10 7 N-s / m2, k = 0.0373
W/(m-K), and Pr = 0.686

E. Flow across tube bundles:

7.19 Air at atmospheric pressure and 30 0 C flows over a bank of tubes consisting 1
cm OD tubes, 10 rows deep. The velocity of air before it enters the bundle is 1
m/s.
(a) Determine the friction factor and the pressure drop and (b) the average heat
transfer coefficient for the following cases:
(i) Tubes are in equilateral-triangular arrangement with ST / D = SD / D = 1.25

296
(ii) Tubes are in square arrangement with ST / D = SL / D = 1.25

7.20. Hot flue gases at 375 0 C flow across a tube bank consisting of 1.25 cm OD
tubes, which are maintained at a uniform surface temperature of 30 0 C by
flowing water through the tubes. The tube bundle is 10 rows deep in the direction
of flow and contains 40 tubes in each row. The tubes are 1 m long and have an
in-line arrangement with SL / D = ST / D = 2. he velocity of the flow gases
entering the tube matrix is 7 m/s. Determine the average heat transfer
coefficient and the total heat transfer rate. Assume that thermo-physical
properties of the flow gases to be same as that of air at any temperature.

A.Hydro-dynamically and Thermally developed flow through ducts


7.1. Solution:-

Water di
At 200C

Mass flow rate = m = 0.015 kg / s ; di = 0.025 m ; L = 100 m ;

Properties of water at 20 0C are: = 1000 kg / m3 ; = 1.006 x 10 6 m2 / s;

Reynolds Number = Red = uav dh / , where uav = average velocity of the fluid in the pipe
and dh = hydraulic diameter for the pipe.

Now m = (di2/4)uav.

4 x 0.015
2
Or uav = (4m) / (di ) = --------------------------- = 0.0305 m / s
1000 x x 0.0252

dh = di for a circular pipe.


0.0305 x 0.025
Therefore Red = ----------------------- = 757.95
1.006 x 10 6

297
Since Red< 2300, flow is laminar. For hydro-dynamically developed laminar flow we
have friction factor as

f = 64 / Red = 64 / 745.5 = 0.084.

Pressure drop for a total length L is given by p = (1/2)f (L/dh) uav2

= x 0.084 x (100 / 0.025) x 1000 x 0.03052 = 156.28 N/m2.

Pumping power = p (m/) = 156.28 x 0.015 / 1000 = 2.34 x 10 3 W

The velocity of the fluid at any radius for fully developed laminar flow through a circular
tube is given by

u(r) = 2uav [1 (r/R)2]

Therefore umax = u(r)r=0 = 2 uav = 2 x 0.0305 = 0.061 m/s

7.2. Solution

Water
At 400C a

Mass flow rate = m = 0.5 kg / s ; a = b = 0.02 m ; L = 12 m ;

Properties of water at 40 0C are: = 994.59 kg / m3 ; = 0.658 x 10 6 m2 / s;

0.5
uav = m / (ab) = -------------------- = 1.26 m / s.
994.59 x 0.022

Hydraulic diameter = dh = 4ab / 2(a + b) = 2ab / (a + b) = 2a2/2a = a = 0.02 m.

1.26 x 0.02
Reynolds number Red = uav dh / = ------------------- = 38299
0.658 x 10 6

Since Red> 2300 flow is turbulent.

For fully developed turbulent flow through a pipe of square cross section the friction factor f
is given by (Moody chart, smooth pipe)

298
f = 0.02175.

Pressure drop = p = (1/2)f (L/dh) uav2 = 0.5 x 0.02175 x (12 / 0.02) x 994.59 x 1.262

= 10303 N / m2

Pumping power = p x m / = 10303 x 0.5 / 994.59 = 5.18 W

For a tube of equilateral triangular cross section, dh = 4 {3 x a2/4} / 3a, where a is the side of
the triangle.

Hence dh = a /3 = 0.02 / 3 = 0.0115 m

0.5
Average velocity = uav = ----------------------------- = 2.9 m/s
994.59 x (3/4) x 0.02 2

2.9 x 0.0115
Reynolds number = Red = --------------------- = 50684
0.658 x 10 6

Since Red> 2300, flow is turbulent. Hence from Moody chart we have

f = 0.02

Pressure drop = p = 0.5 x 994.59 x 2.9 2 x 0.02 x (5 / 0.0115) = 36367.4 N / m2

Pumping power = 36367.4 x 0.5 / 994.59 = 18.28 W.

299
7.3.
Solution:

Tw

Tfi di Tfo

L
Data :- Tfi = 30 C ; Tfo = 70 0C ; Tw = 90 0C ; m = 2 kg / s ; di = 2.5 cm = 0.025 m.
0

To find L , assuming flow is hydrodynamically and thermally developed.

For pipe of circular cross section hydraulic diameter = dh = di = 0.025.

Bulk mean temperature of water = Tm = (Tfi + Tfo) = 0.5 x (30 + 70) = 50 0 C.

Properties of water at 50 0 C are : = 990 kg/m3 ; cp = 4181 J/kg-K ;

k =0.644 W / (m-K) ; = 0.547 x 10 3 kg / (m-s) ; Pr = 3.55

Since nothing has been specified in the problem regarding the type of flow, it is assumed that
the flow is hydro dynamically and thermally developed.

4x2
Average velocity = uav = m / (x di2/4) = ---------------------------- = 4.11 m/s.
990 x x 0.025 2

990 x 4.11 x 0.025


Reynolds number = Red = uav dh / = ----------------------------- = 1.86 x 10 5
0.547 x 10 3

Since Red>2300, flow is turbulent. For fully developed turbulent flow the Nusselt number is
given by

Nud = 0.023 Red0.8 Pr n with n = 0.4 for Tw> Tf.

Therefore Nud = 0.023 x (1.86 x 10 5)0.8 x (3.55) 0.4

= 628

300
Hence the heat transfer coefficient, h = Nud k / dh

= 628 x 0.644 / 0.025 = 16177 W/(m2-K)

To find the length of the tube L, we write the energy balance equation for the entire length of
the tube as

Heat supplied to fluid from the tube wall = Increase of energy of the fluid

Therefore h (diL) Tm = mcp(Tfo Tfi)

L = mcp(Tfo Tfi) /hdi Tm ..(1)

Where Tm = logarithmic mean temperature difference = [Ti To] / ln (Ti / To)

Ti = Tw Tfi = 90 30 = 60 0 C and To = Tw Tfo = 90 70 = 20 0 C.

Therefore Tm = [60 20] / ln(60 / 20) = 36.41 0 C.

[2 x 4181x (70 30)]


Hence L = -------------------------------- = 7.23 m.
16171 x x 0.025 x 36.41

7.4. Solution:
Tw

water Tfi di Tfo

Data :- Tfi = 20 0C ; m = 5 kg / s ; di = 0.05 m ; L = 10 m ; Tw = 80 0 C.

To find Tfo.

Since Tfois not known we cannot determine the bulk fluid mean temperature to know the
properties of the fluid. Hence this problem has to be solved by trial and error method as
shown below

Trial No. 1:- Assume suitable value for Tfo noting that Tfo< Tw.

Let Tfo = Tw = 60 0 C. Hence bulk mean temperature = Tm = (Tfi + Tfo)

301
= 0.5 x (20 + 60) = 40 0 C.

Properties of water at 40 0 C are : = 994.59 kg/m3 ; cp = 4178.4 J/kg-K ; Pr = 4.34 ;

= 0.658 x 10 6 m2 / s ; k = 0.628 W /(m-K).

4x5
2
Average velocity of water = uav = 4m /( di ) = --------------------------
x (0.05)2 x 994.59

= 2.56 m/s.

For a circular tube dh = di = 0.05 m.


2.56 x 0.05
Reynolds number = Red = uavdh / = ------------------------ = 1.945 x 10 5
0.658 x 10 6

Since Red> 2300, flow is turbulent.

Assuming the flow to be thermally and hydrodynamically developed,

Nud = 0.023 Red0.8 Pr n with n = 0.4 (as the fluid is heated)

= 0.023 x (1.945 x 10 5) 0.8 (4.34) 0.4

= 704.5
704.5 x 0.628
Heat transfer coefficient = h = Nud k / dh = ---------------------- = 8848.5 W /(m2 K).
0.05

Heat balance equation for the total length of the tube can be written as

h diL Tm = mcp[Tfo Tfi]

or h diL [Ti To]


---------------------------- = mcp [Ti To]
ln[Ti /To]

or To = Ti /exp {(h diL)/(mcp)}

[80 20]
= ----------------------------------------------------
exp{(8848.5 x x 0.05 x 10) / (5 x 4178.4)}

= 30.85 0 C.

302
Therefore Tfo = 80 30.85 = 49.15 0 C. This temperature is quite away from the
assumed value and hence one more iteration is required.

Trial 2:- Assume Tfo = 49 0C. Therefore Tm = (49+20)/2 = 34.5 0 C.

Properties of water at 34.5 0 C are : = 996.22 kg/m3; cp = 4179.3 J/kg-K; Pr = 5.077;

k = 0.6195 W/(m-K); = 0.7537 x 10 6m2/s.

4x5 2.556 x 0.05


uav = ------------------ ------ = 2.556 m/s; Red = -------------------- = 1.696 x 10 5
x 0.05 2 x 996.22 0.7537 x 10 6

Therefore Nud = 0.023 x (1.696 x 10 5) 0.8 x (5.077) 0.4 = 672.2

Hence h = 672.2 x 0.6195 / 0.05 = 8328.6 W/(m2 K)

[80 20]
Therefore Tfo = 80 ---------------------------------------------------------
exp {(8328.6 x x 0.05 x 10) / (5 x 4179.3)}
0
= 48 C which is very close to the assumed value of Tfo. Hence the
iteration can be stopped.

7.5. Solution:
qw
a

Tfi Tfo b

L
Data :- Fluid is air ; Tfi = 27 0 C ; m = 0.01 kg/s; a = 0.01 m; b = 0.006 m; L = 2 m;

qw = 5000 W / m2.

find (i) Tfo ; (ii) Tw|z = L

Energy balance equation for total length of the tube can be written as

mcp(Tfo Tfi) = qw 2(a+b)L

Or Tfo = Tfi + [qw 2(a+b)L] / (mcp)

303
Since Tfo is not known the property cp is read at Tfi.

Therefore cp = 1005.7 J/kg-K.


2 x (0.01 + 0.006) x 2 x 5000
Therefore Tfo = 27 + --------------------------------------- = 58.8 0 C.
0.01 x 1005.7

Therefore Tm = (Tfi + Tfo) = 0.5 x (27 + 58.8) = 42.9 0 C.

Properties of air at 42.9 0 C are : = 1.12 kg / m3; 1006.8 J/kg-K; = 17.30 x 10 6 m2/s;

Pr = 0.7045 ; k = 0.02745 W/(m-K).(It should be noted that the variation of cp with


temperature between Tfi and Tm is very negligible and hence this problem does not require
trial and error solution)

4ab 2ab 2 x 0.01 x 0.006


Hydraulic diameter = dh = ------------ = ------------- = ------------------------- = 0.0075 m.
2[a + b] [a + b] [0.01 + 0.006]

0.01
Average velocity = uav = m / [ab] = ------------------------- = 149 m /s.
1.12 x 0.01 x 0.006
149 x 0.0075
Reynolds number = Red = uavdh / = --------------------- = 64595
17.3 x 10 6

Since Red>2300 flow is turbulent. Assuming that the flow is hydrodynamically and thermally
developed we have
Nud = 0.023 Red 0.8Pr n, with n = 0.4 as air is being heated.

Therefore Nud = 0.023 x [64595] 0.8 x [0.7045] 0.4= 140.9

140.9 x 0.02745
Heat transfer coefficient = h = Nud k / dh = ---------------------- = 515.7 W/(m2 K).
0.0075

At the exit of the tube we have qw = h [Tw|z=l Tfo]

Therefore Tw|z=l = qw / h + Tfo = 5000 / 515.7 + 58.8 = 68.5 0 C

7.6.Solution:

304
Data: Fluid is liquid sodium; m = 3/60 =0.05 kg/s; Tfi = 400 0 C; Tfo = 500 0C;di = 0.05 m

Ti = Tm = To= 30 0 C; = 846.7 kg/m3; k = 68.34 W/(m-K); Pr = 0.00468

cp = 1274 J/kg-K; = 0.2937 x 10 6m2/s.

4 x 0.05
2
Average velocity = uav = 4m / (di ) = ------------------------ = 0.03 m/s.
846.7 x x 0.05 2

0.03 x 0.05
Reynolds number = Red = uavdh / = ----------------------- = 5107
0.2937 x 10 6

Since Red> 2300, flow is turbulent.Assuming the flow to be hydrodynamically and thermally
developed and since Pr << 1 (Liquid metal), the Nusselt number for constant wall heat flux
condition is given by

Nud = 4.82 + 0.0185 (Red Pr) 0.827

= 4.82 + 0.0185 x [5107 x 0.00468] 0.827 = 5.075

5.075 x 68.34
Heat transfer coefficient = h = Nud k / dh = ------------------------ = 6936.5 W/(m2 K).
0.05

Energy balance equation for the total length of the tube can be written as

h (diL)Tm = mcp(Tfo Tfi)

mcp(Tfo Tfi) 0.05 x 1274 x (500 400)


or L = -------------------- = ----------------------------------- = 0.195 m
h (di)Tm 6936.5 x x 0.05 x 30
7.7. Solution:

The Dittus-Boetler correlation for hydrodynamically and thermally developed flow is given
by
Nud = hdh/k = 0.023 Red0.8 Pr n ..(1)

4M
For a circular tube of diameter D, Red = uavD / = --------------
D

Hence Eq.(1) can be written as h1 D

305
----- = 0.023 [4M/( D)] 0.8 Pr n
k

Or h1 = 0.023k[4M/( )] 0.8 Pr nD 1.8.(2)

Similarly when the diameter of the tube is reduced to D/2, for the same mass flow rate the
heat transfer coefficient is given by

h2 = 0.023k[4M/( )] 0.8 Pr n(D/2) 1.8..(3)

Dividing Eq.(3) by Eq.(2) we get


1.8
h2 / h1 = 2 = 3.5 or h2 = 3.5 h1.
6.8. Solution:

Tw

di
Tfi Tfo

Data:- Fluid is mercury(Liquid metal) ; Tfi = 1000C; Tfo = 2000C;

di = dh = 0.0125m;uav = 1 m/s;Tw = 2500C;

To find L

Bulk mean temperature of mercury = Tm = (Tfi + Tfo) = 0.5 x (100 + 200) = 150 0C.

Properties of mercury at 150 0C are:- = 13230 kg/m3; cp = 137.3 J/kg-K;

= 0.0865 x 10 6 m2 / s; k = 9.65 W/(m-K) ; Pr = 0.0162

1.0 x 0.0125
Reynolds number = Red = uavdh / = ----------------------- = 1.445 x 10 5
0.0865 x 10 6

Therefore Peclet number = Pe = Red Pr= 1.445 x 10 5 x 0.0162 = 2341

306
Since Red> 2300, flow is turbulent. Therefore for liquid metal flow subjected to uniform
wall-temperature Nusselt number is given by

Nud = 5.0 + 0.025 Pe 0.8

= 5.0 + 0.025 x 2341 0.8 = 17.4

17.4 x 11.425
Heat transfer coefficient, h = Nudk/dh = -------------------- = 15904.5 W/(m2-K)
0.0125

Ti = Tw Tfi = 250 100 = 150 0 C ; To = Tw Tfo = 250 200 = 50 0 C.

Mean temperature difference = Tm = [Ti To] / ln(Ti / To)

= [150 50] / ln(150 / 50) = 91 0 C

Mass flow rate of mercury = m = (di2/4)uav = 15904.5 x ( x 0.0125 2/4) x 1.0

= 1.952 kg/s

Energy balance equation for the total length of the pipe is given by

h diL Tm = mcp(Tfo Tfi)

mcp(Tfo Tfi) 1.952 x 137.3 x (200 100)


Therefore L = ------------------ = ------------------------------------
h di Tm 15904.5 x x 0.0125 x 91

= 0.4715 m

B Hydrodynamic and Thermal Entry Lengths

6.9. Solution:
Data:- Tfi = 60 0C ; m = 0.015 kg / s; di = dh = 0.025 m.

(i) Fluid is water. Hence at 60 0 C, = 985.46 kg / m3 ; = 0.478 x 10 6 m2 / s.

4 x 0.015
Average velocity = uav = 4m / di2 = -------------------------- = 0.032 m/s.
985.46 x x 0.025 2

0.032 x 0.025
Reynolds number = Red = uavdh / = --------------------- = 1728
0.478 x 10 6

307
Since Red< 2300, flow is laminar. Hence the hydrodynamic entrance length Lh for a circular
pipe is given by

Lh / dh
-------- = 0.056
Red

Therefore Lh = 0.056 Red dh = 0.056 x 1728 x 0.025

= 2.419 m
(ii)Fluid is ethylene glycol: = 1087.66 kg/m3 ; = 4.75 x 10 6 m2/s.

4 x 0.015
Average velocity = uav = 4m / di2 = -------------------------- = 0.0281 m/s.
1087.66 x x 0.025 2

0.0281 x 0.025
Reynolds number = Red = ---------------------------- = 147.9
4.75 x 10 6

Since Red< 2300, flow is laminar.

Therefore Lh = 0.056 Red dh = 0.056 x 147.9 x 0.025

= 0.21 m.
(iii) Fluid is engine oil; = 864.04 kg/m3 ; = 0.839 x 10 4 m2/s.

4 x 0.015
Average velocity = uav = 4m / di2 = -------------------------- = 0.0354 m/s.
864.04 x x 0.025 2

0.0354 x 0.025
Reynolds number = Red = ---------------------------- = 10.55
0.839 x 10 4

Therefore Lh = 0.056 Red dh = 0.056 x 10.55 x 0.025 = 0.013 m.

6.10.Solution: Data: Fluid is engine oil ; Tfi = 60 0C; m = 0.01 kg/s;


square duct with a = 0.01m ; = 864 kg/m 3; cp = 2047 J/kg-K; k = 0.14 W/(m-K);
= 0.0725 kg/(m-s); Pr = 1050.

To find (i) Lh ;(ii)Lt ; (iii) h for fully developed flow.

(i) Hydraulic diameter = dh = 4a 2 /(4a) = a = 0.01 m.

308
0.01
2
Average velocity = uav = m / (a ) = --------------------- = 0.1157 m/s.
864 x (0.01) 2

864 x 0.1157 x 0.01


Reynolds number = Red = (uavdh) / = ------------------------- = 13.8
0.0725

Since Red< 2300, flow is laminar. Hence (Lh / dh)


---------- = 0.09 for a tube of square section.
Red

Therefore Lh = 0.09 Red dh = 0.09 x 13.8 x 0.01 = 0.0124 m.

(ii) For constant wall temperature condition we have

(Lt / dh)
---------- = 0.041
Pe

Hence Lt = 0.041 Pe dh = 0.041 x (13.8 x 1050) x 0.01 = 5.94 m

(iii)For fully developed laminar flow through a square tube Nusselt number is given by

Nu T = hdh / k = 2.976

Hence h = 2.976 k / dh = 2.976 x 0.14 / 0.01 = 41.66 W/(m2-K)

Consider the flow of water at a rate of 0.01 kg/s through an equilateral triangular duct of
sides 2 cm and whose walls are kept at a uniform temperature of 100 0C. Assuming the flow
to be hydrodynamically and thermally developed, determine the length of the tube required to
heat the water from 200C to 70 0C.

Solution:

309
Tw = 100 0C

Water
Tfo=700C
0
Tfi = 20 C
m = 0.01 kg/s

L=?

a = 0.02 m

Bulk mean temperature of water = (Tw + T) = 0.5x (20 + 70) = 45 0C.

Properties of water at bulk mean temperature are: = 992.3075 kg/m3; Pr = 4.01


= 0.598 x 10 6 m2/s; k = 0.63375 W/(m-K); cp = 4179.9J/kg-K

For an equilateral triangular tube, area of flow = A = (3 / 4)a 2 = (3 / 4) x 0.02 2

= 1.732 x 10 4 m 2

4 [(3 / 4)a 2] a 0.02


Hydraulic diameter = dh = --------------------- = ------------- = ------------ = 0.01155 m
3a 3 3
m 0.01
Average velocity of water = uav = ----------------- = -------------------------------
A 992.3075 x 1.732 x 10 4

=0.0582 m/s

0.0582 x 0.01155
Reynolds number = Red = uavdh / = -------------------------- = 1124
0.598 x 10 6

Since Red<2300 flow is laminar.For thermally developed laminar flow with constant wall-
temperature the Nusselt number is given by

Nud = hav dh / k = 2.47

2.47 x 0.63375
Therefore hav = ------------------ = 135.53 W/(m2-K)
0.01155

Mean temperature difference between the surface and the bulk fluid is given by

310
Tm = [Ti To] / ln[Ti / To]

Now Ti = Tw Tfi = 100 20 = 800C; To = Tw Tfo = 100 70 = 30

Hence Tm = [80 30] / ln{80/30} = 50.1 0C.

Rate of heat transfer to water = Q = mcp(Tfo Tfi) = 0.01 x 4179.9 x (70 20)

= 2090 W

But Q = havPL Tm, where P is the perimeter of the duct = 3a

Hence Q = hav 3aLTm

Q 2090
Or L = -------------------- = -------------------------------- = 5.13 m
hav 3aTm 135.53 x 3 x 0.02 x 50.1

C Flow over a flat plate:

6.11. Solution:

u, T
Tw

x
L

Data:- Fluid is air; u = 5 m/s; T = 25 0 C; L = 1 m; Tw = 75 0 C.

Mean temperature = (Tw + T) = 0.5 x (75 + 25) = 50 0 C.

Properties of air at 50 0 C are : = 1.093 kg/m3; = 18.02 x 10 6 m2 /s; Pr =0.703

k = 0.028 W/(m-K).

a) 1) Reynolds number at the trailing edge = ReL = (uL) / = 5 x 1 / (18.02 x 10 6)

= 2.775 x 10 5

311
Assuming the critical Reynolds number to be 5 x 10 5, the flow is laminar at the trailing edge.
Therefore from heat transfer data hand book we have

(x)|x=L = 5 L ReL 0.5

= 5x 1 x (2.775 x 10 5) 0.5

= 0.0088 m.

2) Local drag coefficient at the trailing edge is given by

Cx|x=L = 0.664 ReL 0.5 = 0.664 x (2.775 x 10 5) 0.5

= 1.26 x 10 3

Therefore w(x)|x= L = (u2) Cx|x=L = x 1.093 x 5 2 x 1.26 x 10 3

Or w(x)|x= L = 0.0173 N/m 2.

3) Local Nusselt number at the trailing edge is given by

Nux|x=L = 0.332 ReL 0.5 Pr 0.333

= 0.332 x (2.775 x 10 5) 0.5 x (0.703) 0.333

= 155.5

Hence local heat transfer coefficient at the trailing edge is given by

hx|x=L = (Nux|x=L k) / L = 155.5 x 0.028 / 1

= 4.354 W/(m2-K)

Heat flux at the trailing edge = qw|x=L = [hx|x=L] (Tw T)

= 4.354 x (75 25) = 217.7 W/(m2 K)

(4) Drag force: Fd = {w}avg .A= 2 x w(x)|x= L x 1= 2x 0.0173= 0.0346 N

Therefore average heat transfer coefficient is havg = 2hx|x=L = 4.354 x 2

=8.708 W/(m2 K) and

Average heat flus = qw= [havg] (Tw T)= 8.708 x 50 = 435.4 W/(m2 K)

312
6.12. Solution:-

u = 10 m/s (air)

T = 30 0 C Tw = 130 0 C

x
W=1 m
L=4m

To find :- (a) qw(x)|x=L; (b) Qlaminar; (c) Qtotal assuming Recr = 2 x 10 5 (d) Qturbulent

Mean film temperature = (Tw + T) = 0.5 x (130 + 30) = 80 0C.

Properties of air at mean film temperature are: =21.48 x 10 6 m2/s; Pr = 0.692


k = 0.03047 W /(m-K).

10 x 4
ReL = uL / = -------------------------- = 1.86 x 10 6
21.48 x 10 6

Since ReL> Recr flow is turbulent at the trailing edge.

For turbulent flow of air over a flat plate the local Nusselt number is given by

Nux = 0.0296 Rex 0.8 Pr 1/3.

Hence Nux|x=L = 0.0296 x [1.86 x 10 6] 0.8 (0.692) 1/3 = 2714

2714 x 0.03047
Therefore [hx|x=L] L / k = 2803 or hx|x=L = ---------------------- = 20.67 W /(m2-K)
4

Heat flux at the trailing edge = qw(x)|x=L = [hx|x=L] (Tw T)

= 20.67 x (130 30) = 2067 W / m2.

(b) Recr = u xcr / .

313
Recr 2 x 10 5 x 21.48 x 10 6
Or xcr = --------- = ------------------------------- = 0.4296 m
u 10

Hence flow is laminar up to xcr.

Average Nusselt number for the laminar region is given by

[Nuav] laminar = 0.664 Recr0.5 Pr 1/3

= 0.664 x [2 x 10 5] 0.5(0.697) 1/3 = 263.3

Hence average heat transfer coefficient for the laminar region is


263.3 x 0.03003
[hav]laminar = [Nuav] laminar k / xcr = -----------------------
0.4296
= 18.40 W /(m2 K)

Heat transfer rate from laminar portion = Qlaminar = [hav]laminar [x cr W] (Tw T)

= 18.4 x [ 0.4296 x 1] x (130 30)

= 790.5.0 W
(c)Average Nusselt number for the entire length of the plate is given by

Nuav = Pr 1/3 [ 0.037ReL0.8 A]

Where A = 0.037Recr 0.8 0.664Recr0.5.

For this problem A = 0.037 x [2 x 10 5] 0.8 0.664 x [2 x 10 5] 0.5 = 356

Therefore Nuav = (0.697) 1/3 [0.037 x {1.86 x 10 6} 0.8 356]= 3085

3085 x 0.03047
Hence hav = Nuav k / L = ----------------------- = 23.5 W/(m2 K)
4

Qtotal = hav (LW)(Tw T) = 23.5 x (4 x 1) x (130 30) = 9400 W

Qturbulent = Qtotal Qlaminar = 9400 790.5 = 8609.5 W

6.13. Solution:

314
Hot air at T1 =250 0 C, u1 = 50 m/s
Tw = (T1+T2)
= 150 0 C

L=2m

Cold air at T2 = 50 0C, u2 = 15 m / s

Additional data:- Recr = 2 x 10 5

To find (i) hav for the hot surface ; (ii) hav for the cold surface ; (iii) qw(x)|x=L/2

(i) Mean film temperature for the hot fluid = (Tw + T1) = 0.5 x (150 + 250) = 200 0C.

Properties of air at 200 0C are: = 34.85 x 10 6 m2 / s;k = 0.03931 W/(m-K);Pr = 0.68

Reynolds number at the trailing edge of the plate = ReL = u1L /

50 x 2
Or ReL = ------------------- = 2.869 x 10 6
34.85 x 10 6

Since ReL> Recr, flow is partly laminar and partly turbulent. Therefore the average Nusselt
number is given by

Nuav = Pr 1/3[0.037 ReL0.8 A]

Where A = 0.037 Recr0.8 0.664 Recr 0.5.

Or A = 0.037 x [2 x 10 5]0.8 0.664 x [2 x 10 5] 0.5 = 347.25

Hence Nuav = (0.68) 1/3 [0.037 x (2.869 x 10 6) 0.8 347.25] = 4465

4465 x 0.0391
Hence for the hot surface [hav] hot = Nuav k / L = ---------------------- = 87.29 W/(m2-K)
2

(ii) Mean film temperature for the cold surface = (150 + 50) = 100 0C.

Properties of air at the mean film temperature are: = 23.33 x 10 6 m2/s; Pr = 0.693
k = 0.03184 W/(m-K)
15 x 2
ReL = u2L / = -------------------------- = 1.286 x 10 6
23.33 x 10 6

315
Therefore Nuav = (0.693) 1/3[0.037 x (1.286 x 10 6) 0.8 347.25] = 2219

2219 x 0.03184
Hence for the cold surface [hav]cold = -------------------- = 35.33 W/(m2-K).
2

(iii) The rate of heat transfer from the hot air stream to cold air stream is given by

Q = (T1 - T2) / [Rc1 + R + Rc2]

Where Rc1 = Thermal resistance offered by hot surface for convection,

Rc2 = Thermal resistance offered by cold surface for convection,

and R = Thermal resistance offered by the plate for conduction.

1
Now Rc1 = 1 / [hav]hot A = ------------------- = 0.00573 m2 K / W.
87.29 x (2 x 1)
1
Similarly Rc2 = 1 / [hav]cold A = -------------------- = 0.01415 m2 K / W.
35.33 x (2 x 1)

R = L / Ak . Since k is not given it is assumed that k is very large i.e R = 0.

Therefore Q = [250 50] / [ 0.00573 + 0.01415] = 10,060 W.

(iv) At mid point of the plate x = L / 2.

Therefore for the hot fluid Re|x=L/2 = ReL = 0.5 x 2.865 x 10 6

= 1.4325 x 10 6 which is > Recr.

Therefore flow is turbulent at mid point of the plate.

Hence [Nu|x=L/2]hot = 0.037 [Re|x=L/2] 0.8 Pr 1/3

= 0.037 x [1.4325 x 10 6] 0.8 x (0.6815) 1/3 = 2739

Hence [hx|x=L/2]hot = 2739 x 0.03931 / 1 = 107.7 W/(m2-K).

Similarly [Nu|x=L/2]cold = 0.037 x [0.5 x 1.286 x 10 6] 0.8 x (0.693) 1/3= 1451

Hence [hx|x=L/2]cold = 1451 x 0.03184 / 1 = 46.2

Heat flux at the mid point of the plate is given by

316
qw(x)|x=L/2 = [hx|x=L/2]hot [T1 Tw] = [hx|x=L/2]cold [Tw - T2]

CT1 + T2 [hx|x=L/2]hot 107.7


Therefore Tw = ----------------------- ; C = ---------------- = ---------- = 2.33
1+C [hx|x=L/2]cold 46.2

(2.33 x 250) + 50
Hence Tw = -------------------------- = 189.940C
1 + 2.33

Therefore qw(x)|x=L/2 = 107.7 x [250 189.94] = 6468.5 W/m2

Also check forqw(x)|x=L/2 ; qw(x)|x=L/2 = 46.2 x (189.94 50) = 6465.2 W/m2

6.14. Solution:

Air with u = 30 m/s,T = 25 0C


Tw = 150 0 C

10 mm

X1 = 700 mm Insulated

X2 = 750 mm

Properties of the module: km = 5.2 W/(m-K); cpm = 320 kJ/kg-K;


m = 2300 kg/m3.

Mean film temperature = (Tw + T) = 0.5 x(25 + 150) = 87.5 0C.


Properties of air at 87.5 0C are : = 21.79 x 10 6 m2/s; k = 0.03075 W/(m-K);
Pr = 0.695. Assume Recr = 5 x 10 5

Reynolds number at x1 = Rex1 = ux1/ = 30 x 0.7 / (21.79 x 10 6) = 9.637 x 10 5.

Similarly Rex2 = ux2 / = 30 x 0.75 / (21.79 x 10 6) = 10.325 x 10 5

Since Rex1> Recr, the flow is turbulent at x1 and the flow will be turbulent over the module
under consideration. Therefore the average heat transfer coefficient for the module can be
written as x2
[hav]m = {1 / (x2 x1)} hxdx
x1

317
= {1 / (x2 x1)} (Nux k/x)dx

x2
= {1 / (x2 x1)} {k[ 0.037 (Rex)0.8 871]Pr 1/3 /x}dx
x1

k Pr 1/3 x2

= ---------------- { [0.037 (u/)0.8 x 0.2 871/x]dx }


(x2 x1) x1

k Pr 1/3
= ---------------- {0.037(u/)0.8 x 0.2 dx - 871 (dx /x)}
(x2 x1)
k Pr 1/3
= ---------------- { 0.04625[Rex20.8 Rex10.8] 871 ln (x2 / x1)}
(x2 x1)

0.03075 x (0.695)1/3{0.04625[(10.325 x105)0.8 (9.325 X 105)0.8] 871 ln (0.75/0.7)}


= -------------------------------------------------------------------------------------------------------
(0.75 0.7)

= 95.16 W / (m2-K)

For the module, power generation = qw = [hav]m {Tw - T} = 95.16 x (150 25)

= 11895 W/m2 = 11.895 kW/ m2

(b) Since the bottom surface of the module is insulated, all the heat generated in the module
is transferred to air from the top surface of the module. Hence if q is the heat generated per
unit volume then
q (x2 x1)W = qw(x2 x1)W,

where is the thickness of the module.

Therefore q = qw / = 11895 / 0.001 = 11.895 x 10 6 W/m3.

For the module the governing conduction equation is

d2T/dy2 + q / k = 0 .(a)

where y is the coordinate measured in the direction of the thickness of the module.

The boundary conditions are (i) at y = 0, the surface is insulated i.e. dT/dy = 0

and at y = , T = Tw. The solution of Eq.(a) subject to the boundary conditions is given by

318
T(y) + qy2 / 2k = Tw + q2 / 2k

Since the bottom surface is insulated, the maximum temperature of the module will be at the
bottom surface (y = 0) and is therefore given by

Tmax = Tw + q2 / 2k

11.895 x 106 x (0.001)2


= 150 + ----------------------------- = 151.1 0C
2 x 5.2

D) Flow across a cylinder

6.15. Solution:

u = 15 m/s; T = 25 0 C
Tw = 1150C

L d = 0.025 m

Mean film temperature = (Tw + T) = 0.5 x (115 + 25) = 70 0C.

Properties of air at 70 0C are: = 1.0231 kg/m3; = 20.05 x 10 6m2/s; Pr = 0.699;

k = 0.0295 W/(m-K); To find (i) Drag force FD ; (ii) Q

(i) Reynolds number = Red = ud / = 15 x 0.025 / 20.05 x 10 6

= 18703

From the chart of drag coefficient versus Reynolds number we get


the drag coefficient ,CD = 1.2.

Therefore drag force = FD = (u2)LD CD = 0.5 x 1.0231 x 152 x 1 x 0.025 x 1.2 = 3.453 N

319
(ii) From Heat transfer data hand book for gases the average Nusselt number is given by
(w= can be considered for air )

Nuav = havD/k = (0.4Red0.5 + 0.06 Red 2/3) Pr0.4

= (0.4 x 18703 0.5 + 0.06 x 18703 2/3) x (0.699) 0.4= 84.04

84.04 x 0.0295
Therefore hav = Nuav k/d = -------------------- = 99.17 W/(m2-K)
0.025

Heat transfer = Q = havdL(Tw T) = 99.17 x x 0.025 x 1 x (115 25) = 701 W

6.16 Solution:

u = 10 m/s; T = 27 0C

Qw

L D = 0.01 m

Given:- Qw = 1000 W/m; ks = 240 W/(m-K); s = 2700 kg/m3; Cps = 900 kJ/kg-K

To find (i) Tw under steady state condition

(ii) time t required for the surface to reach a temperature of (Tw 10) 0C

Since Tw is not known it is not possible to read the properties at the mean film temperature.
Hence the problem has to be solved by trail and error procedure.

Trial 1:- Calculations are started using the properties of air at T.


Properties of air at 270C are:

= 1.1774 kg/m3; cp = 1.0057 kJ/kg-K; = 15.68 x 10 6 m2/s; k = 0.02624 W/(m-K);

Pr = 0.708
10 x 0.01

320
Red = uD / = ---------------------- = 6377.5
15.68 x 10 6

( w= can be considered for air )

Therefore Nuav = havD/k = (0.4Red0.5 + 0.06 Red 2/3) Pr0.4

= (0.4 x 6377.50.5 + 0.06 x 6377.5 2/3) x (0.708) 0.4 = 45.79

45.79 x 0.02624
Therefore hav = Nuav k/D = ------------------------- = 120.15 W/(m2- K)
0.01

Now Qw = hav DL (Tw T)

Or Tw = T + Qw /hav DL = 27 + 1000 / (120.15 x x 0.01 x 1) = 291.930 C

Trial 2:-Assume Tw = 291.93 0C .Mean film temperature = (291.93 +27) = 159.5 0C.

Properties of air at 159.50C are: = 30.09 x 10 6 m2/s; k = 0.03640 W/(m-K); Pr = 0.682.

10 x 0.01
Red = uD / = ---------------------- = 3323.4
30.09 x 10 6

Nuav = (0.4 x 3323.4 0.5 + 0.06 x 3323.4 2/3) x (0.682) 0.4 = 31.25

Hence hav = 31.25 x 0.03640 / 0.01 = 113.75 W/(m2-K)

Therefore Tw = 27 + 1000 / (113.75 x x 0.01 x 1) = 306.8 0C. Since this value of Tw is


considerably different from the value got in the first trial, one more iteration is required.

Trial 3:- Assume Tw = 303 0 C. Mean film temperature = 0.5 (303 + 27) = 165 0 C.

Properties of air at 165 0 C are: = 30.88 x 10 6 m2/s; k = 0.0369 W/(m-K);

Pr = 0.682
10 x 0.01
Red = uD / = ---------------------- = 3238.34
30.88x 10 6

Nuav = (0.4 x 3238.34 0.5 + 0.06 x 3238.34 2/3) x (0.682) 0.4 = 30.8

Hence hav = 30.8 x 0.0369 / 0.01 = 113.65W/(m2-K)

321
Therefore Tw = 27 + 1000 / (113.65 x x 0.01 x 1) = 307 0C which is very close to the
assumed value of 303 0 C and hence the iteration can be stopped.
Therefore Tw = 307 0C.

(ii) To determine the time t required for the surface of the cylinder to reach a temperature of
(307 10 = 2970C).

If dT/dt represents the rate of change of temperature of the surface w.r.t. time during
unsteady state, then energy balance equation for the cylinder can be written as follows:

Rate of increase of energy of the cylinder = Heat transfer rate by convection from
the surroundings to the cylinder +

E. Flow across tube bundles

6.19.Solution: Case (i) Square Arrangement

Row 1 Row 2 Row 3


u,T

D
ST

SL

Data:- T = 30 0 C; D = 0.01 m; N = 10; u = 1 m/s; ST / D = SL / D = 1.25

To find (i) friction factor,f; (ii) pressure drop p ; (iii) hav

Since the surface temperature of the tubes is not known properties of air are evaluated at T.
Hence properties of air at 30 0C are:

= 1.1774 kg/m3; = 1.983 x 10 5 kg/(m-s); cp = 1005.7 J/kg-K; k = 0.02624 W/(m-K);


Pr = 0.708

(i) For square arrangement the maximum velocity is given by

ST / D 1.25
Umax = u ------------------ = 1 x ---------------- = 5 m/s
[ST / D 1] [ 1.25 1]

322
Gmax = Umax = 1.1774 x 5 = 5.887 kg/(m2-s)

0.01 x 5.887
Reynolds number = Re = DGmax / = ------------------------ = 2969
1.983 x 10 5

From the graph friction factor f = 0.55 and Z = 1 for square arrangement as ST = SL.

N (Gmax) 2 10 x (5.887) 2
Now p = f ---------------- Z = 0.55 x ------------------------- x 1.0 = 161.9 N/ m2
1.1774

(ii) For N 20 the average Nusselt number is given by

Nuav = c2Rem Pr0.36 (Pr/Prw)n

Here c2 = 0.27 ; m = 0.63 ; n = 0.

Therefore Nuav = 0.27 x (2969) 0.63 (0.708)0.36 = 36.74

Since N <20, the above value of Nuav has to be multiplied by a correction factor.

Hence [ Nuav] N=10 = c3[Nuav]N 20

For Re = 2969 and N = 10 from the graph c3 = 0.98

Therefore [ Nuav] N=10 = 0.98 x 36.74 = 36

Therefore hav = 36 x 0.02624 / 0.01 = 94.5 W/(m2-K)

Case (ii); Equilateral Triangular Arrangement:

323
u,T

ST
SD

Given: ST / D = SD / D = 1.25

ST / D
Maximum velocity Umax = u ------------------ = 5 m/s as calculated above
[ST / D 1]

ST / D 1.25
Or Umax = ( )u ------------------ = 0.5 x 1 x ------------- = 2.5 m/s
[SD / D 1] (1.25 1)

We have to choose the higher of the two maximum velocities to calculate the Reynolds
number.Hence Umax and Reynolds number will be same as the above case.

From chart for staggered tube arrangement, f = 0.7and Z = 1

10 x 5.887 2
Therefore p = 0.7 x ----------------- x 1 = 206.1 N/m2
1.1774

From data hand book the average Nusselt number is given by

[Nuav]N20 = c2 Re m Pr 0.36(Pr /Prw) n

For staggered arrangement c2 = 0.35 x (ST / SL) 0.2

Now SL2 = SD2 (ST/2)2 or SL / D = [(SD/D)2 (ST/2D)2]

= (3/2)(ST/D)

Or ST/SL = 2 / 3 = 1.155

324
Hence c2 = 0.35 x [1.155]0.2 = 0.36 ;

For staggered arrangement m = 0.6 ; for air n = 0.

Hence [Nuav]N20 = 0.36 x( 2969)0.6 (0.708)0.36

= 38.54

[ Nuav] N=10 = c3[Nuav]N 20

From the graph c3 = 0.98. Hence [ Nuav] N=10 = 0.98 x 38.54 = 37.8

Therefore hav = 37.8 x 0.02624 / 0.01 = 99.2 W/(m2-K).

6.20. Solution:

Hot gases at ST
u,T D

SL

Data:- T = 375 0C; u = 7 m/s; Tw = 30 0C; Number of rows = N = 10 ;

D = 0.0125 m; number of tubes in each row = m = 40; L = 1 m ;

In-line arrangement; SL/D = ST/D = 2.

To find:- (i) hav ; (ii) Q

Mean film temperature = 0.5 x (375 + 30) = 202.5 0C.

Properties of air at 202.5 0C are: = 0.7424 kg/m3; cp = 1026.6 J/(kg-K);

= 2.606 x 10 5 N-s / m2; k = 0.03948 W/(m-K); Pr = 0.6799

For inline arrangement Umax = u (ST/D) / [(ST/D) 1] = 7 x 2 /(2 1) = 14 m/s.

Mass velocity = Gmax = Umax = 0.7424 x 14 = 10.394 kg/(m2-s)

325
0.0125 x 10.394
Reynolds number = Re = DGmax / = ---------------------- = 4985.6
2.606 x 10 5

The average Nusselt number for N 20 is given by [Nuav]N20 = c2 RemPr0.36

For in-line arrangement from data hand book, c2 = 0.27 and m = 0.63

Therefore [Nuav]N20 = 0.27 x (4985.6) 0.63 x (0.6799)0.36 = 50.19

[ Nuav] N=10 = c3[Nuav]N 20 with c3 = 0.96

Hence [ Nuav] N=10 = 0.96 x 50.19 = 48.18

48.18 x 0.03948
Average heat transfer coefficient = hav = [ Nuav] N=10 k / D = ----------------------
0.0125

= 152.175 W/(m2-K).

Energy balance between the tubes surfaces and hot gases can be written as

Heat transfer from hot gases to the tube surfaces = Q = (DLNm)hav(T - Tw)

Or Q = x 0.0125 x 1 x 10 x 40 x 152.175 x (375 30) = 82.779 x103 W = 82.779 kW

Chapter 7 : Free Convective Heat Transfer


A. Free convection from/to plane surfaces:

7.1. A vertical plate 30 cm high and 1 m wide and maintained at a uniform


temperature of 120 0 C is exposed to quiescent air at 30 0 C.Calculate the
average heat transfer coefficient and the total heat transfer rate from the plate to
air.
7.2. An electrically heated vertical plate of size 25 cm x 25 cm is insulated on one side
and dissipates heat from the other surface at a constant rate of 600 W/m2 by free
convection into quiescent atmospheric air at 30 0 C. Determine the surface
temperature of the plate.
7.3. Determine the heat transfer by free convection from a plate 30 cm x 30 cm
whose surfaces are maintained at 100 0 C and exposed to quiescent air at 20 0 C
for the following conditions: (a) the plate is vertical. (b) Plate is horizontal
7.4. A circular plate of 25 cm diameter with both surfaces maintained at a uniform
temperature of 100 0 C is suspended in horizontal position in atmospheric air at
20 0 C. Determine the heat transfer from the plate.

326
7.5. Consider an electrically heated plate 25 cm x 25 cm in which one surface is
thermally insulated and the other surface is dissipating heat by free convection
into atmospheric air at 30 0 C. The heat flux over the surface is uniform and
results in a mean surface temperature of 50 0 C. The plate is inclined making an
angle of 50 0 from the vertical. Determine the heat loss from the plate for (i)
heated surface facing up and (ii) heated surface facing down.
7.6. A thin electric strip heater of width 20 cm is placed with its width oriented
vertically. It dissipates heat by free convection from both the surfaces into
atmospheric air at 20 0 C. If the surface temperature of the heater is not to
exceed 225 0 C, determine the length of the heater required in order to dissipate
1 kW of energy into the atmospheric air.
7.7. A plate 75 cm x 75 cm is thermally insulated on the one side and subjected to a
solar radiation flux of 720 W/m2 on the other surface. The plate makes an angle
of 60 0 with the vertical such that the hot surface is facing upwards. If the surface
is exposed to quiescent air at 25 0 C and if the heat transfer is by pure free
convection determine the equilibrium temperature of the plate.

B. Free convection from/to Cylinders:

7.8. A 5 cm diameter, 1.5 m long vertical tube at a uniform temperature of 100 0C


is exposed to quiescent air at 20 0 C. calculate the rate of heat transfer from
the surface to air. What would be the heat transfer rate if the tube were kept
horizontally?
7.9. A horizontal electrical cable of 25 mm diameter has a heat dissipation rate of 30
W/m. If the ambient air temperature is 27 0 C, estimate the surface temperature
of the cable.
7.10. An electric immersion heater, 10 mm in diameter and 300 mm long is rated at
550 W. If the heater is horizontally positioned in a large tank of water at 20 0 C,
estimate its surface temperature. What would be its surface temperature if the
heater is accidentally operated in air.

A.Free Convection to or from plane surfaces

7.1. Solution:

327
Tw = 120 0 C

T = 30 0C
L = 0.3m
Mean film temperature of air = 0.5 x (120 +30) = 750C
x
Properties of air at 750C are:

= 1/ (273 + 75) = 2.874 x 10 3 1/K; Pr = 0.693

k = 0.03 W/(m-K) ; = 20.555 x 10 6 m 2/s ;

First we have to establish whether the flow become turbulent within the given length of the
plate by evaluating the Rayleigh number at x = L.

9.81 x 2.874 x 10 3x (120 30) x 0.3 3


GrL = (gTL 3) / 2 = ----------------------------------------------------
20.555 x 10 6

= 1.62 x 10 8

Rayleigh number = RaL = GrLPr = 1.62 x 10 8 x 0.693 = 1.12 x 10 8.

Since RaL< 10 9 flow is laminar for the entire height of the plate. Hence the average Nusselt
number is given by (from data hand book)

Nuav = 0.59 x (RaL) 0.25 = 0.59 x (1.12 x 10 8) 0.25 = 60.695

60.6 x 0.03
Therefore hav= Nuav k / L = ---------------- = 6.069 W /(m2 K).
0.3

Total heat transfer fro both sides of the plate per unit width of the plate is given by

Qtotal = hav(2LW)T = 6.06 x (2 x 0.3 x 1) X (120 30) = 327.726 W/m.

7.2. Solution:

328
Insulated

qw = 600 W/m2
T = 30 0C

L = 0.25 m

Since Tw is not known, it is not possible to determine the mean film temperature at which
fluid properties have to be evaluated. Hence this problem requires a trial and error solution
either by assuming Tw and then calculate Tw by using the heat balance equation and check for
the assumed value or assume a value for hav ,calculate Tw and then calculate hav and check for
the assumed value of hav.Since it is difficult to guess a reasonable value for Tw to reduce the
number of iterations, it is preferable to guess a reasonable value for hav for air as we know
that for air hav varies anywhere between 5 and 15 W/(m2-K).

Trial 1:- Assume hav = 10 W/(m2-K).

Now qw = hav[Tw T] or Tw = T + qw / hav = 30 + 600 / 10 = 900C.

Hence mean film temperature = 0.5 x [90 + 30] = 60 0C.

Properties of air at 60 0C are: = 1 / (60 + 273 ) = 3.003 x 10 3 1/K; Pr = 0.696;

k = 0.02896 W/(m-K); = 18.97 x 10 6m2/s.

9.81 x 3.003 x 10 3x 600 x (0.25)4


RaL* 4 2
= GrL*Pr =[(gqwx ) /(k )]Pr = ----------------------------------------------x 0.696
0.02896 x (18.97 x 10 6) 2

Or RaL* = 4.61 x 10 9.

Since RaL*>10 9 flow is turbulent for the entire length of the plate

Hence Nuav = 1.25 Nux|x=L = 1.25 x 0.17 x (4.61 x 10 9) 0.2 = 55.37


Therefore hav = 55.37 x 0.02896 / 0.25 = 6.41 W/(m2 K)

329
Since the calculated value of hav deviates from the assumed value by about 34 %, one more
iteration is required.

Trial 2:- Assume hav = 6.41 W/(m2-K)

Hence Tw = 30 + 600 / 6.41 = 123.6 0C 120 0 C

Mean film temperature = 0.5 x (120 + 30) = 75 0 C

Properties of air at 75 0C are:- = 1/(75 + 273) = 2.873 x 10 3 1/K. Pr = 0.686

k = 0.03338 W /(m-K); = 25.45 x 10 6m 2 /s.

9.81 x 2.873 x 10 3x 600 x 0.25 4


RaL* = ------------------------------------------- x 0.686 = 2.06 x 10 9
0.03338 x (25.45 x 10 6) 2

Flow is turbulent for the entire length of the plate.

Hence Nuav = 1.25 Nux|x=L = 1.25 x 0.17 x (2.06 x 10 9) 0.25 = 45.27

Therefore hav = 45.27 x 0.03338 / 0.25 = 6.04 W/(m2 K).

Since the calculated value of hav is very close to the assumed value (error <, the iteration is
stopped. The surface temperature of the plate is therefore given by

Tw = 30 + 600 / 6.04 = 129.3 0 C.

7.3. Solution:- Case(i) When the plate is vertical

Data:- Characteristic length = L = height of the plate = 0.3 m; Tw = 100 0C;T= 20 0C;

Mean film temperature = 0.5 x (100 + 20) = 60 0C.

Properties of air at 60 0C are: = 1 / (60 + 273 ) = 3.003 x 10 3 1/K; Pr = 0.696;

k = 0.02896 W/(m-K); = 18.97 x 10 6 m2/s.

RaL = GrLPr =( gTL 3/ 2) Pr

9.81 x 3.003 x 10 3 x (100 20) x (0.3) 3


= ---------------------------------------------------- x 0.696 = 1.23 x 10 8
(18.97 x 10 6 ) 2

From data hand book corresponding to this value of RaL have

330
Nuav = 0.59 x (1.23 x 10 8) 0.25 = 62.13

Therefore hav = 62.13 x 0.02896 / 0.3 = 5.99 W/(m2-K).

Rate of heat transfer = Q = hav(2LW)(T) = 5.99 x (2 x 0.3 x 0.3) x (100 20)

= 86.256 W

Case (ii) When the plate is horizontal

Heated surface facing upwards with heat


T transfer coefficient htop

T
Heated surface facing downwards with
Heat transfer coefficient hbottom

Data:- T = 20 0C; Temperature of both the surfaces = Tw = 100 0C;


Mean film temperature = 0.5 x (100 + 20) = 60 0C;L = W = 0.3 m
Properties of air at 60 0C are: = 1 /(60 + 273) = 3.003 x 10 3 1/K; Pr = 0.696
= 18.97 x 10 6 m2/s; k = 0.02896 W/(m-K)

(a) To find htop:- Characteristic length = L = A/P = LW /{2(L+W)}

=L2 / 4L = L/4 = 0.3 / 4 = 0.075 m

gTL 3 9.81 x (3.003 x 10 3) x (100 20) x (0.075) 3


RaL = ----------- Pr = ----------------------------------------------------------- x 0.696
2 (18.97 x 10 6) 2

Or RaL = 1.923 x 10 6.

From data hand book for heated surface facing upwards with constant surface temperaturethe
average Nusselt number is given by

Nutop = htopL/k = 0.54 x (RaL) 0.25 = 0.54 x (1.923 x 10 6) 0.25= 20.11


Hence htop = 20.11 x 0.02896 / 0.075 = 7.76 W/(m2-K)

331
(b)To find hbottom:- From data hand book for heated surface facing downwards with constant
surface temperature, the average Nusselt number is given by

Nubottom = hbottomL/k = 0.27 x(1.923 x10 6) 0.25 = 10.05

Hbottom = 10.05 x 0.02896 / 0.075 = 3.88 W/(m2-K)

Total heat loss to air = Qtotal = Qtop + Qbottom = (LW)htop T + (LW)hbottomT

= (0.3 x 0.3) x (100 20) x (7.76 + 3.88) = 83.808 W

7.4. Solution: Data:- Horizontal circular plate with D = 0.25 m;Tw = 100 0C ; T = 20 0C

This problem is similar to the previous problem except for the characteristic length. For a
horizontal circular plate of diameter D the characteristic length is given by

L = A/P = (D2/4) / (D) = D/4 = 0.25 / 4 = 0.0625 m

Mean film temperature = 0.5 x (100 + 20) = 60 0C;L = W = 0.25 m

Properties of air at 60 0C are: = 1 /(60 + 273) = 3.003 x 10 3 1/K; Pr = 0.696

= 18.97 x 10 6 m2/s; k = 0.02896 W/(m-K).

gTL 3 9.81 x (3.003 x 10 3) x (100 20) x (0.0625) 3


RaL = ----------- Pr = ----------------------------------------------------------- x 0.696
2 (18.97 x 10 6) 2

Or RaL = 1.112 x 10 6.

From data hand book for heated surface facing upwards with constant surface temperaturethe
average Nusselt number is given by

Nutop = htopL/k = 0.54 x (RaL) 0.25 = 0.54 x (1.112 x 10 6) 0.2 = 17.53

Hence htop = 17.53 x 0.02896 / 0.0625 = 8.12 W/(m2-K)

(b)To find hbottom:- From data hand book for heated surface facing downwards with constant
surface temperature, the average Nusselt number is given by

Nubottom = hbottomL/k = 0.27 x (1.112 x10 6) 0.25 = 8.76

hbottom = 8.76 x 0.02896 / 0.0625 = 4.059 W/(m2-K)

332
Total heat loss to air = Qtotal = Qtop + Qbottom = (DL)htop T + (DL)hbottomT

= ( x 0.25 x 1)x (100 20) x (8.76 + 4.059) = 805.44 W

7.5. Soloution: Data:- L = W = 0.25 m; T = 30 0C; Tw = 50 0C;


Mean film temperature = 50 0.25 x (50 - 30) = 45 0C;Properties of air at 45 0C are:
Pr = 0.6835; k = 0.02791 W/(m-K); = 17.455 x 10 6 m2/s.
= 1/[{30 +0.25x(50-30)} +273] =3.25 x 10 3

(i) Inclined plate with heated surface facing upwards:

Characteristic length = L = 0.25 m


gTL3
GrL= -----------
2
= 50 0
9.81 x (50 30) x 3.25 x 103x(0.25)3
= -------------------------------------------------
(17.455 x 10 6)2
Insulated
= 3.27x 10 7

Hence RaL = 3.27 x 10 7 x 0.6835 = 2.23x107

From data hand book, for inclined plate with


heated surface facing upwards the Nusselt number
is given by

Nuav = 0.145 [RaL1/3 (GrcPr)1/3] + 0.56(GrcPrcos)1/4

The above correlation is valid only if GrL > Grc.

From data hand book for = 50 0, Grc = 4 x 10 8 which is more than GrL.Hence the above
correlation cannot be used. Instead the following correlation has to be used.

Nuav = 0.59 (GrLcos Pr) = 0.59 x (2.23 x 107 x cos 500) = 36.3

Hence hav = 36.3 x 0.02791 / 0.25 = 4.05 W/(m2 K).

Therefore Q = 4.05 x (0.25 x 0.25) x (50 30) = 5.062 W

(ii) Inclined plate with heated surface facing downwards:

The correlation for Nusselt number when the heated surface is facing downwards is given by
Nuav = 0.56 (GrLcos Pr) = 0.56 x (2.23 x 107 x cos 500)

333
= 34.45
hav = 34.45 x 0.02791 / 0.25 = 3.84 W/(m2-K).

Hence Q = 3.84 x (0.25 x 0.25) x (50 30) = 4.8 W

7.6. Solution:

Tw = 225 0 C
W

T=200C

L=0.2 m

Mean film temperature = 0.5 x (225 + 20) = 122.5 0C. Properties of air at 122.50C
are: = 1/(122.5 + 273) = 2.5 x 10 3 1/K; = 25.90 x 10 6 m2/s; Pr = 0.6865;
k = 0.03365 W/(m-K).

9.81 x 2.5 x 10 3 x(225 20) x 0.2 3


GrL = ---------------------------------------------- = 5.99 x 10 7
(25.9 x 10 6) 2

RaL = 5.99 x 10 7 x 0.6865 = 4.11 x 10 7

Hence Nuav = 0.59 x (4.11 x 10 7) 0.25 = 47.24

Therefore hav = 47.24 x 0.03365/0.2 = 7.94 W/(m2-K)

1000
Now Q = havLWT or W = Q / (havLT) = -----------------------------
7.94 x 0.2 x (225 20)

= 3.0718 m

7.7. Solution:

334
qw = qsolar =720 W/m2

= 60 0 T = 25 0 C

Insulated

Since Tw is not known, mean film temperature to evaluate the fluid properties cannot be
determined. Hence the problem requires a trial- and error method by suitably assuming a
value for hav and then check for this assumpotion.

Trial 1:- Assume hav = 5 W/(m2-K):

Tw = T + qw / hav = 25 + 720 / 5 = 169 0C.

Tm = Tw 0.25[Tw T] = 169 0.25 x [169 25] = 133 0C.

Properties of air at 133 0C are: = 26.62 x 10 6 m2/s; k = 0.03413 W/(m-K); Pr = 0.685

Mean temperature to evaluate is given by T = T + 0.25[Tw T]

= 25 + 0.25 x(169 25) = 61 0C


Therefore = 1/[61 + 273] = 2.994 x 10 3 1/K.

9.81 x 2.994 x 10 3 x [169 25] x 0.75 3


GrL = ------------------------------------------------------ = 2.51 x 10 9
(26.62 x 10 6) 2

For = 60 0, Grc = 10 8. Since GrL> Grc the average Nusselt number is given by

Nuav = 0.145 [(GrLPr) 1/3 (GrcPr) 1/3] + 0.56[GrcPr]1/4

= 0.145 x [(2.51 x 10 9x 0.685)1/3 (10 8 x 0.685) 1/3] + 0.56 x ( 10 8 x 0.685)1/4

= 164.45

Therefore hav = 164.45 x 0.03413 / 0.75 = 7.48 W/(m2-K).

335
Since the calculated value of hav is quite different from the assumed value one more iteration
is required.

Trial 2: Assume hav = 7.48 W/(m2-K).

With this assumption, following the steps shown in trial 1 we get

Tw = 121.25 0C;Tm = 97.18 0C 100 0 C;T = 49.06 0C; GrL = 2.311 x 10 9;Nuav = 159.96

Hence hav = 6.88 W/(m2-K). This value agrees with the assumed value within 8 %. Hence the
iteration is stopped and the equilibrium temperature of the plate surface is calculated as

Tw = 25 + 720 /{0.5(7.48 + 6.88)} = 125.27 0C.

B Free convection from/to cylinders

7.8. Solution: (i) When the tube is vertical:

Tw = 100 0C Mean film temperature = 0.5x(100+20)

= 60 0C.
Properties of air at 60 0C are:Pr = 0.696

k = 0.0290 W/(m-K); = 18.97x106

m2/s; = 1/(60 + 273) =3.003x1031/K.


L = 1.5 m T = 20 0C

gTL 3 9.81 x 3.003 x 10 3 x (100 20) x 1.5 3


GrL = ----------- = -------------------------------------------------- = 2.21 x 10 10
2 (18.97x106) 2

(L/d) [ 1.5 / 0.05 ]


Now ------------ = --------------------------- = 0.0078
GrL1/4 [2.21 x 10 10]

336
(L/d)
Since ------- < 0.025, the vertical tube/cylinder can be treated as a vertical flat surface
GrL1/4
For calculating the Nusselt number.
Now RaL = GrLPr = 2.21 x 10 10 x 0.696 = 1.538 x 10 10

Hence Nuav = 0.1 x (1.538 x 10 10) 1/3 = 248.7

Therefore hav = 248.7 x 0.029 / 1.5 = 4.81 W/(m2 K).

Rate of heat transfer = Q = dLhavT = x 0.05 x 1.5 x (100 20) x 4.81 = 90.67 W

(ii) When the pipe is horizontal:- When the pipe is horizontal, the characteristic length is the
diameter. Hence

gTd 3 9.81 x 3.003 x 10 3 x (100 20) x 0.05 3


Grd = ----------- = -------------------------------------------------- = 8.185 x 10 5
2 (18.97x106) 2

Rad = 8.185 x 10 5 x 0.696 = 5.697 x 10 5

From data hand book the average Nusselt number is given by


1/6

Rad
[Nuav] = 0.60 + 0.387 -----------------------------------
[1 + (0.559/Pr)9/16] 8/27
1/6

(5.697x 105)
[Nuav] = 0.60 + 0.387 ----------------------------------- = 4.01 or Nuav = 16.08
[1 + (0.559/0.696)9/16] 8/27

Therefore hav = 16.08 x 0.029 / 0.05 = 9.32W/(m2 K).

Q = x 0.05 x 1.5 x (100 20) x 9.32 = 175.67 W

7.9. Solution:
T = 27 0C Tw = ?

qw = 30 W/m d =.025 m

337
Since Tw is not known, it is not possible to evaluate the fluid properties at the mean film
temperature. Hence the problem has to be solved by trial and error solution by assuming a
suitable value for hav and check for the assumed value.

Trial 1:- Assume hav = 10 W/(m2-K)

qw = dhav[Tw T] or Tw = T + qw /(dhav) = 27+ 30 /( x 0.025 x 10) = 65 0C.

Mean film temperature = 0.5 x (27 + 65) = 46 0C.

Properties of air at 46 0C are: = 1 /(46 + 273) = 3.135 x 10 3 1/K; k = 0.0280 W/(m-K)

Pr = 0.684; = 17.45 x 10 6 m2/s.

9.81 x 3.135 x 10 3x (65 27) x 0.025 3


Grd = ---------------------------------------------------- = 6.0 x 10 4.
[17.45 x 10 6 ] 2

(6.0 x 10 4 x 0.684) 1/6



Hence [Nuav] = 0.60 + 0.387 ------------------------------------ = 2.8
[ 1 + (0.559/0.6985) 9/16] 8/27

Hence Nuav = 7.84 or hav = 7.84 x 0.028 / 0.025 = 8.78 W/(m2 K).

Since the calculated value of hav deviates very much from the assumed value one more
iteration is required.

(ii) Trial 2: Assume hav = 8.78 W/(m2-K).


Proceeding in the same way as in trial 1 we have Tw = 70.5 0C. Hence Tm = 48.75 0 500C
Properties of air at 50 0C are: = 1/(50+273) = 3.05 x 10 3 1/K; Pr = 0.698;
k = 0.02826 W/(m-K); =17.95 x 10 6m2/s.

9.81 x 3.05 x 10 3x (70.5 27) x 0.025 3


Grd = ---------------------------------------------------- = 6.4 x 10 4.
[17.95 x 106 ] 2
1/6
4
(6.4 x 10 x 0.698)
[Nuav] = 0.60 + 0.387 ----------------------------------- = 2.89 or Nuav = 8.35
[1 + (0.559/0.698)9/16] 8/27

Hence hav = 8.35 x 0.02865 / 0.025 = 9.5691 W/(m2-K).

The calculated value of hav agrees with the assumed value within 5 % iteration is stopped.

338
The equilibrium temperature of the surface = Tw = 27 + 30 /( x 0.025 x 9.5691) = 67 0C.

7.10. Solution:
Qw

d
L

Data:- L = 0.3 m; d = 0.01 m; Qw = 550 W; T = 20 0C;

550
Wall heat flux = qw = Qw /(dL)= ---------------------- = 58357 W/m2
( x 0.01 x 0.3)

Since Tw is not known, fluid properties cannot be evaluated at the mean


temperature and hence the problem has to be solved by trial and error procedure
by assuming a suitable value for hav and then check for the assumed value.

Case(i):- When the heater is immersed in water

For free convection in liquids the order of heat transfer coefficient is around 10 to 1000
W/(m2-K).Let us assume hav = 1000 W/(m2-K).

Hence Tw = T + qw / hav = 20 + 58357 / 1000 = 78.4 0C.

Mean film temperature = 0.5 x (20 + 78.4) = 49.2 0C.

Properties of water at 49.2 0C are: = 3.103 x 10 3 1 / K;

Pr = 3.68; k = 0.639 W/(m-K) ; = 0.5675 x 10 6 m2/s;

9.81 x 3.103 x 10 3 x (78.4 20) x (0.01) 3


Grd = ------------------------------------------------------- = 5.519 x 10 6
(0.5675 x 10 6) 2

Rad = 5.519 x 10 6 x 3.68 = 2.03 x 10 7.

For horizontal cylinders Nusselt number is given by

Nud = C Radn with C = 0.125 and n = 1/3 for this value of Rad.

Hence Nud = 0.125 x (2.03 x 10 7) 1/3 = 33.908

339
Hence hav = 33.908 x 0.639 / 0.01 = 2166.21 W/(m2 K)

Trial 2:- Assume hav = 2166.21 W/(m2-K)

Tw = 20 + 58357 / 2166.21 = 46.940C.

Hence mean film temperature =0.5x(20 + 46.94) = 33.47 0C.


Properties of water at 33 0C 300C are : k = 0.6129 W/(m-K); Pr = 5.68; = 3.3 x 10 3; =
0.831 x 10 6 m2 /s.

9.81 x 3.3 x 10 3 x (46.94 20) x (0.01) 3


Grd = ----------------------------------------------------- = 2.2 x 106.
(0.831 x 10 6) 2
Rad = 2.2 x 106 x 5.68 = 1.24 x 10 7
Hence, Nuav = 0.125 x (1.24 x 10 7) 0.333 = 28.77
Therefore, hav = 28.77 x 0.6129 / 0.01 = 1763.3 W/(m2 K)

Trial 3:- Assume hav =1763 W/(m2-K)

Tw = 20 + 58357 / 1763 = 53.1 0C. Mean film temperature = 0.5 x (20 + 53.1) = 36.55 0C.

Properties of water at 400C are: k = 0.6280 W/(m-K); Pr = 4.34; = 3.19 x 10 3 1/K;


= 0.657 x 10 6m2/s.

9.81 x 3.19 x 10 3x (53 20) x (0.01) 3


Rad = ------------------------------------------------- x 4.34 = 1.03 x 10 7
(0.657 x 10 6) 2

Hence, Nuav = 0.125 x (1.03 x 10 7) 0.333 = 27.05


Therefore, hav = 27.05x 0.6280 x / 0.01 = 1698.74 W/(m2-K)

Since the calculated value of hav agrees with the assumed value within 4%, iteration is
stopped and the equilibrium temperature of the heater is calculated as

Tw = 20 + 58357 / 1698.74 = 54.35 0C


Case (ii):- When the heater is exposed to air

When a heated surface is exposed to air the order of heat transfer coefficient varies between 5
and 20 W/(m2-K).

Trial 1:- Assume hav = 20 W/(m2-K)

Tw = 20 + 58357 / 20 = 2938 0C. Mean film temperature = 0.5 x (20 + 2938) = 1479 0C

Properties of air at 1479 0C are : = 1/(1479 +273) = 5.71 x 10 4 1/K; Pr = 0.7045;

340
k = 0.108 W/(m-K); = 294.3 x 10 6m2/s.

9.81 x 5.71 x 10 4x (2938 20) x (0.01) 3


Rad = ------------------------------------------------- --- x 0.7045 = 133
(294.3 x 10 6) 2

Nuav = 0.850 x (133) 0.188 = 2.13

Hence hav = 2.13 x 0.108 / 0.01 = 23 W/(m2-K) This is 13% away from the assumed value
and hence one more iteration is required.

Trial2:- Assume hav = 23 W/(m2-K)

Tw = 20 + 58357 / 23 = 2557 0C ; Mean film temperature = 0.5 x ( 20 + 2557) = 1289 0C

Properties of air at 1289 0C are: = 6.4 x 10 4 1/K; = 244.34 x 10 6 m2/s;Pr = 0.705;

k = 0.0978 W/(m-K)

9.81 x 6.4 x 10 4 x (2557 20) x (0.01) 3


Rad = ------------------------------------------------- --- x 0.705 = 188
(244.34 x 10 6) 2

Nuav = 0.850 x (188) 0.188 = 2.275.Hence hav = 2.275 x 0.0978 / 0.01 = 22.25 W/(m2 K).

This value of hav agrees with the assumed value within 4% and hence the iteration is stopped.
The equilibrium temperature of the heater is therefore given by

Tw = 20 + 58357 / 22.25 = 2643 0C.

341
Chapter 8 : Condensation & Boiling
8.1. Introduction: Knowledge of heat transfer occurring during change of phase i.e. during
condensation and boiling is very useful in a number of ways. For example in all power and
refrigeration cycles, it is necessary to convert a liquid into a vapour and vice-versa. This is
accomplished in boilers or evaporators and condensers.

Heat transfer coefficients in both condensation and boiling are generally


much higher than those encountered in single phase processes. Values greater than 1000
W/(m2-K) are almost always obtained. This fact has been used in several recent
applications where it is desired to transfer high heat fluxes with modest temperature
differences. An example is the heat pipe which is a device capable of transferring a large
quantity of heat with very small temperature differences.

8.2. Film-wise and Drop-wise condensation:- Condensation occurs whenever a vapour


comes into contact with a surface at a temperature lower than the saturation temperature of
the vapour corresponding to its vapour pressure. The nature of condensation depends on
whether the liquid thus formed wets the solid surface or does not wet the surface. If the liquid
wets the surface, the condensate flows on the surface in the form of a film and the process is
called film-wise condensation. If on the other hand, the liquid does not wet the surface, the
condensate collects in the form of droplets, which either grow in size or coalesce with
neighboring droplets and eventually roll of the surface under the influence of gravity. This
type of condensation is called drop-wise condensation.

The rate of heat transfer during the two types of condensation processes
is quite different. For the same temperature difference between the vapour and the surface,
the heat transfer rates in drop-wise condensation are significantly higher than those in film-
wise condensation. Therefore it is preferable to have drop-wise condensation from the
designers point of view if the thermal resistance on the condensing side is a significant part
of the total thermal resistance. However it is generally observed that, although drop-wise
condensation may be obtained on new surfaces, it is difficult to maintain drop-wise
condensation continuously and prolonged condensation results in a change to film-wise
condensation. Therefore it is still the practice to design condensers under the conservative
assumption that the condensation is of film type.

8.3. Nusselts theory for laminar film-wise condensation on a plane vertical surface:-
The problem of laminar film-wise condensation on a plane vertical surface was first
analytically solved by Nusselt in 1916.He made the following simplifying assumptions in his
analysis.
(i) The fluid properties are constant.
(ii) The plane surface is maintained at a uniform temperature, Tw which is less than the
saturation temperature Tv of the vapour.

(iii) The vapour is stationary or has a very low velocity and so it does not exert any drag on
the motion of the condensate: i.e., the shear stress at the liquid-vapour interface is zero.

342
(iv) The flow velocity of the condensate layer is so low that the acceleration of the
condensate is negligible.
(v) The downward flow of the condensate under the action of gravity is laminar.
(vi) Heat transfer across the condensate layer is purely by conduction; hence the liquid
temperature distribution is linear.

[ + (/y)dy]dx
x pdy dx
pdy

dx

(p+dp)dy (p+dp)dy

Ldxdyg vdxdyg
(a) Force balance on a condensate (b) Force balance on a vapour element
element at the same distance x from top
Fig. 8.1 : Laminar film condensation on a vertical plate

Consider the film-wise condensation on a vertical plate as illustrated in Fig.8.1.


Here x is the coordinate measured downwards along the plate, and y is the coordinate
measured normal to the plate from the plate surface. The condensate thickness at any x is
represented by [ = (x)]. The velocity distribution u(y) at any location x can be
determined by making a force balance on a condensate element of dimensions dx and dy in x
and y directions as shown in Fig. 8.1(a). Since it is assumed that there is no acceleration of
the liquid in x direction, Newtons second law in x direction gives

Ldxdyg + pdy + [ + (/y)dy]dx dx (p + dp)dy = 0

or (/y) = (dp/dx) Lg ..(8.1)

Expression for (dp/dx) in terms of vapour density v can be obtained by making a force

balance for a vapour element as shown in Fig. 8.1(b). The force balance gives

vdxdyg + pdy = (p + dp) dy

343
or (dp/dx) = vg

Substituting this expression for dp/dx in Eq. (8.1) we have

(/y) = (vL)g

Since the flow is assumed to be laminar, = L(u/y)

Therefore /y{L(u/y)} = (vL)g

Integrating with respect to y we have L(u/y) = (vL)g y + C1

(vL)g y C1
Or (u/y) = --------------- + ------- (8.2)
L L

Integrating once again with respect to y we get


(vL)g y2 C1 y
u(y) = ---------------- + ------------- + C2 ......(8.3)
2 L L

The boundary conditions for the condensate layer are: (i) at y = 0, u = 0;

(ii) at y = , (u/y) = 0.

Condition (i) in Eq. (8.3) gives C2 = 0 and condition (ii) in Eq. (8.2) gives

(vL)g C1
0 = ------------- + ---------
2 L L

(vL)g
Therefore C1 = --------------
2

Substituting for C1 and C2 in Eq.(8.3) we get the velocity distribution in the condensate layer
as

g(L v)
u(y) = --------------- [ y (y2/2)] (8.4)
L

If m is the mass flow rate of the condensate at any x then

m = Ludy
0

344

m = L{g(L v) / L}[ y (y2/2)]dy
0

gL (L v) 3
= ------------------ ..(8.5)
3 L

gL (L v) 2 d
Hence dm = ----------------------
L

Amount of heat transfer across the condensate element = dq = dm hfg

gL (L v) 2 d hfg
Or dq = ------------------------- .(8.6)
L

Energy balance for the condensate element shown in the figure can be written as

dq = kL(Tv Tw)dx /

gL (L v) 2 d hfg
Or ------------------------------- = kL(Tv Tw)dx / .(8.6)
L

kL L (Tv Tw)dx
3
or d = ----------------------
gL (L v) hfg

Integrating we get
4 kL L (Tv Tw)x
----- = --------------------- + C 3
4 gL (L v) hfg

At x = 0, = 0. Hence C3 = 0.

Therefore 4 kL L (Tv Tw)x


----- = ---------------------
4 gL (L v) hfg

4 kL L (Tv Tw)x
or = [-------------------------- ] 1/ 4(8.7)
g L (L v) hfg

345
Now kL (Tv Tw)dx
----------------- = hx dx [Tv Tw]

kL gL (L v) hfg kL3
Therefore hx = --------- = [ --------------------------] 1 / 4
4 L (Tv Tw)x

g L (L v) hfg kL3
Or hx = 0.707[ --------------------------] 1 / 4 ...............................(8.8)
L (Tv Tw)x

The local Nusselt number Nux can therefore be written as

hxx g L (L v) hfg x3
Nux = ----- = 0.707[ --------------------------] 1 / 4 ...............................(8.8)
kLL (Tv Tw)kL

The average heat transfer coefficient for a length L of the plate is given by
L
hav = (1/L) hxdx (8.9)
0

It can be seen from Eq. (8.8) that hx = C x , where C is a constant given by

g L (L v) hfg kL3
Or C = 0.707[ --------------------------] 1 / 4 (8.10)
L (Tv Tw)

L
Hence hav = (1/L) C x dx = (C / L) (4/3) L = (4/3)C L
0

Substituting for C from Eq. (8.10) we have

g L (L v) hfg kL3
hav = 0.943[ --------------------------] 1 / 4 = (4/3)hx|x = L.........................(8.11)
L (Tv Tw)L

8.4. Condensation on Inclined Surfaces : Nusselt,s analysis given above can readily be
extended to inclined plane surfaces making an angle with the horizontal plane as shown in
Fig. 8.2.

346
y

Fig. 8.2 : Condensation on an


inclined plane surface

g

The component of the gravitational force along the length of the pate is g sin .The
expressions for local and average heat transfer coefficients can therefore be written
as

g sin L (L v) hfg kL3


hx = 0.707[ ------------------------------------] 1 / 4
L (Tv Tw)x
.......(8.12)

g sin L (L v) hfg kL3


and hav = 0.943[ ----------------------------------] 1 / 4 = (4/3)hx|x = L
L (Tv Tw)L

(8.13)

8.5. Condensation on a horizontal tube: The analysis of heat transfer for condensation on
the outside surface of a horizontal tube is more complicated than that for a vertical
surface. Nusselt,s analysis for laminar film-wise condensation on the surface of a horizontal
tube gives the average heat transfer coefficient as

gL (L v) hfg kL3
hav = 0.725 [ --------------------------------- ] 1 / 4 (8.14)
L (Tv Tw) D

where D is the outside diameter of the tube. A comparison of equations (8.11) and (8.14) for
condensation on a vertical tube of length L and a horizontal tube of diameter D gives

[hav]vertical 0.943
--------------- = ------------(D/L) = 1.3 (D/L) 1/4 .....................................(8.15)
[hav]horizontal 0.725

This result implies that for a given value of (Tv Tw), the average heat transfer coefficientfor
a vertical tube of length L and a horizontal tube of diameter D becomes equal when L =

347
2.856 D.For example when L = 100 D, theoretically [hav]horizontal would be 2.44 times
[hav]vertical. Therefore horizontal tube arrangements are generally preferred to vertical tube
arrangements in condenser design.

8.6. Condensation on horizontal tube banks: Condenser design generally involves


horizontal tubes arranged in vertical tiers as shown in Fig. 8.3 in such a way that the

Fig. 8.3 : Film-wise condensation on


horizontal tubes arranged in a vertical
tier.

condensate from one tube drains on to tube just below. If it is assumed that the drainage from
one tube flows smoothly on to the tube below, then for a vertical tier of N tubes each of
diameter D, the average heat transfer coefficient for N tubes is given by

gL(L v)hfg kL3 1


[hav]N tubes = 0.725 [ ----------------------- ] = ------------ [hav] 1 tube (8.16)
L(Tv Tw) N D N 1/ 4

This relation generally gives a conservative value for the heat transfer coefficient. Since
some turbulence and some disturbance of condensate are unavoidable during drainage, the
heat transfer coefficient would be more than that given by the above equation.

8.7. Reynolds number for condensate flow: Although the flow hardly changes to turbulent
flow during condensation on a single horizontal tube, turbulence may start at the lower
portions of a vertical tube. When the turbulence occurs in the condensate film, the average
heat transfer coefficient begins to increase with the length of the tube in contrast to its
decrease with the length for laminar film condensation. To establish a criterion for transition
from laminar to turbulent flow, a Reynolds number for condensate flow is defined as
follows.
L uav Dh
Re = ----------------- ..(8.17)
L

348
where uav is the average velocity of the condensate film and Dh is the hydraulic diameter for
the condensate flow given by
4 x (Cross sectional area for condensate flow) 4A
Dh = --------------------------------------------- --------- = -------
Wetted Perimeter P
4A L uav 4M
Therefore Re = -------------------- = ---------------..(8.18)
P L P L

where M is mass flow rate of condensate at the lowest part of the condensing surface in kg/s.
The wetted perimeter depends on the geometry of the condensing surface and is given as
follows.
D ..For vertical tube of outside diameter D .(8.19 a)
P = 2L ...For horizontal tube of length L (8.19 b)
W .. For vertical or inclined plate of width W...(8.19 c)

Experiments have shown that the transition from laminar to turbulent condensation takes
place at a Reynolds number of 1800. The expression for average heat transfer coefficient
for a vertical surface [Eq.(8.11)] can be expressed as follows.

g L(L v) kL3 hfg


hav = 0.943 [ ----------------------------- ] 1 / 4
L(Tv Tw)

Generally L >> v. Therefore

g L2 kL3 hfg
hav = 0.943 [ ----------------------------- ] 1 / 4 ..(8.20)
L(Tv Tw)

The above equation can be arranged in the form

hav [L2 / (gkL3) ] 1 / 3 = 1.47 Re L 1/ 3 (8.21)

The above equation is valid for ReL< 1800.


It has been observed experimentally that when the value of the film Reynolds number is
greater than 30, there are ripples on the film surface which increase the value of the heat
transfer coefficient. Kutateladze has proposed that the value of the local heat transfer
coefficient be multiplied by 0.8(RE / 4)0.11 to account for the ripples effect. Using this
correction it can be shown that

ReL
(hav / kL)(L2 / g) 1 / 3 = ------------------------ (8.22)
[1.08 ReL1.22 5.2]

349
8.8. Turbulent film condensation: For turbulent condensation on a vertical surface,
Kirkbride has proposed the following empirical correlation based on experimental data.

hav [L2 / (gkL3) ] 1 / 3 = 0.0077 (ReL) 0.4 (8.23)

In the above correlation the physical properties of the condensate should be evaluated at the
arithmetic mean temperature of Tv and Tw.

8.9. Film condensation inside horizontal tubes: In all the correlations mentioned above, it
is assumed that the vapour is either stationary or has a negligible velocity. In practical
applications such as condensers in refrigeration and air conditioning systems, vapour
condenses on the inside surface of the tubes and so has a significant velocity. In such
situations the condensation phenomenon is very complicated and a simple analytical
treatment is not possible. Consider, for example, the film condensation on the inside surface
of a long vertical tube. The upward flow of vapour retards the condensate flow and causes
thickening of the condensate layer, which in turn decreases the condensation heat transfer
coefficient. Conversely the down ward flow of vapour decreases the thickness of the
condensate film and hence increases the heat transfer coefficient.
Chato recommends the following correlation for condensation at low vapour
velocities inside horizontal tubes:

g L(L v) kL3 h*fg


hav = 0.555 [ --------------------------- ] 1 / 4 ..(8.24 a)
L(Tv Tw) D

where h*fg = hfg + (3/8)cp,L(Tv Tw) .(8.24 b)

This result has been developed for the condensation of refrigerants at low Reynolds number
[Rev = (vuvD) / v< 35,000 ; Rev should be evaluated at the inlet conditions.]

For higher flow rates, Akers, Deans and Crosser propose the following
correlation for the average condensation heat transfer coefficient on the inside surface of a
horizontal tube of diameter D:

hav D
------ = 0.026 Pr 1 / 3 [ReL + Rev(L / v) ] 0.8 ..(8.25)
k

where ReL = (4ML) / (DL) : Rev = (4Mv) / (Dv) .(8.26)

The above equation correlates the experimental data within 50 % for ReL> 5000 and

Rev> 20,000.

8.10. Illustrative examples on film wise condensation:

350
Example 8.1: Saturated steam at 1.43 bar condenses on a 1.9 cm OD vertical tube which is
20 cm long. The tube wall is at a uniform temperature of 109 0C. Calculate the average heat
transfer coefficient and the thickness of the condensate film at the bottom of thetube.

Solution: Data:- Tv = Saturation temperature at 1.43 bar = 110 0 C (from steam tables)

Tw = 109 0C ; Characteristic length = L = 0.2 m ; D = 0.019 m ;

To find : (i) hav ; (ii) (x)|x=L;

Mean film temperature of the condensate (water) = 0.5 x (110 + 109) = 109.5 0C.

Properties of water at 109.5 0C are: L = 951.0 kg/m3; L = 258.9 x 10 6 N-s / m2;

k = 0.685 W/(m-K); = 0.2714 x 10 6 m2/s; hfg = 2230 kJ/kg. Also L>>> v.

Let us assume that the condensate flow is laminar and later check for this assumption.

g L2 kL3 hfg
hav = 0.943 [ ----------------------------- ] 1 / 4
L(Tv Tw) L

9.81 x (951)2x (0.685)3 x 2230 x 103


Hence hav = 0.943 x [--------------------------------------------] 1/ 4
258.9 x 10 6 x (110 109) x 0.2

= 17,653 W / (m2-K)

(ii) hav = (4 / 3)hx|x=L or hx|x=L = x hav = 0.75 x 17,653 = 13,240 W/(m2-K).

Therefore (x)|x=L = kL / hx|x=L = 0.685 / 13240 = 5.174 x 10 5 m = 0.0517 mm.

Check for Laminar flow assumption:- The relation between hav and Reynolds number at the
bottom of the tube is given by

hav [L2 / (gkL3) ] 1 / 3 = 1.47 Re L 1/ 3 or ReL = (1.47 / hav)3(gkL3 / L2)

Hence ReL = (1.47 / 17,653) 3 [9.81 x 0.685 3 / {0.2714 x 10 6}2]

= 24.72

Since ReL< 1800, our assumption that condensate flow is laminar is correct.

351
Example 8.2:- Saturated steam at 80 0C condenses as a film on a vertical plate 1 m high. The
plate is maintained at a uniform temperature of 70 0C. Calculate the average heat transfer
coefficient and the rate of condensation. What would be the corresponding values if the effect
of ripples is taken into consideration.

Solution:Data:- Tv = 80 0C; Tw = 70 0C; Mean film temperature =0.5 x (80 + 70) = 75 0C.

Properties of condensate (liquid water) at 75 0C are: L = 974.8 kg/m3;

kL = 0.672 W /(m-K) ; L = 381 x 10 6 N-s/m2; hfg at 80 0C = 2309 kJ/kg-K;

L = 0.391 x 10 6 m2/s.Charecteristic length = L = 1.0 m.

Assuming laminar film condensation the average heat transfer coefficient is given by

g L2 kL3 hfg
hav = 0.943 [ ----------------------------- ] 1 / 4
L(Tv Tw)

9.81 x (974.8)2 x (0.672)3 x 2309 x 103


= 0.943 x [ ------------------------------------------------] 1/ 4 = 6066.6 W /(m2 K).
381.6 x 10 6 x (80 70 ) x 1.0

hav L (Tv Tw) 6066.6 x 1.0 x (80 70)


Condensate rate = M = --------------------- = ------------------------------- 0.0263 kg/s.
hfg 2309 x 10 3

4M
Check for laminar flow assumption :- ReL = --------------, where P = width of the plate for
L P

4 x 0.0263
vertical flat plate. Hence ReL = ------------------------- = 276
381 x 10 6
Since ReL< 1800, the condensate flow is laminar.

Since ReL> 30, it is clear that the effects of ripples have to be considered.
4M 4hav L (Tv Tw)
Now ReL = ------------ = -----------------------
L P L P hfg

ReL L P hfg
Hence hav = ------------------ .(1)
4L(Tv Tw)

352
When the effects of ripples are considered the relation between ReL and hav is given by
Eq.(8.22) as follows:
ReL
1.08 ReL1.22 5.2 = ----------------------- Substituting for hav from Eq.(1) we have
(hav/kL)(L2 /g)1 /3

4L (Tv Tw) kL (g / L2) 1/3


1.08 ReL1.22 5.2 = --------------------------------
L P hfg

4 x 1 x (80 70) x 0.672 x {9.81 /( 0.391 x 10 6)2}1/3


1.08 ReL1.22 5.2 = ----------------------------------------------------------------------
381.6 x 10 6 x 1.0 x 2309 x 10 3

1.08 ReL1.22 5.2 = 1221.3. Or ReL = 319.4

319.4 x 381.6 x 10 6 x 1.0 x 2309 x 10 3


Hence from Eq.(1) hav = ---------------------------------------------------- = 7036 W /(m2 K).
4 x 1.0 x (80 70)

hav L (Tv Tw) 7036 x 1.0 x (80 70)


Hence M = --------------------- = ---------------------------- = 0.03047 kg / s.
hfg 2309 x 10 3

[ It can be seen that the ripples on the surface increase the heat transfer coefficient by about
15 %].

Example 8.3:- Air free saturated steam at 65 0C condenses on the surface of a vertical tube
of OD 2.5 cm. The tube surface is maintained at a uniform temperature of 35 0C. Calculate
the length of the tube required to have a condensate flow rate of 6 x 10 3kg/s.

Solution:Data:- Tv = 65 0C; Tw = 35 0C; D0 = 0.025 m; M = 6 x 10 3 kg/s.

To find length of the tube, L.

Mean film temperature = 0.5 x (65 + 35) = 50 0C.Properties of condensate

(liquid water) at 50 0C are: kL = 0.640 W/(m-K); L = 0.562 x 10 3 N-s/m2; L = 990

kg/m3; At 65 0C, hfg = 2346 x 10 3 J/(kg-K).

4M 4 x 6 x 10 3
Reynolds number = Re = --------------- = ------------------------------- = 544
L Do 0.562 x 10 3 x x 0.025

353
Since Re < 1800 flow is laminar. It is more convenient to use Eq.(8.21)

hav [L2 / (gkL3) ] 1 / 3 = 1.47 Re L 1/ 3

or (gkL3) 1.47 x (544) 1/3 x [9.81 x 0.643] 1/3


hav = 1.47 Re L 1/ 3 1/3
[ ---------- ] = ----------------------------------------------
L2 (0.562 x 10 3/ 990)2

= 3599 W/(m2 K)

Heat balance equation gives M hfg = hav DoL [Tv Tw]

M hfg 6 x 10 3 x 2346 x 10 3
Therefore L = ---------------------- = ------------------------------------
hav Do [Tv Tw] 3599 x x 0.025 x (65 35)

= 1.66 m

Example 8.4:- Air free saturated steam at 85 0C condenses on the outer surfaces of 225
horizontal tubes of 1.27 cm OD, arranged in a 15 x 15 array. Tube surfaces are maintained
at a uniform temperature of 75 0C. Calculate the total condensate rate per one metre length
of the tube.

Solution: Data:- Tv = 85 0C; Tw = 75 0C; Do = 0.0127 m; L = 1 m;

Number of tubes in vertical tier = N = 15 ; Total number of tubes = n = 225;

Mean film temperature = 0.5 x (85 + 75) = 80 0C. Properties of the condensate (liquid

water) are: kL = 0.668 W/(m-K); L = 0.355 x 10 3 N-s/m2; L= 974 kg/m3;

At 85 0C, hfg = 2296 x 10 3 J/(kg-K).

For N horizontal tubes arranged in a vertical tier, hav is given by

g L2 hfg kL3
hav = 0.725 [ --------------------- ] 1 / 4
L(Tv Tw)NDo

0.725 x [9.81 x (974)2 x (0.668)3] 1/4


hav = ------------------------------------------------------ = 7142 W/(m2 K)
[0.355 x 10 3 x (85 75) x 15 x 0.0127]

Q = hav Atotal (Tv Tw) = hav n DoL (Tv Tw)

354
= 7142 x 225 x x 0.0127 x 1 x (85 75) = 641.14 x 10 3 W

Mass flow rate of condensate = M = Q / hfg = 641.14 x 10 3 / 2296 x 10 3 = 0.28 kg/ (s-m)

Example 8.5:- Superheated steam at 1.43 bar and 200 0C condenses on a 1.9 cm OD vertical
tube which is 20 cm long. The tube wall is maintained at a uniform temperature of 109 0C.
Calculate the average heat transfer coefficient and the thickness of the condensate at the
bottom of the tube. Assume cp for super heated steam as 2.01 kJ/(kg-K).

Solution: With a superheated vapour, condensation occurs only when the surface
temperature is less than the saturation temperature corresponding to the vapour pressure.
Therefore for a superheated vapour, the amount of heat to be removed per unit mass to
condense it is given by

Q / M = hfg + cpv(Tv Tsat)

Where cp is the specific heat of superheated steam and Tsat is the saturation temperature
corresponding to the vapour pressure. If it is assumed that the liquid vapour interphase is at
the saturation temperature, then Eq.(8.20 ) still holds good with hfg replaced by
hfg + cpv(Tv Tsat).

Hence g L2 kL3 {hfg + cpv(Tv Tsat)}


hav = 0.943 [ --------------------------------------- ] 1/ 4
L(Tsat Tw)L

At 1.43 bar, Tsat = 110 0C.Mean film temperature = 0.5 x (110 + 109) = 109.5 0C.

Properties of the condensate at 109.5 0C are: kL = 0.685W/(m-K); L = 0.259 x 10 3 N-

s/m2; L= 951 kg/m3;At 1.43 bar, hfg = 2230 x 10 3 J/(kg-K).

9.81 x (951)2 x (0.685)3x {2230 x 103 + 2010 x (200 110)}


hav = 0.943 x [ -------------------------------------------------------------------------- ] 1/ 4
0.259 x 10 3 x (110 109) x 0.2

= 18,000 W /(m2 K).

Hence hx|x=L = () x 18000 = 13,500 W / (m2 K).

(x)|x=L = kL / hx|x=L = 0.685 / 13,500 = 5.07 x 10 5m

Example 8.6:- Air free saturated steam at 70 0C condenses on the outer surface of a 2.5 cm
OD vertical tube whose outer surface is maintained at a uniform temperature of
50 0C. What length of the tube would produce turbulent film condensation?

355
Solution: Data:- Tv = 70 0C; Tw = 50 0C; Do = 0.025 m; Vertical tube.

To find L such that Re = 1800.

Mean film temperature = 0.5 x (70 + 50) = 60 0C. Properties of the condensate (liquid

water) are : kL = 0.659W/(m-K); L = 0.4698 x 10 3 N-s/m2; L= 983.2 kg/m3;

At 70 0C hfg = 2358 x 10 3 J/(kg-K).

Re (LDo) 1800 x 0.4698 x 10 3 x x 0.025


Re = 4M / (LDo) or M = ------------------- = ---------------------------------------------
4 4

= 0.0166 kg / s.

For turbulent flow hav [L2 / (gkL3) ] 1 / 3 = 0.0077 (ReL) 0.4

Or hav = 0.0077 (ReL) 0.4[L2 / (gkL3) ] 1 / 3

Hence hav= 0.0077 x (1800)0.4 x [ (0.4698 x 10 3/983.2)2 / (9.81 x 0.6593) ] 1 / 3

= 3563.4 W / (m2 K).

Heat balance equation is M hfg = hav DoL (Tv Tw)

M hfg 0.0166 x 2358 x 10 3


Hence L = ----------------------- = -------------------------------------
hav Do (Tv Tw) 3563.4 x x 0.025 x (70 50)

=7m

Example 8.7:- Saturated steam at 100 0C condenses on the outer surface of a 2 m long
vertical plate. What is the temperature of the plate below which the condensing film at the
bottom of the plate will become turbulent?

Solution: Data:- Tv = 100 0C; L = 2 m. Since Tw is not known, properties of the condensate
at the mean film temperature cannot be determined and therefore the problem has to be
solved by trial and error procedure as follows:

Trial 1:-The properties of the condensate are read at Tv = 100 0C. The properties are

kL = 0.683 W/(m-K); L = 0.2824 x 10 3 N-s/m2; L= 958.4 kg/m3;

At 100 0C, hfg = 2257 x 10 3 J/kg-K.

356
Since the flow has to become turbulent at the bottom of the plate we have

hav = 0.0077 (ReL) 0.4[L2 / (gkL3) ] 1 / 3 with ReL = 1800

9.81 x 0.683 3
Hence hav = 0.0077 x (1800)0.4 x [ ------------------------------ ] 1 / 3
(0.2824 x 10 3 / 958.4) 2

= 5098 W / (m2 K)

Now M hfg = hav L W (Tv Tw)

Or Tw = Tv (M/W)hfg / (hav L). But ReL = 4M / (LW) or M/W = ReL L / 4.

ReL L hfg 1800 x 0.2824 x 10 3 x 2257 x 10 3


Therefore Tw = Tv -------------------- = 100 ----------------------------------------------
4 hav L 4 x 5098 x 2

= 72 0C

Trial 2:-Assume Tw = 72 0C. Mean film temperature = 0.5 x (100 + 72) = 86 0C.

Properties of the condensate at 86 0C ; kL = 0.677 W/(m-K); L = 0.3349 x 10 3 N-s/m2;

L= 968.5kg/m3; At 100 0C, hfg = 2257 x 10 3 J/(kg-K).

9.81 x 0.677 3
Hence hav = 0.0077 x (1800)0.4 x [ ------------------------------ ] 1 / 3
(0.3349 x 10 3 / 968.5) 2

= 4541 W /(m2 K).

1800 x 0.3349 x 10 3 x 2257 x 10 3


Therefore Tw = 100 ----------------------------------------------- = 60 0C
4 x 4541 x 2

Since the calculated value of Tw is quite different from the assumed value, one more iteration
is required.

Trial 3:-Assuming Tw = 60 0C and proceeding on the same lines as shown in trial 2 we

get hav = 4365 W /(m2 K) and hence Tw = 590C. This value is very close to the value
assumed (difference is within 2 % ). The iteration is stopped. Hence Tw = 59 0C.

357
Example 8.8:- Air free saturated steam at 90 0C condenses on the outer surface of a 2.5 cm
OD, 6 m long vertical tube, whose outer surface is maintained at a uniform temperature of
60 0C. Calculate the total rate of condensation of steam at the tube surface.

Solution: Data:- Tv = 90 0C; Tw = 30 0C; Do = 0.025 m; L = 6 m. Vertical tube.

Mean film temperature = 0.5 x (90 + 60) = 75 0C. Properties of the condensate at 75 0C

are: kL = 0.671 W/(m-K); L = 0.3805 x 10 3 N-s/m2; L= 974.8 kg/m3; At 90 0C, hfg =

2283 x 10 3 J/(kg-K).

We do not know whether the condensate flow is laminar or turbulent. We start the
calculations assuming laminar flow and then check for laminar flow condition.
For laminar flow
g L2 kL3 hfg
hav = 0.943 [ ----------------------------- ] 1 / 4
L(Tv Tw) L

9.81 x (974.8)2 x (0.671)3 x 2283 x 10 3


Hence hav = 0.943 x [--------------------------------------------------] 1 / 4
0.3805 x 10 3 x (90 60) x 6

= 2935.3 W /(m2 K).

For laminar flow hav [L2 / (gkL3) ] 1 / 3 = 1.47 Re L 1/ 3

Or ReL = (1.47 / hav)3(gkL3 / L2)

= (1.47 / 2935.3)3 x [9.81 x 0.6713 x 974.82 / (0.3805 x 10 3)2]

= 2443

Since ReL> 1800, flow is turbulent.

For turbulent flow hav [L2 / (gkL3) ] 1 / 3 = 0.0077 (ReL) 0.4

Or hav = 0.0077 (ReL) 0.4 / [L2 / (gkL3) ] 1 / 3.(1)

ReL = 4M / (LDo). But Mhfg = havDoL (Tv Tw) or M / (Do) = havL (Tv Tw) / hfg

4havL (Tv Tw)


Therefore ReL = ----------------------
hfg L

358
Substituting this expression for ReL in equation (1) we have

4havL (Tv Tw)


hav = 0.0077 [------------------------- ] 0.4 [L2 / (gkL3) ] 1 / 3
hfg L

4L (Tv Tw)
(hav )0.6 = 0.0077 [------------------------- ] 0.4 [L2 / (gkL3) ] 1 / 3
hfg L

4 x 6 x (90 60) (0.3805 x 10 3)2


= 0.0077 x [--------------------------------------- ] 0.4 x [ --------------------------------- ] 1 / 3
2283 x 10 3 x 0.3805 x 10 3 974.82 x 9.81 x (0.671)3

= 192. Hence hav = [192] 1 / 0.6 = 6390 W /(m2 K).

havDoL (Tv Tw) 6390 x x 0.025 x 6 x(90 60)


Therefore M = ----------------------- = ------------------------------------------ = 0.0396 kg/s
hfg 2283 x 10 3

8.11.Dropwise Condensation: Experimental investigations on condensation have indicated


that, if traces of oil are present in steam and the condensing surface is highly polished , the
condensate film breaks into droplets. This type of condensation is called drop wise
condensation. The droplets grow, coalesce and run off the surface, leaving a greater portion
of the condensing surface exposed to the incoming steam. Since the entire condensing
surface is not covered with a continuous layer of liquid film, the heat transfer rate for ideal
drop wise condensation is much higher than that for film wise condensation. The heat
transfer coefficient may be 2 to 3 times greater for drop wise condensation than for film wise
condensation. Hence considerable research has been done with the objective of producing
long lasting drop wise condensation. Various types of chemicals have been tried to promote
drop wise condensation. Continuous drop wise condensation, obtainable with different
promoters varies between 100 to 300 hours with pure steam and are shorter with industrial
steam. Failure occurs because of fouling or oxidation of the surface, or by the flow of the
condensate or by a combination of these effects.
It is unlikely that long lasting drop wise condensation can be produced under
practical conditions by a single treatment of any of the promoters currently available.
Therefore in the analysis of a heat exchanger involving condensation of steam, it is
recommended that film wise condensation be assumed for the condensing surface.

8.12. Boiling Types: When evaporation occurs at a solid-liquid interface, it is called as


boiling. The boiling process occurs when the temperature of the surface Tw exceeds the
saturation temperature Tsat corresponding to the liquid pressure. Heat is transferred from the
solid surface to the liquid, and the appropriate form of Newtons law of cooling is

qw = h [Tw Tsat] = h Te (8.27)

359
Where Te = [Tw - Tsat] and is termed as the excess temperature. The boiling process is
characterized by the formation of vapour bubbles which grow and subsequently detach from
the surface. Vapour bubble growth and dynamics depend, in a complicated manner, on the
excess temperature Te, the nature of the surface, and the thermo-physical properties of the
fluid, such as its surface tension. In turn the dynamics of vapour bubble growthaffect fluid
motion near the surface and therefore strongly influence the heat transfer coefficient.
Boiling may occur under varying conditions. For example if the liquid is
quiescent and if its motion near the surface is due to free convection and due to mixing
induced by bubble growth and detachment, then such a boiling process is called pool
boiling. In contrast in forced convection boiling, the fluid motion is induced by an
external means as well as by free convection and bubble induced mixing. Boiling may also
be classified as sub-cooled boiling and saturated boiling. In sub-cooled boiling, the
temperature of the liquid is below the saturation temperature and the bubbles formed at the
surface may condense in the liquid. In contrast, in saturated boiling, the temperature of the
liquid slightly exceeds the saturation temeperature, Bubbles formed at the surface are then
propelled through the liquid by buoyancy forces, eventually escaping from a free surface.

8.13. Pool Boiling Regimes: The first investigator who established experimentally the
different regimes of pool boiling was Nukiyama. He immersed an electric resistance wire
into a body of saturated water and initiated boiling on the surface of the wire by passing
electric current through it. He determined the heat flux as well as the temperature from the
measurements of current and voltage. Since the work of Nukiyama, a number of
investigations on pool boiling have been reported. Fig. 8.4 illustrates the characteristics of
pool boiling for water at atmospheric pressure. This boiling curve illustrates the variation of
heat flux or the heat transfer coefficient as a function of excess temperature Te. This curve
pertains to water at 1 atm pressure.From Eq. (8.27) it can seen that qw depends on the heat
transfer coefficient h and the excess temperature Te.

Free Convection Regime(up to point A):- Free convection is said to exist if Te 5 0 C. In


this regime there is insufficient vapour in contact with the liquid phase to cause boiling at the
saturation temperature. As the excess temperature is increased, the bubble inception will
eventually occur, but below point A (referred to as onset of nucleate boiling,ONB), fluid
motion is primarily due to free convection effects.Therefore,according to whether the flow is
laminar or turbulent, the heat transfer coefficient h varies as Te1/4 or as Te1/3 respectively
so that qw varies as Te5/4 or as Te4/3.

Nucleate Boiling Regime(Between points A and C):- Nucleate boiling exists in the range 5 0
C Te 30 0 C. In this range, two different flow regimes may be distinguished. In the
region A B, isolated bubbles form at nucleation sites and separate from the surface,
substantially increasing h and qw. In this regime most of the heat exchange is through
direct transfer from the surface to liquid in motion at the surface, and not through vapour
bubbles rising from the surface. As Te is increased beyond 10 0C (Region B-C), the
nucleation sites will be numerous and the bubble generation rate is so high that continuous
columns of vapour appear. As a result very high heat fluxes are obtainable in this region. In
practical applications, the nucleate boiling regime is most desirable, because large heat fluxes

360
are obtainable with small temperature differences. In the nucleate boiling regime, the heat
increases rapidly with increasing excess temperature

120
107

106
qw, W/m2

105

104

103

Te until the peak heat flux is reached. The location of this peak heat flux is called the
burnout point, or departure from nucleate boiling (DNB), or the critical heat flux (CHF). The
reason for calling the critical heat flux the burnout point is apparent from the Fig. 8.4. Such
high values of Te may cause the burning up or melting away of the heating element.
Film Boiling Regime:- It can be seen from Fig. 8.4that after the peak heat flux is reached,
any further increase in Te results in a reduction in heat flux. The reason for this
curiousphenomenon is the blanketing of the heating surface with a vapour film which
restricts liquid flow to the surface and has a low thermal conductivity. This regime is called
the film boiling regime. The film boiling regime can be separated into three distinct regions
namely (i) the unstable film boiling region,(ii) the stable film boilingregion and (iii)
radiation dominating region. In the unstable film boiling region, the vapour film is
unstable, collapsing and reforming under the influence of convective currents and the
surface tension. Here the heat flux decreases as the surface temperature increases, because
the average wetted area of the heater surface decreases. In the stable film boiling region, the
heat flux drops to a minimum, because a continuous vapour film covers the heater surface.In
the radiation dominating region, the heat flux begins to increase as the excess temperature
increases, because the temperature at the heater surface is sufficiently high for thermal
radiation effects to augment heat transfer through the vapour film.

361
8.14. Pool Boiling Correlations:

Correlation for The Nucleate Boiling Regime:- The heat transfer in the nucleate boiling
regime is affected by the nucleation process, the distribution of active nucleation sites on the
surface, and the growth and departure of bubbles.Numerous experimental investigations have
been reported and a number of attempts have been made to correlate the experimental data
corresponding to nucleate boiling regime.The most successful and widely used correlation
was developed by Rohsenow. By analyzing the significance of various parameters in relation
to forced - convection effects. He proposed the following empirical relation to correlate the
heat flux in the entire nucleate boiling regime:

Cpl Te qw _______________
------------- = Csf [ --------- * / {g (l v)} ] 0.33 . (8.28)
hfg Prln (l hfg)

where Cpl = specific heat of saturated liquid, J /(kg -0C)

Csf = constant to be determined from experimental data depending upon


Heating surface fluid combination

hfg = latent heat of vapourization, J / kg

g = acceleration due to gravity, m / s2

Prl = Prandtl number of saturated liquid

qw = boiling heat flux, W / m2

Te = excess temperature as defined in Eq. (8.27)

l = viscosity of saturated liquid, kg / (m s)

l, v = density of liquid and saturated vapour respectively, kg / m3

* = surface tension of liquid vapour interface, N / m.

In Eq. (8.28) the exponent n and the coefficient Csf are the two provisions to adjust the
correlation for the liquid surface combination. Table 8-1 gives the experimentally
determined values Csf for a variety of liquid surface combinations. The value of n should be
taken as 1 for water and 1.7 for all other liquids shown in Table 8 1.

362
Table 8 1: Values of Csf of Eq. (8.28) for various liquid surface combinations

Liquid surface combination Csf

Water Copper 0.0130


Water scored copper 0.0068
Water chemically etched stainless steel 0.0130
Water mechanically polished stainless steel 0.0130
Water ground and polished stainless steel 0.0060
Water brass 0.0060
Water nickel 0.0060
Water platinum 0.0130
n-Pentane polished copper 0.0154
n-Pentane lapped copper 0.0049
Benzene chromium 0.1010
Ethyl alcohol chromium 0.0027

Correlations for Peak Heat Flux:- The correlation given by Eq. (8.28) provides information
for the heat flux in nucleate boiling, but it cannot predict the peak heat flux.Based on stability
considerations, Kutateladze and Zuber veloped the following correlation to calculate the peak
heat flux in pool boiling from an infinite horizontal plate facing up.
* g (l v)
qmax = ----- v hfg [ ---------------------- ] [ 1 + v / l ] .(8.29)
24
v2

where * = surface tension of liquid vapour interface, N / m


g = acceleration due to gravity, m / s2
l, v = density of liquid and vapour respectively, kg / m3
hfg = latent heat of vapourization, J / kg
qmax = peak heat flux, W / m2

It is apparent from this equation that large values of hfg, v, g and * are desirable for a large
value of the peak heat flux. For example, water has a large value of hfg; hence the peak heat
flux obtainable with boiling water is high. This equation also shows that a reduced
gravitational field decreases the peak heat flux. For most situations, the quantity
[ 1 + v / l ] is approximately equal to unity. Hence Eq. (8.29) can be written as

qmax = ----- v1/2hfg [* g (l v) ] ..(8.30)


24

Lienhard and Dhir improved the analysis and considered the effect of the size of the
horizontal plate. They showed that

qmax = 0.149 hfg v1/2 [* g (l v) ] .(8.31)

363
The above expression was shown to be valid as long as the plate is large and the
dimensionless quantity L [g(l v) / * ] 2.7, where L is the characteristic dimension of
the plate. For circular plate L is taken as the diameter, while for a square plate it is taken as
the side of the plate.

For the case of horizontal cylinders of radius R, Lienhard & Sun recommended the
following modified form of Eq. (8.30).

qmax = ----- v1/2hfg [* g (l v) ] f(L) ..(8.32)


24

where f(L) = 0.89 + 2.27 exp { 3.44 L }, and L = R [g (l v) / * ] .

Eq. (8.32) is valid for situations in which L 0.15. This equation cal also be used for large
spheres in which case f(L) = 0.84 and L = 4.26. For small spheres
f(L) = 1.734 / L, where 0.15 L 4.26.

Correlations for Film boiling Regime:- No satisfactory correlation exists for the unstable
film boiling region. For the stable film boiling region, Bromley has derived the following
correlation for horizontal cylinders:

kv3 v (l v) g hfg*
ho = 0.62 [ ----------------------------------- ] (8.33)
D v Te

Where ho = average boiling heat transfer coefficient in the absence of radiation W/(m2 K),
D = outside diameter of the tube,
hfg* = Difference between the enthalpy of the vapour at the mean film temperature,Tm
(Tm = [Tw + Tsat] / 2) and the enthalpy of the liquid at the saturation temperature

hfg+ 0.8 Cpv Te.


In the above correlation, all the vapour properties are evaluated at the mean film temperature
while the liquid density is evaluated at the saturation temperature.
Since radiation is significant in film boiling, the radiation component has to be added
in order to obtain the total heat transfer. Brmoley has shown that the total heat transfer
coefficient h is given by the relation

h = ho [ho / h] 1/3 + hr .(8.34)

where hr is the radiation heat transfer coefficient which is calculated from the formula for the
radiation heat exchange between parallel planes;
1 [ Tw4 Tsat4 ]
hr = ----------------------- ------------------------- (8.35)
[ 1 / + 1 / 1] [Tw Tsat

364
where = absorptivity of liquid and = emissivity of the hot tube .

Eq.(8.34) is difficult to use because a trial and error approach is needed to determine h. When
hr is smaller than ho, Eq. (8.34) can be replaced by

h = ho + hr (8.36)

8.15. Illustrative examples on pool boiling

Example 8.9:- Saturated water at 100 0C is boiled with a copper heating element having a
heating surfacearea of0.04 m2which is maintained at a uniform temperatureof 115 0C.
Calculate the surface heat flux and the rate of evaporation of water. Also calculate the
critical flux.

Solution: Given:- Tsat = 100 0C; Tw = 115 0C; Surface area = A = 0.04 m2;

Properties of liquid water at 100 0C are: Cpl = 4216 J/(kg K); hfg = 2257 x 10 3 J/kg;

l= 960.6 kg/m3; v = 0.5977 kg/m3; Prl = 1.75; l = 282.4 x 10 6 kg/(m s);

* = 58.8 x 10 3 N/m ; Te = 115 100 = 15 0C.

Since Te lies between 5 0C and 30 0C, the boiling is in the nucleate regime.

Csf = 0.0130 for water copper combination and for water n = 1.For nucleate boiling

region we can use Eq. (8.28) which is as follows:

Cpl Te qw _______________
------------- = Csf [ --------- * / {g (l v)} ] 0.33 . (8.28)
hfg Prln (l hfg)

Substituting the given values we have


3
4216 x 15 qw 58.8 x 10
----------------------- = 0.013 x [ --------------------------------- x { -------------------- } ] 0.33
(2257 x 10 3) x 1.75 282.4 x 10 6 x 2257 x 10 3 9.81 x (960.6 0.5977)

Solving for qw we get qw = 4.84 x 10 5 W / m2.

Hence total heat transfer = Q = Aqw = 0.04 x 4.84 x 10 5 = 19.36 x 10 3 W = 19.36 kW.

19.36 x 10 3
Rate of evaporation = M = Q / hfg = ----------------- = 8.58 x 10 3 kg / s.
2257 x 10 3

365
Critical heat flux can be calculated from Eq. (8.31) namely

qmax = 0.149 hfg v1/2 [* g (l v) ] .(8.31)

Hence qmax = 0.149 x 2257 x 10 3 x (0.5977)

x [ 58.8 x 10 3 x 9.81 x (960.6 0.5977) ]

= 1.261 x 10 6 W/m2 = 1.261 MW/m2.

Example 8.10:- A metal clad heating element of 6 mm diameter and emissivity equal to unity
is horizontally immersed in a water bath. The surface temperature of the metal is 255 0C
under steady state boiling conditions. If the water is at atmospheric pressure estimate the
power dissipation per unit length of the heater.

Solution: Given:- Tw = 255 0C ; Tsat = saturation temperature of water at 1 atm = 100 0C;

Te = 255 100 = 155 0C. Since Te > 120 0C, film boiling conditions will prevail. The heat
transfer in this regime is given by Eq.(8.33) namely

kv3 v (l v) g hfg*
ho = 0.62 [ ----------------------------------- ]
D v Te

Properties of water at 100 0C are: l = 957.9 kg/m3; hfg = 2257 x 10 3 J/kg;

v = 4.808 kg/m3; Cpv = 2.56 x 10 3 J/(kg-K); kv = 0.0331 W / (m-0C);

v = 14.85 x 10 6 kg / (m-s).

Substituting these values in the expression for ho we have


3 3 3
(0.0331) x 4.808 x (957.9 4.808) x 9.81 x {2257 x 10 + 0.8 x 2.56 x 10 x 155 }
ho = 0.62 x [---------------------------------------------------------------------- ]
14.85 x 10 6 x 0.006 x 155

= 460 W/(m2 K)

1 {Tw4 Tsat4}
hr = ---------------------- x ------------------------------
[1/ + 1/ 1 ] {Tw Tsat}

1 5.67 x 10 8 x { 528 4 373 4}


= ------------------------ x -------------------------------------------
[1/1+1/11] { 528 373}

366
= 21.3 W / (m2-K).

Now h ho + hr = 460 + x 21.3 = 476 W /(m2 K).

Hence Q = h A Te = 476 x ( x 0.006 x 1)x 155 = 1.36 x 10 3 W/ m.

Example 8.11:- A vessel with a flat bottom and 0.1 m2 in area is used for boiling water at
atmospheric pressure. Find the temperature at which the vessel must be maintained if a
boiling rate of 80 kg/h is desired. Assume that the vessel is made of copper and the boiling is
nucleate boiling. Take v = 0.60 kg/m3.

Solution: Given:- A = 0.1 m2; Tsat = 100 0C; M = 80 kg/h = 0.022 kg/s; Prl = 1.75

hfg = 2257 x10 3 J /kg; Cpl = 4216 J/(kg-K); l = 960.6 kg/m3; * = 58.8 x 10 3 N/m;

l = 282.4 x 10 6 kg / (m-s);n = 1; For water-copper combination Csf = 0.0130;

M hfg 0.022 x 2257 x 10 3


qw = Q / A = -------- = ---------------------------- = 4.965 x 10 3 W/m2
A 0.1

For nucleate boiling Eq.(8.28) is used to calculate the excess temperature .Te

Cpl Te qw _______________
------------- = Csf [ --------- * / {g (l v)} ] 0.33
hfg Prln (l hfg)

4216 x Te
----------------------- = 0.013 x {4.965 x 10 5/(282.4 x 10 6x 2257 x 103)
2257 x 10 3 x 1.75 _____________________________
x 58.8 x 10 3 / [9.81 x (960.6 0.6)] } 0.33

Or Te = 15.2 0C

Hence Tw = 100 + 15.2 = 115.2 0C.

Example 8.12:- Calculate the heat transfer coefficient during stable film boiling of water
from a 0.9 cm diameter horizontal carbon tube. The water is saturated and at 100 0C and the
tube surface is at 1000 0C. Take the emissivity of the carbon surface to be 0.8 and assume
that at the average film temperature, the steam has the following properties.
kv = 0.0616 W/(m-K); v = 0.266 kg/m3; v = 28.7 x 10 6 kg/(m-s); Cpv = 2168 J/(kg-K);
l = 958.4 kg/m3

367
Solution: Given:- D = 0.009 m; Te = Tw Tsat = 1000 100 = 900 0C; = 0.8; = 1.0

hfg* = hfg + 0.8 Cpv Te = 2257 x 10 3 + 0.8 x 2168 x 900 = 3818 x 10 3 J/kg.

For stable film boiling the convection coefficient is given by Eq.(8.33)

kv3 v (l v) g hfg*
ho = 0.62 [ ----------------------------------- ]
D v Te

(0.0616)3 x 0.266 x (958.4 0.266) x 9.81 x 3818 x 10 3


ho = 0.62 x [ ---------------------------------------------------------------------- ]
0.009x (28.7 x 10 6) x 900

= 194 W/(m2 K)

Radiation heat transfer coefficient is given by


1 (Tw4 Tsat4)
hr = ---------------------- -------------------------
[ 1/ + 1/ 1] (Tw Tsat)

1 5.67 x 10 8 (1273 4 373 4)


hr = ---------------------- x --------------------------------------
[ 1/0.8 + 1/1 1] (1273 373)

= 131.4 W/(m2 K).

Hence h = ho + hr = 194 + x 131.4 = 292.5

CHAPTER 9

HEAT EXCHANGERS
9.1. Introduction: Heat exchangers are devices in which heat transfer takes place between
two or more fluids.Over the period of time many different types of heat exchangers have
been developed to suit particular applications. We find the use of heat exchangers in steam

368
power plants, chemical processing plants, air conditioning plants, household refrigerators,
automobiles etc.

9.2. Classification of Heat Exchangers: There are many classifications of heat exchangers.
Here we consider classifications based on (i) the heat transfer process, (ii) compactness, (iii)
construction type, (iv) flow arrangement, and (v) heat transfer mechanism.

(i) Classification based on heat transfer process :Based on the heat transfer process, heat
exchangers are classified as
(a) Direct transfer type (or recuperative),
(b) Storage type (or regenerative type)
(c) Direct mixing type.

A direct transfer type of heat exchanger is one in which the cold and hot fluids flow
simultaneously through the exchanger and heat is transferred through a wall separating the
two fluids. This type of heat exchanger is the one which is mostly used.
Cold fluid in

Hot
Fluid in Hot
Fluid out

Cold fluid out

Fig.9.1: Direct transfer type concentric tube heat exchanger


A

simple example of this type is shown in Fig. 9.1. The heat exchanger has a concentric tube
arrangement. One fluid flows through the inner tube, while the other fluid flows through the
annular space between the two tubes. The heat transfer takes place across the wall of the
inner tube. Although both the fluids flow through the exchanger simultaneously, there is no
mixing of the two fluid There are no moving parts.The most serious defect of this type of
heat exchanger is the fact that , with passage of time, scale and dirt tends to accumulate on
the heat transfer surface. This accumulation (called fouling) increases the thermal resistance
to heat flow so that the performance of the heat exchanger deteriorates.
A storage type heat exchanger or regenerator is one in which the heat transfer from
the hot fluid to the cold fluid occurs through a coupling medium in the form of a porous solid
matrix. The hot and cold fluids flow alternately through the matrix, the hot fluid giving heat
to the matrix and the cold fluid extracting heat from the matrix. The arrangement is
illustrated in Fig. 9.2.In many applications, a continuous flow has to be

369
Hot fluid in Cold fluid in

Cold fluid out Hot fluid out

Fig. 9.2: Single matrix storage type heat exchanger

maintained on both the hot and cold sides.In such cases, it is a common practice to use a
rotating disc type matrix, every element of which passes from the hot stream to the cold and
back again during each revolution.. Storage type heat exchangers are commonly used in open
hearth steel melting furnaces and in blast furnaces.It is also used at very low temperatures in
certain cryogenic cycles where a very high degree of performance is required.
Because of alternate flow of hot and cold fluid through the matrix, the storage type
of heat exchanger is self cleaning. This type also results in a much more compact
arrangement than the direct transfer type. The major disadvantages of the storage type are
that some mixing of hot and cold fluids is inevitable and that sealing the hot side from the
cold side in the rotary design presents considerable difficulty.
A direct contact type of heat exchanger is one in which the two fluids come in direct
contact with one another in the heat exchanger. Therefore this type of heat exchanger can be
used only for two immiscible fluids.Cooling towers of steam power plants and scrubbers
used in chemical plants are examples of this type. If heat has to be transferred between a gas
and a liquid, the gas is either bubbled through the liquid or the liquid is sprayed in the form
of droplets into the gas. A schematic diagram of a direct contact type heat exchanger is
shown in Fig. 9.3. Very often in this type of heat exchanger in addition to heat transfer there
will be mass transfer also.

370
Fig. 9.3: Direct mixing type of Heat Exchanger

(ii) Classification based on compactness: The definition of compactness with respect to heat
exchangers is quite arbitrary. The ratio of the maximum heat transfer surface area of the heat
exchanger to its volume called as surface area density, is normally used as the parameter.
A heat exchanger having a surface area density greater than 700 m2 / m3 is arbitrarily referred
to as a compact heat exchanger. Automobile radiators having an area density of the order of
1100 m2 / m3 and the glass ceramic heat exchangers used in vehicular gas-turbine engines
having a surface area density of 6600 m2 / m3 are compact heat exchangers. The human
lungs, with an area density of about 20,000 m2 / m3 , are the most compact heat and mass
exchanger. The very fine matrix regenerator used in sterling engine has an area density
approaching that of the human lung.
The compactness of a heat exchanger becomes very important when heat
exchangers are to be employed in automobiles, marine applications, aircraft, space vehicles,
cryogenic systems, and refrigeration and air conditioning units.

(iii) Classification according to construction: Based on the type of construction, heat


exchangers are classified as (a) tubular type, (b) plate type , (c) plate-fin type, (d) tube-fin
type etc. The tubular type is further classified as (1) concentric tube type (double pipe heat
exchanger) and (2) shell and tube heat exchanger

(iv) Classification based on flow arrangement: Heat exchangers are also classified as (a)
parallel flow, (b) counter flow, (c) cross flow and (d) multi-pass flow.

Parallel flow arrangement: In parallel flow arrangement the hot and cold fluids enter the
same end of the heat exchanger, flow through in the same direction and leave together at the

371
other end as illustrated in Fig. 9.4(a). The temperature distribution along the length of the
exchanger is shown in Fig. 9.4(b) In this figure Thiand Tho are the inlet and exit temperatures

Hot fluid at Thi

Tco Fig. 9.4 (a) Parallel Flow Heat


Exchanger
Tci

T
Hot fluid out at Tho

Thi
Fig 9.4 (b) Temperature
Tho distribution in a parallel flow
T exchanger
Tco

Thi = Inlet Temp. of Hot Fluid


Tci Tci = Inlet Temp. of cold Fluid
Tho = Exit Temp. of Hot Fluid
Tco = Exit Temp. of Cold Fluid
A

of the hot fluid and Tci and Tco are the inlet and exit temperatures of the cold fluid. It is
obvious from the temperature distribution shown that for parallel flow arrangement Tco
cannot exceed Tho in order to satisfy the second law of thermodynamics and in practice it is
necessary that there shall be a finite temperature difference between the two fluids in order to
have a finite amount of heat transfer.Hence Tco is always less than Tho for a parallelflow heat
exchanger.The temperature difference between the two fluids is quite large at one end of the
exchanger and decreases considerably as we proceed towards the other end.

Counter flow arrangement:The schematic and temperature distribution for a counter flow
arrangement are shown in Fig.9.5(a) and Fig. 9.5 (b) respectively. In this type of exchanger
both the fluids flow parallel to, but opposite to each other. It can be seen from Fig. 9.5(b) that

372
Tco has to less than Thi to satisfy the second law. However it is possible to have Tco> Tho in a
counter flow arrangement.

Hot fluid at Thi

Tci Fig. 9.5 (a) Counter Flow


Tco Heat Exchanger

T
Hot fluid out at Tho

Thi
Fig 9.5 (b) Temperature
Tho distribution in a counter flow
Tco
T exchanger

Tci

Cross flow heat exchanger: In this type the two fluids flow through the exchanger at right
angles to each other. The temperature distribution of the two fluids depend on whether each
fluid is mixed or unmixed as it flows through the exchanger. Figure 9.6 illustrates the
direction of flow of the two fluids as well as the temperature distribution for an

373
Cold fluid
in

y
Hot
Fluid
in x

Th = Th (x,y)

Tc = Tc(x,y)
a
a Hot fluid
Outlet
Temperature
distribution

Cold fluid outlet


Fig. 9.6: Unmixed-unmixed cross flow temperature distribution
Heat exchanger

Unmixed-unmixed cross flow heat exchanger. Thermodynamically, the effectiveness for the
cross flow heat exchanger falls in between that for the counter flow and parallel flow
arrangements. The largest temperature difference exists at the corner of the entering hot and
cold fluids such as point a in Fig. 9.6.In a cross flow heat exchanger, a fluid stream is
considered unmixed when it passes through individual flow channels or tubes with no fluid-
mixing between adjacent flow channels.

374
Multipass flow: The multi pass flow arrangement is frequently used in heat exchanger
design as multipassing increases the overall effectiveness individual effectiveness. Fig.9.7
shows some typical arrangements of multipass flow heat exchangers. Fig.9.7(a) is a one shell
pass two tube pass heat exchanger, where as Fig.9.7(b) is a two shell pass four tube pass heat
exchanger. Shell side fluid in

(a) (b)

Shell side fluid out


Fig.9.7: Multipass flow arrangement:- (a) one shell pass two tube pass arrangement;
(b) Two shell pass four tube pass arrangement.

9.3. Overall Heat Transfer Coefficient and Fouling Factor: In the thermal analysis of heat
exchangers,various thermal resistances in the path of the heat flow from the hot fluid to the
cold fluid are combined in the form of overall heat transfer coefficient, U.For example, for a
double pipe heat exchanger, the total thermal resistance is the sum of the individual components;
i.e., resistance of the inside flow, the conduction resistance in the tube material, and the outside
convective resistance, given by

Rtotal = 1 / (Aihi) + t / (kAln) + 1 / (Aoh0) ...(9.1)

Where Ai = diL, ..(9.2a)


Ao = doL, ..(9.2b)
t = (d0 di) / 2 ...(9.2c)
Aiand Ao refer to inner and outer heat-transfer surface areas, respectively, t is the wall thickness,
and Alnis the logarithmic mean heat transfer area, defined as

(Ao Ai)
Aln = ----------------- (9.2d)
ln (Ao / Ai)

and hi and ho are inside and outside heat transfer coefficients respectively.
The total heat transfer resistance can be defined in terms of overall heat transfer
coefficient based on either outer or inner areas, as long as the basis is clearly spelled out.

375
For example, based on outer area, we have

+


Rtotal = + ............................. (9.3)

which after simplifying yields the overall heat transfer coefficient based on inner and
outer areas, respectively as

1
Ui = --------------------------------------------------------------- .(9.4a)
(1/hi)+ [di/(2k)] ln (do/di) + (di/do)( (1/hi)

1
Uo = --------------------------------------------------------------- .(9.4b)
(do/di)(1/hi)+ [d0/(2k)] ln (do/di) + (1/ho)

We note from the above equations that if the wall thickness is negligible Di Do.For example, in
thin tube heat exchangers or the thermal conductivity of the tube material is very high, the
conduction resistance through the tube may be neglected in Equations(9.4a) and (9.4b) to give

= +


..(9.5)

The convection coefficients for the inlet and outlet side of the heat exchanger tube can be
estimated using empirical correlations appropriate for the flow geometry and conditions. During
normal heat exchanger operation, surfaces are subjected to fouling by fluid impurities, rust
formation, and scale depositions, which can markedly increase the resistance to heat transfer
between the fluids. For such situations, one would add the fouling resistance (inside and/or
outside-side) to Equation (9.4a and 9.4 b) to give

1
Ui = -------------------------------------------------------------------------- .(9.6a)
(1/hi) + Fi + [di/(2k)] ln (do/di) +(di/do)Fo + (di/do) (1/ho)

and
1
Uo = ------------------------------------------------------------------------ .(9.6b)
(do/di)(1/hi) + (do/di)Fi + [d0/(2k)] ln (do/di) + Fo + 1/ho
Where di = inside diameter of the inner tube, do = outside diameter of the inner tube,
Fi = Fouling factor for the inside surface of the inner tube,and Fo = Fouling factor for the
outside surface of the inner tube.Values of the fouling factor have been determined
experimentally for a number of situations by conducting tests with clean and fouled surfaces.
Some representative values obatained after running the heat exchangers for long periods of
time are given in Table 9.1.

376
Table 9.1 : Representative values of the fouling factor

Fluid Fouling Factor (m2 K/W)


Distilled water 0.0001
City water (below 500C) 0.0002
City water (above 500C) 0.0004
Sea water & treated boiler feed water (below 500C) 0.0001
Sea water & treated boiler feed water (above 500C) 0.0002
Refrigerating liquids 0.0002
Fuel oil,crude oil 0.001

9.4. Mean Temperature Difference:Consider an elemental area dA of a wall across which


heat is being transferred in a direct transfer type heat exchanger (Fig.9.8). The rate of heat
transfer across the area is given by

dQ = U T dA ..(9.7)

where T = (Th Tc) is the temperature difference between the bulk mean temperatures of
the two fluids on the two sides of the area dA.

Hot Fluid at ThdA

dQ

Cold fluid at Tc

Fig. 9.8: Heat transfer across an elemental area dA

The total heat transfer rate for the heat exchanger is obtained by integrating Eq. (9.7).
A
Hence Q = U T dA
0

If U is assumed to be a constant, then

A
Q= U T dA .(9.8)
0

The mean temperature difference Tm for a heat exchanger is defined such that

Q = UA Tm ...(9.9)

It follows from Eq.(9.8) and Eq.(9.9) that

377
Tm = (1 / A) T dA (9.10)
0

Eq. (9.10) is an important equation describing the performance of a heat exchanger.

For a circular tube the surface area of the heat exchanger can either be inside surface area Ai
or the outside surface area A0. In that case Eq. (9.10) and (9.11) has to be modified as
follows:

Q = U0 A0 Tm ...(9.11a)

Or Q = Ui Ai Tm ...(9.11b)

Where Ui and U0 are the overall heat transfer coefficients based on the outside surface area
and inside surface area respectively.
9.4.1.Mean temperature difference for a parallel flow heat exchanger:The form of the mean
temperature difference, Tm, may be determined by applying an energy balance to differential
control volumes (elements) in the hot and cold fluids. As shown in Fig. 9.9, for the case of
parallel flow arrangement, each element is of length dx and the heat transfer surface area is
dA.Energy balance equation for a surface area dA can be written as follows:

Thi

Tho

Tco

Tci

0 A
Area

Fig.9.9: Mean temperature difference


in Parallel flow

dQ = mhCph = Ch dTh..(9.12 a)

The heat transfer across the surface area dA can be expressed in terms of the overall heat transfer
coefficient as:
dQ = UdA dT .(9.12)

378
where dT = Th Tc is the local temperature difference between the hot and cold fluids.

From Eq.(9.11) we have dTh = dQ / Ch and dTc = dQ / Cc.

dTh dTc = U{ +


Hence } dA (Th Tc)

= U{ + } dA


Or .(9.13)

Integrating Eq. (9.13) between A=0 and A we have

= U{ + } dA


. (9.14)

= + .

Or ln

(9.15)



Now Q = Cc (Tco Tci) or = ------------------------------- (9.16a)



Similarly = ------------------------------- (9.16b)

Substituting these expressions in Eq.(9.15) and rearranging we get


ln = { }

Or Q = UA Tm (9.17)


Where Tm = ln / (9.18a)

and To = Tho Tco and Ti = Thi Tci..(9.18b)

It can be seen from Eq.(9.18a) that Tm is logarithmic mean of and therefore it is


normally referred to as Logarithmic Mean Temperature Difference (LMTD in short.

9.4.2.Mean temperature difference for a counterl flow heat exchanger:

379
Fig.9.9: Mean temperature difference
in Counter flow

For a counterflow heat exchanger, the energy balance equations for an elemental area dA is given
by

dQ = Ch dTh = Cc dTc ..(9.20)

The heat transfer across the surface area dA can be expressed in terms of the overall heat transfer
coefficient as:
dQ = UdA dT .(9.12)

where dT = Th Tc is the local temperature difference between the hot and cold fluids.

From Eq.(9.11) we have dTh = dQ / Ch and dTc = dQ / Cc.

Hence dTh dTc = U {1/Ch 1/Cc} dA (Th Tc)

Or d(Th Tc) / (Th Tc) = U{1/Ch 1/Cc} dA

Integrating Eq. (9.13) between A=0 and A we have

ln {(Tho Tci) / (Thi Tco)} = UA{1/Ch 1/Cc}...(9.21)

380
Substituting for 1/Ch and 1/Cc from Eq.(9.16a) and (9.16b) and rearranging we get

ln{(Tho Tci) / (Thi Tco)} = UA {(Thi Tho) / Q (Tco Tci)/ Q)

Or Q = UA {(Tho Tci) {(Thi Tco)} / ln{(Tho Tci) / (Thi Tco)} ...(9.22)

Or Q = UA Tm


Tm = ln
/
Where (9.23a)

and To = Tho Tci and Ti = Thi Tco ..(9.23b)

The expression for Tm for counter flow [Eq.(9.23)] is identical with that for parallel flow
except that the the symbols Ti and To represent different quantities in each case.

9.4.3.Mean temperature difference for special cases:

Case (i): When one of the fluids is undergoing a change of phase in the heat exchanger:

This situation may arise in the case of a boiler or a condenser. In the boiler the cold fluid will be
undergoing the change of phase at constant pressure so that Tco = Tci as shown in Fig 9.10(a). In
this case the mean temperature difference is same both for parallel flow and counter flow
arrangements.Similarly in a condenser the hot fluid will be undergoing a change of phase so that
Tho = Thi as shown in Fig 9.10(b) and hence in this case also the mean temperature difference is
same both for parallel flow and counter flow arrangements.

381
Fig.9.10(a): Mean temperature difference when Fig.9.10(b) Meantemperature difference when
cold fluid is undergoing a change of phase hot fluid is undergoing a change of phase

Case (ii ): When Ti = To: This situation arises only for counterflow arrangement. When Ti =
To, Eq. (9.23a) gives

Tm= 0 / 0 (Indeterminate).

Hence Tmhas to be obtained using LHospitals rule.

Let Ti / To = p. Then Tm= Lt {To (p 1) /ln p}


p1

Differentiating the numerator and denominator with respect to p we get

Tm= Lt {To ( 1) / (1/p)} = To


p1

Thus the mean temperature difference is equal to the difference in temperature between the hot
and cold side at either end of the heat exchanger. It can be shown that in this case both Th and Tc
vary linearly with the same slope.Consequently the value of Tis the same all along the length of
the heat exchanger.

9.4.4.Mean temperature difference for a cross flow heat exchanger: In both parallel flow
and counter flow arrangements, the temperature on both sides of the wall separating the two
fluids vary along the length of the heat exchanger,x and therefore are functions of a single

382
variable. That is both Th and Tc are functions of x only. This is not the case for cross flow
heat exchanger.
Case(i): consider a single pass cross flow situation in which both fluids are unmixed (see
Fig.9.6). Here the area A across which the heat transfer takes place is in the plane of the
paper with two fluids flowing on either side, one in the x-direction and the other in the y
direction. It is obvious that both Th and Tc are functions of x and y and the exit profile
temperatures are not uniform.
Consider an elemental surface area dA (dA = dx dy). The rate of heat transfer across this area
is given by
dQ = U(Th Tc)dA = U(Th Tc) dx dy ..(9.24)

If U is assumed to be constant then

Q = dQ = U (Th Tc) dx dy (9.25)

Comparing Eq. (9.25) with Eq.(9.9) we have

Tm = (1 / A) (Th Tc) dx dy .(9.26)

The determination of Tm therefore involves double integration and is rather complicated.

Case(ii): If mixing occurs on one side or on both sides the situation will be different from
that in case(i).By mixing we mean a secondary flow transverse to the flow direction takes
place and this permits a fluid to mix with itself and consequently evens out the temperature
of the fluid in the transverse direction. Therefore if the cold fluid is mixed and the hot fluid is
unmixed and if we assume the cold fluid is flowing in x direction and hot fluid is flowing in
y direction then Tc = f(x) and Th = f(x,y).

Case(iii): If both the fluids are mixed then Tc = f(x) and Th = f(y).

The computation for the mean temperature difference for the three cases of cross flow
have ben done by Bowman, Mueller and Nagle.They have represented their results in the
form of a correction factor,F which is defined as follows:

Mean temperature difference in cross flow


F = ------------------------------------------------------------------------------
Mean temperature difference if the arrangement is counter flow

Tm
i.e. F = --------------------------- (9.27)
(Tm)counter flow

Thus the rate of heat transfer for a cross flow heat exchanger can be written as :

Q = UAF (Tm)counter flow (9.28)

383
F is expressed as a function of two parameters R and P where R and P are given by the
following expressions:

(T1 T2)(t2 t1)


R = --------------- and P = ----------------
(t2 t1) (T1 t1)

The parameter R is the ratio of the change in temperatures of the two fluids which is also
equal to the ratio of the heat capacities of the two fluids. P is the ratio of the change in
temperature of one of the fluids to the difference of inlet temperatures of the two fluids.By
definition it follows that R 0 and 0 P 1. Foa cases (i) and (iii) it is immaterial whether T
stands for temperature for hot fluid or cold fluid , but for case (ii) where one fluid is mixed
and the other unmixed,T stands for the temperature of the mixed fluid whether it is hot fluid
or cold fluid and t stands for the temperature of the unmixed fluid

Example 9.1:Water at 25 0 C and a velocity of 1.5 m/s enters a brass condenser tube 6 m long, 1.34
cm ID, 1.58 cm OD and k = 110 W/(m-K). Steam is condensing on the outer surface of the tube with
a heat transfer coefficient of 12,000 W/(m2 K). The fouling factors for the inner and outer surfaces
are both equal to 0.00018 (m2 K) / W. Calculate the overall heat transfer coefficient based on (i) the
inside surface area and (ii) the outside surface area.

Solution:

384
ho=12000 W/m2k
Fo=0.00018 m2-k/W

water hi , Fi = 0.00018 m2-k/W do


o
di
Tfi = 25 C
Uav = 1.5 m

k=110 W/m-k
di=0.0134m
do=0.0158m

To find : (i) Ui ; (ii) Uo

(i) Overall heat transfer coefficient based on inside surface area is given by

1
Ui = -----------------------------------------------------------------------------------
(1/hi )+Fi +[di/2k]loge(do/di)+(di/do)Fo + (di/do)(1/ho)

To find hi: Properties of water at 250C are:

k = 0.6805 W/(m-K) ; = 0.945x 10 6 m 2/s ; Pr = 6.22

Red = Uavdi/ = 1.5x0.0134/0.945x10 6 = 21270

Since Red > 2300, flow is turbulent.Assuming that the flow is hydrodynamically and
thermally developed we have

Nud = hidi/k = 0.023(Red)0.8 x (Pr)0.4 = 138.5

hi = 138.5 x 0.6085 / 0.0134 = 6289.5 W/(m2-K)

1
Ui = ------------------------------------------------------------------------------------------------------

385
(1/6289.5)+0.00018+[0.0134/2x110]loge(0.0158/0.0134)+(0.0134/0.0158)x0.00018

Ui = 1747 W/(m2-K)

(ii) UiAi = Uo Ao

Hence Ui di L = Uo do L

Or Uo = Ui (di/do)

= 1747 x (0.0134/0.0158)

= 1481.6 W/(m2-K)

Example 9.2: In a heat exchanger hot fluid enters at 60 0 C and leaves at 48 0 C, where as the cold
fluid enters at 35 0 C and leaves at 44 0 C. Calculate the mean temperature difference for (a) parallel
flow, (b) counter flow, (c) single pass cross flow (both fluids unmixed), (d) single pass cross flow
(hot side fluid mixed, cold side fluid unmixed), and (e) single pass cross flow (hot side fluid unmixed,
cold side fluid mixed).

Given: Thi = 60 0C ; Tho = 480C ; Tci = 350C ; Tco = 440C

To find: (a) MTD for parallel flow ; (b) MTD for counter flow; (c) MTD for single pass cross flow
with both fluids mixed; (d) single path cross flow with hot fluid mixed and cold fluid unmixed.

(a) MTD for parallel flow:

To = Tho - Tco
Ti = Thi - Tci

Tco

Ti = Thi - Tci =60 35 =250C ;

To = Tho - Tco = 48 44 = 40C

(Ti To) (25 4)

386
For parallel flow, MTD = ------------------- = ---------------- = 11.5 0C
ln(Ti / To) ln (25/4)

(b) MTD for counter flow:

Ti = Thi - Tco = 60 44 = 16o C

To = Tho - Tci =48 35 = 13o C

(Ti To) (16 13)


Hence MTD = ----------------- = ---------------- = 14.45 o C
ln(Ti / To) loge (16/13)

(c) MTD for cross flow with both fluids mixed: MTD = F (MTD)counter flow.

The correction factor F is obtained from the chart as follows:


Since both the fluids are mixed, it is immaterial whether T stands for hot fluid or cold fluid.We take
that T stands for hot fluid and t stands for cold fluid. Then T1 = Thi, T2 = Tho, t1 = Tci and t2 = Tco.

T1

t1 t2

T2
(T1 T2) (60 48) (t2 - t1)(44 35)
Therefore R = ------------- = --------------- = 1.33 ; P = ---------- = -------------- = 0.36
(t2 t1) (44 35) (T1 t1) (60 35)

From chart , F = 0.90.

Hence MTD = 0.90 x 14.45 = 13.010C

(d) MTD for cross flow with cold fluid mixed and hot fluid unmixed:

Since cold fluid is mixed, T stands for temperature ofcold fluid and t stands for temperature of hot
fluid. Therefore T1 = Tci = 35 0C , T2 = Tco = 44 0C, t1 = Thi = 60 0C and t2 = Tho = 48 0C.

(35 44) (48 35)


Therfore R = ---------------- = 0.75 ; P = --------------- = 0.52
(48 60) (35 60)

From chart F = 0.92. Therefore MTD = 0.92 x 14.45 = 13.3 0C

Example 9.3: A simple heat exchanger consisting of two concentric flow passages is used for heating

387
1110 kg/h of oil (Cp = 2.1 kJ/kg-K) from a temperature of 27 0 C to 49 0C.The oil flows through the
inner pipe made of copper (k = 350 W/m-K) with2.54 cm IDand 2.86 cm OD. The surface heat
transfer coefficient on the oil side is 635 W/m2 K. The oil is heated by water supplied at a rate of 390
kg/h and at an inlet temperature of 93 0 C. The water side heat transfer coefficient is 1270 W/m2-K.
the fouling factors o the oil side and water side are 0.0001 and 0.0004 m2-K/W respectively. What is
the length of the heat exchanger required for (i) parallel flow, (ii) counter flow arrangement.

Given : di=0.0254m ; do=0.0286m ; k = 350 W/(m K) ;

Cold fluid : oil mc = (1110 /3600) = 0.3055 kg/s;Cpc = 2100 J/(kg-K) ; Tci = 27 0C ;

Tco = 49 0C; hi = 635 W / (m2 K); Fi = 0.0001 m2-K/W;

Hot fluid : water ; mh = 390/3600 = 0.1083 kg/s; Cph = 4200 J/kg-K ;Thi = 93oC ;

ho = 1270 W/ (m2-K) ; Fo = 0.0004 (m2-K) / W

ho,Fo
water

oil hi , Fi do
di

To find : To find L for (i) parallel flow (ii) counter flow:

Solution: Overall heat transfer coefficient based on outside area of the inner tube is given by
1
Uo = ----------------------------------------------------------------------------------------------------
(do/di) (1/hi) +(do/di) Fi + (do/2k) loge(do/di) + Fo + (1/ho)

1
= ----------------------------------------------------------------------------------------------------------------------------
(0.0286/0.0254)(1/635)+(0.0286/0.0254)0.0001+(0.0286/700)loge(0.0286/0.0254)+0.0004+ (1/1270)

= 325 W/(m2-K)

Heat balance equation can be written as

388
Q = mc x Cpc (Tco Tci) = 0.3083 x 2100 ( 49 27 ) = 14243.5 W

Also Q = mh Cph (Thi Tho)

Or Tho = Thi Q/mh Cph = 93 [14243.5/(0.1083x4200)] = 61.7oC

(i) Parallel flow arrangement :

Ti = Thi Tci = 93 27 = 66oC

To = Tho Tco = 61.7 49 = 12.7oC

hence MTD = Tm = Ti - To / loge (66/12.7) = 32.34 o C

Q = Uo do L Tm

Or L = Q / (Uo doTm) = 14243.5/(325 x x 0.0286 x 32.34) = 15.1 m.

(ii) Counter flow arrangement :In this case Ti = Thi Tco = 93 49 = 440C.

To = Tho Tci = 61.7 27 = 34.7 0C.

Ti To (44 34.7)
Hence Tm = ------------------- = -------------------- = 39.20C.
ln (Ti / To) ln (44 / 34.7)

Hence L = Q / (Uo doTm) = 14243.5 / (325 x x 0.0286 x 39.2) = 12.5 m

9.5.Effectiveness NTU method:

9.5.1. Limitations of Mean Temperature Difference method:- There are generally two types of
problems encountered with heat exchangers. The first type (known as the design type) is the design of
heat exchanger for a given heat load i.e.,for the two fluids, the flow rates, the inlet and exit
temperatures are specified and the heat transfer area required for the heat exchanger is to be
calculated. This type of problem has been solved in Example 9.3 using the mean temperature
difference method. In the second type of problem(known as the rating problem of heat exchanger), for
a given heat exchanger and the two given fluids , only the flow rates and the inlet tempetratures of the
two fluids are specified and the exit temperatures of the two fluids have to be determined.In the mean
temperature difference method,for this type of problem, the exit temperatures of the two fluids have
to be determined by solving two simulataneous equations for both parallel flow and counter flow
arrangements, where as for cross flow and multi pass arrangements the exit temperatures have to be
determined using a trial and error method as illustrated below:

Case (i):Solution for Parallel Flow arrangement:From Eq. (9.15) we have

Tho Tco = exp { UA (1 / Ch + 1 / Cc)}9.29(a)

389
From heat balance equation we have

(Ch/Cc)Tho + Tco = (Ch/Cc)Thi + Tci .9.29(b)

The RHS of both the equations 9.29(a) and 9.29(b) can bedetermined from the given information and
hence these two rquations can be solved for Tho and Tco

Case(ii) Solution for counter Flow arrangement:

Eq.(9.22) can be written as follows:

(Tho Tci)
-------------- = exp { UA [(1 / Ch) (1 / Cc)]
(Thi Tco)

or

Tho + exp { UA [(1 / Ch) (1 / Cc)] Tco = exp { UA [(1 / Ch) (1 / Cc)] Thi + Tci 9.29(c)

Eq.9.29(b) holds good for counter flow arrangement also. Hence Eq.9.29(b) and 9.29(c) can be solved
simultaneously to get Tho and Tco.

Case (iii): Solution for crossflow and multi pass arrangements: Unfortunately the solution for these
type of heat exchangers are not as straight forward as the above two types ,as the mean temperature
difference involves the correction factor, F which requires the knowledge of the exit temperatures.
Hence the problem has to be solved by trial and error method[ ] as shown below.

Calculate R ,assume a value for F and read the value of P from the chart for the particular
arrangement.Knowing P, calculate Tho, Tco and Q . Then calculate Tm for counterflow arrangement
and check for theassumed value of F by calculating F from the equation

Q
F = -------------------------
UA (Tm)counter flow

The procedure is repeated until the solution converges. The method is illustrated for all the three cases
mentioned above with the following example.

Example 9.4. In an oil to water heat exchanger, the oil[Cp = 2000 J/(kg-k)] enters the exchanger at
1000C with aheat capacity rate of 3700 W/K. Water(Cp= 4190 J/kg-K) enters the exchanger at 150C
with a mass flow rate of 0.6 kg/s.Determine the exit temperatures of the two fluids for (i) parallel flow
arrangement, (ii) counterflow arrangement and (iii)on shell pass two tube pass arrangement.Assume
the total surface area of the heat exchanger is 10 m2 and the overall heat transfer coefficient is 500
W/(m2-K)

Given: Hot fluid is oil with: Thi = 100 0C; Ch = 3700 W / K;

Cold Fluid is water with : Tci = 150C ; mc = 0.6 kg/s ; Cpc = 4190 J/(kg-K)

U = 500 W/(m2 K) ; A = 10 m2

390
Solution: (a) Parallel flow arrangement:Cc = mc Cpc = 0.6 x 4190 = 2514 W/K

From Eq.9.29(a) we have Tho Tco = (100 15) exp { 500 x 10 (1 / 3700 + 1 / 2514)}

Or Tho Tco = 3.01 0C .(i)

From Eq.9.29(b) we have (3700/2514) Tho + Tco = (3700/2514) x100 + 15

Or 1.47Tho + Tco = 162.20 .(ii)

Solving for Tho and Tco from Eq.(i) and Eq.(ii) we get

Tho = 66.890C , Tco = 63.88 0C and Q = 3700 x (100 66.89) = 122,507W

(b) Counter flow arrangement:

From Eq.9.29 (c) we have

Tho + exp { 500x10 [(1 / 3700) (1 / 2514)} Tco = exp { 500x10 [(1 / 3700) (1 / 2514)}x 100

+ 15

Or Tho + 1.892 Tco = 204.20C ..(iii)

Solving for Tho and Tco from (ii) and (iii) we get Tho = 57.60C ,Tco = 77.470 C and

Q = 2514 x (77.47 15) = 157,050 W

(c) One shell pass two tube pass arrangement:

Trial 1: Assume F = 0.8 (This generally is a mimum value for good design)

T1 T2 Tco TciCh 3771


R = -------- = --------------- = -------- = ------------ = 1.50
t2 t1 Thi Tho Cc 2514

From chart, corresponding to F = 0.8 and R = 1.50, P = 0.40

Hence ( t2 t1) / (T1 t1) = 0.40

t2 = t1 + 0.40(T1 t1) = 100 + 0.40 x (15 100)

Or t2 = 66.00C = Tho

Q = 3771 x (100 66.0) = 128,214 W

Hence Tco = Tci + Q / Cc = 15 + 128214 / 2514 = 66.000C

(100 66.0) (66 15)

391
(Tm)counter flow = -------------------------------------------------- = 41.930C
ln {(100 66.0) / (66 15)}

Q 128214
Hence F = ------------------------ = --------------------- = 0.612.
U A (Tm)counter flow 500 x 10 x 41.93

The calculated value of F is about 31 % lower than the assumed value. Hence further iteration has to
becarried out. The Table 2 shows the difference between the assumed value of F and calculated value
in % for first 16 iterations.

Table 2: Iterative solution for 1-2 heat exchanger of Example 9.4

Trial F P Tho Tco F (calculated) F in %


(Assumed)
1 0.80 0.4052 65.56 65.69 0.6080 + 31.58
2 0.6080 0.4513 61.64 71.46 0.7701 21.05
3 0.7701 0.4163 64.61 67.09 0.6435 + 19.67
4 0.6435 0.4456 62.13 70.75 0.7477 13.94
5 0.7477 0.4234 64.01 67.98 0.6672 + 12.07
6 0.6672 0.4417 62.46 70.26 0.7328 8.95
7 0.7328 0.4273 63.68 68.46 0.6806 + 7.67
8 0.6806 0.4394 62.65 69.98 0.7243 6.03
9 0.7243 0.4291 63.53 68.68 0.6868 + 5.46
10 0.6868 0.4375 62.81 69.74 0.7173 4.25
11 0.7173 0.4314 63.33 68.98 0.6949 + 3.12
12 0.6949 0.4363 62.92 69.58 0.7125 2.47
13 0.7125 0.4322 63.26 69.08 0.6979 + 2.09
14 0.6979 0.4359 62.95 69.54 0.7113 1.88
15 0.7113 0.4329 63.20 69.17 0.7005 + 1.54
16 0.7005 0.4355 62.98 69.49 0.7099 1.32

It can be seen from the above table that the convergence of the iterations is slow and F continues to
fluctuate, but it will converge. This rating problem can be guaranteed convergent using the Newton-
Raphson method (i.e. the solution of f(x) = 0 where function f is non linear in the present case).

The above example illustrates the limitations of the mean temperature difference method for solving
the rating problem of a given heat exchanger. Though the method is straight forward for parallel flow
and counterflow arrangement, but it requires solving two simultaneous equations to find Tho and
Tco..For crossflow and multi pass type heat exchangers the method involves a trial and error solution
wherein the solution converges very slowly iterating on F.This problem can be solved relatively
straight forward by effectiveness-NTU method.

9.5.2 Definitions of Terms Used in Effectiveness NTU Method:

(i) Capacity ratio,C: It is defined as the ratio of minimum heat capacity to the maximum capacity. i.e.

392
Cmin
C = ---------- .(9.30)
Cmax

Cc (Thi Tho)
If Cc< Ch then C = ------------- = -------------------- ..(9.31a)
Ch (Tco Tci)

Ch (Tco Tci)
If Cc> Ch then C = ------------- = -------------------- ..(9.31a)
Cc (Thi Tho)
(ii) Number of transfer units, NTU:
UA
NTU = --------------- ..(9.32)
Cmin

UA
If Cc< Ch, then NTU = ----------------- (9.33a)
Cc

UA
If Cc> Ch, then NTU = ----------------- (9.33b)
Ch

(iii) Effectiveness, : This defined as the ratio of the actual rate of heat transfer in the exchanger to
the maximumpossible rate of heat transfer.

Exercise Problems:

A. Overall heat transfer coefficient:

9.1.A stainless steel tube [k = 45 W/(m-K)] of inner and outer diameters of 22 mm and
27 mm respectively, is used in a cross flow heat exchanger (see Fig. P 9.2). The
fouling factors for the inner and outer surfaces are estimated to be 0.0004 and
0.0002 (m2K) /W respectively

393
Water at
75 0C & Di = 22 mm D0 = 27 mm
0.5 m/s

Air at 15 0 C & 20 m/s

Fig. P 9.2 : Schematic for problem 9.2

Determine the overall heat transfer coefficient based on the outside surface area
of the tube. Compare the thermal resistances due to convection, tube-wall
conduction and fouling and make comments.

B. Mean Temperature Difference Method:

9.2. A one-shell pass, two-tube pass exchanger is to be designed to heat 0.5 kg/s of
water entering the shell side at 10 0 C. The hot fluid oil enters the tube at 80 0 C
with a mass flow rate of 0.3 kg/s and leaves the exchanger at 30 0 C. The overall
heat transfer coefficient is 250 W/m2-K. Assuming the specific heat of oil to be 2
kJ/kg-K, calculate the surface area of the heat exchanger required.

9.3. A single pass cross flow heat exchanger uses hot gases (mixed) to heat water
(unmixed) from 30 0 C to 80 0 C at a rate of 3 kg/s.The exhaust gases, having
thermo-physical properties similar to air enter and leave the exchanger at 225
and 100 0 C respectively. If the overall heat transfer coefficient is 200 W/m 2-K,
determine the required surface area of the exchanger.

9.4. A two-shell pass, four-tube pass heat exchanger is used to heat water with oil.
Water enters the tubes at a flow rate of 2 kg/s and at 20 0 C and leaves at 80 0
C. Oil enters the shell side at 140 0 C and leaves at 90 0 C. If the overall heat
transfer coefficient is 300 W/m2-K, calculate the heat transfer area required.

9.5. A shell and tube heat exchanger is to be designed for heating water from 25 0 C
to 50 0 C with the help of steam condensing at atmospheric pressure. The water
flows through the tubes (2.5 cm ID, 2.9 cm OD and 2 m long) and the steam
condenses on the outside of the tubes. Calculate the number of tubes required if
the water flow rate is 500 kg/min and the individual heat transfer coefficients on

394
the steam and water side are 8000 and 3000 W/ m2-K respectively. Neglect all
other resistances.

C. Effectiveness NTU method:

9.6. Show that for counter flow heat exchanger with capacity ratio C = 1, the
effectiveness is given by

= NTU / (1 + NTU)

9.7. The following data refer to a heat exchanger.


Mass flow rate of the hot fluid = 4 kg/min.
Mass flow rate of the cold fluid = 8 kg/min.
Specific heat of hot fluid = 4.20 kJ/kg-K.
Specific heat of the cold fluid = 2.52 kJ/kg-K.
Inlet temperature of hot fluid = 100 0 C.
Inlet temperature of cold fluid = 20 0 C.

What is the maximum possible effectiveness if the arrangement is (i) parallel flow
and (ii) counter flow?

9.8. Calculate the exit temperature of the hot fluid and inlet temperature of the cold
fluid for a counter flow heat exchanger having the following specifications.
Mass flow rate of hot fluid = 3 kg/s.
Mass flow rate of cold fluid = 0.75 kg/s.
Cp for hot fluid = 1.05 kJ/kg-K.
Cp for cold fluid = 4.2 kJ/kg-K.
Inlet temperature of hot fluid = 500 0 C.
Exit temperature of cold fluid = 85 0 C.
Overall heat transfer coefficient = 450 W/m2-K.
Total surface area of the exchanger = 1 m2.

9.9. In a gas turbine power plant, heat is transferred in an exchanger from the hot
gases leaving the turbine to the air leaving the compressor. The air flow rate is
5000 kg/h and the fuel-air ratio is 0.015 kg/kg. The inlet temperatures on the air
side and the gas side are 170 0 C and 450 0 C respectively. The overall heat
transfer coefficient for the exchanger is 52 W/m2-K and the surface area of the
exchanger is 50 m 2. If the arrangement is cross flow with both fluids unmixed
determine the exit temperatures of both the fluids and the rate of heat transfer.
Take the specific heats of both the fluids as 1.05 kJ/kg-K.

9.10. A concentric-tube heat exchanger operates on the counter flow mode. The fluid
flowing in the annular space enters the exchanger at 20 0 and leaves at 70 0 C.
The fluid flowing through the inner tube enters at 110 0 C and leaves at 65 0 C.
The length of the exchanger is 30 m. It is desired to increase the outlet
temperature of the cold fluid to 80 0 C by increasing only the length while
maintaining the same mass flow rates, inlet temperatures and tube diameters.
Make any justifiable assumption and calculate the new length.

395
9.11. It is proposed to cool 1000 kg/h of oil from150 0 C to 50 0 C in a heat exchanger
using 1667 kg/h of water at an inlet temperature of 30 0 C. Calculate the surface
required assuming a single pass cross flow arrangement in which the oil is mixed
and the water unmixed. Assume Cp for oil to be 2.087 kJ/kg-K and the overall
heat transfer coefficient to be 550 W/m2-K. Solve the problem by the mean
temperature difference method as well as by the NTU method.

9.12. Two identical counter flow heat exchangers are available. Water at the rate of
5000 kg/h and at 30 0 C is to be heated by cooling an oil (Cp = 2.1 kJ/kg-K) at 90
0
C. The oil flow rate is 2000 kg/h. The heat transfer area is 3 m 2. From the point
of view of maximizing the heat transfer rate, which of the following is the best
arrangement?
(a) Both the fluids flow in series.
(b)The oil flow is split up equally between the two exchangers, while the water flows
in series.
(c) Both oil and water flows are split up equally in both the exchangers.

9.13. A counter flow double pipe heat exchanger is used to heat 1.25 kg/s of water
from 35 0 C to 80 0 C by cooling an oil (Cp = 2.0 kJ/kg-K) from 150 0 C to 85 0 C.
The overall heat transfer coefficient is 850 W/m2-K. A similar arrangement is to
be built at another location, but it is desired to compare the performance of the
single counter flow heat exchanger with two smaller counter flow heat
exchangers connected in series on the water side and in parallel on the oil side as
shown in Fig. P 9.16. The oil flow is split equally between the two exchangers and
it may be assumed that the overall heat transfer coefficient for the smaller
exchangers is the same as for the large exchanger. If the smaller exchanger
costs 20 % more per unit surface area, which would be the most economical
arrangement the one large exchanger or the two equal-sized small exchangers?

A.Overall heat transfer coefficient:

9.1 Solution:

396
ho
Fo= 0.0002 m2-k/W
Di= 0.022m
Do= 0.027m

water hi , Fi = 0.0004 m2-k/W Di Do


o
Q
Tfi = 75 C
Uav = 0.5 m

k=45 W/m-k

Air at T = 15oC
Uav = 20m/s

To find hi: Properties of water at 750C are:

k = 0.6715 W/(m-K) ; = 0.39x 10 6 m 2/s ; Pr = 2.38

Red = Uavdi/ = 0.5x0.022/0.39 x 10 6 = 28205

Since Red > 2300, flow is turbulent.

Nud = hidi/k = 0.023(Red)0.8 x (Pr)0.4 = 118.2

hi = 118.2 x 0.6715/ 0.022 = 3608 W/(m2-K)

Rci = 1/(hiAi) = 1/ hi di L = 1/ ( x 0.022 x 1 x 3608)

= 4.01 x 10 3 k/W

To find ho:

Air is flowing across the cylinder.

Properties of air at 150C are :

397
k = 0.0255 W/(m-K) ; = 14.61x 10 6 m 2/s ; Pr = 0.704

Red = Udo/ = 20 x 0.027/14.61 x 10 6 = 36961

From data hand book,

Nud = hodo/k = [ 0.4 Red0.5 + 0.06Red 0.6667


] x Pr0.4x ( / w )0.25

For gases, ( / w ) = 1

Hence hodo/k = [ 0.4 x (36961)0.5 + 0.06(39961)0.6667] x 0.7040.4 = 130.4

ho = 130.4 x 0.0255/0.027 = 123 W/(m2-K)

Rco = 1/(h0A0) = 1/ h0 d0 L = 1/ ( x 0.027 x 1 x 123)

= 0.096 k/W

Rfi = Fi/ Ai = Fi / di L = 0.004 / ( x 0.022 x 1) = 0.0579 k/W

R = ( 1/ 2 L k ) x log e (do/di ) = ( 1 / 2 x x 1 x 45 ) log e (0.027/0.022)

= 7.24 x 10-3 k/W

Rfo = Fo / Ao = Fo /( do L) = 0.0002 / ( x 0.027 x 1)= 2.36 x 10-3 k/W

Total thermal resistance = R = 4.01 x 10-3 + 0.0579 + 7.24 x 10-3 + 2.36 x 10-3
+ 0.096

= 0.1675 k/W

If Uo is the overall heat transfer coefficient based on outside area then,

Uo Ao = 1/ R

Uo = 1 / Ao R

= 1 / ( x 0.027 x 0.1675)

= 70.4 W/(m2-K)

Comparison between various resistances:

Rci Thermal resistance for 4 x 10-3


convection at the inside
surface.

398
Rfi Resistance due to fouling at 0.0579
the inside surface
R Resistance of the tube wall 7.24 x 10-3
for conduction
Rfo Resistance due to fouling at 2.36 x 10-3
the outside surface
Rco Thermal resistance for 96 x 10-3
convection at the outside
surface

The comparison shows that the thermal resistance for convection heat transfer from the outer
surface of the tube due to air is very large compared to the other resistances i.e,

R Rco

9.2 Solution:

Shell side fluid : water :

mc = 0.5 kg/s ; Cpc = 4200 J/kg-K ;Tci = 10oC ;

Tube side fluid : Oil :

mh = 0.3 kg/s ; Cph = 2000 J/kg-K ;Thi = 80oC ; Tho = 30oC

U = 250 W/(m2-K)

Heat balance equation is

Q = mh Cph (Thi Tho) = 0.3 x 2000 x (80 30)

= 30000 W
Also Q = mc x Cpc (Tco Tci)

Or Tco = Tci + Q/ mc Cpc = 10 + 30000/(0.5 x 4200)

= 24.3oC

(Tm)c.f = (80 24.3) (30 10) / loge [(80 24.3) / (30 10)]

= 34.35oC

Single shell pass - two tube pass HE :

399
T1 = 10oC ; T2 = 24.3oC ; t1 = 80oC ; t2 = 30oC

R = T1 T2 / t2 t1 = 10 24.3 / 30 80 = 0.29

P = t2 t1 / T1 t1 = 30 80 / 10 80 = 0.714

From chart , F = 0.875

Hence (Tm) = F x (MTD)c.f = 0.875 x 34.85 = 30.5oC

Q = U A (Tm)

Or A = Q / U (Tm) = 30000/(250 x 30.5) = 3.93 m2

9.3 Solution:

Cross flow HE :Cold water (Unmixed) : mc = 3 kg/s ; Cpc = 4200 J/kg-K ;Tci = 30oC ;
Tco = 80oC ;

Hot gases (Mixed) : Cph = 1005 J/kg-K ;Thi = 225oC ; Tho = 100oC ; U = 200 W/(m2/K)

Q = mc x Cpc (Tco Tci) = 3 x 4200 (80 30 ) = 630000 W

Also

P = t2 t1 / T1 t1 = 80 30 / 225 30 = 0.256

R = T1 T2 / t2 t1 = 225 100 / 80 30 = 2.50

From chart, F = 0.93

(Tm)c.f = (225 80) (100 30) / loge [(225 80) / (100 30)] = 54.62oC

(Tm) = 0.93 x 54.62 = 50.8oC

Q = U A (Tm) or A = Q / A (Tm) = 630000 / (200 x 54.62) = 57.7 m2.

9.4 Solution:

Water : mc = 2 kg/s; Cpc = 4200 J/kg-K ; Tci = 20oC ;Tco = 80oC ;

Oil : Thi = 140oC ; Tho = 90oC ; U = 300 W / (m2 K )

400
Q = mc x Cpc (Tco Tci) = 2 x 4200 (80 20 ) = 504000 W

Tube side fluid is water. Hence t1 = 20oC ; t2 = 80oC

Shell side fluid is oil. Hence T1 = 140 oC ; T2 = 90 oC

P = t2 t1 / T1 t1 = 80 20 / 140 20 = 0.5

R = T1 T2 / t2 t1 = 140 90 / 80 20 = 0.83

From chart F = 0.97

(Tm)c.f = (140 80) (90 20) / loge [(140 80) / (90 20)] = 64.87oC

(Tm) = 0.97 x 64.87 = 60.9oC

Q = U A (Tm) or A = Q / A (Tm) = 504000 / (300 x 62.9) = 26.7 m2.

9.5 Solution:

Shell and Tube HE : Cold fluid : Water : mc = 8.33 kg/s ;Tci = 25oC ; Tco = 50oC ;

Hot fluid : Steam condensing at atmospheric pressure.

Hence Thi = Tho = Tsat at atmospheric pressure = 99.6o C (from steam tables)

hfg = 2257 x 103 J / kg K

Tube side fluid is water. Hence hi = 3000 W / m2 K

Shell side fluid is oil. Hence ho = 8000 W / m2 K

Inside dia of tube = di = 0.025 m

Outside dia of tube = do = 0.029 m

Length of the tube = L = 2 m

Overall heat transfer coefficient based on outside surface area is given by


1
Uo = ----------------------------------------------------------------------------------------------------
(do/di) (1/hi) + (do/2k) loge(do/di) + (1/ho)

401
1
= ------------------------------------------------------------------ = 1954 W/(m2-K)
(0.029 / 0.025) (1/3000) + 0 + ( 1 / 8000 )

Since Thi = Tho , both parallel flow and counter flow arrangement will give the same

value of (Tm).

Hence (Tm) = (99.6 25) (99.6 50 ) / loge [(99.6 25) / (99.6 50 ) ] = 61.2 oC

Q = mc x Cpc (Tco Tci) = 8.33 x 4200 (50 25 ) = 874650 W

Q = Uo Ao (Tm) or Ao = Q / Uo (Tm) = 874650 / (1954 x 61.2) = 7.314 m2.

Surface area of each tube = ao = do L = x 0.029 x 2 = 0.1822 m2

Hence number of tubes = n = Ao / ao = 7.314 / 0.1822 = 40.14 41

B. Effectiveness - NTU method:

9.6 Solution:

For a counter flow HE effectiveness is given by

1 - [ - (1 c ) NTU ]
= ------------------------------
1 - c[ - (1 c ) NTU]

when c = 1 , the above expression gives

1 - 0
= ------------------ = 0 / 0 = indeterminate.
1 1 x 0

Hence we have to find using Lhospitals rule.

d/dc [1 - e[ - (1 c ) NTU ] ]
= Ltc--> 1 -------------------------------------
d/dc [1 - ce[ - (1 c ) NTU] ]

0 NTU - (1 c ) NTU
= Ltc--> 1 ------------------------------------------------------------------
0 { - (1 c ) NTU x 1 + c (NTU)- (1 c ) NTU }

402
0 NTU NTU
= --------------------------- = --------------------
0 { 1 + NTU } 1 + NTU

9.7 Solution:

mh = 0.067 kg/s ; Cph = 4200 J/kg-K ;Thi = 100oC ;

mc = 0.133 kg/s; Cpc = 2520 J/kg-K ; Tci = 20oC ;

mhCph = 0.067 x 4200 = 281.4 J / s K

mcCpc = 0.133 x 2520 = 335.16 J / s K

mhCph 281.4
Since mhCph< mcCpc , hence c = -------------- = ------------ = 0.84
mcCpc 335.16

For a parallel flow HE ,

1 - e[ - (1 + c ) NTU ]
= ------------------------------ .. (1)
1+c

For a given value of c , will be max if [d/d(NTU)] = 0

From (1),

1 x [ 0 + (1+c)e (1+c)NTU]
[d/d(NTU)] = ------------------------------------- = 0
1+c

Therefore, e (1+c)NTU= 0 or NTU =

Substituting this condition in eqn (1) we have

1 - [ - (1 + c ) ] 10 1 1
max = ---------------------- = --------------- = ----------- = --------------- = 0.5435
1+c 1+c 1+c 1.84

403
For a counter flow HE,

1 - [ - (1 c ) NTU ]
= ------------------------------ (2)
1 - c[ - (1 c ) NTU]

d [1 - c[ - (1 c ) NTU]] [(1-c)- (1 c ) NTU] [1- [ - (1 c ) NTU]][c- (1 c ) NTU(1-c)]


------------ = -------------------------------------------------------------------------------------------
d(NTU) [1 - c[ - (1 c ) NTU]]2

Evaluating, we get 1 c = 0 or c = 1 ; substituting this value of c, we have

= 1 [ from (2) ]

9.8 Solution:

Counter flow HE :

mh = 3 kg/s ; Cph = 1050 J/kg-K ;Thi = 500oC ;

mc = 0.75 kg/s; Cpc = 4200 J/kg-K ; Tco = 85oC ;

U = 450 W / (m2- K )

mhCph = 3 x 1050 = 3150 J / s K

mcCpc = 0.75 x 4200 = 3150 J / s K

mhCph = mcCpc Hence Thi Tho = Tco Tci and c = 1

UA UA
Also NTU = ------------------ or ---------------------
mcCpc mhCph

450 x 1
= -------------- = 0.1428
3150

For a counter flow HE,

404
1 - [ - (1 c ) NTU ]
= ------------------------------
1 - c[ - (1 c ) NTU]

NTU 0.1428
Since c = 1, = ---------------- = ------------------- = 0.1250
1 + NTU 1.1428

Thi Tho Tco Tci


But = --------------- or ------------------ when c = 1
Thi Tci Thi Tci
85 - Tci
Hence 0.1250 = -----------------
500 - Tci

Or 62.5 0.125Tci = 85 - Tci

85 62.5
Or Tci = ---------------------- = 25.7oC
(1 0.125)

Also Thi Tho = Tco Tci when c = 1

Hence 500 - Tho = 85 25.7

Tho = 440.7oC

9.9 Solution:

Hot gases : mh = mc (1 + 0.015) = 1.015mc= 1.015 x (5000/3600) = 1.41 kg/s

Thi = 450oC ;

Cold fluid : Air : mc = 1.39 kg/s; Cpc = Cph = 1050 J/kg-K ; Tci = 170oC ;

U = 52 W / (m2- K ) ; A = 50 m2

mhCph = 1.39 x 1050 = 1459.5 J / s K

mcCpc = 1.41 x 1050 = 1480.5 J / s K

405
mcCpc 1459.5
Since mhCph> mcCpc , hence c = -------------- = ------------ = 0.986
mhCph 1480.5

UA 52 x 50
NTU = ------------------ = -------------- = 1.78
mcCpc 1459.5

From chart for cross flow with both fluids unmixed,

= 0.6

when mhCph> mcCpc , Tco Tci


= ------------------
Thi Tci

Or Tco = Tci + (Thi Tci )

= 170 + 0.6 x [450 170 ]

= 338oC

Also mc x Cpc (Tco Tci) = mh x Cph (Thi Tho)

mcCpc
Hence Tho = Thi ------------- x [ Tco Tci ]
mh Cph

= 450 - 0.986 [ 338 170 ] = 284.35oC

9.10 Solution:

Counter flow HE :

case (i) Thi = 110o C ; Tho = 65o C ; Tci = 20o C ; Tco = 77oC ; L = 30 m ; U1

case (ii) Tci = 20o C ; Tco = 80oC ; Thi = 110o C ; U2 = U1 (assumed)

For case (i) Q1 = mc x Cpc (Tco Tci) = mc x Cpc ( 70 20 )

For case (ii) Q2 = mc x Cpc ( 80 20 )

406
Q1 mc x Cpc ( 70 20 ) 5
Hence, ------ = -------------------------------- = ------
Q2 mc x Cpc ( 80 20 )6

Also for case (i) mc x Cpc ( 70 20 ) = mh x Cph ( 110 65 )

Also for case (ii) mc x Cpc ( 80 20 ) = mh x Cph [ ( 110 (Tho)2 ]

110 65 70 20
Hence --------------------- = --------------
110 - (Tho)2 80 20

hence (Tho)2 = 110 (6/5) [ 110 65 ] = 56o C

(110 70 ) (65 20 )
Hence for case (i) (Tm)1 = ----------------------------------------- = 42.45oC
loge [(110 70 ) / (65 20 )]

(110 80 ) (56 20 )
Hence for case (ii) (Tm)2 = ----------------------------------------- = 32.9oC
loge [(110 80 ) / (56 20 )]

Q1 U1 d L1 (Tm)15
hence , --------- = ---------------------- = -------
Q2 U2 d L2 (Tm)2 6

L1 (Tm)1 5
Hence --------------------------- = ----------
L2 (Tm)2 6

Or L2 = (6/5) x [ (Tm)1 / (Tm)2 ] x L1

= (6/5) x (42.45/32.90) x 30 = 46.45 m

9.11 Solution:

Solution by MTD method:

407
Cross flow HE with oil mixed and water unmixed;

Hot fluid : oil : mh = 0.278 kg/s ; Cph = 2087 J/kg-K ;Thi = 150oC ; Tho = 50oC

Cold fluid : water : mc = 0.463 kg/s ; Cpc = 4200 J/kg-K ;Tci = 30oC;

U = 550 W / (m2 K )

Q = mc x Cpc (Tco Tci) = 0.278 x 2087 [150 50 ] = 58019 W

Also Q = mh x Cph (Thi Tho)

Or Tco = Tci + (Q / mc Cpc)

= 30 + [58019 / (0.463 x 4200 ) ] = 59.8oC

(Tm)c.f = (150 59.8) (50 30) / loge [(150 59.8) / (50 30) ]

= 46.6oC

R = (150 50 ) / (59.8 30 ) = 3.35

P = (59.8 30 ) / ( 150 30 ) = 0.25

From chart F = 0.80

(Tm) = 0.8 x 46.6 = 37.3oC

Q 58019
A = ------------- = --------------- = 2.83 m2
U (Tm) 550 x 37.3

Solution by NTU method :

mhCph = 0.278 x 2087 = 580.2 J / s K

mcCpc = 0.463 x 4200 = 1944.6 J / s K

mhCph 580.2
Since mhCph< mcCpc , hence c = -------------- = ------------ = 0.29
mcCpc 1944.6
when mhCph< mcCpc ,

Thi Tho 150 50

408
= ------------------ = ---------------- = 0.83
Thi Tci 150 30

From chart, NTU = 2.8 = UA / (mCp)min = UA / mhCph

Hence A = 2.8 x 530.2 / 550 = 2.95 m2

9.12. Solution:
Cold: water: mc= 5000/3600 = 1.39 kg/s ; Tci= 30 oC ;Cpc= 4200 J/(kg-K) assumed

Hot : oil : mh = 2000/3600 = 0.555 kg/s ;Cph = 2100 J/(kg-K) ;Thi= 90oC

A= 3m2 ; U = 465 W/(m2-K)

Case (1) Both the fluids flow in series.


The arrangement for this case will be as shown with figure below:

Tco Thi , mhcph


EXCHANGER 2

T c*
T h*
Tci , mhcph

EXCHANGER 1
Tho
As mcCpc = (5000/3600) x 4200 = 5833.3 J/(s-K)

And mhCph= (2000/3600) x 2100 = 1166.7 J/(s-K)

therefore mcCpc< mhCph

Therefore C= 1166.7/5833.3 = 0.20

409
UA 465*3
NTU = = = 1.196
mhC ph 1166.7
1 exp[(1 C ) NTU ]
=
1 C exp[(1 C ) NTU ]
1 exp[(1 0.2)1.196] 1 0.3841
= =
1 0.2 exp[(1 0.2)1.196] 1 0.2*0.3841
= 0.667
Case (2) When oil is split up equally between the two heat exchangers:

In this case mh= 1000/3600 = 0.278 kg/s

Therefore mh Cph = 0.278x2100= 583.8 J/(s-K)

Therefore C= 583.8/5833.3 = 0.10

UA 465*3
NTU = = = 2.4
mhC ph 583.8
1 exp[(1 C ) NTU ]
=
1 C exp[(1 C ) NTU ]
1 exp[(1 0.1)2.4] 1 0.1153
= =
1 0.1exp[(1 0.1)2.4] 1 0.1*0.1153
= 0.895

Case (3) Both the oil and water flows are split up equally:

In this case mcCpc = (2500/3600) x 4200 = 2917 J/(s-K)


mhCph= 0.278 x 2100 = 583.8 J/(s-K)

Therefore C= 583.8/2917 = 0.20


UA 465 * 3
N TU = = = 2.4
m h C ph 583.8
1 exp[ (1 C ) N TU ]
=
1 C exp[ (1 C ) N TU ]
1 exp[ (1 0.2)2.4] 1 0.1467
= =
1 0.2 exp[ (1 0.2)2.4] 1 0.0293
= 0.879

Since is the highest in case (2), this arrangement is the best for maximizing heat transfer

410
9.13Solution:

Oil (mh), Cph=2000 J/kg-k


EXCHANGER 1 Thi= 150oC

mh/2 mh/2
Tci = 35oC
mc =1.25 kg/s
Tco=80oC

T c*
Thi Th1* Th2*

EXCHANGER 2
Tho=85oC

U1 = U2 = U = 850 W / (m2 K )

Case (i) single counter flow HE :

Q = mc x Cpc (Tco Tci) = 1.25 x 4200 [80 35 ] = 236250 W

Q = mh x Cph (Thi Tho)

Q 236250
Hence mh = ------------------------ = ---------------------------- = 1.817 kg/s
Cp [Thi Tho] 2000 x [150 85 ]

mhCph = 1.817 x 2000 = 3634.6 J / s K

mcCpc = 1.25 x 4200 = 5250 J / s K

mhCph 3634.6
Since mhCph< mcCpc , hence c = -------------- = ------------ = 0.7
mcCpc 5250

Thi Tho 150 85


= ------------------ = ------------------- = 0.565
Thi Tci 150 35

From chart, NTU = 1.0 = UA / (mCp)min = UA / mhCph

Hence A = 1 x 3634.6 / 850 = 4.276 m2

411
Case (ii) When two smaller heat exchangers are used:

For this case mh = 1.317 / 2 = 0.9085 kg/s

mhCph = 0.9085 x 2000 = 1817 J / s K

mcCpc = 1.25 x 4200 = 5250 J / s K

Hence c = 1817 / 5250 = 0.35

To find , we should know the exit temperatures of hot and cold fluids for at least one

HE. Since UA and (mCp)min is the same for both the exchangers NTU should be same

for both the exchangers.

Thi Th1* Thi Th2*


Thus 1 = ------------------ = 2 = ----------------
Thi Tci Thi Tc*

150 Thi 150 Th 2*


Therefore = = 1 = 2 ..............................(1)
150 35 150 Tc*

Since the oil flow is the same in each exchanger and the average exit oil temperature must be
85oC ..it follows that
Th1* + Th 2*
= 85o C........................................(2)
2
Energy balance on the second heat exchanger gives

5250(80 Tc* ) = 1817(150 Th 2* )..................................(3)

Equations 1,2 ,3 may be solved for the three unknowns Th1* , Th2* , Tc*. The solutions are as
follows:
Eqn 1can be rearranged after cross multiplying as:

150Th1* 115Th 2* + 150Tc* Th1*Tc* = 5250........................(4)


Eqn 2 Th1* + Th 2* = 170........................................(5)
Eqn 3.. Th 2* 2.9Tc* = 82.....................................(6)

From eqn 6
T * + 82
Tc* = h 2 = 0.345Th 2* + 28.3
2.9

412
From eqn 5. Th1* = 170 Th 2*
Substituting these expressions in Eqn 4 we have:
150(170 Th 2* ) 115Th 2* + 150(0.345Th 2* + 28.3) (170 Th 2* )(0.345Th 2* + 28.3) = 5250

Or Th 2*2 706Th 2* + 57055 = 0


* 706 7062 4*57055
Therefore T =h2 = 93or 613
2
Th2* cannot be 613oC.

Therefore. Th 2* = 93o C

Therefore.. Th1* = 170 93 = 77 o C

And.. Tc* = 0.345*93 + 28.3 = 60.4o C

150 77
Therefore. 1 = 2 = = 0.635
150 33

Therefore from chart, NTU = 1.16

1817 *1.16
Therefore.. A1 = = 2.48m 2 = A2
850

Therefore total area required to meet the heat load = 2.482*2= 4.92m2

This is more than the 4.276m2 required in the one larger heat exchanger. In addition the cost
per unit area is greater so that the most economical choice from the heat transfer point of
view would be the single large heat exchanger.

413
CHAPTER: 10

RADIATION HEAT TRANSFER

10.1. INTRODUCTORY CONCEPTS AND DEFINITIONS

10.1.1 THERMAL RADIATION

When a body is placed in an enclosure whose walls are at temperatures below that of
the body, the temperature of the body will decrease even if the enclosure is evacuated. This
process by which heat is transferred from a body by virtue of its temperature, without the aid
of any intersecting medium is called THERMAL RADIATION. The actual mechanism of
radiation is not yet completely understood. There are at present two theories by which
radiation phenomenon is explained. According to Maxwells electromagnetic theory,
Radiation is treated as electromagnetic waves, while Max Plancks theory treats radiation as
photons or quanta of energy. Neither theory completely describes all observed
phenomena. It is however known that radiation travels with the speed of light, c (c = 3x108
m/s) in a vacuum. This speed is equal to the product of the frequency of the radiation and the
wavelength of this radiation,

OR c = .(10.1)

Where = wavelength of radiation (m) and = frequency (1/s).

Usually, it is more convenient to specify wavelength in micrometer, which is equal to


10-6 m.

From the viewpoint of electromagnetic wave theory, the waves travel at the speed of
light, while from the quantum theory point of view, energy is transported by photons which
travel at the speed of light. Although all the photons have the same velocity, there is always a
distribution of energy among them. The energy associated with a photon, ep = h where h is
the Plancks constant equal to 6.6256 x 10-34 Js. The entire energy spectrum can also be
described in terms of the wavelength of radiation.

Radiation phenomena are usually classified by their characteristic wavelength, . At


temperatures encountered in most engineering applications, the bulk of the thermal energy
emitted by a body lies in the wavelengths between = 0.1 and 100 m. for this reason, the
portion of the wavelength spectrum between = 0.1 and 100 mm is generally referred to as
THERMAL RADIATION. The wavelength spectrum in the range l= 0.1 and 0.7 mm is
visible to the naked eye, and this is called light rays. The wavelength spectrum of radiation is
illustrated in Fig 10.1

414
2
Ultra violet radiation (10 to 0.4 m) 3
Infrared Radiation (0.7 to 10 m )

Thermal Radiation
( 0.1 m to 100 m )

5 4 3 2 1 0 2 3 4 5
10 10 10 10 10 10 10 10 10 10 10

Light Rays (0.1 m to 0.3 m)

, m

Fig. 10.1 Typical Spectrum of electromagnetic radiation

In the study of radiation transfer, a distinction should be made between bodies semi-
transparent to radiation and those which are opaque. If the material is semitransparent to
radiation, such as Glass, salt crystals, and gases at elevated temperatures, then the radiation
leaving the body from its outer surfaces results from emissions at all depths within the
material. The emission of radiation for such cases is a BULK or a VOLUMETRIC
PHENOMENON. If the material is opaque to thermal radiation, such as metals, wood, rock
etc. then the radiation emitted by the interior regions of the body cannot reach the surface. In
such cases, the radiation emitted by the body originates from the material at the immediate
vicinity of the surface (i.e. within about 1mm) and the emission is regarded as a SURFACE
PHENOMENON. It should also be noted that a material may behave as a semi transparent
medium for certain temperature ranges, and as opaque for other temperatures. Glass is a
typical example for such behaviour. It is semi transparent to thermal radiation at elevated
temperatures and opaque at intermediate and low temperatures.

10.1.2 DEFINITIONS OF TERMS USED IN THERMAL RADIATION

Monochromatic Emissive Power (E): The monochromatic emissive power of a surface at


any temperature T and wavelength is defined as the quantity which when multiplied by
d gives the radiant flux in the wavelength range to + d.

Emissive Power (E): The emissive power of a surface is the energy emitted by the surface at
a given temperature per unit time per unit area for the entire wavelength range, from =0 to

415

E = Ed (10.2)
0

Absorptivity, Reflectivity and Transmissibility of a body:

Qt Q

Fig.10.2: Effects of radiation incident on a surface

When a radiant energy strikes a material surface, part of the radiation is reflected, part is
absorbed, and part is transmitted, as shown in Fig. 10.2. Reflectivity () is defined as the
fraction of energy which is reflected, absorptivity () as the fraction absorbed, and
transmissivity () as the fraction transmitted. Thus + + = 1.

Most solid bodies do not transmit thermal radiation, so that for many applied problems,
the transmissivity may be taken as zero. Then

+ = 1 (10.3)

416
Specular Radiation and Diffuse Radiation:

1 2

2 = 1

(a) Specular Radiation (b) Diffuse Radiation

Fig.10.3: Specular and Diffuse Radiation

When radiation strikes a surface, two types of reflection phenomena may be observed. If
the angle of incidence is equal to the angle of reflection, the radiation is called Specular.
On the other hand, when an incident beam is distributed uniformly in all directions after
reflection, the radiation is called Diffuse Radiation. The two types of radiation are
depicted in Fig. 10.3. Ordinarily, no real surface is either specular or diffuse. An ordinary
mirror is specular for visible light, but would not necessarily be specular over the entire
wavelength range.A rough surface exhibits diffuse behaviour better than a highly
polished surface. Similarly, a highly polished surface is more specular than a rough
surface.

Black Body:

A body which absorbs all incident radiation falling on it is called a blackbody. For a
blackbody, = 1, = = 0. For a given temperature and wavelength, no other body at the
same temperature and wavelength, can emit more radiation than a blackbody. Blackbody
radiation at any temperature T is the maximum possible emission at that temperature. A
blackbody or ideal radiator is a theoretical concept which sets an upper limit to the
emission f radiation. It is a standard with which the radiation characteristics of other
media are compared.

Emissivity of a Surface (): The emissivity of a surface is the ratio of the emissive
power of the surface to the emissive power of a black surface at the same

417
temperature. It is denoted by the symbol .

i.e. = [E/Eb]T.

Monochromatic Emissivity of a Surface ():

The monochromatic emissivity of a surface is the ratio of the monochromatic emissive


power of the surface to the monochromatic emissive power of a black surface at the same
temperature and same wavelength.
= [E /Eb ] ,T.
Gray Body:A gray body is a body having the same value of monochromatic emissivity
for all wavelengths. i.e.
=, for a gray body.
Radiosity of A Surface (J):

This is defined as the total energy leaving the surface per unit time per unit area of the
surface. This definition includes the energy reflected by the surface due to some radiation
falling on it.

Irradiation of a surface(G):

This is defined as the radiant energy falling on a surface per unit time, per unit area of the
surface.

Therefore if E is the emissive power, J is the radiosity, e is the irradiation and r the
reflectivity of a surface, then,
J = E + G
For an opaque surface + = 1 or = (1 )

J = E + (1-)G . (10.4)

10.2 LAWS OF RADIATION

10.2.1 STEFAN BOLTZMANN LAW:

This law states that the emissive power of a blackbody is directly proportional to the
fourth power of the absolute temperature of the body. i.e.

Eb T4
Or Eb = T4 ---------------------------------- (10.5)
where is called the Stefan Boltzmann constant.
In SI units = 5.669x10-8 W/(m2-K4).

418
10.2.2 PLANCKS LAW:

This law states that the monochromatic power of a blackbody is given by

C1
Eb = ------------------------------ ..(10.6)
5 [ exp (C2 / T) 1]
where C1 and C2 are constants whose values are found from experimental data; C1 = 3.745 x
10-16 Wm2 and C2 = 1.4388 x 10-2 m-K. is the wavelength and T is the absolute temperature
in K.

10.2.3 WEINS DISPLACEMENT LAW:

It can be seen from Eq. 10.6 that at a given temperature, Eb depends only on .
Therefore this vale of which gives max value for Eb can be obtained by differentiating
Eq(10.6) w.r.t and equating it to zero.

Let C2/T = y. Then Eq. (10.6) reduces to

C1
Eb = -----------------------------
[C2 / (yT)] 5 [ e y 1]

Then dEb C1 d / dy {[C2 / (yT)] 5 [ e y 1]}


------ = ------------------------------------------
dy {[C2 / (yT)] 5 [ e y 1]} 2

or d / dy {[C2 / (yT)] 5 (e y 1)} = 0

Or ey(5 y) = 5

By trial and error, y = 4.965

Therefore, if m denotes the value of which gives maximum Eb, then

C2/mT = 4.965

Or mT = C2/4.965 = 1.4388x10-2 /4.965

mT = 0.002898 m-K .(10.7)

Equation (10.7) is called the Weins displacement law. From this equation it can be seen that
the wavelength at which the monochromatic emissive power is a maximum decreases with
increasing temperature. This is also illustrated in Fig 10.4(a). Fig 10.4(b) gives a

419
comparison of monochromatic emissive powers for different surfaces at a particular
temperature for different wavelengths.

Eb

Wave length, (m)

Fig. 10.4 (a) Black body emissive power as a function of wave length
and Temperature

Fig. 10.4 (b) Comparison of emissive powers of different types o


surfaces as a function of wavelength at a given temperature

420
10.2.4 KIRCHOFFS LAW:

This law states that the emissivity of a surface is equal to its absorptivity when the
surface is in thermal equilibrium with the surroundings.

Proof: Consider a perfect black enclosure i.e. the one which absorbs all the incident radiation
falling on it (see Fig 10.5). Now let the radiant flux from this enclosure per unit area arriving
at some area be qi W/m2.

Black Enclosure
EA

Sample

qi A

Fig. 10.5 : Model used for deriving Kirchoof law

Now suppose that a body is placed inside the enclosure and allowed to come to thermal
equilibrium with it. At equilibrium, the energy absorbed by the body must be equal to the
energy emitted; otherwise there would be an energy flow into or out of the body, which
would raise or lower its temperature. At thermal equilibrium we may write

EA = qiA .(10.8)

If we now replace the body in the enclosure with a black body of the same size and shape and
allow it to come to thermal equilibrium with the enclosure,

EbA = qiA(10.9)

Since = 1 for a blackbody, if Eq. 10.8 is divided by Eq. 10.9 we get

E/Eb =

421
But by definition E/Eb = e, the emissivity of the body, so that

= .(10.10)
Equation 10.10 is called Kirchoffs law and is valid only when the body is in thermal
equilibrium with the surroundings. However, while analyzing radiation problems in practice
we assume that Kirchoffs law holds good even if the body is not in thermal equilibrium with
the surroundings, as the error involved is not very significant.

10.3 ILLUSTRATIVE EXAMPLES ON BASIC CONCEPTS

Example 10.1: The emission of radiation from a surface can be approximated as blackbody
radiation at 1000K.

(a) What fraction of the total energy emitted is below = 5m


(b) What is the wavelength below which the emission is 10.5% of the total emission at
1000K.
(c) What is the wavelength at which the maximum spectral emission occurs at 1000K.

Solution: The radiation flux emitted by the blackbody over the wavelength interval 0 l is
given by

[Eb]0 l = Ebd
0
The integration required in the above equation has been done numerically and the results are
presented in the form of a table. The table gives the value of D 0- where

Ebd 1
D0- = ---0------------ = ------- Eb d
40
Ebd T
0

(a) From Table of Radiation properties, for T = 5 x 1000 = 5000, D0- = 0.6337. This
means that 63.37 % of the total emission occurs below = 5 m.

(b) From the same table, for D0- = 0.105, T = 2222.


Hence = 2222/1000 = 2.222 m.

(c) From Weins displacement law, mT = 0.002898.


Hence for T = 1000 K, m =0.002898 / 1000 = 2.898 x 10 6 m = 2.898 m.

Example 10.2: The monochromatic emissivity of a surface varies with the wavelength in the
following manner:

422
= 0 for < 0.3m
= 0.9 for 0.3m < < 1m
= 0 for > 1m
Calculate the heat flux emitted by the surface if it is at a temperature of 1500K

Solution:
E = Eb
0.3 m 1 m
Therefore E = Eb = 0.0 Ebd + 0.9 Eb d+ 0.0 Ebd
0 0 0.3m 1 m

1 m 1 m 0.3 m
= 0.9 Eb d = 0.9 [ Eb d Eb d ]
0.3 m0 0

= 0.9 T 4 [ D0-1 D0 0.3]

For = 1m, T = 1500 m-K, therefore D0-1 = (0.01972 + 0.00779) = 0.93755

For = 0.3m, T = 450 m-K, therefore D0-3 = 0

Thus E = 0.9x5.67x10-8x15004 [0.013755 0] = 3553 W/m2

Example 10.3: Calculate the heat flux emitted due to thermal radiation from a black surface
at 60000 C. At what wavelength is the monochromatic emissive power maximum and what is
the maximum value?

Solution:Temp of the black surface = 6273K

Heat Flux emitted Eb = T4 = 5.67x10-8x62734 = 87798 KW/m2

Wavelength corresponding to max monochromatic emissive power is given by

mT = 0.002898 m-K

m = 0.002898/6273 = 4.62x10-7 m
The maximum monochromatic emissive power is given by

2 C1
(Eb)max = ------------------------------------
max [ exp {C2 / (maxT)} 1]

2 x x 0.596 x 10 16
= ---------------------------------------------------------------
(4.62 x 10 7) 5 x [ exp{ 0.014387 / 0.002898} 1]

423
= 1.251 x 10 14 W / m2

Example 10.4: The spectral hemispherical emissivity (monochromatic emissivity) of fire


brick at 750K as a function of wavelength is as follows:

1 = 0.1 for 0 < < 2m


2 = 0.6 for 2m < < 14m
3 = 0.8 for14 < <
Calculate the hemispherical emissivity, for all wavelengths.

Solution:

E Eb d 1 1 2 3
0
= ------ = - ------------- = ------ [ 1 Eb d + 2 Eb d + 3 Eb d ]
1 2
Eb T4 T4 0

Where 1 = 2m, 2 = 14m, 3 =

Thus = 1D0-1 + 2[D0-2 D0-1] + 3[D0- D0-2]

Now, 1T = 2x750 = 1500; D0-1 = 0.013

2T = 14x750 = 10500; D0-2 = 0.9243T = ; D0-3 = 1

Hence = 0.1 x 0.013 + 0.6 x [ 0.924 0.013] + 0.8 x [1 0.924] = 0.609

Example 10.5: the filament of a light bulb is assumed to emit radiation as a black body at
2400K. if the bulb glass has a transmissivity of 0.90 for radiation in the visible range,
calculate the percentage of the total energy emitted by the filament that reaches the ambient
as visible light.Assume the wave length rabge corresponding to the visible range as0.38 m
to 0.76 m.

Solution: The wavelength range corresponding to the visible range is taken as


1 = 0.38m to 2 = 0.76m. Therefore the fraction F of the total energy emitted in this range
is given by
2
Eb d 2 1
F = [1--------- ] = [ Eb d Eb d ] / Eb
0 0
Eb
= [D0-2 D0-1].

Now 1T = 0.38 x 2400 = 912. Hence D0-1 = 0.0002

424
and 2T = 0.76 x 2400 = 1824. Hence D0-2 = 0.0436

Therefore F = 0.9 x [0.0436 0.0002] =0.039 .

Only 3.9 % of the total energy enters the ambient as light. The remaining energy produces
heating.

10.4 RADIATION HEAT EXCHANGE BETWEEN INFINITE PARALLEL


SURFACES IN THE PRESENCE OF NON PARTICIPATING MEDIUM

Assumptions:
(i) The medium does not participate in radiation heat exchange between the two
surfaces.
(ii) The surfaces are flat and are at specified uniform temperatures.

10.4.1: RADIATION EXCHANGE BETWEEN TWO PARALLEL BLACK SURFACES

J1 = Eb1
T1 G1 = J2

G2 = J1 J2 = Eb2
T2

Fig: 10.6 Radiation heat exchange between two parallel black surfaces.

Since both surfaces are parallel, flat and infinite, radiosity of surface 1 = irradiation of
surface 2 and vice versa. i.e. J1 = G2 and J2 = G1. Since both the surfaces are black, J1 = Eb1 =
T14 and J2 = Eb2 = T24

Net radiation leaving A1 = Qr1 = A1(J1 G1)All this energy will reach A2.

425
Net radiation leaving A1 and reaching A2 is given by

Q1-2 = Qr1 = A1(J1 G1) = A1[J1 J2]

Or Q1-2 = A1[Eb1 Eb2]

Or Q1-2 = A1[T14 T24] (10.11)

10.4.2 RADIATION HEAT EXCHANGE BETWEEN TWO PARALLEL INFINITE


GRAYSURFACES:

J1
T1, 1, 1,A1 G1 = J2

G2 = J1 J2
T2, 2 ,2, A2

Fig: 10.7 Radiation Heat Exchange Between 2 Parallel Infinite Gray Surfaces.

Since the net radiation leaving A1 will reach A2,


Q1-2 = Qr1 = A1[J1 G1] (10.12a)

J1 = E1 + (1-1)G1 (10.12b)

J2 = E2 + (1-2)G2 (10.12c)

J1 = G2 (10.12d)

J2 = G1 (10.12e)

Equation (10.12b) can be written as

426
J1 (1 1)G1 = E1 ..(4.12f)

Equation (4.12c) with the help of Eqns. (10.12d) and Eqns. (10.12e) can be rewritten as

(1 2)J1 + G1 = E2 (10.12g)

Solving for J1 and G1 from Eq. (10.12f) and (10.12g) we get

E1 + (1 1) E2
J1 = ----------------------------
1 (1 1)(1 2)

1E b1 + (1 1) 2E b2
Or J1 = ----------------------------- ..(10.13a)
1 (1 1)(1 2)

2E b2 + (1 2) 1E b1
and G1 = ----------------------------- ..(10.13b)
1 (1 1)(1 2)

Substituting these expressions for J1 and G1 in Eq.( 10.12a) we get

A1
Q1-2 = -------------------------- [1E b1 + (1 1) 2E b2 2E b2 (1 2) 1E b1]
[1 (1 1)(1 2)]

A1 [2 1Eb1 1 2Eb2 ]
Or Q1-2 = ------------------------------------
[1 (1 1)(1 2)]

Substituting for Eb1 and Eb2 in terms of temperatures we get

A1 [2 1T14 12T2 4]
Or Q1-2 = ------------------------------------ .(10.14)
[1 (1 1)(1 2)]

427
If Kirchoffs law holds good then 1 = 1 and 2 = 2.

A1 [1 2T14 12T2 4]
Hence Q1-2 = ------------------------------------
[1 (1 1)(1 2)]

A1 (T14 T24)
Or Q1-2 = --------------------------- (10.15)
[ 1 / 1 + 1 / 2 1 ]

10.4.3 PLANE RADIATION SHIELDS: It is possible to reduce the net radiation heat
exchange between two infinite parallel gray surfaces by introducing a third surface in
between them. If the third surface, known as the radiation shield is assumed to be very thin,
then both sides of this surface can be assumed to be at the same temperature.
Fig.10.7 shows a scheme for radiation heat exchange between two parallel infinite gray
surfaces at two different temperatures T1 and T2 in presence of a radiation shield at a uniform
temperature, T3.

Now Q1-3 (T14 T34)


------ = --------------------------- ..(10.16a)
A1 [ 1 / 1 + 1 / 13 1]

Q3-2 (T34 T24)


And ------ = --------------------------- ..(10.16b)
A1 [ 1 / 32 + 1 / 2 1]

428
T1, 1= 1,A1
T3, 13, A3 = A1=A2

T3, 23, A3 T2, 2 = 2, A2

Fig: 10.7 Radiation Heat Exchange Between TwoParallel Infinite Gray


surfaces in presence of a radiation shield

For steady state conditions, these two must be equal..Therefore we have

(T14 T34) (T34 T24)


--------------------------- = ----------------------------
[ 1 / 1 + 1 / 13 1] [ 1 / 32 + 1 / 2 1]

Let X = [ 1 / 1 + 1 / 13 1] and Y = [ 1 / 32 + 1 / 2 1]

Then
(T14 T34) (T34 T24)
---------------= ------------------
X Y

Solving for T3 we get


T14 + (X/Y)T24
T3 = [ ----------------------------- ] 1 / 4 .(10.17)
(1 + X /Y)

Substituting this value of T3 in Eq. (10.16a) we get

429
Q1-3 / A1 = Q3-2 / A1 = (Q1-2 / A)1 Rad.Shield = { T24 [{T14 +(X/Y)T24}/(1 + X/Y)] } / X

.(10.17a)

Special case: When 1 = 2 = 13 = 32 = , then X = Y = 2 1

Hence T3 = [(T14+ T24) / 2 ] ..(10.18a)

{ T14 [(T14 + T24) / 2 ]


and [Q1-2 / A ]1 rad shield = -------------------------------------
[2 / 1]

[ T14 T24 ]
= ------------------------ (10.18b)
2 [2 / 1]

It can be seen from the above equation that when the emissivities of all surfaces are equal,
the net radiation heat exchange between the surfaces in the presence of single radiation shield
is 50% of the radiation heat exchange between the same two surfaces without the presence of
a radiation shield. This statement can be generalised for N radiation shields as follows:

1
[Q1-2 / A]N shields = --------- [Q1-2 / A] without shield (10.18c)
(N + 1)

10.5: ILLUSTRATIVE EXAMPLES ON PLANE RADIATION SHIELDS

Example 10.6:Two parallel infinite grey surfaces of emissivities 0.5 are at temperatures of
400K and 300K. Determine the net radiation heat flux between the two surfaces. Also
determine the reduction in radiation flux when a plane radiation shield having emissivity of
0.5 on both its surfaces is placed between the two grey surfaces. Also determine the steady
state temperature of the shield.
Solution:The radiation flux between two grey surfaces is given by

(T1 4 T24)
q = Q1-2 / A = -------------------------- .
[ 1 / 1 + 1 / 2 1]
5.67 x 10 8 x (400 4 300 4)
Since 1 = 2 = = 0.5, we have q = -------------------------------------
(2 / 0.5 1)

430
Or q = 330.75 W / m2.

When a radiation shield of same emissivity is placed between two grey surfaces, the
temperature of the shield T3is given by

T3 = [(T14 + T24) / 2 ]

Hence T3 = [(400 4 + 300 4) / 2]

= 360.3 K

Also, since the emissivities of the plates and shield are equal we have

(q)1 shield = q / 2 = 330.75 / 2 = 163.375 W /m2

Example 10.7: Two parallel plates are at temperatures T1 and T2 and have emissivities
1 = 0.8 and 2 = 0.5. A radiation shield having the same emissivity 3 on both sides is placed
between the plates. Calculate the emissivity 3 of the shield in order to reduce the radiation
heat loss from the system to one tenth of that without shield.

Solution: Radiation flux between the two plates without the presence of a radiation shield is
given by

[ T14 T24 ] [ T14 T24 ]


q1-2 = ------------------------- = ---------------------------
[ 1 / 1 + 1 / 2 1 ] [ 1 / 0.8 + 1 / 0.5 1]

[ T14 T24 ]
or q1-2 = -------------------- (1)
2.25

When a shield is placed between the plates, the radiation flux is given by

431
T 1 4 + ( x / y ) T 24
T14
1+ x / y
( q )1 r a d s h i e l d =
x
1 1 1 1 1
x = + 1 = + 1 = 0 .2 5 +
1 3 0 .8 3 3
1 1 1 1 1
y = + 1 = + 1 = 1+
3 2 3 0 .5 3
(1 + x / y ) T 1 4 T 1 4 ( x / y ) T 24
( q )1 r a d s h i e l d =
x (1 + x / y )

(q )1 r a d s h i e l d =
(T 1
4
T 24 )
x + y

But (q)1radshield = 0.1xq1-2

(T 1
4
T 24 )= 1 (T 1
4
T 24 ) x + y = 2 2 .5 0 .2 5 +
1
+1+
1
= 2 2 .5
x + y 10 2 .2 5 3 3
2 2
= 2 1 .2 5 3 = = 0 .0 9 4
3 2 1 .2 5
Example 10.8: Two large parallel plates are at 800K and 600K have emissivities of 0.5 and
0.8 respectively. A radiation shield having emissivity of 0.1 on the surface facing 800K plate
and 0.05 on the surface facing 600K plate is placed between the plates. Calculate the heat
transfer rate per m2 with and without the shield. Also calculate the temperature of the shield.

Solution:

The radiation flux without the radiation shield is given by

(T14 T24) 5.67 x 10 8x (800 4 600 4)


q = ------------------------ = -------------------------------------- = 7056 W / m2
[ 1 / 1 + 1 / 2 1] [ 1 / 0.5 + 1 / 0.8 1]

When a radiation field is placed between thick plates the radiation flux is given by

(T14 T34)
(q)1 Rad. shield = ------------------------- with x = [ 1 / 1 + 1 / 13 1]
X

T14 + (x/y) T24


and T3 = [ ------------------ ] with y = [ 1 / 32 + 1 / 2 1]
1 + (x/y)

Now x = [ 1 / 0.5 + 1 / 0.1 1] = 11 and y = [1 / 0.05 + 1 / 0.8 1] = 20.25

432
800 4 + (11/20.25) x 600 4
Therefore T3 = [------------------------------- ] 1/4
1 + (11 / 20.25)

= 746.8 K

5.67 x 10 8 x (800 4 746.8 4)


Hence (q)1 Rad. shield = ---------------------------------------- = 508 W / m 2
11

Example 10.9: Find an expression for the net radiant flux between two infinite parallel
diffuse grey surfaces at temperatures T1 and T2 degrees Kelvin when an infinite opaque plate
of thickness b and thermal conductivity K is placed between them. Assume that all surfaces
have the same emissivity.

Solution: The schematic for the problem is shown in Fig. E10.9. For steady state heat
transfer we have

q = Q1-2 / A = Q1-3 / A = Q3-4 / A = Q4-2 / A

(T14 T34)
Now Q1-3 / A = ------------------------ .(1)
(2 / 1)

Q3-4 / A = k (T3 T4) / b (2)

433
T1,

T3 ,

T4 ,
Plate of thickness b &
Conductivity k T2 ,

Fig. E10.9: Schematic for example 10.9

(T44 T24)
Q4-2 / A = ------------------------ .(3)
(2 / 1)

From Eq. (1), T3 = [ T14 (q / )(2 / 1)] .

Similarly from Eq. (3) we get T4 = [ T24 (q / )(2 / 1)] .

Substituting these expressions for T3 and T4 in Eq. (2) we get

q = (k / b)[{T14 (q / )(2 / 1)}1/4 {T24 + (q / )(2 / 1)}1/4]

Or (qb) / k = [{T14 (q / )(2 / 1)}1/4 {T24 + (q / )(2 / 1)}1/4]

Example 10.10: Calculate the steady heat flow through the composite slab of Fig P10.10
consisting of two large plane walls with an evacuated space in between. The thicknesses
of the walls are 20 and 30cm, they have thermal conductivities of 1.0 and 0.5 W/m-k and the
emissivities of the surfaces facing each other are 0.5 and 0.4 respectively.

Solution :

If is the heat flux through the composite slab then

Q (Ti T1) (473 T1) (473 T1)


q = ------- = ---------- = ----------------------- = ------------------- (a)
A Rci + R1 (1 / 20 + 0.2 /1) 0.25

434
L1 L2

Surface in
contact surface in
with fluid T1 contact
at Ti, hi k1 k2 with fluid
T2 at T0, h0

vacuum
Fig. E10.10: Schematic for example 10.10

(T14 T24) 5.67 x 10 8 x [ T14 T24]


Also q = ---------------------------- = -----------------------------------
(1 / 1 + 1 / 2 1) [ 1 / 0.5 + 1 / 0.4 1]

5.67 x 10 8 x [ T14 T24]


Or q = ------------------------------------ (b)
3.5
(T2 T0) (T2 313) (T2 313)
and q = --------------------- = --------------------------- = --------------- ..(c)
R2 + Rco (0.3 / 0.5) + (1 /10) 0.7

From Eq. (a) T1 = 473 0.25 q, and from Eq. (c) T2 = 313 + 0.7q.

Substituing these expressions for T1 and T2 in Eq. (b) we get

5.67 x 10 8 [ (473 0.25q) 4 (313 + 0.7q) 4]


q = ------------------------------------------------------------
3.5

Solving the above equatin by trial and error method we get q = 139 W / m 2.

435
10.6 VIEW FACTOR OR CONFIGURATION FACTOR:

In engineering applications, we come across problems involving radiation heat


exchange between two or more finite surfaces. When the surfaces are separated from each
other by a non participating medium that does not absorb, emit or reflect radiation, then the
radiation heat exchange is not affected by the medium. A vacuum is a perfect non
participating medium. However, air and many gases closely approximate this condition. For
any two surfaces, the orientation of them with respect to each other affects the fraction of the
radiation energy leaving one surface and striking the other directly. The concept of VIEW
FACTOR (also called as CONFIGURATION FACTOR/SHAPE FACTOR) has been
utilised to formalise the effects of orientation in the radiation heat exchange between
surfaces. Before the concept of view factor is introduced, two more terms have to be defined.

10.6.1 SOLID ANGLE AND INTENSITY OF RADIATION:

Solid Angle: The solid angle dw subtended by an elemental area dA surrounding point P
with respect to any other point O in space is defined as the component of the area dA in the
direction OP divided by the square of the distance between O and P. This is illustrated in Fig.
10.9. Solid angle is measured in Steradian (Sr).
n
dA
P
n

d r
r

d = dA / r2 d = dA cos / r2

Fig. 10.9 : Definition of solid angle


Based on this definition, it can be readily inferred that the solid angle subtended by a
hemispherical surface from its centre is 2 (dw = 2xr2/r2) and by a full glass sphere from its
centre is 4.
Intensity of Radiation: The total intensity of radiation emitted by the surface in a given
direction is equal to the radiant flux passing in that direction per unit solid angle. If I is the
intensity of radiation and E is the total emissive power, then by definition

I = dE/dw (10.19a)

E = Idw (10.19b)

436
Where the integration is carried out over all directions encompassed by a hemisphere.

Consider an elemental area dA1 whose total emissive power is E1. This total radiant
energy emitted by dA1 can be intercepted by a hemisphere as shown in Fig 10.10.

dAn

P

OP = r

O

d
dA1

Fig 10.10: Radiation from a differential area dA1 into surrounding hemisphere
centered at dA1.

If I is the intensity of radiation at any point P on the surface of the hemisphere due to
emission by an elemental area dA1 at O, then

dA
E1 = I 1 cos dw = I 1 c o s 2n
r

2 I 1 c o s r s in d r d
E1 =
2
=0 =0 r2
Assuming that I1 is same in all directions (Lamberts Law)

2
E1 = I1 2
c o s s in d d (10.20)
=0 =0

E1 = I1

If the surface is a black surface then

Eb = Ib (10.21)

10.6.2: VIEW FACTOR BETWEEN TWO ELEMENTAL SURFACES.

437
Consider 2 elemental surfaces of area dA1 and dA2 as shown in Fig. 10.11. Let their
normals n1 and n2 make angles 1 and 2 with the line joining the centroid of the two
elemental areas. Let dw2 be the solid angle subtended by dA2 at dA1 and dw1 be the solid
angle subtended by dA1 at dA2.

n1 n2

2
dA2

dA1

Fig. 10.11: View factor between two elemental areas

Let I1 be the intensity of radiation from dA1 striking dA2 and I2 be the intensity of radiation
from dA2 striking dA1. If dq1->2 is the radiant energy leaving dA1 and striking dA2 then
d q 1 2 = I 1 d A1 c o s 1 d w 2
d A2 co s 2
O r d q 1 2 = I 1 d A1 c o s 1
r2
Radiation energy leaving dA1 = dqr1 = E1dA1

Fraction of energy leaving dA1 and striking dA2 is defined as the view factor of dA2 with
respect to dA1 and is denoted by dF1-2

dq1-2 I1 dA1 dA2 cos 1 cos 2


Therefore dF1-2 = ------------- = -----------------------------------
dqr1 r2 E1 dA1

Using the relation E1 = I1 we have

dA2 cos 1 cos 2


dF1-2 = ------------------------- (10.22)
r2

438
Similarly, the view factor of dA1 with respect to dA2 is denoted by dF2-1 and given by

dA1 cos 1 cos 2


dF2-1 = ------------------------- (10.23)
r2
It follows from Eq. 10.22 and 10.23 that

dA1dF1-2 = dA2dF2-1 (10.24)

10.6.3: VIEW FACTOR FOR FINITE SURFACES:

n1

n2
2 dA2

r A2

A1 1

dA1

Fig.10.12: View Factor between two finite areas


Consider two finite

surfaces of areas A1 and A2 as shown in Fig 10.12. If n1 and n2 are the


Normals for elemental areas dA1 and dA2 then energy leaving dA1 and reaching dA2 is given
by

dq12 = I1 dA1 cos 1 dA2 cos 2 / r2

Hence Q12 = [{ I1 dA1 dA2 cos 2} / r2].


A1 A2

439
Total radiation emitted by A1 = Qr1 = E1A1 = I1 A1

Fraction of enrgy which leaves A1 and reaches A2 is given by

Q12 [{ I1 dA1 dA2 cos 2} / r2].


F1-2 = -------- = ---------------------------------------
Qr1 I1 A1

1
Or F1-2 = ------[ { I1 dA1 dA2 cos 2}/ ( r2)] (10.25a)
A1

1
Similarly F2-1 = ------[ { I1 dA1 dA2 cos 2}/ ( r2)] (10.25b)
A2

It follows from Equations (10.25a) and (10.25b) that

A1F1-2 = A2F2-1 .(10.26)

Properties of view factor: Consider an enclosure consisting of N zones, each of surface area
Ai (i = 1,2,3 N). The surface of each zone may be plane, convex or concave. For the
enclosure, the following relations hold good.

1. AiFi-j = AjFj-i , i= 1,2,3 N, j = 1,2,3 N

2. Fi-i = 0 if Ai is plane or convex (i.e. Ai cannot see itself)


0 if Ai is concave.
3. F1-1 + F1-2 + + F1-N = 1
F2-1 + F2-2 + + F2-N = 1
|
|
FN-1 + FN-2 + + FN-N = 1
N
In short, Fi,j = 1, i = 1,2,3 N
j=1

4. When there are two surfaces, one surface say A1 is completely enclosed by A2 and
if A1 cannot see itself then, F1-2 = 1 and F2-1 = A1/A2
5. The view factor F1-2 between surfaces A1 and A2 (Fig. 10.13) is equal to the sum
of the view factors F1-3 and F1-4 if the two areas A3 and A4 together make up the
area A2.

440
A2 = A3 + A4

A3
A4

A1

Fig. 10.13 : Additive relation between view factors

i.e., F1-2 = F1-3 + F1-4 . It should however be noted that

F2-1 F3-1 + F3-4

View factors for standard configurations: The determination of view factors has been the
object of considerable research. In cases where the integrals in Eq. 10.25 and Eq. 10.26
cannot be solved analytically, numerical methods have been used. Some of these results are
represented graphically for certain standard configurations like
(i) Shape factors between parallel rectangles of equal size.
(ii) Shape factors between rectangles perpendicular to each other and having a
common edge
(iii) Shape factor from an elemental area dA1 to a rectangular area A2
(iv) Shape factor between two coaxial parallel discs
(v) Shape factors for concentric cylinders of finite length etc.

With the help of those charts and View Factor algebra, shape factors between surfaces not
covered above can be determined.

441
10.7 ILLUSTRATIVE EXAMPLES ON VIEW FACTORS:

Example 10.11: Determine the view factor between an elemental area A1 and a circular disc
A2 of radius R. The two areas are parallel to each other and positioned at a distance L form
each other such that the perpendicular to A1 passes through the centre of A2.

L 1
r

Fig.E10.11: Figure for example 10.11

d A1 d A 2 c o s 1 c o s 2
A 1 F1 2 =
A1 A 2
r2
S in c e A1 < < A 2 a n d 1 = 2 = , th e a b o v e e x p c a n b e w r it te n a s
R
d A2 c o s 2 2 d co s 2
A 1 F1 2 = A1
A2
r2
F1 2 =
0
r2
L
N o w c o s 1 = c o s = and r = L2 + 2
2 2
L +
R R
2 d L2 2 d
F1 2 = ( L2 + 2 ) ( L2 + 2
)
= L2 2
0 0 (L 2
+ 2
)
R2
F1 2 = 2 2
= s in 2
L + R

Example 10.12: Obtain an expression for the shape factor for a conical cavity with respect to
itself. The height of the cavity is H and the semi vertex angle of the cavity is

442
(See Fig. E10.12a)

Solution:

A2

H dz

z

r
ds
Fig. E10.12(a) : Figure for
Example 10.12
Fig. E10.12 (b):

Refering to Fig. 10.12(b) we have A1 and A2 form an enclosure.

Hence F1-1 + F1-2 = 1, and F2-1 + F2-2 = 1.

Since A2 cannot see itself, F2-2 = 0. Hence F2-1 = 1.0

Also A1F1-2 = A2F2-1. Therefore F1-2 = A2 / A1.


___________
From Fig. E10.12(b), dA1 = 2 r ds = 2 (dr2 + dz2)
____________
Or dA1 = 2 z tan dz [(dr / dz)2 + 1]
_______________
= 2 z tan dz [ tan 2 + 1 ]

= 2 z dz tan sec
H
Therefore A1 = 2 z dz tan sec = H2 tan sec
0

H 2 tan 2
Hence F1-1 = 1 F1-2 = 1 ------------------------- = 1 sin
H2 tan sec

443
Example 10.13: Consider 3 small surfaces each of area dA1 = dA2 = dA3 = 2 cm2 as shown
in fig E10.13. (a) Calculate the solid angle subtended by dA2 with respect to a point on dA1
(b) The solid angle subtended by dA3 with respect to a point on dA1 and (c) The elemental
Diffuse factors dFdA1-dA2 and dFdA1-dA3.

1 = 30 0; 2 = 45 0 ;

r12 = 300 cm ; r13 = 200 cm ;


dA2
dA1 = dA2 = dA3 = 2 cm2 ;
2
dA3

r12
n3
n2
n1 r13
1
Fig. E10.13: Figure for example 10.13
dA1

If d w 2 1 i s t h e s o li d a n g le s u b t e n d e d b y d A 2 w . r .t a p o i n t o n d A 1 t h e n
d A2 co s 2 2 cos 450
d w 2 1 = = = 1 .5 7 1 0 5 s r
r1 22 3002
d A3 c o s 3
S i m i la r ly d w 3 1 = 2
(b u t 3 = 0 0 )
r1 3
d A3 2
d w 3 1 = 2
= = 5 1 0 5 sr
r1 3 2002
d A2 c o s 12 c o s 2 2 cos 30 0 cos 450
d F d A1 d A 2 = = = 4 .3 3 1 0 6
( r1 2 )
2
300 2

d A3 c o s 13 c o s 3 2 cos2 0 0
S i m i la r ly d F d A1 d A 3 = = = 1 .5 9 1 0 5
r1 23 2002

Example 10.14: Determine the view factor F1-2 between an elemental surface dA1 and the
finite rectangular surface A2 for the geometric arrangements shown in Fig E10.14

444
4m

8m 8m

3m A2 3m A2 A2
6m

4m 4m

3m
3m
3m

(a) (b) (c)

Fig. E10.14: Figures for example 10.14

Soluiton : (a) The configuration in Fig. E10.14(a) is a standard configuration for which the
analytical expression for F1-2 is given by

1 x y y x
1 1
F1-2 = ---- [ ------------ tan {------------- } + ------------- tan { ----------------} ]
2 (1 + x2) (1 + x2) (1 + y2) (1 + y2)

Where x = L1 / D = 3 /3 = 1 ; and y = L2 / D = 4 / 3 = 1.33

Substituting these values in the expression for F1-2 we get

1 1 1.33 1.33 1
1 1
F1-2 = ---- [ ------------ tan {------------- } + ------------- tan { ----------------} ]
2 (1 + 12) (1 + 12) (1 + 1.332) (1 + 1.332)

Or F1-2 = 0.154

445
(a) The area A2 is divided into two equal areas A3 and A4 both having the common edge
of width L1 = 3 m.Then

F1-2 = F1-3 + F1-4 = 2 F1-3 (Because F1-3 = F1-4)

= 2 x 0.15 v4 = 0.308

(c) In this case A2 is divided into three equal areas A3,A4, and A5 as shown below

Now F1-2 = F1-3 + F1-4 + F1-5

A4 A5 = 3 F1-3

A3 = 3 x 0.154

= 0.462

Example 10.15: Determine the Shape factor F1-2 for the configuration shown in Fig E10.15

446
3m

A1

1m

1m
A2

1m 1m

Fig. E10.15: Figure for example 10.15

Solution:

A6 F6-1 A4 F4-1
F1-2 = F1-6 F1-4= ---------- --------------
A1 A1

A6 A4
= ------ [ F6-5 F6-3 ] ---------- [ F4-5 F4-3 ]
A1 A1

Values of F6-5, F6-3, F4-5 and F4-3 can be obtained from chart as follows.

To find F6-5:

447
A5 = A6 = 3 x 2 = 6 m 2 ;
2m
L1 / W = L2/W = 2/3 = 0.667;

3m From chart F6-5 = 0.22. Similarly we


A5
Get F6-3 = 0.16; F4-5 = 0.32 ; F4-3 = 0.27

2m
A6

6 3x1
Hence F1-2 = ------- [ 0.22 0.16 ] ------- [ 0.32 0.27 ] = 0.07
1x3 3x1

Example 10.16:Find F1-2 for the configuration shown in Fig. E10.16

448
2m

2m
2m
A2
A1

2m

Fig. E10.16: Figure for example 10.16

Refering to Fig. E10.16(a),let

A6 = A2 + A4 = 4 x 2 = 6 m2 and

A4 A5 = A1 + A3 4 x 2 = 6 m2.

A1 = A2 = A3 = A4 = 2 x 2 = 4 m2.
A2
A1
F1-2 = F1-6 F1-4

= (A6F6-1) / A1 F1-4
A3
= (A6/A1)[F6-5 F6-3] F1-4
Fig. E10.16(a)
= (A6/A1)F6-5 {(A3F3-6)/A1} F1-4

Or F1-2 = (A6/A1)F6-5 (A3/A1) [ F3-2 + F3-4] F1-4

But F3-2 = F1-4 and F3-4 = F1-2.

Hence F1-2 = (A6/A1)F6-5 (A3/A1)F1-4 (A3/A1) F1-2 F1-4

A6F6-5 A3F1-4 A1F1-4 2A1F6-5 2A1F1-4


Or F1-2 = ---------------------------------- = -------------------------- = F6-5 F1-4
(A1 + A3) 2A1
To find F6-5:- L2 = L1 = 2 m; W = 4 m.

449
L1 / W = L2 / W = 2 / 4 = 0.5.

From chart F5-6 = 0.25 = F6-5 since A5 = A6. Similarly F1-4 = 0.2.

Hence F1-2 = 0.25 0.20 = 0.05

Example 10.17: Find the Shape Factor F2-1 for the configuration shown in fig E10.17

A2
2m

2m
A1

Fig. E10.17: Figure for example 10.17

0.5
2m

Solution: Refer to Fig. 10.17 (a)

F2-1 = F2-3 + F2-5 = F2-3 + (A5 F5-2) / A2

= F2-3 + (A5 / A2) [ F5-6 F5-4]

= F2-3 + (A6F6-5) / A2 (A5F5-4) / A2

= F2-3 + (A6 / A2) [F6-1 F6-3] (A5F5-4) / A2

= F2-3 + (A6 / A2)F6-1 (A3F3-6) / A2 (A5F5-4) / A2

= F2-3 + (A6 / A2)F6-1 (A3F2-1) / A2 (A5F5-4) / A2

Or [( 1 + (A3 / A2) ] F2-1 = F2-3 + (A6 / A2)F6-1 (A5F5-4) / A2

450
A1 = A3 + A5; A6 = A2 + A4
A4
A2
A2 = A3 = 0.5 x 2 = 1 m 2

A4 = A5 = 1.5 x 2 = 3 m 2

A1 = A6 = 2 x 2 = 4 m2

A3 A5

Fig. E10.17(a)

F2-3 + (A6 / A2)F6-1 (A5F5-4) / A2


Or F2-1 = -----------------------------------------------------
[( 1 + (A3 / A2) ]

From chart : F2-3 = 0.06 ; F5-4 = 0.17 ; F6-1 = 0.2

0.06 + 4 x 0.2 3 x 0.17


Hence F2-1 = --------------------------------- = 0.175
( 1 + 1)

Example 10.18: Find the Shape factor F1-2 for the configuration shown in Fig E10.18

Solution: Refer Fig. E10.18(a).

F1-2 = F1-8 + F1-4 + F1-6. But F1-4 = F1-6.

Hence F1-2 = F1-8 + 2F1-4

= F1-8 + 2 (A4F4-1) / A1

From example 10.16 we have F4-1 = F7-10 F4-3

Hence F1-2 = F1-8 + 2 (A4 /A1) [F7-10 F4-3]

From chart F1-8 =0.15 ; F7-10 = 0.23 ; F4-3 = 0.2

451
a/2
a

A1
a
A2

a Fig. E10.18: Figure for example 10.18

A7 = A1 + A3 ;
A3
A10 = A4 + A8

A1 A4

A5 A8

A6

Fig. E10.18 (a)

a2
Hence F1-2 = 0.15 + 2 x -------------- x [0.23 0.20] = 0.27
[(a/2) x a]

Example 10.19 A1 and A2 are two rectangular flat surfaces having a common edge and
inclined at an arbitrary angle to each other. They are very long along the common edge and
have lengths of ab and ac respectively in the other direction. Show that

452
F1-2 = (ab +ac) bc
2ab

Solution:Refer Fig. E10.19

c A1,A2 and A3 form an enclosure.

A2 Hence F1-1 + F1-2 + F1-3 = 1.


A3 But F1-1 = 0. Hence
F1-2 + F1-3 = 1 .(a)
similarly
F2-1 + F2-3 = 1 .(b)
a b
A1 F3-1 + F3-2 = 1..(c)
Fig. E10.19
From Eq. (a) we get F1-2 = 1 F1-3 ...(d)

From (c) we get F3-1 = 1 F3-2

Or (A1F1-3) / A3 = 1 (A2F2-3)/A3

Or F1-3 = (A3 / A1) (A2F2-3) / A1

= (A3 / A1) (A2 / A1) [1 F2-1]

= (A3 / A1) (A2 / A1) + F1-2

Substituting this expression in Eq. (d) we get

F1-2 = 1 [(A3 / A1) (A2 / A1) + F1-2]

Solving for F1-2 we get


(A1 + A2) A3 (ab x 1) + (ac x 1) (bc x 1)
F1-2 = ---------------------- = -----------------------------------
2A1 2 x (ab x 1)

(ab + ac bc)
= -------------------
2 ab

Example 10.20:(Hottels cross string formula) Obtain an expression for the view factor
between two flat surfaces, which extend to infinity in one direction.

Solution: Refer Fig. E10.20

453
A2 = L2 x 1

A4 = L4 x 1

A6 = L6 x 1

A3 = L3 x 1 A1 = L1 x1

A5 = L5 x 1

Fig. E10.20 : Figure for example 10.20

Consider unit width perpendicular to the plane of the paper. A1, A2, A3 and A4 form an
enclosure. Hence we have
F1-1 + F1-2 + F1-3 + F1-4 = 1 and F1-1 = 0.

Therefore F1-2 + F1-3 + F1-4 = 1 (a)

Similarly A1, A3 and A5 form an enclosure. Therefore we have

F1-3 + F1-5 = 1 ..(b)

Also F3-1 + F3-5 = 1 ..(c)

and F5-1 + F5-3 = 1 ..(d)

From Eq. (b) we have F1-3 = 1 F1-5

= 1 (A5F5-1) / A1

= 1 (A5 / A1) [ 1 F5-3]

= 1 (A5 / A1)+ (A3F3-5) / A1

Or F1-3 = 1 (A5 / A1) + (A3 / A1) [1 F3-1]

= 1 (A5 / A1) + (A3 / A1) F1-3

454
1 (A5 / A1) + (A3 / A1) A1 A5 + A3
Hence F1-3 = --------------------------------- = ---------------------------
2 2A1

A1 A6 + A4
Similarly F1-4 = ------------------------
2A1

Now from Eq. (a) we have F1-2 = 1 [F1-3 + F1-4]

Substituting the expressions obtained for F1-3 and F1-4 we get

A1 A5 + A3 A1 A6 + A4
F1-2 = 1 ------------------- -------------------
2A1 2A1

(A5 + A6) (A3 + A4) (L5 + L6) (L3 + L4)


= ---------------------------- = ----------------------------
2A1 2L1

Example 10.21: A truncated cone has top and bottom diameters of 10cm and 20cm and a
height of 10cm. Calculate the shape factor between the top surface and the side and the side
and itself.

Solution: To find (i) F2-3 and (ii) F3-3. Refer to Fig. E10.21.

(i) F2-1 = (A1F1-2) / A2. F1-2 can be directly obtained fom chart as follows:

L1 / r= 10 / 10 = 1 ; and L2 / r = 5 / 10 = 0.5.

Hence fromchart F1-2 = 0.12.

x (10)2
Therefore F2-1 = -------------- x 0.12 = 0.48; F2-1 + F2-2 + F2-3 = 1 and F2-2 = 0
x (5)2

Hence F2-3 = 1 F2-1 = 1 0.48 = 0.52

455
A2
r2
A3

r1 = 10 cm

r2 = 5 cm
L
L = 10 cm

r1
A1

Fig. E10.21: Figure for example 10.21

(ii) F1-3 = 1 F1-2 = 1 0.12 = 0.88

x (10 2)
Hence F3-1 = (A1F1-3) / A3 = ------------- x 0.88 = 0.525
526.9

Therefore F3-3 = 1 [F3-1 + F3-2] = 1 [0.525 + 0.0775] = 0.397

Example 10.22: Determine the shape factors for the geometries shown in Fig. E10.23(a) to
E10.23(i)

456
Long groove, A1 Surroundings, A2
A1
Long duct

A2

Find F1-2, F2-1 and F1-1 Find F1-2, F2-1 and F1-1

Fig. E 10.23 (a) Fig. E 10.23 (b)

A1 1.0 Surroundings, A3
Find F1-1 and F1-3

Fig. E 10.23 (c)


2.0

A1 is covered by a
A1 hemispherical surface A2 of
radius 1.5. Find F1-2, F1-3,
1.0 F2-1 and F2-2

Fig. E 10.23 (d)

457
A1

A2
2.0 A2
A1

1.0

Fig.E10.23(e): F1-3,F2-3,F2-2,F1-2 for Fig.E10.23 (f): F1-2,F2-1 for sphere


disc surrounded by a short cylinder on infinite plane

A1
A2
A1 A3
1.0
A2
1.5

1.5
1.0
3.0

Fig.E10.23 (g): F1-2,F2-1 ,F2-3 Fig.E10.23(h): F1-2,F2-1 ,F1-3, F2-3

A1

A2

Fig. E10.23 (i) : F1-2, F2-1, F1-3;


1.0 hemispherical shell, A1 enclosing a
3.0 small sphere,A2 surrrounded by a
large enclosure A3

458
Solution:Refer Fig. E10.23(a):

A1 and A2 form an enclosure. Hence F1-1 + F1-2 = 1


and F2-1+ F2-2 = 1.
Since A2 is a flat surface F2-2 = 0. Hence F2-1 = 1. But A1F1-2 = A2F2-1.
Therefore F1-2 = (A2 / A1)F2-1 = (A2 / A1) =

Refer to Fig. E10.23(b) :


All the radiation from (1) which goes to the surroundings can be intercepted by an imaginary
surface 21 as shown in Fig. Now 1 and 21 form an enclosure. Therefore from the above
example
1 1
F21 = F21 = 1; F1 2 = F1 2 = ; F11 =

Refer to Fig. E10.23(c):

All the radiation from (1) which goes to the surroundings can be intercepted by an imaginary
surface 2 and 4 as shown in Fig. Now, 1, 2 and 4 form an enclosure.

F11 + F1 2 + F1 4 = 1
but F1 4 = F1 2

459
F11 = 1 2 F1 2 (a )
also F21 + F2 2 + F2 4 = 1 and F2 2 = 0
F21 + F24 = 1 F21 = 1 F2 4
from chart for two parallel coaxial discs ( 2 ) and ( 4 )
F2 4 = 0.383
F21 = 1 0.383 = 0.617
A2 F21 R 2 1.0
F1 2 = = 0.617 = 0.617 = 0.309
A1 2 RL 2 1.0
F11 = 1 2 0.309 = 0.383
F13 = 1 F11 = 1 0.383 = 0.617

Refer to Fig. E10.23(d)

F11 + F1 2 + F1 3 = 1 and F11 = 0; F1 3 = 0


F1 2 = 1 ____________________( a )
F2 1 + F2 2 + F2 3 = 1 ___________(b )
F31 + F3 2 + F3 3 = 1 and F3 3 = F31 = 0
F3 2 = 1 ____________________(c )
A3 F3 2 = A2 F2 3
1.52 0.52
F3 2 =
A3
F2 3 = 1 = 0.444
A2 2 (1.5 2 )
A2 F2 1 = A1 F1 2
A1 ( 0.52 )
F2 1 = F1 2 = 1 = 0.055
A2 2 (1.5 2 )
From eq (b), F2 2 = 1 ( F2 1 + F2 3 )
= 1 ( 0.055 + 0.444 )
F2-2 =0.5

460
Refer to Fig. E10.23(e):
Surroundings (3) can be replaced by imaginary surfaces (5) and (4) as shown in
Fig.below.Now, 1, 2, 4 and 5 form an enclosure.Hence

F1-1 + F1-2 + F1-4 + F1-5 = 1, and F1-1 = F1-4 = 0.

Hence F1-2+ F1-5 = 1.(a)

A5 F2-1 + F2-2 + F2-4 + F2-5 = 1, and F2-5 = F2-4 + F2-4


A2
Hence F2-2+ 2F2-5 = 1.(b)
A1

F4-1 + F4-2 + F4-4 + F4-5 = 1, and F4-1 = F4-4 = 0.

Hence F4-2+ F4-5 = 1(c)


A4
F5-1 + F5-2 + F5-4 + F5-5 = 1, and F5-5 = 0.

Hence F5-1 + F5-2 + F5-4 = 1

Or F5-2 + F5-(1+4) = 1..(d)

To find F1-5: L/r1 = 2.0 / 0.5 = 4 ; r5 / L = 1.5 / 2 = 0.75 ;


From chart F1-5 = 0.35.

From Eq.(a) we have F1-2= 1 0.35 = 0.65

To find F5(1+ 4) :
d 3
R= = = 0.75
2L 2 2
2 R 2 + 1 2 0.752 + 1
X= = = 3.78
R2 0.752
X X 2 4 3.78 3.782 4
F(1+ 4 )5 = F5(1+ 4) = =
2 2
or F5(1+ 4) = 0.286
From Eqn (d) F5 2 = 1 0.286 = 0.714

461
AF (1.5 ) 2

F2 5 = 5 5 2 = 0.714 = 0.268
A2 2 1.5 2
From (b) F2 2 = 1 2 0.268 = 0.465
A1 F15
Now F5 4 = F5(1+ 4 ) F51 = F5(1+ 4)
A5
( 0.52 )
= 0.286 0.35 = 0.247
(1.52 )

AF (1.5 ) 2

F4 5 = 5 5 4 = 0.247 = 0.278
A4 (1.52 0.52 )
From (c) F4 2 = 1 0.278 = 0.722
A4 F4 2 (1.5 0.5 )
2 2

F2 4 = = 0.722 = 0.24
A2 2 1.5 2
F21 = F2 5 F2 4 = 0.268 0.240
F2 1 = 0.028
F23 = F2 5 + F2 (1+ 4) = 2 F2 5 = 2 0.268
F2 3 = 0.536

10.8 RADIATION HEAT EXCHANGE BETWEEN FINITE SURFACES

10.8.1 RADIATION HEAT EXCHANGE BETWEEN FINITE BLACK SURFACES

Consider two black surfaces of area A1 and A2 and at temperatures T1 and T2 as shown in Fig
10.14. Then radiation leaving A1 and reaching A2 can be written as

462
A2,T2, Eb2

A1, T1,Eb1

Fig. 10.14: Radiation Heat Exchange Between Two Black Surfaces

Q1->2 = A1J1F1-2 Since A1 is a black surface, J1 = Eb1 thus,

Q1->2 = A1Eb1F1-2

Similarly radiation leaving A2 and reaching A1 is given by


Q2->1 = A2Eb2F2-1

Net heat exchange between A1 and A2 can be written as


Q12 = Q1->2 Q2->1
= A1F1-2Eb1 A2F2-1Eb2

But A1F1-2 = A2F2-1

Thus Q1-2 = A1F1-2 (Eb1 Eb2)

Q1-2 = A1F1-2(T14 T24) -----------------(10.27)

10.8.2 RADIATION HEAT EXCHANGE BETWEEN FINITE GREY SURFACES (NET


WORK METHOD)

The calculation of radiation heat transfer between black surfaces is relatively easy because all
the radiant energy which strikes a surface is absorbed by it. When non black bodies are
involved, the situation is much more complex because all the energy striking a surface will
not be absorbed: part will be reflected back to another heat transfer surface, and part may be

463
reflected out of the system entirely. The problem can become complicated because the
radiant energy can be reflected back and forth between heat transfer surfaces several times.
While deriving the expression for radiation exchange between any two finite grey
surfaces the following assumptions are made
i. All the surfaces are diffuse and uniform in temperature.
ii. The reflection and emissive properties are constant over all the surface
iii. Radiosity and irradiation are uniform over each surface. This assumption is not
strictly correct even for ideal grey diffuse surfaces, but the problems become
exceedingly complex when this restriction is not imposed.
iv. The surfaces are opaque (i.e. Transmissivity is zero)

Now the net radiation from a surface is given by

Qr = A(J G).
But J = E + (1 )G.
Assuming = , J = Eb + (1 )G. Or G = (J - Eb) / (1 )

Hence expression for Qr can be written as follows:

Qr = A [J (J - Eb) / (1 )]

(Eb J) (Eb J)
Qr = ---------------- = ---------------- ..(10.28)
(1 )/(A) R

Eq. 10.28 can be interpreted as follows (Eb-J) can be thought of as thermal potential, R =
(1-)/A can be thought of as thermal resistance offered by the surface for radiation, as Qr is
the radiation heat flow rate. Therefore a radiating surface can be replaced by an element as
shown in Fig 10.15

Qr

Eb R = (1 )/(A) J

Fig 10.15: Element representing surface resistance in the Radiation network method

Now let us consider the radiation heat exchange between two surfaces A1 and A2. Radiation
which leaves A1 and strikes A2 is given by

Q12 = A1J1F1-2

Similarly Radiation which leaves A2 and strikes A1 is given by

464
Q21 = A2J2F2-1

Therefore net radiation heat transfer from A1 to A2 is given by

Q1-2 = Q12 Q21 = A1J1F1-2 - A2J2F2-1

But A1F1-2 = A2F2-1.


(J1 J2)
Hence Q1-2 = A1F1-2(J1 J2) = ---------------- (10.29)
1 / (A1F1-2)

Eq. (10.29) can be represented by an element as shown in Fig 10.16

J1 J2

R12 = 1 / (A1F12)

Fig. 10.16: Element representing space resistance in radiation network method

The two network elements shown in Fig 10.15 and 10.16 represent the essentials of radiation
network method. To construct a network for a particular radiation heat transfer problem we
need only to connect a surface resistance (1-)/A to each surface and a space resistance
1/(AiFi-j) between the radiosity potential potentials. This is illustrated below.

10.8.3 NETWORK METHOD FOR RADIATION HEAT EXCHANGE BETWEEN TWO


PARALLEL INFINITE GREY SURFACES

The radiation network for the above problem will be as shown in Fig 10.17

J1
T1, 1= 1,A1 G1 = J2

G2 = J1 J2
T2, 2 = 2, A2

Eb1 J1 J2 Eb2

Qr1 Q12 R12 Q21 R2 Qr2


R1

Fig 10.17 Radiation network for two parallel infinite grey surfaces

465
(Eb1 Eb2)
From Fig. 10.17 we can write Qr1 = Q12 = ------------------------
(R1 + R12 + R2)

(T14 T24)
Or Q12 = ---------------------------------------------------------------------
{(1 1) / (A11) + 1 / (A1F12) + (1 2) / (A22)}

For two infinite parallel surfaces A1 = A2 and F12 = 1.0. Hence the above expression
simplifies to

A1 (T14 T24) A1 (T14 T24)


Q12 = ----------------------------------------- = --------------------------------
{(1 1) / 1 + 1 + (1 2) / 2} { (1 / 1) + (1 / 2) 1}

an expression which has already been derived (Refer Eq. 10.15).

10.8.4: NETWORK FOR RADIATION HEAT EXCHANGE BETWEEN TWO


PARALLEL INFINITE GRAY SURFACES IN PRESENCE OF A RADIATION SHIELD

T1, 1= 1,A1
T3, 13, A3 = A1=A2

T3, 23, A3 T2, 2 = 2, A2

R1 R13 R3 R3 R32 R2
Qr1 J1 J13 Eb3 J32 J2 Eb2
Eb1

Fig: 10.18 Radiation Net work forHeat Exchange Between Two Parallel Infinite Gray
surfaces in presence of a radiation shield

From Fig 10.18 the net radiation heat transfer from A1 to A2 is given by

(Eb1 Eb2)
Q12 = ----------------------------------------------
[R1 + R13 + R3 + R3 + R32 + R2]

466
(Eb1 Eb2)
= ---------------------------------------------------------------------------------------------------
[(1 1) / (A11) + 1 / (A1F13) + (1 13) / (A313) + (1 32) / (A332) + 1 / (A3F32)

+ (1 2) / (A22)]

But F13 = F32 = 1 and A1 = A2 = A3. Therefore we have

A1(T14 T24)
Q12 = ------------------------------------------------------------------------------------------
[{(1 1) / 1} + 1 + {(1 13) / 13} + {(1 32) / 32}+ 1 + {(1 2) / 2}]

A1(T14 T24)
= -------------------------------------------------------------------- ..(10.30)
[ { (1 / 1) + (1 / 2) 1 } + { (1 / 13) + (1 / 32) 1 }]

10.8.5: RADIATION NETWORK FOR HEAT EXCHANGE BETWEEN TWO


SURFACES, ONE SURFACE COMPLETELY ENCLOSING THE OTHER (The
enclosed surface cannot see itself)

A2, T2, 2
A1, T1, 1

Fig. 10.19(a): A1 is completely enclosed by A2 and A1 cannot see itself

Let A1 be completely enclosed by A2 as shown in Fig.10.19(a). Then we have

F1-1 + F1-2 = 1 and F1-1 = 0.

Hence F1-2 = 1 and therefore

F2-1 = A1/A2. The net work is shown in Fig.10.19(b)

467
(Eb1 Eb2) (T14 T24)
Qr1 = Q1-2 = -------------------- = -----------------------------------------------------------
(R1 + R12 + R2) [(1 1)/(A11) + 1/(A1F1-2) + [(1 2)/(A22)]

(T14 T24)
= ------------------------------------- (10.31)
[(1/1) + (A1/A2){(1/2) 1}]

Eb1 J1 J2 Eb2

Qr1 Q12 R12 Q21 R2 Qr2


R1

Fig. 10.19 (b) : Radiation network for the configuration shown in Fig. 10.19(a)

10.8.6: NETWORK METHOD FOR THREE ZONE ENCLOSURE

The network method described above can be readily generalised to enclosures


involving three or more zones. However when there are more than three zones, the analysis
becomes more involved and it is preferable to use the more direct Radiosity Matrix
method. The radiation network for a three zone enclosure shown in Fig 10.20(a) is shown in
Fig 10.20(b)

A1, 1, T1

A2, 2 T2

A3, 3 T3

Fig. 10.20 (a) : Radiation in a three zone enclosure

468
Fig. 10.20 (b) : Radiation Network for a three zone enclosure

Reradiating Surface: In many practical situations one of the zones may be thermally
insulated. In such a case, the net radiation heat flux in that particular zone is zero, because
that surface emits as much energy as it receives by radiation from the surrounding zones.
Such a zone is called a RERADIATION ZONE or an ADIABATIC ZONE. Fig 10.21(a)
represents a three zone enclosure with surface (3) being the reradiating surface and Fig
10.21(b) the corresponding radiation network.

A1, 1, T1

A2, 2 T2

Reradiating
surface ,A3, T3

Fig. 10.21 (a) : Radiation in a three zone enclosure with A3 being a reradiating surface

469
Fig. 10.21 (b) : Radiation network for the configuration shown in Fig. 10.21(a)

For a three zone enclosure under steady state conditions, by I law of thermodynamics.

Qr1 + Qr2 + Qr3 = 0 ..(10.32)

If A3 is a reradiating surface, then Qr3 = 0.

Therefore Qr1 = - Qr2 = (Eb1 Eb2) / Req .(10.33a)

Where Req = R1 + [(1/R1) + {1/ (R13 + R23)}] 1

(1 1) (1 2)
i.e Req = ----------- + [ A1F12 + {1/(A1F1-3) + 1/(A2F2-3)} 1 ] 1 + ------------
A11 A22

. (10.33b)

10.9 ILLUSTRATIVE EXAMPLES ON NETWORK METHOD:

Example 10.24: Two square plates 1m x 1m are parallel to and directly opposite to each
other at a distance of 1m. The hot plate is at 800K and has an emissivity of 0.8. The clod
plate is at 600K and also has an emissivity of 0.8. The radiation heat exchange takes place
between the plates as well as the ambient at 300K through the opening between the plates.
Calculate the net radiation at each plate and the ambient.

Solution:

470
1m

A1, 1, T1 1m

1m 3

A2, 2, T2

Given: T1 = 800 K ; 1 = 0.8 ; T2 = 600 K; 2 = 0.8; T3 = 300 K


To find: Qr1 ; Qr2; Qr3

Solution: (1 0.8)
R1 = (1 1) / (A11) = ----------------- = 0.25
(1 x 1) x 0.8
(1 0.8)
Similarly R2 = (1 2) / (A22) = ----------------- = 0.25
(1 x 1) x 0.8
R3 = (1 3) / (A33)
A3 is the area of the surroundings which is very large. Hence R3 = 0.The radiation net work
for this problem is shown in Fig.E 10.24.

From chart F1-2 = F2-1 = 0.20. But F1-1 + F1-2 + F1-3 = 1 and F1-1 = 0.
Hence F1-3 = 1 F1-2 = 1 0.2 = 0.8 = F2-3.

R12 = 1 / (A1F1-2) = 1 / {(1x1) x 0.2} = 5.0. R13 = 1 / (A1F1-3) = 1 / {(1x1) x 0.8} = 1.25

R23 = 1 / (A2F2-3) = 1 / {(1x1) x 0.8} = 1.25

Eb1 = T14 = 5.67 x 10 8 x 800 4 = 23224 W / m2 = 23.224 kW/m2.

Eb2 = T24 = 5.67 x 10 8 x 600 4 = 7348 W / m2 = 7.348 kW/m2.

Eb3 = T234 = 5.67 x 10 8 x 300 4 = 459 W / m2 = 0.459 kW/m2.


For steady state radiation, radiation energy cannot accumulate at nodes.Hence

Qr1 = Q1-2 + Q1-3

Or Qr1 Q1-2 Q1-3 = 0

471

Eb1 J1

( J 1 J 2 ) J1 J 3 = 0
R1 R12 R13
23.224 J1 ( J1 J 2 ) ( J1 0.459 )
or = 0 _______ ( a )
0.25 5 1.25
similarly Qr2 = Q21 + Q23 Qr2 Q21 Q23 = 0
Eb2 J 2

( J 2 J1 ) J 2 Eb3
=0
R2 R12 R23
7.348 J 2 ( J 2 J1 ) ( J 2 0.459 )
= 0 _______ ( b )
0.25 5 1.25
Solving Eqn (a) and (b) simultaneously we get
J1 = 18.921KW / m 2 ; J 2 = 6.709 KW / m 2
Eb1 J1 23.224 18.921
Qr1 = = = 17.212 KW
R1 0.25
Eb2 J 2 7.348 6.709
Qr2 = = = 2.557 KW
R2 0.25
But Qr1 + Qr2 + Qr3 = 0 Qr3 = Qr1 + Qr2 = [17.212 + 2.557 ]
Qr3 = 19.769 KW

Example 10.25 The configuration of a furnace can be approximated as an equilateral


triangular duct which is sufficiently long that the end efforts are negligible. The hot wall is at
900K with an emissivity of 0.8 and the cold wall is at 400K with emissivity of 0.8. The third
wall is a reradiating wall. Determine the net radiation flux leaving the hot wall.

Solution:
A1 = A2 = A3 = 1m2
T1 =900K, 1 = 0.8
T2 = 400K, 2 = 0.8

The radiation network for the above problem will be as shown for example 10.24

472
1 1 1 0.8
R1 = = = 0.25
A1 1 1 0.8
1 2 1 0.8
R2 = = = 0.25
A2 2 1 0.8
Using Hottel's cross string formula, we have

F1 2 =
( A1 + A2 ) A3 = (1 + 1) 1 = 0.5 = F = F2 3
13
2 A1 2 1
1 1
R12 = = = 2 = R23 = R13
A1 F1 2 1 0.5
1
1 1
Req = R1 + + + R2
R
12 R13 + R23
1
1 1
= 0.25 + + + 0.25 = 1.833
2 2 + 2
Eb1 Eb2 (T14 T24 ) 5.67 108 9004 4004
Qr1 = = =
Req Req 1.833
= 19503 W / m 2
Qr1 + Qr2 + Qr3 = 0 and Qr3 = 0 Qr2 = Qr1 = 19503 W / m 2

Example 10 .26 A short cylindrical enclosure is maintained at the temperatures as shown in


Fig P4.26. Assuming 2= 3=1; 1=0.8 determineQr1 and Qr2
From chart, F1-2 = 0.175 = F2-1 (A2=A1)
Also F1-1 + F1-2 + F1-3 = 1 and F1-1 = 0
So F1-3 = 1 F1-2 = 1 0.175 or
F1-3 = 0.825 = F2-3

The radiation network for the above problem will be as shown for example 10.24

473
1 1 1 0.8
R1 = = = 0.318
A11 ( 0.52 ) 0.8
1 2 11
R2 = = =0
A2 2 A2 1
1 1
R12 = = = 7.3
A1 F1 2 ( 0.5 ) 0.175
2

1 1
R13 = = = 1.54
A1 F1 2 ( 0.5 ) 0.825
2

1 1
R23 = = = 1.54
A2 F23 ( 0.5 ) 0.825
2

1
1 1
Req = R1 + + + R2
R12 R13 + R23
1
1 1
= 0.318 + + + 0 = 2.484
7.3 1.54 + 1.54
Eb1 Eb 2 (T1 T2 ) 5.67 10 2000 1000
4 4 8 4 4

Qr1 = = =
Req Req 2.484
= 329.14 103 W = 329.14 KW
Qr1 + Qr 2 + Qr 3 = 0 and Qr 3 = 0
Qr 2 = Qr1 = 329.14 KW

Example 10.27 A spherical tank with diameter 40cm fixed with a cryogenic fluid at 100K is
placed inside a spherical container of diameter 60cm and is maintained at 300K. The
emissivities of the inner and outer tanks are 0.15 and 0.2 respectively. A spherical radiation
shield of diameter 50cm and having an emissivity of 0.05 on both sides is placed between the
spheres. Calculate the rate of heat loss from the system by radiation and find also the rate of
evaporation of the cryogenic liquid if the latent heat of vaporization of the fluid is 2.1x105
W-s/Kg

Solution: The schematic and the corresponding network for the problem will be as shown in
Fig P.4.27

474
Eb1 Eb 2
Qr1 = Q12 =
R1 + R13 + 2 R3 + R32 + R2
(T14 T24 )
=
1 1 1 1 3 1 1 2
+ +2 + +
A11 A1 F13 A3 3 A3 F3 2 A2 2
A1 (T14 T24 )
=
1 A1 2 A1 1
+ 1 + 1
1 A3 3 A2 2
5.67 108 4 ( 0.22 ) 1004 3004
= 2 2
= 6.83 W
1 40 2 40 1
+ 1 + 1
0.15 50 0.05 60 0.2
Q 6.83
Evaporation Ratio = r1 = 5
= 3.25 105 Kg / s
hfg 2.1 10

475
4.10 RADIOSITY- MATRIX METHOD FOR RADIATION IN ENCLOSURES

The network method is quite easy to apply for determining the radiation exchange in
enclosures having not more than 3 zones. As the number of zones forming the enclosure
increases, the manipulations involved in the network method becomes enormous and the
method is not so practical. Whereas the radiosity-matrix method is very straight forward and
the method transforms the problem to the solution of an algebraic matrix equation for the
unknown radiosities Ji [i=1,2,3,.N] once these equations are solved for Ji then the net
radiation flux or the zone temperature at any zone (i) can be immediately determined. This is
illustrated below.
Consider an enclosure made up of N zones.Let Gi be the irradiation at zone i
Let Ji be the Radiosity at zone i .Net radiation heat flux from zone i is given by

qi = Qri / Ai .(10.34)

Radiation leaving Aj and striking Ai is given by

Qj i = AjFj-i Jj = AiFi-j Jj

Radiation leaving all zones and reaching zone i is therefore given by


N N
Qj-I = Ai Fi-j Jj
j=1 j=1

N
Therefore irradiation for zone i = Gi = Qj I / Ai = Fi-j Jj .(10.35)
j=1

Now qi = Ji Gi. Substituting for Gi from Eq. (10.35) we get


N
qi = Ji - Fi-j Jj .(10.36)
j=1

(Ebi Ji)
Also qi = ----------------- (10.36a)
{(1-i) / i}

From Eq. (10.36) and Eq. (10.36a) it follows that


N
Ji (1 i)[Ji Fi-j Jj
j=1

Ebi = ----------------------------------------- ..(10.37)


i
i= 1,2,3,N

476
Equations (4.36) and (4.37) provide the fundamental relations for obtaining a system of N
algebraic equations to determine the N unknown radiosities. Once the radiosities are known
the net radiation heat flux qi at any zone (i) can be computed using either Eq. 4.36 or 4.36a

The solution depends on the prescribed conditions for each of the zones. Two
situations are of practical interest
1. Temperatures are prescribed for each of the N zones
2. Temperatures are prescribed for some of the zones and the net radiation flux are
prescribed for the remaining zones

i. Temperatures prescribed for all the zones

Consider Eq. 10.37


N
J i (1 i ) Fi j J j
j=1
Ebi = ( i = 1,2,3 ... N )
i
And Ebi = Ti4 is known because Tis are prescribed. Therefore the above set of equations
can be solved for unknown radiosities J i ( i = 1, 2,3 ... N ) and knowing Ji the net radiation
flux qi can be determined from Eq. 4.36a

Eq 10.37 can be rewritten as follows:


N
( Ji / i) {(1 i) / i}Fi-jJj = Ti4 (10.38)
j=1

Eq. (10.38) is of the form


[M] {J} = {T} .(10.39a)

m11 m12 m13 .m1N


Where m21 m22 m23 .m2N
[M] = - (10.39b)
-
-
mN1 mN2 mN3 .mNN

J1
J2
{J} = - (10.39c)
-
JN

T1
T2
-
{T} = - (10.39d)

477
-
TN

The elements mij of matrix [M] can be determined from the following:
ij (1 i) Fi-j
mij = ------------------------- , i = 1 to N and j = 1 toN.(10.39e)
i

Where ij = 1 for i = j and = 0 for i j. (10.39f)

ij is known as Kronecker delta.

ii. Temperature prescribed for some zones and net heat flux prescribed for others
In many practical situations, temperatures are prescribed for some of the zones and
net heat fluxes for the remaining zones of an enclosure. In such problems we have to
determine the net heat fluxes for the zones for which temperatures are specified and
temperatures for the zones for which the net heat fluxes are prescribed. This can be done
using the same equations (10.36) and (1037) and illustrated below.
Let us assume that temperatures Ti are prescribed for zones i=1, 2, 3 .k and the net heat
fluxes qi are prescribed for the remaining zones i=k+1, k+2.n
For zones 1 to k since temperatures are prescribed we can use eq. 10.38 that is

N
( Ji / i) {(1 i) / i}Fi-jJj = Ti4; i= 1 to k (10.40)
j
=1

For zones i = k+1, k+2, N, with prescribed heat fluxes we can use Eq (4.36) namely
N
qi = Ji - Fi-j Jj ; 1 = k+1, k+2, N (10.40a))
j=1

It is more convenient to express Eq. (10.40) and (10.40a) in matrix form as follows.

[M] {J} = {S} (10.41a)


Where [M] is given by Eq.(10.39b) with

ij (1 i) Fi-j
mij = ------------------------- , i = 1 to k and j = 1 to k..(10.41b)
i

mij = ij Fi-j; 1 = k+1 to N and j = k+1 to N .(10.41c)

{J} is as given by Eq.(10.39c) . {S} is given by


T 1 4
T 2 4
-
-
{S} = Tk4 ..(10.41d)

478
qk+1
qk+2
-
qN
Ones these equations are solved for unknown radiosities Ji, then the unknown radiation
fluxes can be determined from the equation

i
qi = --------- [Ebi Ji] ; i = 1,2,3, .k
(1 i)
.(10.41e)
10.10: ILLUSTRATIVE EXAMPLES ON RADIOSITY MATRIX METHOD

Example 10.28:- Solve Example 10.24 using radiosity matrix method.

Solution:Given: T1 = 800K, 1 = 0.8; T2 = 600K, 2 = 0.8;T3 = 300K, J3 = Eb3 = T34

F1-2 = F2-1 = 0.2;F1-1 = F2-2 =0;F1-3 = F2-3 = 0.8

Since J3 is already known, we have to solve only for J1 and J2. Thus the matrix form of
equation for radiosities J1 and J2 can be written as

479
m11 m12 J1 T14
m =
21 m22 J 2 T24
ij (1 i ) Fij
mij =
i
where ij = 1 for i = j
= 0 for i j
1 (1 0.8 ) 0
m11 = = 1.25; T14 = 5.67 108 8004 = 23224W / m 2
0.8
0 (1 0.8 ) 0.2
m12 = = 0.05; T24 = 5.67 108 6004 = 7348W / m 2
0.8
0 (1 0.8 ) 0.2
m21 = = 0.05
0.8
1 (1 0.8 ) 0
m22 = = 1.25
0.8
1.25 0.05 J1 23224
=
0.05 1.25 J 2 7348
i.e,
1.25 J1 0.05 J 2 = 23224
0.05 J1 + 1.25 J 2 = 7348
Solving for J1 and J 2 we get J 2 = 6632W / m 2 and J1 = 18845W / m 2
Eb1 J1 23224 18845
qr1 = = = 17516W / m 2
1 1 1 0.8

1 0.8
E J 2 7348 6632
qr 2 = b 2 = = 2864W / m 2
1 2 1 0.8

2 0.8
qr1 + qr 2 + qr 3 = 0 qr 3 = [17516 + 2864] = 20380W / m 2
Example 10.29:- Solve example 10.25 by radiosity-matrix method.

Solution: Given A1 = A2 = A3 = 1 m2 ; T1 = 900 K ; T2 = 400 K ; Qr3 = 0 ; 1 = 0.8 ;

2 = 0.8;
This is a case wherein temperature is specified for 2 zones and radiation flux is specified for
the remaining zone.
ij (1 i) Fij
mij = ------------------------ for i = 1,2 and mij = ij Fij for i = 3
i
0 (1 0.8) x 0.5
Therefore m11 = (1 0) / 0.8 = 1.25 ; m12 = ------------------------- = - 0.125 = m21
0.8

480
(1 0) 0 (1 0.8) x 0.5
m22 = ------------ = 1.25 ; m13 = ------------------------ = 0.125 = m23
0.8 0.8

m31= 0 F31 = 0.5 ; m32 = 0 F32 = 0.5 ; m33 = 1 F33 = 1.0

The radiosity matrix equation can now be written as follows:

T1 4
[mij]{Ji} = {S} where {S} = T24
0
In expanded form the above equation can be written as:

m11J1 + m12J2 + m13J3 = T14

m21J1 + m22J2 + m23J3 = T24

m31 J1 + m32 J2 + m33 J3 = 0

Substituting the numerical values fo mij , T1, T2 and the above three equations can be
solved for J1, J2 and J3.

481
CHAPTER 11: MASS TRANSFER
11.1 INTRODUCTION: Mass transfer can result from several different phenomena. There
is a mass transfer associated with convection, where in, mass is transported from one place to
another in the flow system. This type of mass transfer occurs on a macroscopic level and is
normally treated in the subject of fluid mechanics. When a mixture of gases or liquids is
contained such that, there exists a concentration gradient of one or more of the constituents of
the system, there will be a mass transfer on a microscopic level as the result of diffusion from
regions of high concentration to regions of low concentration.
This chapter deals with some of the simple relations which may be used to calculate mass
diffusion and their relation to heat transfer. However it should be remembered that the
general subject of mass transfer encompasses both mass diffusion on a molecular scale and
the bulk mass transport which may result from a convection process.
Mass diffusion not only occurs on a molecular scale, but also in turbulent systems
where accelerated diffusion rates will occur as a result of the rapid-eddy mixing process.
Mass diffusion may also result from a temperature gradient in a system; this is called
thermal diffusion. Similarly, a concentration gradient can give rise to a temperature gradient
and a consequent heat transfer.

11.2. FICKS LAW OF DIFFUSION: Consider the system shown in Fig. 11.1. A thin

A thin partition separates two gases A and B.


When the partition is removed, the two gases
diffuse through each other until equilibrium
A B is established and the concentration of the
gases is uniform throughout the box. The rate
at which the diffusion rate takes place is given
Fig. 11.1: Diffusion of gas A by Ficks law of diffusion, which states that
into gas B the mass flux off of a constituent per unit area
is proportional to the concentration gradient.

Therefore mA
------ = D (CA / x) (11.1)
A

Where D = constant of proportionality and is called diffusion coefficient,


expressed in m2/s ;
mA = mass flux of component A per unit time, kg/s;

CA = mass concentration of component A per unit volume, kg/m3.

482
An expression similar to Eq.(11.1) could also be written for the diffusion of the constituent
A in either y or z direction.

Similarity between Eq.(11.1) and the Fourier law of heat conduction

(Q /A)x = k (T / x)

and the equation for shear stress between fluid layers,

= (u / y)

can be noticed. The heat conduction equation describes the transport of energy, the viscous-
shear equation describes the transport of momentum across the fluid layers, and the Ficks
diffusion equation describes the transport of mass.

To understand the physical mechanism of diffusion, consider the imaginary plane


shown by the dashed line in Fig. 11.2.The concentration of component A is greater

CA

(mA / A)x = D(CA/x)

Fig.11.2: Sketch illustrating diffusion dependence on


concentration profile

on the left hand side of this plane than on the right hand side. A higher concentration means
that there are more molecules per unit volume. If the system is a gas or a liquid, the
molecules are moving in a random fashion, and the higher the concentration, more
molecules will cross a given plane per unit time. Thus, on the average, more molecules are
moving from left to right across the plane than in the opposite direction. This results in a net
mass transfer from the region of high concentration to the region of low concentration.
In gases the diffusion rates are clearly dependent on the molecular
speed, and consequently we should expect a dependence of the diffusion coefficient on
temperature since temperature indicates the average molecular speed.

483
11.3. DIFFUSION IN GASES: GILLILAND has proposed a semi-empirical equation for
the diffusion coefficient in gases:

T3/2 __________________
D = 435.7 ---------------------------- [ (1 / MA) + (1 / MB) ]
p [ VA1/3 + VB1/3 ]

.(11.2)

In the above equation D is in cm 2/s, T is in degrees Kelvin, p is the total system pressure in
pascals, and VA and VB are the molecular volumes of constituents A and B and MA and MB
are the molecular weights of constituents A and B.

Equation (11.2) offers a convenient expression for calculating the diffusion


coefficients for various compounds and mixtures, but should not be used as a substitute for
experimental values of the diffusion coefficients when they are available for a particular
system.
With reference to Fig.11.1, it can be stated that the diffusion process is
occurring in two ways at the same time; that is gas A is diffusing into gas B and at the same
time gas B is diffusing into gas A. We , therefore, could refer to the diffusion coefficient for
either of these processes.
While working with Eq. (11.1), one can use mass flux per unit area and
mass concentrations, or the equation may be expressed in terms of molal concentrations and
fluxes. There is no general rule to say which type of expression will be most convenient, and
the specific problem under consideration will determine the one to be used. For gases Ficks
law may be expressed in terms of partial pressures by making use of the ideal-gas equation of
state
The ideal-gas equation of state can be written as

p = RT (11.3)

The density represents the mass concentration to be used in Ficks law. The gas _
constant R for a particular gas may expressed in terms of the universal gas constant R and the
molecular weight of the gas: _
R
RA = ------- .(11.4)
MA

_
Where R = 8314 J/(kg-mol-K)
_
or R = 0.08205 m3.atm/(kg-mol K)

484
Consequently Ficks law of diffusion for component A into component B can be written as
follows:
_
CA = A = (pAMA) / (R T)
_
Hence ( CA / x) = [MA / (R T)] (dpA/dx)

if isothermal diffusion is considered. Substituting this expression in Eq.(11.1) we have

mA _
------ = DAB[MA / (R T)] (dpA/dx) ..(11.5)
A

Similarly we can also write for the diffusion of component B into component A as

mB _
------ = DBA[MB / (R T)] (dpB/dx) ..(11.6)
A
Equation (11.6) also assumes isothermal diffusion process.

Equimolal counter diffusion in gases:- Consider a physical situation as shown in Fig. 11.3:
NA and NB represent the steady-state molal diffusion rates of components A and B,

Reservoir of A Reservoir of B

NA NB

x
1 2

Fig. 11.3: Sketch illustrating equimolal diffusion

respectively. In this steady-state situation each molecule of A is replaced by a molecule of B,


and vice versa. The molal diffusion rates are given by

mA _
NA =------ = DAB [A / (R T)] (dpA/dx)
MA

485
mB_
NB =------ = DBA[A / (R T)] (dpB/dx)
MB

The total pressure of the system p remains constant at steady state, so that

p = pA + pB = constant

where pA and pB are partial pressures of gas A and gas B respectively.

Therefore it follows that (dpA/dx) + (dpB/dx) = 0

or (dpA/dx) = (dpB/dx) .(11.7)

Since each molecule of A is replacing a molecule of B, the molal diffusion rates are equal:

N A = NB

Therefore DAB [A / (R T)] (dpA/dx) = DBA[A / (R T)] (dpB/dx)

Using Eq. (11.7), the above equation can be written as

DAB = DBA = D (11.8)

Integration of Eq.(11.5) between section 1 and section 2 of Fig. 11.3 we get

mA DMA (pA2 pA1)


----- = ---_---- ------------------ ..(11.9)
A RT x

Illustrative example 11.1:- Consider two large vessels, each containing uniform mixtures of
nitrogen and carbon dioxide at 1 atm, and 288.9 K, but at different concentrations. Vessel 1
contains 90 mole percent N2 and 10 mole percent CO2, where as vessel 2 contains 20 mole
percent N2 and 80 mole percent CO2. The two vessels are connected by a duct of 0.1524 m
inside diameter and length of 1.22 m. Determine the rate of transfer of nitrogen between the
two vessels by assuming that steady-state transfer takes place. The mass diffusivity for the N2
CO2 mixture at 1 atm and 288.9 K is 0.16 x 10 4 m 2/s.

Solution: In this mass transfer process, N2 is transferred from vessel 1 containing higher
concentration of N2 to vessel 2 having lower concentration. In the early stages of mass
transfer, the partial pressure of N2 in both vessels is considered to remain constant so that
steady-state transfer can be assumed. Then the mass transfer process can be characterized as
an equimolal counter diffusion as described above. Hence the mass flux of N2 can be
determined by using Eq. (11.9)

486
Data:- d= 0.1524 m ; length = x = 1.22 m; D = 0.16 x 10 4 m2/s;
_
R = 0.08205 m3.atm/(kg-mol K) ; (pN2)1 = 0.9 x 1 = 0.9 atm; (pN2)2 = 0.2 x 1 = 0.2 atm

The mass transfer rate of N2 is given by

DMN2 A[(pN2)2 - (pN2)1]


mN2 = ---_---- -----------------------
RT x

0.16 x 10 4 x 28.013 x (/4)x 0.1524 2 x [0.2 0.9]


= --------------------------------------------------------------------- = 2.012 x 10 7 kg/s
0.08205 x 288.9 x 1.22

11.4. Diffusion of vapour through a stationary gas (STEFAN Flow):Many engineering


applications such as heat pipes, cooling ponds, and the familiar perspiration involve
condensation, evaporation, and transpiration in the presence of a non condensable gas, and
this process is nothing but diffusion of a vapour through a stationary (stagnant) gas.To
analyse such a process, consider the isothermal evaporation of water from a surface and the
subsequent diffusion through a stagnant layer as shown in Fig. 11.4. The free surface of the
water is exposed to air in the tank, as shown. We assume that the system is isothermal and
the total pressure of the system remains constant. We further assume that the system is in
steady state. This requires that there be a slight air movement over the top of the tank to
remove the water vapour that diffuses to that point. Whatever
x
Air
2

pw pA

1
Water
p

Fig. 11.4: Diffusion of water vapour into air

air movement may be necessary to accomplish this, it is assumed that it does alter the
concentration profiles in the air in the tank.We further assume that air and water vapour
behave as ideal gases.

487
As the water evaporates, it will diffuse upward through the air, and at
steady state this upward movement must be balanced by a downward diffusion of air so that
the concentration at any x will remain constant. But at the surface of the water there can be
no net mass movement of air downward. Consequently there must be a bulk mass movement
upward with a velocity just large enough to balance the diffusion of air
downward. This bulk mass movement then produces an additional mass flux of water vapour
upward. The diffusion of air downward is given by (see Eq. 11.5)

_
mA = DAB A[MA / (R T)] (dpA/dx) ..(11.10)

where A denotes the cross sectional area of the tank. This must be balanced by the bulk-mass
transfer upward. If u is the bulk-mass velocity then
pAMA
mA = AAu = -------- Au
RT

DAW
or u = -------- (dPA/dx) .(11.11)
pA

Mass diffusion of water vapour is given by

_
(mw)diffusion = DWA A[MW / (R T)] (dpW/dx) (11.12)

and bulk transport of water vapour is given by

pWMW
(mw)bulk = WAu = ---_-------- Au
RT

Substituting for u we get


pWMW DAW
(mw)bulk = WAu = ---_------------ (dpA/dx) A
RT pA

(mW)total = (mw)diffusion + (mw)bulk

_ pWMWDAW
= DWA A[MW / (R T)] (dpW/dx) + --------_-------------(dpA/dx) A
R T pA

488
Also p = pW + pA, p = constant. Hence (dpA/dx) = (dpW/dx) ; DWA = DAW = D. Hence the
expression for (mW)total reduces to

D A MW
(mW)total = --_-------- (dpW/dx) = [pA + pW] / pA
RT

D A MW
or (mW)total = --_-------- (dpW/dx) p/ (p pW) ..(11.13)
RT

Equation 11.13 is called Stefans law. It may be integrated to give

D p MW AD p MW A
(mW)total = --_---------------- ln [(p pw2) / (p pw1) = --_-------------- ln [pA2 / pA1]
R T (x2 x1) R T (x2 x1)

.(11.14)

Illustrative example 11.1:- Estimate the diffusion rate of water from the bottom of a test tube
10 mm in diameter and 15 cm long into dry atmospheric air at 250C.
_
Solution:Data:- d = 0.01 m; (x2 x1) = 0.15 m; T = 25 + 273 = 298 K;R = 8314.3N-m/kMol-
K ;p = 1atm = 101.32 kPa ;pw1 = saturation pressure at 250C = 3.166 kPa

From tables for diffusion of water into air D = 0.256 cm 2/s;


We use Eq.(11.14) to calculate the mass flux. The partial pressure of water vapour at the
bottom of the test tube is the saturation temperature of water at 250C and at
the top the partial pressure of water vapour is zero as the air is dry (i.e. pw2 = 0).
Substituting the values in Eq.(11.14) we have

(0.256 x 10 4) x 18 x ( x 0.005 2)
(mw)total = ------------------------------------------- ln [(101.32 0) / (101.32 3.166)]
8314.3 x298 x 0.15

= 3.131 x 10 10 kg/s

11.5. Steady State Diffusion in Liquids:

Steady state Equimolal Counter Diffusion in Liquids:- The analysis for steady state counter
diffusion in liquids follows the same approach as that for gases.
In terms of number of mole flux of species A ,Ficks law can be written as

NA = (mA / MA) = D (CA / x) ..(11.15)

489
The integration of Eq.(11.15) from x = x1 to x = x2 with CA varying from CA1 to CA2,gives

(CA1 CA2 )
NA = D ---------------- .(11.16)
(x2 x1)

The above equation can also be expressed in terms of mole fractions of species
at locations x1 and x2 as follows.

Mole fraction at x1 = (CA1 / C ) = A1; Mole fraction at x2 = A2 = CA2 /C.

Where C = Total molal concentration of the mixture.

Eq.(11.16) can be written in terms of mole fraction as

(A1 A2)
NA = DC -------------- kg-mol /(m2-s) (11.17)
(x2 x1)

Steady State Unidirectional Diffusion in liquids:- The analysis for the determination of the
concentration distribution during steady state unidirectional diffusion in liquids is exactly
same as that for gases except that in the case of liquids the equation is derived in terms of
molal concentrations CA and CB instead of partial pressures.

If NA = molal flux of species A per unit area, then

C ( CB2 CB1) C ( CA1 CA2)


NA = D ---------------- ------------------ = D ----------------------------------- (11.18 a)
( x 2 x1 ) CB ln ( x2 x1 ) CB ln

Where the logarithmic mean concentration CB ln of component B is defined as

( CB2 CB1)
CB ln = -------------------- ..(11.18 b)
ln (CB2 / CB1)

Also C = CA + CB and CB2 CB1 = ( CA1 CA2). Equations (11.18) can also be written in
terms ox mole fractions:

C ( B2 B1) C ( A1 A2)
NA = D ---------------- ------------------ = D ----------------------------------- (11.19 a)
( x 2 x1 ) B ln ( x 2 x1 ) B ln

490
Where the logarithmic mean mole fraction B ln of component B is defined as

( B2 B1)
B ln = -------------------- ..(11.19 b)
ln (B2 / B1)

In Eqs.(11.18) and (11.19), the units of various quantities may be taken as

Ci = molal concentration, i = A, or B, kg-mol/m3

i = Ci / C= mole fraction of component i

C= molal concentration of the mixture, kg-mol/m3

D = mass diffusivity, m2/s

NA = molal flux of species A, kg-mol /(m2-s)

X = distance,m

11.6. Solution to tutorial problems:

11.1 Solution: Data:- p = 1 atm; T = 273 K; (CA)1 = 0.8;

(Cso2)1 = 0.20; (CA)2 = 0.30; (Cso2)1 = 0.70; d = 0.1 m ; x = 1.8 m

Now pA1 / p = (CA)1. Hence pA1 = 0.8 x 1 = 0.8 atm. Similarly pA2 = 0.3 x 1 = 0.3 atm
_
R = 0.08205 m3 atm / (kg mol K); MA = 28.86; D = 0.122

DMAA[pA2 - pA1]
mA = --- _---- -----------------------
RT x

0.122 x 10 4 x 28.86 x ( x 0.052) x [ 0.8 0.3]


= -------------------------------------------------------------
0.08205 x 273 x 1.8

= 3.43 x 10 8 kg / s.

11.2. Solution: Data:- T = 298 K; x2 x1 = 3 m; D = 0.0962 cm 2/s; pA1 = 0.01 atm;


A = 1 m2; p = 1 atm.MB = Molecular weight of benzene = 78

D p MB A
mB = _-------------- ln [pA2 / pA1]

491
R T (x2 x1)

0.0962 x 10 4x1.0 x 78 x 1.0


= ---------------------------------------------- ln [(1.0-0) /(1.0 0.01)]
0.08205 x 298 x 3

= 1.03 x 10 7 kg / (s m2)

492
493

Вам также может понравиться